Planetary Energy Balance

by Judith Curry

By Judith Curry

Here is another attempt at trying to untangle the Skydragons’ misunderstanding about the greenhouse effect and the planetary energy balance.

To clarify the planetary energy balance equation and its (mis) application to interpreting the greenhouse effect, I am posting section 14.3 [planetary energy balance] from my text Thermodynamics of Atmospheres and Oceans.  If you have been following the Postma thread, you should be familiar with the starting point in this section.

The planetary energy balance equation has its greatest utility in the context of comparative planetology.  It is not needed to prove the existence of the greenhouse effect, although it is conceptually useful in explaining and understanding the greenhouse effect.

With regards to the planetary energy balance equation 12.1:

The temperature Te is not necessarily the actual surface or atmospheric temperature of the planet; it is simply the equivalent blackbody emission temperature a planet requires to balance the solar radiation that it absorbs.  Using a value of ap = 0.31, we obtain from (12.1c) a value of Te = 254 K.  Note that this temperature is much less than the observed global mean surface temperature T0= 288 K.  The difference between Te and T0 arises from the emission of thermal radiation by atmospheric gases and clouds at temperatures colder than T0.

The ratio of Te to T0 provides an indication of the magnitude of the infrared optical depth, which is referred to as n.   n includes contributions from both clouds and gases.

So far nothing new.   In section 14.3.2  I introduce the concept of planetary time scales, which I think gets to the heart of the problem that Postma has with equation 12.1.

If radiative transfer is the only process occurring in a planetary atmosphere, the surface temperatures of the planets would be determined solely by the net radiation at the top of the atmosphere.  Thus, the equatorial regions would be warmest and the poles would be extremely cold.  Whereas this is the case on Mars and to a lesser extent on Earth, other planets such as Venus and all of the Jovian planets have little or no equator-to-pole temperature gradients at the surface or deep in their interiors.

Simple arguments allow us to understand why there are differences in horizontal temperature gradients among the planets.  There are two fundamental time scales that determine how a planetary atmosphere transfers heat.  The radiative timescale, trad, is the time it would take for an atmosphere above some pressure level p0 to reduce its temperature by 1/e of its initial value via radiative cooling if solar radiation were turned off.  The dynamic timescale, tdyn, is the time required to move a parcel over a characteristic distance in the atmosphere and, in so doing, transport heat from one location to another.

Introducing a variable  e = tdyn trad .   There are several different regimes for e:

(i)    If e >> 1, the dynamic time scale is much greater than the radiative time scale, and hence radiative processes dominate.  This regime is characteristic of Mars. Consequently, on a planet with e >> 1, the equator-to-pole temperature gradient is very large, in response the latitudinal variations of solar radiation. Also, as the radiative cooling of the atmospheric parcel is so efficient, a very large diurnal variation of temperature is expected.

(ii)  If e << 1, the dynamical transports of heat dominate over radiative cooling.  This regime, characteristic of Venus, Jupiter, and Saturn.  Thus, for e << 1 a very small or negligible equator-to-pole temperature difference is expected.  For similar reasons, the diurnal variation of temperature on such a planet would be negligibly small.

(iii) For e ~ 1, there is parity between the two time scales.  Such a situation occurs if the mass of the atmosphere is moderate and the velocities on the planet are relatively weak, which is the case for Earth. On such a planet,  one would expect a pole-to-equator temperature difference but one where the temperature of a parcel at a particular latitude is not in radiative equilibrium with the net radiative fluxes at the top of the atmosphere.  Furthermore, a moderate diurnal temperature is expected.

In this context, equation 12.1 provides a good representation for Venus, with little diurnal variation and pole-to-equator temperature gradient (Jupiter and Saturn have large internal heat sources, so equation 12.1 is not applicable).   For Venus, a time integration or spatially varying energy balance model would add nothing.

What Postma proposes (if implemented correctly) might make sense for Mars, with a large diurnal cycle and pole-to-equator gradient, with local thermodynamic balances in a column.  However, the fact that equation 12.1 does a good job of simulating the actual surface temperature of Mars (which has an optically thin atmosphere) implies that  added complexity is not necessary.

For the Earth, the issue with equation 12.1 is how to interpret Te and n.  There is no simple way to deconvolute the gaseous greenhouse effect from the radiative emission from clouds. Postma’s proposed model doesn’t help at all with the Earth, because of the complex coupling between the radiation and dynamics.  Which is why a dynamical climate model with interactive radiative transfer, water vapor and clouds is needed to get insights much beyond Equation 12.1.  Which is what climate scientists have been doing for several decades.

Note to Skydragons:  Exercises such as Postma that use (or misuse) the planetary energy balance equation cannot disprove the existence of the Earth’s greenhouse effect.  If you want to disprove the existence of the greenhouse effect, you need some sort of alternative explanation for a whole lot of fundamental physics that explains the infrared emission spectra as measured at the Earth’s surface and high above the emitting portion of the atmosphere (see Pierrehumbert).  With regards to the magnitude of the Earth’s greenhouse effect, for a given atmospheric thermodynamic state and gaseous composition, the clear-sky (cloudless)  radiative fluxes are accurately calculated using sophisticated radiative transfer models that have been validated extensively by the Atmospheric Radiation Measurement program (see this previous thread).

Trying to take the planetary energy balance equation and torture it into something that it was never intended to do is pointless  and misleading at best.  The planetary energy balance equation is the simplest possible climate model.  On the other hand there are the general circulation global climate models, which are of enormous complexity.  In between  is a whole class of energy balance climate models, which were very popular in the 1950’s – 1980’s, the Wikipedia gives a good introduction to this class of models, which includes box models, radiative-convective models, higher dimension versions of the zero-dimensional planetary energy balance model, and Earth system models of intermediate complexity (statistical/dynamical models).  Different models can be used depending  the nature of questions asked and the pertinent time scales.

So starting from the simplest zero-dimensional model (planetary energy balance model) and saying that it does not answer all relevant questions and resolve issues related to the diurnal cycle, and therefore the greenhouse effect doesn’t exist, is well, I can’t even find a word for this.  Use a higher dimensional model and play around with it, but if the model has the correct physics, the greenhouse effect is not going to disappear.  Trying to demonstrate that the greenhouse effect doesn’t exist with one of these models, particularly by applying a particular model for something that is beyond the assumptions built into the model, makes no sense.

989 responses to “Planetary Energy Balance

  1. Earth is tightly linked to the neutron star at the core of the Sun [1.2]

    1. “Earth’s Heat Source – The Sun”,
    Energy and Environment 20, 131-144 (2009);
    http://arxiv.org/pdf/0905.0704

    2. “Neutron Repulsion”, The
    APEIRON Journal, in press, 19 pages (2011);
    http://arxiv.org/pdf/1102.1499v1

    • Raymond T. Pierrehumbert
      “In a single second, Earth absorbs 1.22 × 1017 joules of energy from the Sun. Distributed uniformly over the mass of the planet, the absorbed energy would raise Earth’s temperature to nearly 800 000 K after a billion years, if Earth had no way of getting rid of it. ”
      http://climateclash.com/2011/01/15/g6-infrared-radiation-and-planetary-temperature/

      This is incorrect.
      You can’t increase the temperature of something higher than source of heat. The sun surface temperature is about 5778 K, and that is as high as the earth temperature could get.

      • gbaikie,

        it’s important to remember that Ray is saying that the earth would be that warm if it had no way to get rid of that energy. So as sunlight is falling on the earth, the amount of energy contained in the earth system would just keep increasing. Since temperature is a proxy for some types of energy (which would also be increasing), it should increase to dramatic levels as well over a long enough period of time.

        Also, the interior of the sun is not 6000 K. It’s much, much, much hotter than that precisely because there are physical mechanism that prevent the material in those parts of the sun from losing their energy as quickly as the outer layers exposed to the deep freeze of space.

        Does that make a little more sense?

      • “It’s important to remember that Ray is saying that the earth would be that warm if it had no way to get rid of that energy. ”

        My point is simply, earth temperature can not exceed the sun temperature, unless Ray wants to violate more rules of this universe.
        So what he said is incorrect and it’s stupid to say it.

        Relate to this is chemical rocket fuel has limit of what temperature it can attain- depending on the particular rocket fuel. Hydrogen and oxygen gets pretty hot, gun powder not as hot. This is one reason LH2&LOX is used in rockets, rather than gun powder. It also means you can’t heat up anything higher than max temperature of whatever fuel you are using- regardless of magic substances such as CO2 or smoke&mirrors.

        The idiot[s] that said fire doesn’t melt steel, was vaguely close to being correct. The problem is that kerosene could melt steel and steel is weakened in it’s structural strength if it gets, say 50% of the temperature needed to melt it. Kerosene is used to fly airplanes and rockets.

        “Also, the interior of the sun is not 6000 K. It’s much, much, much hotter than that precisely because there are physical mechanism that prevent the material in those parts of the sun”

        So you trying to help Ray.
        If the sun surface reaches 800,000 K, it won’t take a billion years to heat up the earth. And btw, most of it’s energy will be in Xrays or something like that and probably double it’s size. And since interior of the sun is about 15 million degrees and if sun gets to this temperature, someone would have to do the math but time period would be seconds rather than a year or billions of years.

      • Right.

        Got it.

        So irrelevant projection from extrapolation of a trend used in an argument from absurdity is absurd for more than one reason.

        One presumes that had the initial argument said more accurately that the Earth would approach the heat of the Sun and remain there for several billion years, it would overcome this really nitpicky (in this case) objection, and still result in a successful argument from absurdity that would still prove persuasively to any reader that the Earth has some mechanism for getting rid of incoming energy from the Sun.

        Are we satisfied with such an amendment, or does it still distract from the point in some other unimportant (at this point) way?

      • gbaikie- Raypierre is correct. If Earth could not get rid of the energy it absorbed, its temperature would eventually exceed that of the heat source. Is there a violation of the laws of thermodynamics? Of course, and that was part of Raypierre’s point – a failure to shed heat violates those laws. Your argument is false.

      • “gbaikie- Raypierre is correct. If Earth could not get rid of the energy it absorbed, its temperature would eventually exceed that of the heat source. Is there a violation of the laws of thermodynamics? Of course, and that was part of Raypierre’s point – a failure to shed heat violates those laws. Your argument is false.”

        You could surround earth with a sphere which was heated to same temperature or slightly hotter than earth, and have hole/window in that sphere that let in the sunlight. Thereby without violating the laws of this universe you meet the condition of Earth not able to get rid of energy absorbed- and Earth’s temperature would never exceed the sun’s temperature.

        But we don’t need to use the entire Earth- just use mirrors and see if you concentrate the sun energy hotter than 6000 K. Not expensive, and you could win a prize.

      • gbaikie – I believe you’re still wrong. Your example would allow the Earth to shed heat. For the setup to prevent all heat loss, the “hole” would have to allow only one way energy transfer, which again is the point of Raypierre’s argument. I don’t see any way of getting around his argument.

      • “I believe you’re still wrong. Your example would allow the Earth to shed heat. For the setup to prevent all heat loss, the “hole” would have to allow only one way energy transfer, which again is the point of Raypierre’s argument. I don’t see any way of getting around his argument.”
        The hole to allow sunlight in could be glass [or diamond] and glass could also be heated.
        But I think the point I am making is it is at least a debatable point- and Ray is simply asserting something which isn’t scientifically accepted.
        I did google search and here is discussion on this exact point. And there is wonderful video posted there:
        http://www.youtube.com/watch?v=z0_nuvPKIi8&feature=player_embedded
        Posted here:
        http://www.reddit.com/r/askscience/comments/ej61k/what_is_the_maximum_temperature_you_can_achieve/
        In the video the sunlight is concentrated to a very high temperature but is no where near the temperature of the sun.

      • gbaikie,

        ‘But I think the point I am making is it is at least a debatable point- and Ray is simply asserting something which isn’t scientifically accepted.’

        Yes, Ray is asserting something that is not scientifically accepted as a part of reality…on purpose.

        In reality, there are mechanisms by which the energy deposited into the earth’s climate system by the sun can be dissipated back to space. The most important of those mechanisms is radiative dissipation back to space at the top of the atmosphere.

        But in Ray’s ‘thought experiment’, there are no dissipative channels. So that the energy deposited into the earth’s climate system just continues to build up and build up, forever essentially.

        Think of it this way. I have a really big bucket I’m trying to hold up. As I’m holding it in the air, the bucket is being filled with water. In order to hold it up (I’m only so strong), I have to poke some holes in the bucket so that, as water is pouring in, some is pouring out.

        If I make enough or large enough holes, I can make the rate of water coming out of the holes equal to the rate of water entering the bucket. That way, even though water is still pouring into the bucket, the bucket is not getting any heavier because the same amount of water is leaking out of the holes I’ve made.

        This is essentially the situation with the earth system with energy taking the place of water.

        Now imagine that somehow I grow a third arm and am able to plug all the holes I’ve made. The water does not stop from entering the bucket, but I have now stopped the flow out of the bucket. So now the bucket will get heavier and heavier and heavier as time goes on (let’s a assume it’s an extraordinarily large bucket).

        If we were to plug up the ‘holes’ for the earth by ‘turning off’ the energy dissipation mechanisms, then energy, just like the water in the example above, would continue to build up in the climate system. This energy inevitably manifests itself as an increase in the temperature of the surface and atmosphere.

        But your instincts are correct. All of that is just hypothetical. The earth has ways to dissipate energy which is part of the reason we’re here even talking about this.

        Does that make a little more sense?

        As to exact what temperature the earth could realistically get to, I’m not sure. I’m assume that you’re thinking of the sun as a heat engine from which you’re using the engine efficiency equation to figure the exact temperature out?

        I would say it’s not a bad place to start, but I would imagine, and I could be wrong, that figure out the exact number would be a bear of a problem. But I bet you could get a physics PhD doing it…

      • “If we were to plug up the ‘holes’ for the earth by ‘turning off’ the energy dissipation mechanisms, then energy, just like the water in the example above, would continue to build up in the climate system. This energy inevitably manifests itself as an increase in the temperature of the surface and atmosphere. ”

        I don’t think this water analogy is adding any understanding.
        But I can’t resist:
        At some point the bucket is going to have more mass than the planet, and you will be “lifting” the planet rather than the bucket.
        So if you dying to go towards infinite values, the planet’s mass is more important than the bucket.
        The planet in this case is the Sun’s 6000 K. And Ray’s number is 800,000 K. Which is analogous standing on very low gravity object holding the bucket and being concerned about the weight of the bucket.

      • Yes, eventually the metaphor will fall apart, as it’s intended to do by virtue of the fact that it’s meant to show the absurdity of the proposal that Earth might not have any means to get rid of energy, in order to focus the reader’s attention on those mechanisms of getting rid of energy that appear to exist.

        Along the way the metaphor might convert from an Earth-sized body approximating a black hole — which would be so awesomely fascinating that its climate would be utterly forgotten in the rush to study the anomalies that ensued — to one that, after absorbing enough energy that its mass grew sufficiently would be a black hole of dubious physical origin (but as state has no memory, who cares), and we’d all be able to forget about climate on the black hole entirely.

        Erm, so, are we all agreed, some energy leaves the planet?

      • “Erm, so, are we all agreed, some energy leaves the planet?”

        wiki:”The Earth receives 174 petawatts (PW) of incoming solar radiation (insolation) at the upper atmosphere”

        So each second the earth receives 174 petawatts of energy from the Sun, and receives energy from the Moon in terms of tidal energy, and receives energy from radioactive decay in the Earth’s core.
        And all that energy is radiated from earth into space every second.
        So we have a rather large amount of energy leaving earth.
        wiki:”The Earth receives 174 petawatts (PW) of incoming solar radiation (insolation) at the upper atmosphere. Approximately 30% is reflected back to space while the rest is absorbed by clouds, oceans and land masses.”

        And considerable amount of solar energy is reflected.
        Since we doing analogies and thought experiments. I will do couple more.
        What if instead 30% the Earth reflected 100%?
        One might reasonably object that 100% is impossible, that only 99. [point something ] would only be possible.
        Now can you reflect 100 or 99% if you cover the earth surface with mirrors or do the mirrors need to be above the Earth’s atmosphere?

        If the atmosphere had no clouds or dust or any kind particle in the air, and if you covered to Earth with mirrors which were capable of reflecting 99.8% of all sunlight hitting them, what percentage of solar energy would reflected back into space?
        Or what would the difference be between mirrors being on the earth surface as compared to mirrors being above the earth’s atmosphere reflecting sunlight?
        Is there much difference between mirror on the ground compared to mirrors above earth atmosphere? Is it a difference of say 10% or much higher percentage?

        Suppose you at equator with sun directly above you [noon] and the day was without any clouds. And you were at sea level in terms of elevation.
        You are going to get about 1000 watts per square meter, meaning one could only reflect 1000 watts per meter. If you were 200 miles higher in same location and time, you could reflect about 1300 watts per meter.
        So at 200 miles you reflect about 300 watts per square meter more than if on the surface of earth.
        Can we also assume that the sunlight reflected from the earth surface, will lose around another 300 watt before reaches someone at 200 miles?
        So guy at 200 miles can reflect with same mirror, 1300 watts per square meter whereas on the ground only 700 watts per square meter reaches space.
        And instead sun being directly above you, say it’s 9 am or 3 pm, the mirror on the ground will have more losses.
        So if the earth surface is completely covered with mirrors, one should receive about 700 watts per meter of sunlight from earth if you at 200 miles above earth and receive 1300 watts if facing the sun.

        So that difference between mirror on the ground as compared to mirrors in space would be significantly different- not 10% or less, but rather around 50% less.
        Now you have a earth covered with mirrors with all particles removed from atmosphere [or in the above ignoring them].
        Now, let’s add in clouds. Would the addition of normal levels of cloud cover increase, decrease, or more no difference in terms of reflecting sunlight from an earth covered in mirrors?

      • “The presumption that started the entire conversation is that Pierrehumbert was pointing out that the earth contains channels it uses to dissipate away energy absorbed via sunlight. To make his point, he shows that in the case there were no such channels, the earth’s temperature would be so much hotter than we observe it. ”

        No, you are wrong. Since I start this particular conservation [Judith Curry started the main thread, obviously] And I was responding to link she provided in the main article. And topic wasn’t “channels” by which I assume you mean long wave length infrared.
        Addressing your issue of “channels”, the earth is a very large object and a large object can radiate into space immense amounts of energy.
        In space, huge objects are huge refrigerators space. More specifically in terms refrigerators in space, it’s mainly about amount surface area which can radiate the heat- more surface area more heat you can get rid of.

        Conversely, the smaller the object [and less surface area it has] the less energy it can get rid of- via radiation, conduction, and/or convection.
        So you take a dime and a blowtorch and you can heat the dime to the same temperature as the blowtorch. You put the dime on a brick [a brick doesn’t conduct heat very well nor is affected much by blowtorch] and you heat the dime to the temperature blowtorch, but you can not heat the dime above the temperature of the blowtorch- by using that blowtorch or much bigger blowtorch.

      • Fred Moolten: Nothing in the universe allows “only one way energy transfer,” other than a black hole. Thus, Raypierre’s argument is pure nonsense.

        gbaikie: thanks for that illustrative video. Now, let’s consider what happens when we concentrate the “backradiation” claimed to have TWICE THE POWER OF THE SUN – IT CAUSES COOLING

        HAHAHAHAHAHAHAHAHAHAHA

        http://solarcooking.org/research/McGuire-Jones.mht

        http://chriscolose.wordpress.com/2010/05/08/stoat-taking-science-by-the-throat-latest-posts-archives-about-rss-contact-profile-me-my-family-and-me-more-make-sure-youre-familiar-with-the-comment-polic/#comment-2363

        http://hockeyschtick.blogspot.com/2010/06/why-conventional-greenhouse-theory.html

      • Good God, folks…This isn’t that hard to understand. Look up the word “counter-factual” in the dictionary. Ray proposed a counter-factual for the sole purpose of giving the physicist audience he was addressing an order-of-magnitude idea of how much energy the earth receives from the sun.

        It is really no more mysterious than that. Of course it’s not realistic…It wasn’t meant to be. It was simply meant to illustrate the amount of
        energy that the earth is receiving from the sun. In particular, it gets people off on the right foot of realizing that there is no particular temperature associated with the amount of power coming from the sun…but rather that the temperature would increase without bound if the
        earth did not radiate energy back out into space. Hence, the average temperature of the earth is determined by the balance between what it receives from the sun and what it radiates back out into space.

      • This discussion appears mired in a clash of cultures.

        Some, though not all, math and science thinking is influenced by repeated presentations of examinations of fact and proofs in exactly the form Ray P. uses: “If A implies B, and B is absurd, then A is false.”

        I myself spent four years repeating such logic in two or more lessons a day, five days a week, thirty eight weeks a year. I’m indoctrinated to the form. It’s like second nature, and requires an effort to step back from and examine from the point of view of someone who wasn’t so deeply inculcated to the practices of formal logic.

        This practice of formalism — which I haven’t kept up and no longer am very rigorous about — leaves a mark in one’s habits of thought. It only takes a few moments to spot those who are encultured to it from those who just aren’t.

        Which may be why this discussion involves so much talking past one another.

        Or there really may be people who don’t understand the concept of a black hole, where energy can’t escape, and a model that allows 800,000K temperatures internal to black holes, or that S-B wouldn’t apply normally within a black hole, where many ordinary precepts of physics are suspended.

      • Bart R 8/23/11, 11:35 am, Planetary energy …

        BR: This discussion appears mired in a clash of cultures. [¶] Some, though not all, math and science thinking is influenced by repeated presentations of examinations of fact and proofs in exactly the form Ray P. uses: “If A implies B, and B is absurd, then A is false.”

        The formula you recite is called modus tollens (MT) in the propositional calculus of symbolic logic, where it is called an axiom or a rule of inference. Like all axioms and rules of logic, it is an axiom of science.

        If your equation did not hold, digital circuitry and digital computers would fail to work. If you find verbal writing or speech where MT is invalidated, you are looking at a sample that is not science (an application of MT).

        We have for the heat transferred by radiation between a body at the temperature θ and walls (of an enclosure) at θ_sub_W, Q_dot = Aασ(θ_sub_ W^4-θ^4), where α refers to the temperature θ Zemansky, 4th ed, p. 106. A is the area and α is the absorptivity. [Example Hypothesis] In science, [Example Conclusion] the back radiation illuminating an object minus the object’s forward radiation warms the object.

        Your subject post in response to Fred Moolten is well-placed. Dr. Moolten is saturating this blog with the claim that back radiation supplies thermal energy. What he teaches negates the Example Conclusion. Others mired in back radiation give credence to his error. Apply MT to conclude that none of them is practicing science. It’s a clash between science and non-science. E.g.:

        The emissivity relevant to heat transfer into the ocean from back radiation relates to the thermal IR range. Week in Review: April 2, 2011 thread, FM, 4/6/11, 10:57 am.

        That back radiation contributes more than solar SW direct radiation is well accepted in the literature based on data such as the T-F-K energy budget data cited earlier. Id., 4/7/11, 3:09 pm.

        Rather, during a radiative imbalance, surface temperature rises in response to back radiation until incoming absorbed energy and IR flux to space again balance. Slaying the Greenhouse Dragon, Part IV, FM, 8/13/11. 4:11 pm.

        Back radiation changes the temperature at which any given lapse rate operates, and so it can increase surface temperature in response to CO2 without a change in lapse rate. Id., 4:55 pm.

        Back radiation, which is substantial and has been measured, is an important variable in climate dynamics because of its contribution to the thermal energy of land and oceans. Upward IR radiation is of course equally important, as is the difference between them – the net IR radiation, which is upward at about 60 W/m^2, and thereby helps to balance incoming solar radiation. Merely knowing the latter 60 W figure, however, is inadequate for a full understanding, because it is equally important to realize that it’s the difference between two large fluxes with different sources and destinations, and to know how each flux originates and the temperature with which it’s associated. Planetary Energy Balance thread, FM, 8/22/11, 8:53 pm.

        which references an earlier post,

        The net IR flow is upward (in a cooling direction – approximately 60 W/m^2), as is necessary for balance, because solar irradiance is downward. Id., 8/23/11, 9:54 am.

        In the Kiehl and Trenberth (1997) energy budget, the initial condition that defines IPCC’s radiative forcing paradigm, (AR4, FAQ 1.1, Figure 1, p. 96), the net longwave (IR) flux is 66 W/m^2, corresponding to Dr. Moolten’s 60 W/m^2. It is less than a fifth of the incident average solar radiation at the top of the atmosphere (342 W/m^2). The upward IR is 390, with 350 going into the atmosphere, and the Back Radiation is 324 W/m^2. The following creates a fully equivalent diagram and budget. Reduce the Back Radiation, the Surface Radiation, and the unnamed 350 W/m^2 going into the atmosphere of gases and clouds by any arbitrary amount, X. The bases for the radiation is as arbitrary as 273.15º for converting Kelvin to Centigrade.

        Now make X = 324 W/m^2, delete the original Back Radiation along with its arrow, change Surface Radiation to the net of 66 W/M^2, and the unnamed radiation to the atmosphere becomes 26 W/m^2. The diagram turns into a heat diagram instead of a hybrid of heat and radiation. The net radiation is both necessary and sufficient.

        Back radiation … has been measured”, but only by calculation when in equilibrium with some forward radiation, such as the detector in an IR thermometer.

        If forward and back radiation could be separated from one another as Dr. Moolten does, it would violate the Second Law of Thermodynamics. He would be Maxwell’s Demon (1867).

      • Bart R 8/23/11, 11:35 am, Planetary energy … Errata

        The last 2 paragraphs should have read:

        Back radiation … has been measure, but only by calculation when in equilibrium with some forward radiation, such as the detector in an IR thermometer.

        If forward and back radiation could be separated from one another as Dr. Moolten does, it would violate the Second Law of Thermodynamics. He would be Maxwell’s Demon (1867).

      • Jeff, Chris Colose makes the same error. He even describes the upwelling a back radiation components as two separate fluxes

        I really wonder about this. Who taught them these falsehoods?

      • Jeff Glassman and tallbloke

        Here you bring to mind another cultural distinction within the handling of formal arguments: changing topics in mid-argument.

        There’s a latin phrase for this sort of hijacking trains of thought, too, but I’m not predisposed to fancy up straight talk at the moment.

        Although it’s perfectly valid to question one argument by invalidating it through examining weaknesses in an assumed element of the first argument, there are more and less scientific ways to do so.

        So, the first argument being Ray P’s proof that the Earth must lose heat by some mechanism, the underlying assumption being that 800,000K is an absurd temperature for the Earth, some are attacking that underlying claim.

        However, this attack is not meant to amend the first argument, which it appears everyone agrees correct by MT, simply having a fine dispute about the nature of the absurdity they’ll allow.

        At which juncture, I’d expect some of my professors to fling chalk at them and shout, “Stick to the point!”

        Were Ray P’s argument framed in terms of black holes, we would understand that the singularity of a black hole would invalidate application of S-B, Thermodynamics, and even ultimately causality, and no one would be taken seriously for dissecting the absurdity.

        As to this new point, meh.

        Thermodynamics is difficult enough where strict formalism is observed. The myriad little logical slips and rhetorical tricks introduced by mixing in blog-saturation ratios and silly unphysical models half in one argument and half in another simply leads me to the conclusion that there is no truth to be gained in carrying on the discussion in this vein.

        If you want to rehash Fred M’s backradiation, I recommend start fresh, with a clean argument and explicitly stated terms, in some forum that provides interfaces for animations and graphs, and the ability to use the formal symbols of calculus, because these things are best expressed as partial integrals in some places.. and you’re all going nowhere fast with text.

        Having skimmed what Fred’s said elsewhere on the blog, I opine that in most cases he’s fairly solid on both formal logic and Thermodynamics, though from time to time he has held to different conclusions than I would make. I won’t pretend to make his argument for him, since he’s certainly brighter than I am and demonstrably better versed in the subject at hand.

      • Regarding statements made below in this long column, if one wants to measure the back radiation and the upwelling radiation separately, all that is required is to turn the Pyrgeometer to face upward for the former and downward for the latter.

      • I should have referred to statements that appeared above where my last comment ended up.

      • Fred Moolten,

        No your argument is false. Why? Because you violated radiation laws, in particular Stefan-Boltzmann’s law. Its a common misconcept for the GHE and is a common mistake by the warmists.

        ” If Earth could not get rid of the energy it absorbed, its temperature would eventually exceed that of the heat source.” You have a big wrong assumption here that if the Earth reached 6000K, the Earth could continue receiving energy from the Sun. No, this assumption violated S-B law.

        A lot of people here trying to tell you that GHE violates radiation laws and thermodynamic laws.

      • It’s this type of physically nonsensical comment we get for promoting the work of the dragons. Now others are realizing that if they just throw a whole bunch of jargon into a comment, some readers might take their overall point more seriously.

        I think we can all agree that the dragons are completely mistaken in their promotion of the Johnson and Postma’s work. This work adds nothing to science or individuals’ understanding of physical reality.

        In my mind, the battle has been won and because of this fact we just need to move on. If someone in the future is interested in assessing the value of the work that dragons are pursuing, these threads will always be here to help.

        But continuing to give a soapbox to a group of people who would just use Boltzmann’s name without giving any respect to the physics he so cared about is a travesty I’ve had enough of at this point.

      • maxwell,

        Violation of radiation laws and thermodynamic laws, you called them nonsense. Here you are and thats what you are.

      • No Sam,

        what makes what you’ve said total nonsense is that you no have context in the current situation. The presumption that started the entire conversation is that Pierrehumbert was pointing out that the earth contains channels it uses to dissipate away energy absorbed via sunlight. To make his point, he shows that in the case there were no such channels, the earth’s temperature would be so much hotter than we observe it.

        So instead of actually looking into the context of the actual conversation, you’ve just assumed you knew what you were saying, even though, in the context of the conversation, your point is meaningless.

        That’s what’s nonsensical about your point. You’ve done so little work in establishing why Fred would have made that comment that you think he’s saying it as though that’s the way the earth works. That’s obviously incorrect to anyone who has been taking part in the conversation since its beginning.

        And it’s that attitude that somehow, even without reference to where the conversation has come from, you think that you can insert your opinion as though no one else has made that point is exactly the type of lazy behavior that has lead to the misapplication or misinterpretation of so much basic physics in thread and similar threads.

      • maxwell,

        No maxwell,

        “what makes what you’ve said total nonsense is that you no have context in the current situation.” You totally fit to it and not me.

        “The presumption that started the entire conversation is that Pierrehumbert was pointing out that the earth contains channels it uses to dissipate away energy absorbed via sunlight. To make his point, he shows that in the case there were no such channels, the earth’s temperature would be so much hotter than we observe it. ” I agreed to the point that theoretically without the channels it can reach 5778K. Anything beyond is absurd and violated radiation laws, I guess you have not spent enough time to understand them at all.

        “So instead of actually looking into the context of the actual conversation, you’ve just assumed you knew what you were saying, even though, in the context of the conversation, your point is meaningless. ” These statements are meaningless and nonsense. You assumed you knew better was just as nonsense as Fred had said.

        “That’s what’s nonsensical about your point. You’ve done so little work in establishing why Fred would have made that comment that you think he’s saying it as though that’s the way the earth works. That’s obviously incorrect to anyone who has been taking part in the conversation since its beginning.” You are the true denier of radiation laws and thermodynamics.

        “And it’s that attitude that somehow, even without reference to where the conversation has come from, you think that you can insert your opinion as though no one else has made that point is exactly the type of lazy behavior that has lead to the misapplication or misinterpretation of so much basic physics in thread and similar threads.” Thats nonsense. You were very lasy to understand radiation laws and thermodynamics, I cannot help you unless you try hard.

      • The corona that surrounds the “solar surface” is much hotter (T ~ 10^6 K) than that glowing sphere of waste products (91% H, 9% He) that emits photons (photosphere) and hides the material beneath.

        The core of the Sun is nuclear material in a pulsar, not atomic material with kinetic energy that can be measured as temperature. The nuclear material is energized by neutron repulsion (see ref. #2 above) – like violent resentments and anger of the ferocious Hindu Goddess, Kali:

        http://www.muktinath.org/hinduism/kali.htm

        Surrounding the pulsar is atomic material with the same composition as Earth and ordinary meteorites: Fe, O, Ni, Si, S, Mg and Ca.

        Most of those nuclei have zero or even spin, satisfy Bose-Einstein statistics, and may become a super-fluid super-conductor.

        Barry W. Ninham, “Charged Bose gas in astrophysics”, Physics Letters 4 (1963) 278-279.

        Sunspots and solar eruptions arise from deep-seated magnetic fields:

        a.) The neutron star in the solar core, or the
        b.) Super-conducting, iron-rich material around the solar core

        “Super-fluidity in the solar interior: Implications for solar eruptions and climate”, Journal of Fusion Energy 21 (2002) 193-198:

        http://arxiv.org/pdf/astro-ph/0501441

        AGW is built on the false premise that the Sun is Earth’s steady source of input heat.

        http://dl.dropbox.com/u/10640850/20110722_Climategate_Roots.pdf

      • Oliver,

        “The corona that surrounds the “solar surface” is much hotter (T ~ 10^6 K) than that glowing sphere of waste products (91% H, 9% He) that emits photons (photosphere) and hides the material beneath.” I am an ignorant of the Sun’s matter beneath the hydrogen-helium atmosphere. You have just opened up my mind that:

        “Surrounding the pulsar is atomic material with the same composition as Earth and ordinary meteorites: Fe, O, Ni, Si, S, Mg and Ca.”

        I thought the Sun composed of hydrogen and helium only. Thanks.

        Presumably, the solar system planets and meteorites were spinned off from the Sun due to a large explosion.

      • “The sun surface temperature is about 5778 K, and that is as high as the earth temperature could get”

        The explain how the solar atmosphere can be in excess of 1 million degrees?

      • Yes, I would also like to know.

        It seems to me there is contradiction on the theory that the Sun’s surface temperature is only 5778K and at this temperature, nuclear fusion is not possible unless the surface is over one million degrees.

      • If the Sun’s core is over 1 million degrees, then within the radius of the Sun + the Sun’s atmosphere, the temperature cools down to 5778K. A huge temperature gradient on the Sun. The Sun’s temperature gradient is larger if the Sun’s ground (if there is one) surface is 1 million degrees. Gravitational force is huge on the Sun to confine hydrogen at 5778K within the Sun’s atmosphere.

        Is the Sun’s core made up of compacted neutrons and protons only? Nothing to do with hydrogen and helium?

      • “explain how the solar atmosphere can be in excess of 1 million degrees?”
        I don’t think there is a known answer- perhaps someone would like to suggest the is some CO2 near the sun.
        But there is also a very hot atmosphere above earth, call the thermosphere:
        “Among the four atmospheric temperature-defined layers, the thermosphere is located highest above Earth’s surface, beginning at about 57 mi (90 km) above Earth, and reaching into about 300 mi (500 km) height. The name of this layer, thermosphere, originates from the Greek thermo, meaning heat, because in this layer the temperature increases with altitude reaching temperatures higher than 1830°F
        (1000°C).”
        http://www.enotes.com/earth-science/thermosphere

        So ISS is flying thru the thermosphere- and the heat isn’t unbearable:)

        If I had to guess about the Sun’s corona- but first this is what wiki says:
        “The exact mechanism by which the corona is heated is still the subject of some debate, but likely possibilities include induction by the Sun’s magnetic field and sonic pressure waves from below (the latter being less probable now that coronae are known to be present in early-type, highly magnetic stars)”
        I think a relation to sun’s magnetic field is likely [and gives spectum of Fe at 1 million degrees] but would also throw in possibility that atoms could in orbit around the sun. And orbital speed that near the sun is very fast. And there is enough of them there that they could hitting each other. They could also be hitting particles of solar eruption [CME] and solar wind in general.
        So collision and/or magnetic fields crossing another magnetic fields [and this imparts energy]. So if mostly collisions then one has a faint ghost of what happens in a Quasars and accretion disc [though it’s not disc- and not disc because {I would I guess} atoms are constantly being added {and lost}].

    • Diviner observes cooling during June 15 total eclipse
      “The following plot shows data taken during successive orbits over a unit of lunar mare situated between 32 and 33 degrees north. The orbit path progressed from east to west (right to left), with each orbit separated by roughly two hours. The first two data swaths were taken before the eclipse, the three center swaths were taken during the eclipse, and the last two swaths were taken after the Moon had reemerged from Earth’s shadow. The data show an average decrease in surface temperature during the eclipse of around 100K, with some locations remaining warmer than others.”
      http://www.diviner.ucla.edu/blog/

    • Dr. Curry,
      If you can’t even comprehend the difference between the ‘Slayers’ (GHE critics) and the ‘Sky Dragons’ themselves (GHE believers) what hope do we have of persuading you that you’ve erred on the science?
      This further confirms to me you’ve not even read our book. Not good.

  2. I hope somebody has a list of the quantities that have actual measurements? Or link.

    • This was discussed on the confidence in radiative transfer models. see http://www.arm.gov

      • Dear Dr. Curry:
        With all due respect, I have a problem with interpretations in Section 14.3, planetary energy balance: Why is the difference between Te* and T0 is attributed to greenhouse effect and not something else?

      • Dr. Curry:
        You have not addressed my question above, and I have another question :We learned at school that atmospheric layers, or slabs, cannot exchange radiations between them for there is no interface between them. The atmospheric air is one homogeneous air mixture and acts as such. Atmospheric air can only exchange heat, within, by convection and conduction. How come, in Section 14.3 Planetary energy balance, you are assuming that atmospheric layers or slabs exchange solar radiations between them? Where is the physics that supports your assumption?

        Does surface water layers, or slabs, exchange radiations between them?

      • Nabil, if you want the whole enchilada, look at general circulation climate models with interactive radiative transfer, thermodynamics and dynamics. If you want simple models to aid conceptual understanding, then you need to accept some approximations.

        This statement is completely false, and if you learned this in school, it was a lousy school:

        “We learned at school that atmospheric layers, or slabs, cannot exchange radiations between them for there is no interface between them. The atmospheric air is one homogeneous air mixture and acts as such. Atmospheric air can only exchange heat, within, by convection and conduction.”

        Molecules in the atmosphere with a favorable molecular structure (e.g. H2O and CO2) emit and absorb infrared radiation. Read Pierrehumbert, and numerous previous posts, I am not going to explain how this works here. Until the skydragons can understand and accept this, we can’t get anywhere with talking about thermodynamics etc.

      • We just had a long and interesting discussion on WUWT regarding the inability of longwave radiation to penetrate the ocean. What proportion of the energy in downwelling longwave do the GCM’s have being propagated into the ocean bulk from the surface by mechanisms such as turbulent convection?

      • Dear Dr. Curry:
        My schools are excellent and produced Fermi, Marconi, and Copernicus, to name a few. I am confident that they taught me a correct science.

      • Nabil Swedan

        One must admit cause to believe none of Fermi, Marconi, or Copernicus ever said of their schools (as excellent an education as Bologna might rightly claim to offer) that they were confident they learned from them correct science.

        Else, which of them would have gone on to so overthrow the science that came before them? ;)

        Do they still teach the quality of critical thinking that allows one to, for oneself, examine a proposition by logic and reason, and through observation and experiment, determine one’s own thoughts on the matter without reference to authority?

      • “Molecules in the atmosphere with a favorable molecular structure (e.g. H2O and CO2) emit and absorb infrared radiation. Read Pierrehumbert, and numerous previous posts, I am not going to explain how this works here. Until the skydragons can understand and accept this, we can’t get anywhere with talking about thermodynamics etc.”

        Your reference doesn’t claim that oxygen and nitrogen don’t also emit and absorb infrared radiation. Or that there is any substance in existence that doesn’t emit and absorb infrared radiation.

        Btw, I wondering why the plank curve is for 260 K.
        Earth average temperature is closer to 290 K.
        And Venus obviously higher and Mars obviously colder.
        Could a possible answer be that actual scientist aren’t interested in such concepts as “average global temperature”- meaning it’s irrelevant
        in terms of scientific inquiry?

      • Arfur Bryant

        Dr Curry,

        [“Molecules in the atmosphere with a favorable molecular structure (e.g. H2O and CO2) emit and absorb infrared radiation. Read Pierrehumbert, and numerous previous posts, I am not going to explain how this works here. Until the skydragons can understand and accept this, we can’t get anywhere with talking about thermodynamics etc.”]

        The problem is not with accepting the ability of CO2/H2O to emit and absorb radiation, it is, in terms of cAGW, the AMOUNT of contribution these gasses have to the GHE. Pierrehumbert gives a far greater contribution to CO2 than any existing data can support. In the dry atmosphere. 99.96% of the atmosphere cannot contribute to GHE within the relevant wavelengths. The other 0.04% consists of the trace dry radiative gasses, mostly CO2. If you add water vapour – say 2.5% on average, making a total of 102.5% – then you have 2.54% able to either emit or absorb radiation (although I’m not sure that water emits much in the relevant wavelengths, but it is certainly the largest absorber) but still have 99.96% of the atmosphere unable to contribute. Either way, the contribution of dry radiative GHGs to the GHE is minimal. This can be seen using any dataset of sufficient history, such as HadCRUt. If, as Pierrehumbert says, CO2 contributes 30% to the GHE, then how has that contribution been almost entirely negated following a significant increase in CO2? ‘Negative feedbacks’ and ‘Thermal inertial lag’ do not explain this anomaly.

        If no-one can answer the question ‘How much does CO2 contribute to the GHE?’, then how can anyone make statements which consistently assume that the contribution is significant?

        I don’t know anyone who seriously believes there is no such thing as a GHE. It is the assumption that the GHE is largely due to CO2 that I, for one, find hard to accept.

        The basis upon which the entire cAGW debate has been formulated requires a complete rethink from the start. Arrhenius may have had a point in stating CO2 would contribute to GHE, but he has been hopelessly inaccurate in any quantitative sense.

      • Pierrehumbert has some work on the cited claim, such as his 2007 paper, and there have been other experiments along similar lines (Kiehl and Trenberth 1997, and most recently Schmidt et al 2010). They all give relative contributions of CO2 at ~20% in a cloud atmosphere and a bit more in clear skies.

      • Arfur Bryant

        And a minute’s thought will tell you that they are all way wrong. How many of those papers are based on experimental data and, if so, where is it?
        .
        Here’s the rub. In 1850 the CO2 atmospheric concentration was appx 280 ppmv. The global temperature was appx 0.8 deg C less than it is today. This means the GHE was appx 32.2 deg C (given that most players think it is 33 deg C today). Today the CO2 concentration is 395 ppmv, an increase of 40%.
        .
        So, if Pierrehumbert and K&T and Schmidt (and Lacis) are correct, the contribution of CO2 was appx 6.5deg C in 1850 and 6.6 deg C today!.
        So, a 40% increase in the largest trace dry radiative gas (and similar increases in the others) has led to a 0.1 deg C increase in the GHE in 160 years!
        .
        So what’s all the fuss about? The logical answer is that the radiative theory, upon which the cAGW hype is based, is demonstrably nonsense in any quantitative sense.
        .
        What did Arrhenius predict? A sensitivity of 4 to 6 deg C? Yeah, right…

      • Arfur Bryant | August 20, 2011 at 4:37 pm |

        And then had to revise that to 1.5-2º C

        Svante Erroneous

      • GHE = green house effect or global heat engine?

      • Arfur Bryant

        Green house effect. Sorry, should have made it clear.

      • Given how little CO2 there is in the atmosphere, a mere 400 ppmv or 0.04%, it is understandable that people find it hard to believe that such a tiny amount could have any significant impact on surface temperature.

        The reason a very small amount can have a big effect is that it is not the quantity of CO2 that is relevant but its percentage rate of change.

        When CO2 increases at a given percentage rate, that is, in geometric progression, it causes the surface temperature to rise at a fixed rate, that is, in arithmetic progression. Surface temperature increases logarithmically with CO2.

        This means that an increase from 100 ppmv to 150 ppmv raises the temperature by about the same amount as an increase from 1000 ppmv to 1500 ppmv, which in turn is about the same amount as an increase from 100000 ppmv to 150000 ppmv, that is, from 10% to 15% of the atmosphere.

        If you can accept that increasing CO2 from 10% to 15% of the atmosphere would have a significant effect, then you should expect that increasing it from 0.01% to 0.015% will have the same effect.

        The global warming of the past few decades is not the result of CO2 having reached an outrageous value like 0.5% of the atmosphere. It is the result of CO2 increasing lately at the outrageous rate of 0.5% per year, up from 0.3% per year four decades ago. Putting it more constantly, anthropogenic CO2, defined as the surplus over the natural base of 280 ppmv, has been increasing by a steady 2.2% a year since 1958 when the CO2 observatory at Mauna Loa was established.

        There is no record anywhere, neither in books nor the rocks, of CO2 ever having risen at this rate before, whether a hundred, a thousand, a million, or a billion years ago. This is an astonishing rate, and it is what is responsible for the last few decades of increasing temperature.

        The temperature rise expected from such a rate of increase can be predicted from our understanding of the absorption properties of CO2. From 1970 to 2000, CO2 rose 13%. Assuming an immediate response to CO2 and a climate sensitivity of 1 °C per doubling, the temperature would therefore rise 1*lb(1.13) = 0.17 °C, while with 3 °C/doubling the rise would be 0.5 °C. Since the observed rise over that period, was 0.5 °C, somewhere between zero and 0.33 °C of that rise must be of some other origin.

        But the IPCC has suggested a delay of 20 years in its concept of transient climate response, meaning the time it takes CO2 to have its warming effect. With that assumption we should base these calculations on the CO2 rise not from 1970 to 2000 but from 1950 to 1980, which was around 8.7%. This shrinks the above two numbers to 1*lb(1.087) = 0.12 °C and 3*lb(1.087) = 0.36 °C. In that case we would have between 0.14 °C and 0.38 °C not accounted for by CO2.

        These defects should not be at all surprising in light of the large variations in temperature observed during 1850 to 1950, which can hardly be the fault of CO2 given that in 1900 we were producing CO2 at only 6% of our present rate.

        One can then ask whether it is possible to pin down both climate sensitivity and natural variation any more accurately than this.

        I believe we can, and that very close to two-thirds of the observed temperature increase from 1970 to 2000 was of anthropogenic origin. This is a preliminary estimate based on a study of the global land-sea temperature records since 1850 in combination with estimates since 1751 of combustion of gas, liquid, and solid forms of carbon based fuel including gas flaring, plus CO2 from cement production.

        If 1970-2000 saw CO2 contributing 66% of total warming, one can ask what percentage it contributed from 1940 to 1970. Well, in fact the observed temperature declined some 0.08 °C over that period. Had the CO2 level remained constant I estimate that it would have declined by 0.26 °C. So CO2 merely prevented what would have been even more significant cooling.

        More on this later (preliminary ≠ final).

      • Arfur Bryant

        Hello Vaughan!

        Once again, my old internet friend, you are arguing based on an assumption.

        [“This is an astonishing rate, and it is what is responsible for the last few decades of increasing temperature.”]

        You ignore the fact that there has been no increased warming for the latter (nearly) half of your ‘last three decades’ period, during which you say the effect of CO2 should be increasing at an accelerative rate. If your 20-year lag was a valid argument, we should be seeing some increase in the global temperature due to CO2. You assertion that any current warming is due to CO2 is a triumph of hope over objectivity.

        You also state that, during the period 1970-2000, CO2 produced 66% of global warming. But, in that case, the period 1910-1945 something else produced 100% of the global warming (0.8 C). Was there no effect from CO2 then? Had CO2 not been produced for more than 20 years?

        You are, in my opinion, correct in your assertion that any warming is due to a combination of natural and anthropogenic factors. However, what neither you – nor anyone else – can state is the relative contributions of either. Observed data indicates that there have been several warming periods since accurate records began in 1850. Observed data indicates that CO2 has increased at a slightly accelerative rate since the Mauna Loa dataset was started. What has not been observed is any correlating acceleration (other than short-term intermediate periods) between global temperature and CO2.

        It is quite simple. Instead of regurgitating various spurious reasons why the anticipated effect of CO2 has not lived up to Arrhenius’s expectations, why not simply address the logical possibility that he was just plain wrong in any quantitative sense?

        Regards,

      • You ignore the fact that there has been no increased warming for the latter (nearly) half of your ‘last three decades’ period, during which you say the effect of CO2 should be increasing at an accelerative rate.

        Evidently you haven’t heard of El Nino solar cycles, Arfur. These are strong swings that go up and down every five and eleven years respectively. They have no significance whatsoever for long-term climate. They have been happening for hundreds of years and in all probability for millions of years.

        You are simply exploiting the most recent El Nino and solar cycle to claim that temperature must be declining. Claims based on short-term events have no relevance to long-term projections.

        Unless you filter out short-term transients like El Nino and solar cycles, you will be able to prove on an upswing that the temperature is going up and on a downswing that it is going down. These swings are meaningless for projections twenty years hence.

      • Evidently you haven’t heard of El Nino solar cycles, Arfur. These are strong swings that go up and down every five and eleven years respectively. They have no significance whatsoever for long-term climate. They have been happening for hundreds of years and in all probability for millions of years.

        A statement that requires some validation,

      • Arfur Bryant

        [” August 21, 2011 at 3:28 am |”]
        I’ll ignore the sarcasm Vaughan. What you evidently ignore is that – by your own admission – there have been several El-Whatsit cycles since 1850 and yet the global temperature has increased by only 0.8 deg C.

        I didn’t hear any warmist say, in 1998, that the Hockey Stick peak “was just an example of an El Nino and the temperatures will not increase at all over the next thirteen years…”

        You, along with all the others who believe in the cAGW theory without question still refuse to answer the basic questions: “What is the contribution of CO2 to the GHE?” and “Why is there no acceleration in warming in spite of the slight acceleration in CO2 concentration?”

        The overall trend in 160 years is 0.06 deg C per decade (HadCRUt). It is far lower than it was in 1888!

      • What you evidently ignore is that – by your own admission – there have been several El-Whatsit cycles since 1850 and yet the global temperature has increased by only 0.8 deg C.

        Arfur, your logic has gotten even worse lately. You can’t infer from the fact that someone has admitted something that they are ignoring it.

        The 3-7 year El-Nino events and episodes have no evident long-term impact on climate, and should therefore be removed from the temperature record in order to see more clearly those factors that do have a long term impact.

        I didn’t hear any warmist say, in 1998, that the Hockey Stick peak “was just an example of an El Nino and the temperatures will not increase at all over the next thirteen years…”

        Neither did I. It would be completely illogical to say such a thing about a rise that took place over a 30-year period.

        You, along with all the others who believe in the cAGW theory without question

        It is illogical to say that someone who is attacked for questioning cAGW theory does not question it.

        still refuse to answer the basic questions: “What is the contribution of CO2 to the GHE?” and “Why is there no acceleration in warming in spite of the slight acceleration in CO2 concentration?”

        That’s like saying that Charlie Brown still refuses to kick Lucy van Pelt’s football.

        Arfur, we had a great many exchanges in the past where I patiently tried to answer your questions, and to explain why your reasoning was illogical. If anyone refused it was you: you refused to accept that you were being illogical, and continued to trot out the same illogic, not just to me but to everyone silly enough to engage with you. I now refuse to answer further questions from you, there is no point. The argument you love the most is the following.

        The overall trend in 160 years is 0.06 deg C per decade (HadCRUt). It is far lower than it was in 1888!

        Officer: “I clocked you at 100 mph, let me see your license.”

        You: “But officer, my average speed over the last 24 hours has been only 3 mph, far lower than the 30 mph I was doing 22 hours ago.”

        Officer: “You argue like that chap Arfur Bryant over on Climate Etc.”

      • Arfur Bryant

        Vaughan,

        Yes, we had many exchanges in the past. And the illogic shown was yours, not mine. Your hubris knows few bounds. It is a shame that you refuse to answer the most basic of questions and yet you can proudly insist that I am the one who is illogical. I am very happy for you not to engage with me, however there are a few points that need qualifying lest you continue to inhabit the warmist pseudo-world without the requirement for objectivity. Nice try at deflection but no cigar I’m afraid.

        “The 3-7 year El-Nino events and episodes have no evident long-term impact on climate, and should therefore be removed from the temperature record in order to see more clearly those factors that do have a long term impact.”

        It was you that brought up the El Nino etc cycles. Of course they are irrelevant to the long term climate, that was my point! Since 1998 there have been two3-7 year cycles and yet the temperature is still lower than 1998 – hence your use of them as an excuse was false. This makes your 30-year argument invalid. There is NO correlation between accelerating CO2 and global temperature. Get it?

        “It is illogical to say that someone who is attacked for questioning cAGW theory does not question it.

        Again, that is my point. I attack it and question it, you do not. Your belief in the theory based on radiative forcing is just a belief. There is nothing scientific about your approach.

        “That’s like saying that Charlie Brown still refuses to kick Lucy van Pelt’s football.”

        This statement, like your following attempt at sarcasm-based humour is well beneath you, Vaughan. Once again you try to deflect the argument because you cannot counter-argue using either logic, science or intellect. You really need to understand the following: The overall trend is 0.06 deg C per decade. There is NO acceleration in this trend apart from short-term, interim periods. The overall (nb) trend has been much greater than 0.06 deg C per decade, particularly in the late nineteenth century. However, the point is the OVERALL trend is much lower today. This means your warmist assertion that CO2 leads to an acceleration of global warming is NOT supported by evidence. No amount of deflection on your part will change this simple fact.

        If, Instead of providing evidence, all you have is a puerile attempt at sanctimonious ridicule, then you are not worthy of debating. Stick to the facts, stick to at least some semblance of objectivity and you might see the light.

        Please note that I, unlike you, feel no desire or requirement to resort to pathetic sarcasm and cutesy (and erroneous) little analogies.

      • It was you that brought up the El Nino etc cycles. Of course they are irrelevant to the long term climate

        Good, then you should have no objection to removing short-term events like the 3-7 year El Nino and 11-year solar cycles. This is easily done by smoothing the temperature record with a moving average of 11 years, as a definition of long term climate.

        The overall trend is 0.06 deg C per decade. There is NO acceleration in this trend apart from short-term, interim periods.

        The long term climate record does not bear this out, having been on a steady rise since 1970, with insignificant downturns in the vicinity of 1980, 1987, and 2005.

      • Arfur Bryant

        That’s better, Vaughan. Isn’t it nicer to debate without the yucky personal stuff?

        “Good, then you should have no objection to removing short-term events like the 3-7 year El Nino and 11-year solar cycles. This is easily done by smoothing the temperature record with a moving average of 11 years, as a definition of long term climate.”

        A nice attempt at moving the goalposts Vaughan. I never said anything about 11-year solar cycles but I can see why you would want to sneak that little gem into the discussion. OK, You like to remove the cycles by smoothing over ELEVEN years? Wow. Personally, I prefer to look at the actual data and accept that the cycles may or may not exist. I don’t hide them, I can interpret the overall trend without smoothing. Look at the same data without any smoothing:

        http://www.woodfortrees.org/plot/hadcrut3gl/mean:1

        See how the two graphs look so different? Same data, different picture. Except mine is unaltered. Unadjusted. The simple truth.

        So lets address your graph, the one with an eleven year smoothing. Look carefully at your graph. You stated earlier that the warming of the last 30 years is due to accelerated CO2 concentration rate (as opposed to simply the concentration). Could you explain what caused the warming that occurred (in your graph) between about 1910 and 1940? Measure it. The warming is EXACTLY the same as the later warming from 1970! Amazing. No CO2 acceleration but exactly the same warming! SO, according to your graph ‘something’ caused an exactly similar warming period but then ‘something’ also caused a sharp drop in temperature between 1940 and 1960. All this before your quote:

        “The long term climate record does not bear this out, having been on a steady rise since 1970, with insignificant downturns in the vicinity of 1980, 1987, and 2005.”

        So your ‘long term climate record’ says a lot more than your interpretation without your cherry-picking of interim periods.
        .
        So what do you think that ‘something’ is, Vaughan? How do you explain an exact similarity without your cause? How do you know that CO2 caused the latter warming period but not the earlier warming period?

        Now look at my graph. Now look at yours. They both show the simple fact that the TOTAL warming is a mere 0.8 deg C in 160 years. They both show that there have been periods of warming in the past, as well as periods of cooling (and certainly not insignificant ones, as you suggest). They also both show that the warming trend is about 0.06 deg C per decade. You wish – because you want to maintain your belief in the radiative forcing theory of cAGW – that the latter warming is due to CO2. What you’re not prepared to do is look objectively at the data and embrace the logical possibility that you CANNOT say with any certainty that CO2 is responsible for global warming.

      • You stated earlier that the warming of the last 30 years is due to accelerated CO2 concentration rate (as opposed to simply the concentration).

        You aren’t paying attention. It’s not all CO2. What I wrote in this thread on Saturday, that prompted your response, was that
        very close to two-thirds of the observed temperature increase from 1970 to 2000 was of anthropogenic origin. I don’t know what proportion of that two-thirds is due to CO2 but I would be surprised if it were less than a half. I estimate the other one-third to be mainly due to the same natural processes that contributed in part to the 1910 to 1940 rise, to which anthropogenic influences also contributed though not as much as now.

        They also both show that the warming trend is about 0.06 deg C per decade.

        You aren’t paying attention. What I wrote earlier was that the same line of reasoning shows that an average speed of 3 mph over a day proves that you couldn’t have been speeding at 100 mph recently. The relevance of this to CO2 and our other emissions is that for most of the past 160 years we haven’t been emitting much CO2, in fact two-thirds of emitted CO2 has been in the past one-quarter of that period. In those four decades we’ve been pushing the pedal to the metal and now we’re being pulled over. And you’re arguing with a straight face that the average over sixteen decades constitutes mitigating circumstances?

        If your logic works differently then it’s too advanced for me, I’m unable to distinguish it from magic. I’m afraid I will have to throw in the towel.

      • I didn’t respond to the following exchange at first because I couldn’t make head or tail of it.

        VP: It is illogical to say that someone who is attacked for questioning cAGW theory does not question it.

        AB: Again, that is my point. I attack it and question it, you do not.

        I kept trying to understand it, and finally concluded that your reply is such a non sequitur that I have to wonder if you can even read English.

        It is a complete waste of time arguing with someone who can’t even understand what the other side said.

      • Arfur Bryant

        Vaughan, Vaughan…(sigh)!

        Of course you can throw in the towel but don’t pretend the reason is anything other than you cannot either formulate a reasoned debate or that you have no answer for the points or questions that I raise. Before I go I need to clarify your attempts at deflection…

        “You aren’t paying attention. It’s not all CO2. What I wrote in this thread on Saturday, that prompted your response, was that
very close to two-thirds of the observed temperature increase from 1970 to 2000 was of anthropogenic origin. I don’t know what proportion of that two-thirds is due to CO2 but I would be surprised if it were less than a half.”

        No Vaughan, it is you that is not paying attention. You also wrote this:
        “There is no record anywhere, neither in books nor the rocks, of CO2 ever having risen at this rate before, whether a hundred, a thousand, a million, or a billion years ago. This is an astonishing rate, and it is what is responsible for the last few decades of increasing temperature.

        So you clearly have laid the fault of global warming at the doorstep of the rate of increase of CO2.

        But lets examine your figures more closely…
        You state that 66% of the warming between 1970 and 2000 is anthropogenic and, of that, at least 50% (your estimate, not mine) of that 66% is due to CO2. Ok, you’re the Maths professor but I reckon that makes CO2 responsible for 33% of the total warming in that period. Now, according to your beloved ELEVEN year smoothing graph of that period…
        http://www.woodfortrees.org/plot/hadcrut3gl/from:1970/to:2000/mean:132
        … the warming is 0.3 deg C (If you look at the monthly data it is actually about 0.2 C. This makes CO2 responsible for a mind-blowingly huge temperature increase of, er… 0.1 deg C. Wow! Just Wow! And, what’s more, seeing as how you insist that anthropogenic factors, including CO2 would have a much smaller contribution for all earlier warmings since ‘accurate data’ began, then you are effectively saying that CO2 cannot be responsible for more than – what, 0.2 C in the period 1850-2000? Obviously, the rate of CO2 increase, having latterly been much more pronounced, the contribution is much greater in the last 30 years (1980-2010) – so 33% of 0.3 C = 0.1 C but some of that is already included in the 1970-2000 period – which means that the total contribution of CO2 by your own admission is about 0.25 deg C in 160 years! Again, Wow!

        When you have more time and you can refrain from the snarky comments, maybe you could go into more detail about how you think the radiative properties of a trace gas existing at less than 400 ppmv in the atmosphere can possibly be the “responsible for the last few decades of increasing temperature”.

        Seriously, I’d love to hear it!

        As to your references to the genuinely ridiculous traffic analogy, you are just wrong. Its not a line of reasoning; it is a fact. The overall trend is 0.06 C per decade. If you disagree, instead of using sarcasm, why not just state what you think it is?

        Finally, as for this:
        “VP: It is illogical to say that someone who is attacked for questioning cAGW theory does not question it.”
        I don’t understand it either – are you saying it is YOU who questions cAGW? I think not, chum.

        The rest of your post is just infantile snark, and falls far below the level of debate expected here. IMO.

      • You also wrote this: “There is no record anywhere, neither in books nor the rocks, of CO2 ever having risen at this rate before, whether a hundred, a thousand, a million, or a billion years ago. This is an astonishing rate, and it is what is responsible for the last few decades of increasing temperature.

        So you clearly have laid the fault of global warming at the doorstep of the rate of increase of CO2.

        Touché. Had I been choosing my words more carefully I’d have written “and CO2 and other human emissions are what is responsible for two thirds of the last few decades of increasing temperature.” I apologize for my sloppy wording.

        But lets examine your figures more closely…
        You state that 66% of the warming between 1970 and 2000 is anthropogenic and, of that, at least 50% (your estimate, not mine) of that 66% is due to CO2. Ok, you’re the Maths professor but I reckon that makes CO2 responsible for 33% of the total warming in that period.

        Now you’re the one being sloppy. But if you replace “33%” by “at least 33%”, then yes, your math is spot on.

        Now, according to your beloved ELEVEN year smoothing graph of that period…
        http://www.woodfortrees.org/plot/hadcrut3gl/from:1970/to:2000/mean:132
        … the warming is 0.3 deg C (If you look at the monthly data it is actually about 0.2 C. This makes CO2 responsible for a mind-blowingly huge temperature increase of, er… 0.1 deg C.

        Hey, I appreciate your m.o. there, Arfur. Your argument is worth a 5 on a scale that gives a 6 to Easterbrook’s famous proof that MWP is warmer than “now” and an 8 to Huffman’s less famous but very deserving proof that CO2 isn’t warming Venus.

        What you’re exploiting here is the obscure fact that an 11-year moving average knocks 11 years off the window. Hence your alleged graph for 1970-2000 (it was never mine) only shows the 19 years 1975.5-1994.5 (look at the x-axis if you don’t believe me, 1970 and 2000 are nowhere to be seen). Naturally 19 years didn’t warm as much as 30 years, and not even in proportion given that the last 5.5 years are steeper than the rest.

        The correct graph for the full 30 years from 1970 to 2000 is this one. It shows a rise of 0.49 °C over those 30 years.

        I really like your graph, it’s a big improvement over the point you and Girma keep making that the global temperature has only been rising at 0.06 °C per decade when averaged over the last 160 years, which is worth a 2 at best. You’ve clearly raised your game and are approaching the level of professionals like Easterbrook and Huffman.

        The rest of your post is just infantile snark, and falls far below the level of debate expected here. IMO.

        Ah, you’ve mastered Internet-speak. Go to the next level.

      • Arfur Bryant

        Vaughan,

        You introduced the eleven year smoothing, so don’t complain when I throw it back at you. Its really cute to keep changing the goalposts but you might want to check the warming since your cherry-picked period of 1970-2000 (or 1965.5-20005.5001) and check how much warming could have been attributed to CO2 at at least 33.3%.
        .
        Anyway, what is wrong with Girma and I pointing out the average trend over 260 years = 0.06 C per decade? Do you have another figure?

      • Dear Mr. Bryant:
        An observation preceded good science, not the GHE science.You will find in my comments later that the GHE exists only on paper and does not exist in the real word. It is an imaginative concept that invented out of the blue to explain global warming. I do not believe in GHE science at all, it is flawed, and has devoured quite a bit of our resources already. Look at the price of corn, it has jumped from less than two dollars to over seven dollars in a short period of time. GHE suggests that there will be a run-away surface temperature and dooms’ day, which is not going to happen. In fact this warming trend will cease on its own without human intervention. These are some of the costs of adopting a false science.

      • Arfur Bryant

        Dear Mr Swedan,

        Thank you for your post. I am very sorry not to have replied earlier – I was too engrossed in my discussion with Vaughan and actually missed your input! I will read your other posts but, in the meantime, I agree that ‘GHE science’ has been invented to engender a false fear in order to enforce governmental policies. I find it hard to believe so many so-called scientists can be prepared to religiously believe in a concept that has virtually no observational evidence to support it, and no logical or intellectual foundation in its expected impact.

        Regards,

      • Nullius in Verba

        Radiation can be exchanged between different parts of a gas. An opaque gas emits and absorbs like any material. The division into “slabs” is purely for intuitive convenience. The same result arises (but less easily) treating them as continua.

        Surface water layers do indeed exchange radiation between them, As do the layers of solid opaque materials, too. These internal exchanges, being bidirectional between materials at virtually the same temperature and with the same properties, cancel out, and only the surface exchanges have any net effect. But the internal exchanges do exist. Treating a material as if it only emits at its surface boundary is another one of those intuitive approximations.

      • Dear Verba,

        Solid layers exchange radiations between them??!!! Do you have a reference book or the name of the discoverer of this phenomenon? I have never heard of this. I know that radiation occures between two surfaces that have a space between them. When these surfaces come to be in contact with each other, radiation heat transfer process ceases, and conduction is the only heat transfer process between the two surfaces.

        The same is true for gases and liquid, their layers have no interface between them, and only conduction and convection can occur within a liquid or gas mass.

      • Nullius in Verba

        How would a book help, if you don’t trust them?

        Radiation and absorption are atomic/molecular processes. A molecule in the middle of a body of fluid (or solid) does not behave any differently from one at the surface – if it is excited by thermal collisions/vibrations, it can radiate the energy.

        In an opaque material, the radiation doesn’t get far, getting re-absorbed within a few molecules distance. Since the energy flows equally in all directions, it cancels out, and can be safely neglected. A lot of elementary introductions to the subject do so – and no doubt that’s what you’ve seen. But if you suspend a detector in an opaque fluid, or a light source, radiation and absorption are found to work the same way throughout the bulk as they do at the surface.

        At an intermolecular scale, there is always a gap.

        What did you think was stopping molecules inside a fluid radiating?

      • Dear Verba:
        At the molecular level, the molecules of a homogeneous gas or liquid are at the same temperature. No heat transfer of any sort is expected between these molecules.

      • Nullius in Verba

        “No heat transfer of any sort is expected between these molecules.”

        Quite so. But you agree they will still radiate, yes?

      • NiV, while I fully agree with you that radiation exists within solids, I’d be very interested to know the typical energy ratio between phonons and photons in a solid like NaCl crystals or cast iron. I’ve plowed through some 15 volumes of the Solid State Physics series in the hope of running across a hint, so far without success.

      • Nullius in Verba

        “I’d be very interested to know the typical energy ratio between phonons and photons in a solid like NaCl crystals or cast iron.”

        I don’t know. I’ve an idea radiative heat transport tends to be only practically important for semi-transparent and porous materials at high temperatures – most of the research I’ve heard of is for application in glass furnaces, although there’s a certain amount for work for use on lasers, ablative coatings, protective coatings for high temperature applications, etc. The nearest to the optically thick domain I can think of would be Rosseland theory for photon transport deep in stellar cores. There may be something there you can use.

        But I would guess that the spectrum of internal photoemission is much the same as at the surface, and you could determine the radiative energy density by treating it as slabs of material just thick enough to be opaque.

        Opacity isn’t a binary property. If you spread even a metal, like gold, out into a thin enough layer on glass, you can see through it. So the principle is the same as for a ruby, but with a much, much shorter mean free path.

      • Opacity isn’t a binary property. If you spread even a metal, like gold, out into a thin enough layer on glass, you can see through it.

        Indeed. The same principle applies to titanium dioxide, which in any reasonable thickness makes a wonderful whitener, but which is also used as a thin film in alternating layers of high and low refractive index to form antireflective coatings, optical filters, heat reflecting windows, etc. Its very high refractive index suits it to all these purposes, including scattering when used in thick films so as to minimize absorption of the scattered light whence its bright whiteness.

      • If I may interject, the example of the solid state laser comes to mind, or more especially of the original solid ruby laser; radiation but not heat, though of course the end product was a special kind of radiation.

        I think perhaps there was a paper by an Austrian about it in my grandfather’s time. What was his name? Albert something.

        As for the sophism of heat for radiation, one admits to not understanding the cause of this confusion.

      • Bart, NiV was talking about opaque solids. Ruby is not opaque.

      • Vaughan Pratt

        Nothing is opaque to all of everything, and even transparent things are opaque to some of some things.

        Red ruby is somewhat opaque to some green light. Though NiV’s proposal above looks kinda fun, and may even be true.

        Incandescence gives us another clue, in that even tungsten — quite opaque — will glow from within, at least if we tust our eyes.

        Some photonic metamaterials (http://en.wikipedia.org/wiki/Photonic_metamaterial) exhibit fun surface emissive radiative effects (such as twelvefold increase in efficiency of old-fashioned lightbulbs http://www.azom.com/article.aspx?ArticleID=1538), however (see also http://wakalix.com/tech/PhDdissertationBrianTSchwartz.pdf) so you may be right and I remain agog for discussion of the surface/internal radiation model of exotic opaque solids.

        I wouldn’t expect we’d need consider the case of the photon tree falling in the blackbody forest.

      • Red ruby is somewhat opaque to some green light.

        Good point. NiV’s comment about radiation traveling a short distance in opaque solids takes two parameters, the solid and the wavelength. I imagine green photons can pass many thousands of ruby atoms before being captured, easily inferred from the transmission curve for ruby. So even green light in a ruby would not be as extreme as what NiV had in mind.

        What I’d love to see is transmission curves for much more opaque objects, where a photon has a mean free path on the order of just a few atoms. There seems to be a lot less data in those cases.

      • Dear Dr. Curry:

        I have another question for you: In Section 14.3, planetary energy balance, you have assumed that air is transparent to visible light. We know that this is an incorrect assumption. We see visible light absorption by the atmosphere every morning, afternoon, and sunset, without measurements. If you wish to measure it please do so, it does not cost a lot.

        Therefore, you, incorrectly, assumed that the atmosphere is transparent to visible radiations; but, you also assumed that the atmospheric layers, or slabs, exchange radiations between them, which is inconsistent. You, in fact, crated a radiative effect out of the blue. This effect appears to be what is now called greenhouse gas effect. The effect can not be real for it is based on incorrect physics, it is fiction and does not exist in the real world applications.

      • Nullius in Verba

        Scattering isn’t absorption.

      • However 25% extinction most certainly is absorption.

      • Like i said, if you want a complicated radiative transfer model, look at the general circulation models. It is pointless for me to discuss this with you any further.

      • Dr. Curry:
        Thank you for your time, and now I know that you referenced mother’s work.

      • Sorry, I meant others work.

    • steven mosher

      Now that you have the link I hope you read it and return to say your mind was changed. or present an argument

  3. NASA Science News for August 18, 2011

    For the first time, a spacecraft far from Earth has turned and watched a solar storm engulf our planet. The movie, released today during a NASA press conference, has galvanized solar physicists, who say it could lead to important advances in space weather forecasting.

    FULL STORY at
    http://science.nasa.gov/science-news/science-at-nasa/2011/18aug_cmemovie/

  4. Judith,

    Introducing a variable e = trad / tdyn. There are several different regimes for e:

    (i) If e >> 1, the dynamic time scale is much greater than the radiative time scale, and hence radiative processes dominate.

    It appears that there is an error here. Your text would continue logically, if the value of e is taken as the inverse of that given by the definition.

  5. Claes Johnson

    Judy: You attribute the difference between 254 and 288 to radiation, but this is not correct scince it results from an interaction between thermodynamics and radiation. An interaction in which doubling of CO2 cannot be argued to have other than a small effect on global temp, less than plus/minus 0.3 C. Why do you mislead by forgetting thermodynamics?
    PS Postma’s model is not mine.

    • Claes, a zero dimensional model that includes only radiation tells you only about radiation. The actual interpretation of the effective radiating temperature requires thermodynamics (the effective height of the effective radiating temperature depends on thermodynamics). So the point of my post is that for e ~1, you need a more complicated model to interpret all this in any detail. And I’ve provided a whole list of more complicated models to use. Bottom line: the existence of the greenhouse effect is demonstrated by the observed infrared spectra, and its interpretation by the physics that goes into radiative transfer codes. The zero-dimensional model is not used to determine the magnitude of the greenhouse effect, nor can it be used to deny the existence of the greenhouse effect. in the context of comparative planetology, it does support the existence of the greenhouse effect.

    • Here is another attempt at trying to untangle the Skydragons’ misunderstanding about the greenhouse effect and the planetary energy balance.

      Are we any closer to the goal?

      Can we ever get closer?

      • If it is an actual disagreement, as opposed to a misunderstanding, then the goal is misstated. Nor are a few lines from a textbook likely to resolve the disagreement. In The Structure of Scientific Revolutions, Kuhn describes how proponents of conflicting paradigms often talk past one another. They cannot seem to come to grips with their differences. We may have here a classic case of this deep confusion. (I did my doctoral thesis on this problem, formulating a new theory of scientific language in the process.)

      • Again David, you’re supposing that your ‘expertise’ in issue analysis can supplant real technical expertise in physics. This is fallacious reasoning given the fact that Miskolczi, Johnson and Postma offer no other ‘paradigm’ in place of quantum mechanics, on which all of the physics that leads to the GHE is based.

        So there is no issue related to how I, Pekka or Judy ‘come to grips with their differences…’ because there are no ‘differences’ other than these other ‘theories’ being incorrect.

        Maybe you can’t see that because you lack technical expertise, but you shouldn’t confuse your lack of understanding with the existence a of real ‘issue’. Unless these guys predict observables in a particular quantum mechanics experiment, it’s all garbage.

      • maxwell
        Miskolczi was employed by NASA as a theoretical physicist
        Johnson and Postma are certainly well educated enough to form opinions with sound reasons for doing so.
        You are almost claiming infallibility for proponents of the Greenhouse Theory

      • Bryan,

        I’m not claiming the infallibility of anything. I’m saying that there is clear observational evidence that proves the theories presented by the ‘researchers’ named above as totally and utterly incorrect. There is no way around this fact, which is why each of them makes such an enormous deal about the way that specific equations are derived or code is written.

        As I have said before, I have no doubt that these particular men can do math correctly, but when applied to physics, there is nothing but failure so far. I would go into greater detail of this observational evidence, but it’s found elsewhere I’ve written it and your combative attitude on other threads leads me to believe that it would be a futile attempt at educating.

        So go on believing whatever you’d like. The physics of real world could care less.

      • Maxwell says

        ….”So go on believing whatever you’d like. The physics of real world could care less.”….

        There is a growing number of well qualified mathematicians and scientists(particularly physicists) who have grave doubts about about the AGW story.
        The list runs into several thousand.
        If you are such an expert why don’t you stick to pointing out the flaws of (lets say) Postma.
        Instead you come across as wanting to stop dialog!
        My rejoinder to your quote at top is;

        ….”So go on sticking your head in the sand. The physics of real world could not care less.”….

      • Bryan,

        How can my head be in the sand while I’m standing at the front lines of science challenging currently held theories?

        I’m actually doing what Claes, Miskolczi and Postma trying to do, but I’m at least trying to use correct physics to do it. Making predictions that are testable and then re-assessing my theoretical contributions as the results come in.

        I’ll be honest. I’m more wrong than right, but I am also willing to acknowledge I’m wrong and fix the problems. We have seen no such behavior of any of three researchers whose work has been featured here and elsewhere. They change their story, refuse to answer to serious concerns or will simply stop engaging in a conversation with particular contributors, as has become the case between Claes and myself.

        If anyone is sticking their head in the sand it’s these three guys. No criticism is worthy of note nor are the standard accepted physical interpretations of the real world.

        The stubbornness they display is noteworthy only for it’s amazing fortitude and little else at this point.

        Moreover, it seems rather ludicrous to me, and maybe others who have witnessed the exchanges, that you should claim I’m trying to stifle dialog. There are extensive comments in which I was totally engaged with Claes, as well as others, from months ago. Without looking at this conversations, your point is moot.

        And when I made my points, Claes just put his fingers in his ears, as he did with the criticisms from other contributors.

        So he’s been stifling dialog. Go look at the comments. Rather than clearly explaining the meaning of his theory, he resorts to poorly thought out analogies and improperly done calculus. One cannot reason with on these points. I’ve tried for going on a year now.

        So again, rather than trying to figure out what the correct context is in this conversation, you’ve assumed that you’ve seen all that is to be seen and accounted for, despite the fact that you clearly have not. Please start at the beginning of the Dragon’s Slayers threads here on this blog, read all the comments (more than 5000 at this point I would presume) and then come back and talk to me.

        I’ve now put in a great deal of effort trying to talk about meaningful physics with a group of people dead set on denying well documented and understood physical law (all from quantum mechanics, mind you) in order to maintain a specific position about global warming. When they continue to ignore my attempts, why should I continue to engage with them?

        ‘Insanity is repeating the same thing over and over again and expecting the different results.’

      • What’s with this Claes character? I paged through his huge opus on applied math called “Body and Soul” and haven’t a clue as to what he intended to accomplish with it. (other than to use it as an example of his being constantly misunderstood :)

      • Maxwell, what you are ignoring is the hundreds of articulate responses these folks have given to the criticisms and arguments against them. Claiming there is no issue here is unrealistic.

      • David,

        You are unbelievable. Do you really believe that there was anything of value in their responses?

        That’s the problem that I and many scientists have with philosophers of science. They think often that their formal thinking can supersede scientific knowledge.

      • simon abingdon

        Pekka, you guys convince me altogether on the physics. But the sensitivity issue remains unresolved and I don’t trust the accuracy of models in principle. So where are we?

      • As I’m a physicist, not a climate scientist, I have personal understanding only to a point. Essentially I trust in my understanding of the idealized models that explain the general physics and structure of the atmosphere, a typical strength of the GHE (not exactly that of the real Earth, but something similar), the radiative forcing, and the non-feedback climate sensitivity (although I don’t really like the concept).

        Going beyond that I’m a layman with some ability to make educated judgments on how far I should trust, what working climate scientists tell.

        (I’m also a systems scientist with fair research level knowledge on modeling and on some fields of economics. I was physicist first, the other expertize is more recent. And one more point. I have worked very much with engineers, and I have taught university level engineering students.)

      • David,

        ‘…what you are ignoring is the hundreds of articulate responses these folks have given to the criticisms and arguments against them.’

        That’s exactly it. You have such a complete lack of technical knowledge in physics and physical science that you are easily confused between the totally nonsensical arguments that have been proposed to criticism and ‘articulate responses’.

        For Johnson to say that his theory ‘comes from Maxwell’s equations’ was the most I could get him to say to me with respect to the actual physics behind his theory. To the untrained eye, like yours, that may seem ‘articulate’ (although I think my chihuahua would see through it). To me, the trained optical physicist, such a statement is totally nonsensical. Of course he used Maxwell’s equations. He’s deriving dynamics of radiation. If he didn’t use those equations (and I’m still not sure how he’s using them), then his theory would be even more incorrect.

        But there is more important point. You have several times in these threads asked for observational evidence of the incorrectness of these theories to which several commenters, including myself, have provided ample evidence to show. Yet you still want to claim that there is an ‘issue’ here. It’s total ridiculous.

        And here’s what you told Pekka on a thread just recently,

        ‘Perhaps you should stick to physics and leave communication theory to the experts.’

        yet, you are constantly insisting that as a non-expert in physics, you can include yourself in a discussion as what is and what is not relevant in terms of the physical theory of the greenhouse effect.

        So physicists should stay out of communication theory, yet a communication theorist should be allowed to do physics?

        It’s this kind of backward reasoning that is going to marginalize your opinion on these and other topics on this blog in the time ahead of us. Although you still have time to change course.

      • David said:

        ‘Perhaps you should stick to physics and leave communication theory to the experts.’

        I get that criticism as well. According to some people, everything you write needs a Carl Sagan version to complement the scholarly work. If you can’t write like Carl Sagan, they expect you to hire a writer. Yet they don’t realize that the ghost-writer has to be able to understand what you have written and communicate it without distorting the meaning.
        The reality is that share the knowledge with other scientists to the best of your ability and we will work it out.

      • You are ignoring the context. Pekka was propounding a theory of communication that did not work.

      • Maxwell, your disdain for my science does not impress me. I don’t want to do any physics. I just want to map the statements to see the lines of argument, or rather I am pointing out that they are mappable. I don’t have the money to do the mapping. .Something you clearly do not understand. In fact if I did map the issue tree you would be able to point to the specific errors you claim they are making.

      • David,

        ‘…your disdain for my science does not impress me.’

        What science? Philosophy is not science. Knowledge mapping is not science. ‘Issue analysis’ is not science.

        Let’s be clear.

        I think that what you do has importance. I’m not diminishing the value of your expertise or its necessity in the appropriate context.

        But the question still remains whether you have the appropriate expertise to be able to to say that there is a real technical debate going on between the proponents of the standard understanding of the GHE and the ‘dragons’. Based your responses so far, my assessment as someone who has explicit expertise in molecular, optical and chemical physics, is that you do not have the requisite expertise.

        To say something along the lines that I have ignored the ‘articulate responses’ of the ‘slayers’ is enough for me to know that you are vastly out of your league in technical knowledge for your ‘mapping’ to be any importance. I agree that would be able to find where the errors were in such a mapping, but it would a vast waste of your and my time to even get to that level of development. There is nothing of physical value contained in what Johnson or Postma have offered so far. Their complete lack of engagement with contributors here, including myself, makes me believe that there is no hope for such engagement.

        As for your quote, I understood the context quite well. Pekka was making a point concerning the quality of communication of a contributor’s thesis which you were having a hard time understanding. Based on that fact, you thought it would suit your position to appeal to your own authority, which I do not see as an issue in that context.

        In the same vein, I am appealing to my own authority in the realm of physical science and explicitly telling you that there is no ‘talking past’, ‘revolution’ or ‘ungraspable differences’ between my interpretation of extremely well understood physical law and that of Johnson and Postma. They are simply mistaken in their theories.

        End of story.

        Yet again, I feel that you should strongly consider accepting this notion as fact at this point or go ahead and learn the actual physics yourself at the level to which I am capable of discussing it in order to prove me wrong.

        If not, then what is your point? That one can make a good argument with incorrect physics? Is that a point worth making here?

      • steven mosher

        I’m sorry but I havent seen any articulate responses from them. I have seen changing the argument, ignoring the criticism, answering the wrong question, failure to clearly articulate testable predictions from their “theories”. The Slayers engage in “gainsaying’ not dialog. That’s clear. That you don’t see it is puzzling.

      • Mosh, the kind of reasoning moves you are talking about are not what I mean by articulate. I just mean well formed sentences making points of obvious substance. Note that whether they are true or false is irrelevant as far as building the issue tree is concerned. Moreover, a lot of what you describe sounds exactly like talking past.

      • I’ll deny that they make points of obvious substance. I am 112 feet tall.
        is well formed, and appears to make a point that is obvious, but it’s missing something. It lacks warrant.

        I think the tree works fine for understanding a conversation where good faith is practiced. This is not one of these cases. You either have people who are 112 feet tall, deluded, willfully ignorant, lying, or come up with your own explantion

      • Maxwell, Good point. I see it as people that try to orchestrate the analysis so that it meets the goals that they want to achieve. The beauty in real citizen-based science is taking the theory and the data and analyzing it and then seeing where it takes you. It could reveal new ways of thinking, it could lead to new inventions, or it could lead to you completely changing your original mindset. Scientific thinking is always about keeping an open mind.

      • My fledglings keep falling out of their nests. I’ll re-post:

        Well, here’s an interesting juxtaposition:

        There may not be any [heavyweights with substantial past academic or applied credentials, with demonstrated meta-cognitive abilities to reason about diverse scientific problems and communicate that reasoning.] Most of our experts are relatively incoherent. This is not unusual. One of the central roles of the university is to shelter such people.

        and

        (I did my doctoral thesis on this problem, formulating a new theory of scientific language in the process.)

      • David,

        Do you believe in full relativism in natural sciences?

      • Pekka, Does full relativism mean there are no rational reasons to believe in one theory versus another? Or does it mean that one’s understanding depends on the context on the person’s worldview? Or is it something else?

      • The first sentence on relativism in Wiki tells essentially, what I had in mind:

        Relativism is the concept that points of view have no absolute truth or validity, having only relative, subjective value according to differences in perception and consideration.

        I didn’t have in mind any deep philosophical analysis, just a loose interpretation of such formulations as implying that anybody’s subjective view is of equal value in judging what’s true physics.

      • Pekka
        Thanks for raising that issue. Such relativism is directly contrary to the scientific method which requires observable evidence against which models can be objectively evaluated.

      • It’s thought provoking to notice, which participants in the discussion of this site are most eager to write messages about the scientific method, and how it should be interpreted and applied.

        Unfortunately that happens almost exclusively with the aim of discrediting valid science based on ignorance on, how good science is really done and shall be done. Only in exceptional cases is the issue raised in connection with real problems in scientific practices.

      • Pekka, probably not although I do not know what you mean by relativism, much less full relativism. I am certainly not claiming that the dragons are right, or that nobody is right (which is what relativism sometimes means). I take no position on who is right because I have not tried to follow the physics.

        I am a logician and my points have to do with the form of the argument. Logic is not about truth, it is about form, specifically the forms that reasoning may take. I have spent the last 40 years studying, diagnosing and resolving confusions, so I am telling you what I see going on here.

        By way of background, my training is in analytical philosophy. Analytical refers to concept analysis. The original idea, developed a hundred years ago by Russell, Wittgenstein, and many others, was that the classic philosophical problems were conceptual confusions. What I discovered is that there are many conceptual confusions in ordinary discourse as well. So I became an applied concept analyst (probably because I was also an engineer), specializing on confusion in complex issues. I see things going on in this debate that you physicists don’t even know exist, because the logic of debate is not your field. (I have a diagnostic system of 126 kinds of confusion.)

      • David,

        I have read a lot of philosophical literature, and know quite a bit of it, but it’s pointless to start to go deeper in that.

        The point that I hold essential is that, when philosophical considerations lead to explicit conflict with substance knowledge, the error is in the erroneous application of philosophical ideas. That happens too often.

      • Pekka, first of all I am not advancing any philosophical considerations. Issue tree analysis is a science. Like word problems in algebra, I simply map relations between statements, relations that are already there. Second, if there is any conflict with substance knowledge in the slayer case it is in the statements, not the mapping. When I do an issue tree of a complex issue each side gets fair representation. That is the only way to find out what the issue is really all about, and where the confusions are.

        Mind you, I am not doing an issue tree of the dragon slayer issues, because that would be a big job. But I have seen enough to see (based on 38 years of experience) that there is a genuine complex issue here, with many lines of internal reasoning and debate. The slayers may be wrong but they have one or more systematic ideas, and there is a lot of talking past going on.

        I have a rule of thumb called the rule of 2000, which says that one has to work through an issue tree of at least 2000 statements to really grasp a complex issue. This rule clearly applies to the slayer issue. The interesting issue science question is whether it applies to each of the individual slayer arguments, or whether they have a unified position. I suspect the former, which adds greatly to the complexity.

        However, I have found that no two issues have the same structure so I do not try to guess what it is going in. When I go into an organization that is suffering from confusion due to issue complexity it normally takes me a few months to figure out what the underlying problem or conflict is. And I usually need a subject matter expert to help me along the way.

      • David,

        this is so much more simple than what it seems necessitates an ‘issue tree’.

        Claes’ theory clearly predicts that 275 K radar dish detector cannot absorb radiation emitted by a blackbody at a temperature below 275 K. Yet, a radar dish detector can detect the cosmic microwave background radiation from the big bang itself. This corresponds to the blackbody emission of space at a temperature below 5 K.

        So clearly Claes’ theory is wrong.

        There are very similar clear points of contention between the predictions of both Postma’s and Miskolczi’s theories as well. There is no need to assess each and every one of their grammatically correct, yet physically nonsensical responses when these inconsistencies are pointed.

        When are you going to grasp this point?

        Algebraic arguments concerning logic don’t matter when physical reality dictates what is and what is not truth. That is the ultimate failure of logical arguments in this context. Observational evidence is a necessity, which seems to be the failing of your interpretation of this entire enterprise. Without the sufficient technical expertise to be able to assess the claims of these theories in light of well known physical observations, all of your points concerning issue analysis, trees or what have you are totally moot. They don’t matter in this context. There is nothing ‘complex’ happening.

        Why are you struggling so mightily to grasp this point?

        Have you even gone through the first two slayers threads? Have you actually looked at the concerns raised and inconsistencies point out?

      • I have a diagnostic system of 126 kinds of confusion.

        But there is also a diagnostic system of intentionally misleading arguments (the list of fallacious arguments ) and then you have the scoring system for someone who is off the deep-end ( the crackpot index). These strategies are not borne by confusion but they come from human intent of various forms, including that of game theory. How do you account for these kinds of arguments?

      • IPCC seems to have employed a couple of these viz.

        Argument From Adverse Consequences (Appeal To Fear, Scare Tactics):

        Special Pleading (Stacking The Deck):
        using the arguments that support your position, but ignoring or somehow disallowing the arguments against. This is also known as selection bias or cherry-picking.

      • Web & Gyp, indeed there is no lack of fallacies. In fact I spend a lot of time identifying them, as it is something logicians have done for several thousand years. Fallacies are certainly confusion causing factors, which is technically what my taxonomy of confusions lists. But I do not usually concern myself with intent, as that is psychology, which is not my field. I assume everyone is honest unless I have strong evidence otherwise, which occasionally happens.

        I would point out that a lot of what you are talking about is advocacy. Advocacy, or making your best case, is not generally considered to be dishonest, as it is the basis of both our policy and our judicial systems.

      • David,
        Curious why Postma links to Lyndon LaRouche, referring to him as an “American Patriot and polymath”, and then quotes him over a page in length, where LaRouche discusses Platonic ideals. This seems right up your alley, but I can’t make much sense of it.

  6. Claes Johnson

    Judy, you claim there is GHE referring to a model which only includes radiation and no thermodynamics, which you admit does not at all describe climate.

    Yet you claim that the GHE you are describing is a climate GHE. Don’t you see that you are contradicting yourself? Or is your GHE not about climate?

    • Claes,

      the major point has stated very clearly by Judy. The measured radiation in space emitted by the atmosphere corresponds to a greybody at 255 K. The surface, however, is 288 K. The greenhouse effect is the difference because it can theoretically explain the stratification of energy in the vertical and latitudinal directions VIA RADIATION. Part of how that stratification occurs is due to radiation and we can measure it to support the GHE theory as described by Judy in her text, which I would suggest as informative reading on the subject.

      Furthermore, the actual measurement of the atmospheric radiation itself is a falsification of your theory as incorrect given that a warmer blackbody (a detector of some kind) is absorbing radiation from a cooler blackbody (the atmosphere).

      So there you have it.

      • Everyone appears to missing the most obvious The question to be asked is this: If the earth had an atmosphere without CO2, what would the surface temperature be? How much of the 33C warming is due to H2O and how much is due to CO2?

        CO2 contributes much less than H2O, so we ware looiing at a small fraction of 33C due to CO2. Maybe 5C? Now, as we know CO2 is a log contribution, how many doubling of CO2 are there in our current 400 ppm to give us this small fraction of 33C? Maybe 2 ^ 9 or more? So the contribution of each doubling of CO2, maybe 5/9 C max.

        Which explains why temperatures have not increased significantly even as CO2 output has shot up like a rocket due to rapid industrialization in China and India.

      • Lacis et al. (2010) did a model study. The “Control knob” paper showed a reduction of nearly 35 C without CO2. This is because a lot of water froze out and increased the ice albedo too which added a positive feedback. Interesting experiment.

      • What would be the reduction without CO2 AND H2O?

      • About 35C, because w/o CO2 (and other non condensing GHGs, but mostly CO2) the GHE collapses and H2O disappears from the atmosphere. Just like Jim D said.

      • It is circular reasoning. It proved only that a model based on radiation and radiation forcing gets to those results in a flat world without night and day.

      • They used equations for dynamics and physics and time-integrated them in a low-resolution GCM.

      • In a flat world without night and day at double the distance from the sun. Sorry, we have past the flat world image for the earth, but some still hang to it.

      • Is that your model, or are you talking about Postma here?

      • I would be interested in Postma’s computation of the global annual average solar flux at the Earth’s surface. Until he does that, he is not talking in climate terms.

      • Alexander Harvey

        ferd,

        I am not sure that I do know that the logarithmic relationship is thought to hold throughout the progression down to zero.

        To do so with the knowledge that the current value is finite would imply that below some point the sign of the effect would reverse followed by a descent towards an infinitely negative value.

        I do not think that this is the case and infer that at some point the nature of the curve may tend towards the linear and even something more swallow and approach zero just as the concentration approaches zero.

        I could but speculate on the number of halvings that would approximate a logarithmic rule but I can but think that it is not unbounded.

        Alex

      • Norm Kalmanovitch

        The true point that is missed is that it is not gases but water in liquid and solid form i.e. clouds that provide well over 75% of the greenhouse effect.
        Water ingaseous form combined with the effect from clouds provides well over 90% of tyhe greenhouse effect.
        In the absence of CO2 clouds and watervapour could easily account for over 99% of the greenhouse effect but because when no cvlouds are present CO2 first takes its chunk out of the thermal radiation from the Earth over the band centred at 14.77microns providing about 10% of the greenhouse effect.
        Essentially if there is enough food for ten people but an eleventh person comes to the table the same amount of food is consumed with each person consuming less. The same is true for the greenhouse effect; there is only a limited amount of the earth’s radiative energy captured by the atmosphere and if CO2 increases the only thing that changes is that clouds and water vapour trap less.
        Any publication about the greenhouse effect on Earth that does not explicitly state that over 90% is from clouds and water vapour is scientifically fraudulent, because the named greenhouse gases in the Kyoto Accord with the exclusion of CO2 have a combined contribution to the greenhouse effect that is indistinguishable from zero!

      • I concur Norm.

        Ignoring the ‘habits’ of the atmosphere (as a whole) and just taking note of the ‘habits’ of, so called, ‘long lived radiative gasses’ is like “throwing the baby out with the bath water”!

        One can only observe (and ‘mark’) part of the observation. This can only lead to ‘obsfuscation by exclusion’.

        Best regards, Ray.

      • Which explains why temperatures have not increased significantly even as CO2 output has shot up like a rocket due to rapid industrialization in China and India.

        Fred, are you claiming you can see CO2 coming out of China and India that the observatory at Mauna Loa cannot? I would love to learn how to do this, are you willing to divulge your trick or is it a closely guarded secret?

        CO2 shot up “like a rocket” between 1960 and 2000. In that period temperature climbed 0.5 °C. Looks like a strong correlation to me.

  7. Judy, consider the difference between ‘doesn’t exist’ and ‘needn’t exist’. It is creating a strawman to claim Postma’s model is a claim of ‘doesn’t exist’. It’s merely speculating that the GHE need not exist. In my opinion, his effort lies somewhere in the class of information suggesting that the effect of CO2 has been exaggerated. Where in that class? Well, let me check the seating diagram. It’s early in the semester.
    =========

    • Kim the way to go is to break the problem into pieces. Once we can get past the basics of infrared radiative transfer and the fact that the greenhouse effect exists and acts in the direction of warming the planet, then we can get to the more interesting questions like how does earth’s surface and atmosphere respond to this heating thermodynamically, and then integrate the radiation, thermodynamics and dynamics to try to understand the feedbacks.

      So sure, w should all be skeptical of a high confidence in a 3C sensitivity to a doubling of CO2. But the reason for being skeptical about this number should be in the complexity of the coupled thermodynamics and dynamics, not the basic radiative transfer of the clear sky greenhouse effect.

      • Oh, yes, I understand. It seems that Postma’s effort has focussed attention on the inadequacy of the ‘coupled thermodynamics and dynamics’ more so than the basic greenhouse effect. In the conversation, more attention was placed on arguing about the greenhouse effect’s existence, which is, as I say, a strawman. That Postma’s work could imply no greenhouse effect doesn’t mean that it does and I don’t think that it does.

        So can we get on with the ‘coupled thermodynamics and dynamics’? That is where it is all at, and we all know it.

        I think I’ve never heard so loud
        The quiet message in a cloud.
        ============

      • Hmmm.

        Well, we have this:

        In my opinion, his effort lies somewhere in the class of information suggesting that the effect of CO2 has been exaggerated.

        And we have this:

        Joshua | August 17, 2011 at 1:59 pm

        Again – please excuse my ignorance – but as I read it, you are not simply hypothesizing a phenomenon, but you’re also hypothesizing that the phenomenon you describe has an overall, significant impact on global energy balance – to the extent that it negates the understanding of the vast majority of people who study the relevant physcial interactions. You are hypothesizing not only about a phenomenon, but about the degree of impact of that phenomenon. As such, it would seem that you would seek opportunities to use controls to measure that degree of impact.

        Along with this:

        Joseph E Postma | August 17, 2011 at 2:04 pm

        That’s pretty good assessment Joshua, I’d buy that.

        And we also have this:

        Joshua | August 18, 2011 at 11:21 am

        By including your focus on the GHE, and by concluding that you’ve proven that it doesn’t exist, you are wading into the swamp of a much larger argument…

        Along with this:

        Joseph E Postma | August 18, 2011 at 12:05 pm

        I’d buy that assessment Joshua. However it did seem immediately “obvious” at the time that it implied the GHE, as we understand it today, is not likely what actually exists.

        And The Magic 8 ball says: “Situation somewhat murky.”

        Careful there, kiim. You wouldn’t want to be winnowing, would you?

      • Joshua, you emphasize ‘is not likely what actually exists.’ I emphasize ‘as we understand it today’. You want to spin it back to the existence or not of GHE, I want to spin it to today’s understanding.

        I applaud your interest in the science. I can’t help but believe it will help you a lot.
        ===============

      • Getting into the highly nonlinear, coupled thermodynamics, which as you say is where the real analysis is, is so far above Joshua’s and several others heads that they MUST keep bringing the conversation back to a simple concept that they can continue to pretend tells the whole story. Otherwise they might have to admit that the “science isn’t settled”, and that would never do.

      • Bruce too

        It’s the natural reflex of a mathematician when discovering an error in another’s work to resort to first principles to explain the source of the error.

        I have no way of ascertaining the mathematical aptitude of any commentator on this blog; other than to suggest locking them in a room with Chief Hydrologist and asking them to derive a solution for any of the unsolved classic problems of mathematics, and time how long before they pick the lock and escape.

        I do know, however, that glaring errors pop up repeatedly in this discourse, and I have far greater regard for those who see the errors when they’re explained, struggle through reworking their computations, and move on to accept ideas based on correct logic than for those who keep insisting that because their conclusion must be right, the error cannot be there, or it cannot affect the outcome.

      • ‘Would they take to Rebelasian ‘disciplination’?

        Chapter 1.XXIII.—How Gargantua was instructed by Ponocrates, and in such sort disciplinated, that he lost not one hour of the day.

        ‘Afterwards he put himself into such a road and way of studying, that he lost not any one hour in the day, but employed all his time in learning and honest knowledge. Gargantua awaked, then, about four o’clock in the morning. Whilst they were in rubbing of him, there was read unto him some chapter of the holy Scripture aloud and clearly, with a pronunciation fit for the matter, and hereunto was appointed a young page born in Basche, named Anagnostes. According to the purpose and argument of that lesson, he oftentimes gave himself to worship, adore, pray, and send up his supplications to that good God, whose Word did show his majesty and marvellous judgment. Then went he unto the secret places to make excretion of his natural digestions. There his master repeated what had been read, expounding unto him the most obscure and difficult points. In returning, they considered the face of the sky, if it was such as they had observed it the night before, and into what signs the sun was entering, as also the moon for that day. This done, he was apparelled, combed, curled, trimmed, and perfumed, during which time they repeated to him the lessons of the day before. He himself said them by heart, and upon them would ground some practical cases concerning the estate of man, which he would prosecute sometimes two or three hours, but ordinarily they ceased as soon as he was fully clothed. Then for three good hours he had a lecture read unto him. This done they went forth, still conferring of the substance of the lecture, either unto a field near the university called the Brack, or unto the meadows, where they played at the ball, the long-tennis, and at the piletrigone (which is a play wherein we throw a triangular piece of iron at a ring, to pass it), most gallantly exercising their bodies, as formerly they had done their minds. All their play was but in liberty, for they left off when they pleased, and that was commonly when they did sweat over all their body, or were otherwise weary. Then were they very well wiped and rubbed, shifted their shirts, and, walking soberly, went to see if dinner was ready. Whilst they stayed for that, they did clearly and eloquently pronounce some sentences that they had retained of the lecture. In the meantime Master Appetite came, and then very orderly sat they down at table. At the beginning of the meal there was read some pleasant history of the warlike actions of former times, until he had taken a glass of wine. Then, if they thought good, they continued reading, or began to discourse merrily together; speaking first of the virtue, propriety, efficacy, and nature of all that was served in at the table; of bread, of wine, of water, of salt, of fleshes, fishes, fruits, herbs, roots, and of their dressing.’
        http://www.gutenberg.org/files/1200/1200-h/p1.htm#2HCH0005

      • I feel your pain! If you follow this discourse as much as I do, you should have a very low regard for the likes of say, um ……Michael Mann and his apologists. He used (deliberately?) data in his 2008 paper from a Finnish scientist (Tiljander) that was upside down from what the data actually said. When this is pointed out to them, they categorically refuse to acknowledge the error, even going so far as to say the claim is bizarre! And we have a president that actually listens to him!!!

        Interesting times.

      • Bruce too

        Oh, you mean http://arthur.shumwaysmith.com/life/content/wheres_the_fraud vs http://www.bloomberg.com/news/2011-08-22/climate-change-scientist-cleared-in-u-s-data-altering-inquiry.html?

        There we get into the murky distinctions between vindication in the face of persecution, fighting the good fight, going down with the ship or blindly holding to wrong ideas.

        Let’s look at Joachim Seifert (who has been extremely patient with me); his thesis on the degenerate elliptic motion of the planet (libration) and coincidental paleoclimate is interesting, and he is very confident he is right.

        Is JS right? Wrong? Dunno.

        Were JS faced with such an outpouring of condemnation as MM over the past 3 years, would he fold under pressure, or carry on, as MM has done up to his most recent of several substantial vindicating events?

        Dunno.

        So, with all the murk, can’t say I’m in position to declare glaring error per Mann re Tiljander, unless I’m willing to spend a lot of time researching the matter in the hopes I see something everyone else has missed.

        A far cry from what I

      • Bart R

        Had not noticed your reply since it was off margin.

        I guess I should not waste any more of either one of ours’ time. I read the lame articles that you linked to. They are the same old insane denial of facts that alarmists can’t stomach. For all the long winded tripe in the first one, it never did directly answer whether or not Mann used polluted data and whether he inverted the data (that showed cooling not warming). Just lots of red herrings to change the subject in order to prevent having to answer as simple a question as can be asked. ie Did he use data that Tiljander said was corrupted and should not to be used. ie Did he then use the data inverted? The answer to both questions is yes. I put a ? after deliberately in my first post. I only strongly suspect he did it deliberately. But there are people in prison for less proof than this.

        As to the second article pointing out that all the Climategate guys are lily white, well as our host, and many other scientists (the British IOP ie) have said, the inquiries were nothing but whitewashes. Dr. Curry likened it to the fire department being told that there is a fire in the kitchen of a house, only to have them say that they looked and found no fire in the living room. Very funny I thought.

        Bye

      • Dr. Curry likened it to the fire department being told that there is a fire in the kitchen of a house, only to have them say that they looked and found no fire in the living room. Very funny I thought.

        My experience with analogies is that they only work on the choir. Evangelists are better served by recruiting would-be converts to the choir by appealing to their love of singing. A few hymns serve to turn off the choir’s critical faculties, thereby putting them in a state to receive an analogy as though it were just another hymn.

        My two cents on the function of church services. Which I’m all for as long as they’re kept to once a week.

      • Bruce too

        Perhaps having felt the intention of my words were twisted by your use of them sours me on trusting the interpretation you give the interpretation of the words of others, but I feel strongly warned to take what you say not at face value.

        As President George W. Bush said, “There’s an old saying in Tennessee — I know it’s in Texas, probably in Tennessee — that says, fool me once, shame on — uh — shame on you. Fool me — uh.. uhm.. uh.. — You can’t get fooled again. ”

        I don’t know what Tiljander said, and can’t be bothered to look it up and guess which event and context you might mean when you make your remarks. Implicit in your statements is a requirement of trust by the reader which, yes, is a waste of both our times.

        I don’t pretend to know the truth of this Tiljander-Mann controversy, however the opinions you have expressed have only rhetorical merit, and so one can see why links to reports of fact or discussions involving reasoned dialogue and debate would be lame by your calculation.

        Paleoclimatology has extraordinarily low resolution and is doomed to frequently provide contrary-seeming and moot evidence, at best indicating a wide error range and significant uncertainty.

        It tells us that anything we wish to say about global climate before the modern age of instrumentation is so limited that we ought treat such expressions as “Little Ice Age” and “Medieval Warming Period” as the feeblest indicators, and we ought learn utmost respect for the doubts we ought harbor as skeptics about claims on climate in general.

        So while I respect that paleoclimatology provides us this much perspective, and narrows down the probable ranges of expected climate values over the spans of time prior to this quarter millennium, I see zero reason to find it so exciting as yourself and many commentators do.

        These tempests are marginal scandals and infinitessimal tempests in a tiny teacup, and not interesting enough to warrant multiple taxpayer-funded inquiries that all appear to come to the same conclusion, by my admittedly minority point of view.

        But as they _have_ had multiple quasijudicial proceedings, and those proceedings have been concluded with fairly consistent agreement on the points you’re finding contentious, I have to ask, how is this not just sour grapes on your part?

        If you disagree with a finding to the point you have to allege incompetence on the scale of firefighters going to the wrong room and missing the fire — or is it corruption or deceit you accuse all of these official investigations of? — then shouldn’t you as a reasonable person seek every measure of appeal or overthrow, instead of just blogging defamations about it bitterly?

        Me, I believe it’s not worth my time, as the interesting part of this topic isn’t the paleo record, which is principally useful for squashing the intellectual insects who spout LIA-this and MWP-that like bible-thumping pulpit bullies chasing down a Darwinian.

      • “Paleoclimatology has extraordinarily low resolution and is doomed to frequently provide contrary-seeming and moot evidence, at best indicating a wide error range and significant uncertainty.

        It tells us that anything we wish to say about global climate before the modern age of instrumentation is so limited that we ought treat such expressions as “Little Ice Age” and “Medieval Warming Period” as the feeblest indicators, and we ought learn utmost respect for the doubts we ought harbor as skeptics about claims on climate in general.”

        If you are pretending you can measure average temperature in in terms of tenths of degrees hundreds or thousands of years ago, whereas we can’t even really measure tenths of degree in terms of average temperature currently. There are problems.
        There are similar problems when you imagine you able to accurately measure the PPM of global CO2 from Ice cores of the past. You can’t accurately measure current level of CO2 by examining new ice.

        But if you measuring a order magnitude greater, such 1 degree C rather than a 1/10th of C. Or if you are measuring hundreds of PPM of CO2, instead two hundred, fifty two PPM, at this resolution one can get accurate data.
        From Paleoclimatology we can determine that global temperatures of the Little Ice Age were about 2 C cooler than present temperatures. We can’t know if it was 2.13 C cooler or 2.1 cooler. Or that the range was 1.8 to 3.1 C cooler. But the time period of Little Ice Age is “known” because it marks a period global glaciers advancement and ended with glaciers globally receding. You can date the ice in glaciers. There is a lot science related to this. There is little to doubt about this.

        Paleoclimatology also indicates that “Medieval Warming Period” was centuries in length and generally as warm or warmer than the present temperatures.

      • gbaikie

        I’d be delighted to read details of your supporting evidence and the independently audited statistical analyses substantiating your claims.

        Do you have links?

      • Judy,
        You avoided the spirit of dispassionate scientific inquiry from the start because you refuse to accept that Postma has demonstrated that the numbers add up with or without factoring in a GHE.
        The first half of his paper systematically lays out the standard GHE and the numbers and equations involved. He then then uses such numbers when he re-thinks the model paradigm and adds into the equation the ACTUAL insolation numbers from measuring the energy inputs of night and day.
        His approach is CLOSER to reality than the standard GHE model because that OMITS the existence of the inertia for such insolation due to the difference in day/night – plus Postma also includes adiabatic physics. He thus shows that making Earth into a flat disk under 24-hour twilight causes the GHE model to be 33K short in the overall known energy budget.
        He shows that the disparity is a product of the failure in averaging insolation. The definition of an average means you have both higher and lower numbers…simple laws of physics such as the adiabatic distribution tell you that the warm number should be found at the bottom, the cool ones higher up. It all adds together to show how the “average” can be properly calculated to balance with the incoming energy and thus demonstrate we do not need to rely on a mythical GHE of +33k.
        His point is we need to look for the true average from the DEPTH of the atmosphere not at the ground surface. He argues this is where average is *always* found – it is in the middle…no matter what the input forcing is. So if the input forcing is -18C and this is therefore supposed to be the average temperature of the system, a-priori you should not expect to find that at the surface, but in the middle, and the surface-gas will be hotter, all without a GHE in the first place.

        This is the only way we can reasonably considering -18C as an average input figure…with these caveats acknowledged and understood. Otherwise you need to consider the instantaneous inputs and work your way back to the daily average…you learn a lot more physics doing it this way anyway. The “averaging” route appears to have made many people go insane and seemingly forget what the averaging was *about* in the first place.

      • Norm Kalmanovitch

        One cannot have a proper scientific discussion about the greenhouse effect without a proper scientific definition. The discussion taking place is everyone arguing over an effect that does not actually exist but is merely a theoretical number for how much cooler a planet would be if the atmosphere was removed.
        On Mars this is 5.5°C on Venus this is 450°C on mercury which has virtually zero atmosphere this is just a tiny fraction of a degree C
        On Earth this is stated between 33°C and 35°C depending on how the calculation is done and the rounding involved.
        The entire AGW hypothesis is based on turning tghis theoretical number into an active process and then falsely attributing the entire process to fraudulently coined greenhouise gases which have no valid scientific definition.
        The simple truth is that water vapour and clouds account for well over 90% of this effect so by default CO2 at current concentration can account for no more than 10% of the 33°C. If CO2 concentration were to double all that would happen is that less of the effect would be due to water vapour and clouds and more would be due to CO2 but the net effect would still be the same 33°C.
        The monly thing that could change the value of the greenhouse effect is a change to the Earth’s temperature. If the Earth’s surface was hotter there would be more radiation and the same insulating effect of the atmosphere would produce a larger theoretical temperature differential also known as the greenhouse effect.

      • Norm

        I’m with you right up to the word ‘simple’, where I believe you may wish to avail yourself of a dictionary. ;)

        Your 90%, is it the top 90%, or the bottom 90%?

        Is it an independent figure, or does it vary with the dependent variable no more than 10%?

        You assert without explaining a pure negative feedback of CO2 and H2O effects that, apparently, is coincidentally also exactly equal in size for Temperature despite the many differences of the two molecules.

        Or do you? Your last sentence seems at odds with the paragraph it ends.

        One wishes to understand, how did you derive this wondrous function, unless you mean its opposite?

      • If CO2 concentration were to double all that would happen is that less of the effect would be due to water vapour and clouds and more would be due to CO2 but the net effect would still be the same 33°C.

        Fail.

        Like most of the world you assume that the warming effect of GHGs is linear in their level. If you would redo your calculations using the correct law, which is logarithmic, you would have a better chance of arriving at a reasonable estimate of the impact of GHGs.

      • JC says: “Once we can get past the basics of infrared radiative transfer…”

        That could be part of the problem. DavidN on another post indicates that
        the following are equal, SB(T1^4-T2^4)= SB(T1^4) – SB(T2^4)

        Do you think they are equal in radiative heat transfer?

        I donot.

      • Do you realize what you’re saying? You need to go back and review elementary algebra.

      • I do know what I am saying. Is that the way you do radiative heat transfer?

        You have just pointed out a problem in the discussions. There are no radiative heat transfer equations that use the right hand side of the = sign. And why would that be? Because you must first have a temperature grdient and the only way to do that is with the left side.

        Path and gradient are the basics and DavidN you cannot manipulate the temperature until you’ve established the gradient. Algrebra or not.

      • The left and right hand side are equivalent statements. They say the same thing and return the same answer, everywhere. Maybe the reason you don’t see the expression in the right hand form a lot is because the left hand form is a trivial simplification that requires fewer symbols, a simplification that every high school algebra student knows.

        The expression you gave, in either form, contains a temperature difference. I don’t believe the expression is useful in terms of a temperature gradient, becuase the blackbody surface is an idealized surface at the discontinuity between two regions with different optical properties, not a surface within a single medium.

      • David N,

        But it’s more than plain algebra, it’s a formula. The concept is not that hard if you think of synergy and emergence – the sum is greater than the parts type thing.

      • Nonsense. The formula y = c(x1 – x0) is the same formula as y = cx1 – cx0. Plug in any numbers you want.

      • But “SB” is shorthand for the entire Stephan-Bolzmann function; it is not a constant.

      • Please express SB as a function of T, with references, or accept that SB = σ, known as the Stefan-Boltzmann constant.

      • As I said DavidN you have just shown the first and maybe the largest problem. Doing proper radiative heat transfer equations. Once we start buggering up the language then the equations and what is next. If we cannot agree on the proper equations to use then all is lost in discussions.

        Path and gradient and you choose not to find the gradient. How sad.

      • You have not offered any different equations, you’ve only asserted that the same equation written two ways is somehow different, which is absurd.

        The SB Law is defined on a surface with a discontinuity. It’s not a vector formula. A gradient is typically defined over a continuous vector function.

        Instead of making vague reference to “buggering” things up and offering no actual content in your alternative viewpoint why don’t you respond at a equal level of sophistication, define your terms and submit your equations.

      • That’s only true in an operation-preserving isomorphism

      • Like an operation say, multiplying T^4 times a constant?

      • For some reason mkelly and Kermit seem to think that SB(x) is some sort of function applied to x, instead of merely multiplying the Stefan-Boltzmann constant by x. I would be fascinated to know what textbook this appears in.

      • Meanwhile I think I understand the confusion. The phrase “Stefan-Boltzmann” is being applied both to the dependence of power density on temperature and to its associated constant σ = 5.670400 x 10^-8. The former has the form σT^4, which denotes the product of σ and T^4. Anyone who thinks the Stefan-Boltzmann function denotes something else is kindly invited to step forward and explain what they think it denotes and why.

  8. Claes Johnson

    You did not answer my question Judy, an essential question: Is the GHE you know exists a GHE about climate? I think your readers expect an answer.

    • At this point Claes, if you don’t know the answer to this question, you should give up. Dr. Curry has made it crystal clear that she thinks that radiation plays a role in maintaining a higher level of energy at the surface of the earth than at the top of the earth’s atmosphere.

      What more is she to do? Enroll you in the earth sciences program at Georgia Tech for a masters degree?

      If you don’t understand the basic physical concepts involved and how they relate to Dr. Curry’s (or anyone else’) explanation of the greenhouse effect, I believe there is no hope for you. You’re already too far down the rabbit hole.

      • This exchange looks like a fine example of talking past. There is no obvious relation between Maxwell’s response about radiation and Claes’s question about climate. Thus Maxwell must be interpreting Claes’s question, probably in a way that Claes would not agree with. In science, concepts have theories built into them. People who hold conflicting theories have conflicting concepts of the same thing, so they will interpret the same sentences differently.

      • No, Claes will interpret anything said in a way that works for him only. And if he can’t interpret it at all (e.g. the entire discipline of statistical mechanics), then it’s bunk and you’re not allowed to use that part of science to argue with him. This last question doesn’t even make sense. Claes should explain the difference between a “climate GHE” and a non-climate GHE if he expects any kind of answer.

      • I agree the term “climate GHE” is unexplained, although he may have explained it earlier, which the phrasing suggests. The obvious thin to do when someone uses a term that is not understood it to ask what it means, but that sort of reasoning seems in short supply here.

        As for the rest, your extreme claims are diagnostic for talking past due to conceptual confusion.

      • It’s not an extreme claim, it’s the truth. Go find the threads from 6 months ago, or just ask Claes today what he thinks of statistical mechanics.

      • David,

        ‘This exchange looks like a fine example of talking past.’

        …yes, to you, the untrained non-expert in physics. Where was your take on my ongoing exchange with Claes when I explicitly pointed out that his violated the conservation of energy? Or that I pointed out I could easily calculate the response of the atmosphere, radiative and non-radiative, via the Maxwell-Bloch equations?

        I have been having a conversation with Claes for 8 or 9 months now pointing out the enormous physical flaws with his theory. They are clearly documented on this and his own blog.

        And you say,

        ‘Thus Maxwell must be interpreting Claes’s question, probably in a way that Claes would not agree with.’

        The onus is not on me to correctly interpret Claes’ question. I know what is physically relevant in this context. The onus is on Claes to correctly predict what we clearly observe in reality, which his theory cannot do at this point.

        So again, your lack of expert training in the context of the physics important to this discussion, as well as your lack of context in the actual conversation has misled to believe that there is ‘talking past’ by me. If there is any ‘talking past’, it clearly on the side of Claes.

        You should be beginning to make better mental notes of these facts.

      • steven mosher

        I think you are talking past david. he clearly wants to talk about talking past which is his speciality. So for him these discussions have to appear to fit into his system of understanding confusion. However, I wonder if Davids system has a mechanism for understanding deception. for understanding willful IGNORANCE. willful ignorance often looks like confusion, but it’s not.

      • People have a variety of reasons why they throw their hat into the ring and have different priorities driving their approach. The self delusion of belief in ones own perfect objectivity can easily lead to a belief in the wilful ignorance of others. Nobody has a perfect and fully developed theory of climate. The gap between pure physics and the application of physical laws everyone agrees and the complex systems we have imperfectly conceptualized and measured for a short period is such that disagreements about how those physical laws should be applied are inevitable.

        In that situation, those more convinced of the correctness of their approach might come to the conclusion that others who disagree with them are being wilfully ignorant. I suggest that given the uncertainties, we should strive to avoid such *assumptions* and maintain an open and civil discourse using the presumption of good faith.

        This is a pragmatic suggestion rather than a theory of disagreement.

      • TB,

        I would agree with you if we were discussing internal variations of the climate system and analysis with which we would tease out those contributions from the overall behavior of climate.

        But the slayers are not discussing those points. They are proposing that long held, experimentally verified and extremely well understood physical law is incorrect. They do this with poor math, poor physical assumptions and the backing of publishers totally impervious to criticism of any kind.

        I cannot be pragmatic if after 8 or 9 months of continuing to engage in a meaningful physical discussion on these topics these same ‘researchers’ are saying the exact same bogus unphysical answers.

        What is there even to be pragmatic about at this juncture?

        Claes believes that an IR thermometer cannot read the temperature of the atmosphere because it can’t absorb the radiation emitted by the atmosphere via his theory.

        That’s clearly wrong.

        Postma somehow can tell the difference between a ‘physical’ response and an ‘optical’ response, nor does he understand that climate is not an instantaneous response of the earth system.

        When going over the simplest points concerning time averaging to understand climatic change (change in the earth system over decades), he began an argument over who understood the concept of a Watt.

        Really?

        What more is there to do then to say that these men are willing to delude themselves in thinking obviously physically incorrect things? When someone is to say that a blackbody, which my definition absorbs ALL frequencies of light, will stop absorbing some frequencies because of the presence of a cooler blackbody it is not physically connected to, there is nothing to be pragmatic about.

        I’m sorry.

      • maxwell | August 19, 2011 at 3:49 pm | Reply

        TB,

        I’m sorry.

        Now you’re being wilfully disingenuous.

      • “What more is there to do then to say that these men are willing to delude themselves in thinking obviously physically incorrect things?”

        Well, we could say that although your judgement is that of an expert in physics, your abilities as a psychologist may be in doubt. ;)

        “I’m sorry.”

        Accepted, and your frustration will be considered to be a mitigating circumstance. :)

      • mosher,

        whether the ignorance is willful or not is meaningless to me. The point is that there is ignorance on the part of slayers as to what are and what are not important physical concepts, constructs and laws that necessitate critical analysis with respect to science behind global warming. If David is willing to accept that he is trusting ignorance as it has been called to his attention by numerous contributors, I would take that.

        As it stands now, I think he will eventually marginalize his position in the minds of others here.

  9. It matters not . . . might as well pack up the discussion because it is game over. NASA has now determined that Aliens are getting ready to destroy the planet because we have been, bad, bad, badder and soiled our planetary diaper.

    And that is why the NASA budget should be increased . . . because they are doing such stellar work for the good of humanity.

    • Childhood’s End.
      ===========

    • Hank Zentgraf

      The NASA associated scientist that offered the concern regarding the Alien attack is reported to be employed at Penn State University. Is he also closely associated with M Mann? Someone should check the drinking water on campus!

    • “NASA has now determined that Aliens are getting ready to destroy the planet because”

      Talk about misrepresenting the study.

    • NASA talks about finding microbial life on meteorites, searching for organisms in Martian soil, and getting funding to land on the moon again someday, too.

      Rocket jocks have a need to imagineer, and worse, many have little sense of when to stop geeking about it out loud.

      Seems like harmless fantasy to me.

      I mean, can’t you think of anyone you know connected to NASA who seems a little like that?

  10. Why is it felt appropriate to calculate an average blackbody temp for a sphere by averaging the radiation when the formula used has a fourth power in it? (Stefan-Boltzmann)
    First calculating the different temperatures and then averaging the TEMPERATURES makes much more sense to me.

    • Averaging temperatures can seriously mislead. Temperature says little about the actual quantity of heat. The end of the cigar and the ash that falls from it are close to the same temperature but only the cigar has the capacity to burn you.

      Also, when calculating a temperature from average radiation from S-B the expression has a 1/4 power.

      • Don’t follow your explanation.
        Daytime solarradiation can burn you seriously. Nighttime radiation not really afaik.
        Still doesn’t make much sense to me to first spread incoming solar around the earth and expect the SB formula to give a sensible temp for that “average” radiation.

      • Solar radiation burns you because the UV photons are of a much higher energy, high enough to cause cellular damage. The IR photons emitted by the earth are of lower energy and cannot do that.

      • BenAW

        I believe you’re right in your last line. Sort of.

        If someone is doing exactly what you suggest, in the wrong way and for the wrong reasons, and then using their results wrongly, they ought stop.

        However, the 1/4 power among other things means you have to have a very large change in radiant energy to get a very small change in temperature (and of course there are other complicating factors in the real world, such as that S-B is based on an imaginary ideal nonexistent color painted on surfaces).

        It also means that for different levels of energy, the same relative change in energy (or temperature) means a different outcome on the other side of the equation, and even a small temperature level difference makes a huge energy difference.

        Also, temperature density of different materials is very different. If you could transfer the heat in a swimming pool at room temperature into an iron nail, you’d liquify the metal, or some such. Specific heat capacity, maybe Google might have something on that search term.

        Shorter answer, ‘sensible temperature’ is meaningful, but you either have to simplify your model of the world to limit all the complicating factors that lead to errors, or you have to do a huge amount of math and fill books with lists of all the factors and all the ways they interact, to get useful answers.

        This it the nub of the problem so many people have with Claes’ methods.

        Claes is an immensely bright theoretical mathematician. Far better at it than me, certainly. And that’s flattering me immensely.

        Claes has a concept that has something to do with waves.

        It’s not even an incorrect concept, at its heart. Likely.

        It just requires far greater work than Claes appears to be willing to recognize is really needed to produce correct results.

        And so Claes takes shortcuts, ignores parts of systems that turn out to be important later, makes up stuff as he needs it to gloss over the problems caused because there aren’t any brains in the world big enough or fast enough to turn his concept into a real way to describe what happens.

        At least, that’s my take on it.

        Hope it helps.

      • BenAW, if your reasoning about radiation and temperature were right, it would apply to power and voltage as well. In that case no one would use the concept of RMS voltage, they would use average voltage instead.

        Have you ever wondered about the significance of RMS voltage? Happy to explain it if it’s not clear.

  11. No worries, ‘safe & sound’…

    http://www.guardian.co.uk/science/2011/aug/18/aliens-destroy-humanity-protect-civilisations

    Based on what; exactly?

  12. Well, here’s an interesting juxtaposition:

    There may not be any [heavyweights with substantial past academic or applied credentials, with demonstrated meta-cognitive abilities to reason about diverse scientific problems and communicate that reasoning.] Most of our experts are relatively incoherent. This is not unusual. One of the central roles of the university is to shelter such people.

    and

    (I did my doctoral thesis on this problem, formulating a new theory of scientific language in the process.)

  13. Claes Johnson

    Judy, come on now and answer my question. You present a GHE based on radiation alone, and admit that climate is thermodynamics with radiation. Is your GHE about climate?

  14. Professor Curry

    In your equation 12.a it would appear that the calculation is based on the Earth radiating from the “solid Earth”.
    Yet we know that most of the radiation to space occurs from the atmosphere at an average emitting height of 5km.
    Although it makes little difference here, later on this becomes a major point of contention.
    There also appears a typo for the surface of a sphere.

    • Yet we know that most of the radiation to space occurs from the atmosphere at an average emitting height of 5km.

      Citation needed. (Please don’t cite the sentence “This is much colder than the conditions that actually exist at the Earth’s surface (the global mean surface temperature is about 14°C). Instead, the necessary –19°C is found at an altitude about 5 km above the surface.” This does not say what you said.)

  15. Claes,
    I agree. Judy like all GHE believers does not want to allow Postma or any other skeptic to factor into their discredited GHE anything as inconvenient as convection, conduction, adiabatic pressure and, heaven forbid, all that liquid water and the moderating effects of night and day.
    As long as we let the religionists insist on restricting the discussion to radiative effects they can cling onto their discredited notions unincumbered by such inconvenient realities.

  16. Judith Curry 8/18/11, 6:30 am Potential energy balance

    Judith Curry can barely contain her revulsion at the position attributed to Skydragons, followers and authors presumably of the book Slaying the Sky Dragon by John O’Sullivan, Hans Schreuder, Claes Johnson, Alan Siddons, Martin Hertzberg, Joseph Olson, Charles Anderson, and Tim Ball. The book description and the reviews on Amazon say that the book debunks AGW and IPCC along with it, about which little doubt exists. But what Dr. Curry, posters on her blog, and reviewers on Amazon object to is the allegation that thee book denies, per Dr. Curry’s article, the existence of the greenhouse effect. The two positions are not comparable.

    Joel Shore reviewed the book for Amazon, saying

    One example of those who have unfortunately been duped is the reviewer here who states: “The global warming mechanism of carbon dioxide infrared radiation is impossible because heat can not flow from a cooler area (the atmosphere) to a warmer area (the earth surface). If this impossibility was the case it would violate the Clausius statement of the Second Law of Thermodynamics.” I teach thermodynamics and have published many papers in top physics journals in the field of statistical physics, which provides the underpinning of thermodynamics, and I can state categorically that this claim is utterly wrong. In all models of the greenhouse effect, be they toy models that one can work out on the back of an envelope or full-blown climate models that occupy the world’s fastest supercomputers, the flow of heat is from the earth’s (warmer) surface to the (colder) atmosphere as the 2nd Law requires. Bold added.

    Dr. Shore is wrong, not in his physics, but in his reading. The passage does not describe the greenhouse effect. Rather it inarticulately describes the back radiation effect, alleged by AGW supporters here (and there) to have a warming effect, which violates the physics of thermodynamics. Back radiation is not heat (flow of heat is redundant, so let’s not repeat that mistake either). Vaughan Pratt’s block of ice thought experiment was great. Let’s extend it. You sit in an isolated, absorbing room with a huge block of dry ice and between you and the dry ice is an even larger block of water ice. The water ice has the same back radiation everywhere, but the water ice cools you and warms the dry ice. Knowledge of the back radiation alone is insufficient to proclaim warming. QED.

    What might be inferred from Dr. Shore’s critique is that he couldn’t find a passage in Slaying the Sky Dragon that actually denied the existence of the greenhouse effect. I’ll check that myself when I get a free, searchable e-copy.

    Dr. Curry goes too far when she says,

    If you want to disprove the existence of the greenhouse effect, you need some sort of alternative explanation for a whole lot of fundamental physics that explains the infrared emission spectra as measured at the Earth’s surface and high above the emitting portion of the atmosphere (see Pierrehumbert).

    Denying the existence of the greenhouse effect means invalidating the Beer-Lambert Law and the foundations of thermodynamics. If IPCC and the AGW model rely on either, it is not only implicit but well-hidden. Certain gasses in the atmosphere absorb radiant heat, acting in the most elementary model as a blanket, keeping Earth from cooling by radiation to deep space. That is sufficient as a statement of the greenhouse effect. We only need worry about the total absorption from the surface to TOA, and not lapse rates or layering.

    The subject of the article is energy balance. The fact that Earth is always warming or cooling on climate scales is evidence that it is never actually in energy balance. As a hypothesis, nothing is wrong with the assumption. “If Earth were in energy balance, here is what one might expect: … .” Dr. Curry suggests as much when she writes,

    The planetary energy balance equation has its greatest utility in the context of comparative planetology.

    (This passage would be more suggestive of an assumption without the first The .) Still, Dr. Curry then goes through a model for the optical depth of the atmosphere. This model is weak and insufficient. Regardless that atmospheric temperature at the boundary is the parameter of interest in global warming, the atmosphere does not regulate its own temperature. It is but one fluid distributing thermal energy, the other being the ocean, and their relative importance is proportional to their relative heat capacity, estimated to be on the order of 1000 to 1 in favor of the ocean. Earth’s atmosphere is a byproduct of the ocean in relevant composition and temperature on global climate scales. A proper planetary comparison would be other planets with substantial surface water in the liquid phase. Perhaps that will be available in the next millennium.

    The lead article gives the impression that Slaying the Sky Dragon, having denied the existence of the greenhouse effect (did it?), is wrong because AGW is substantially valid. That fits the fallacious model used by Askolnick and Dr. Shore on this blog, and by such sources as skepticalscience.org, that debunking erroneous anti-AGW claims tends to validate AGW. This fallacy gives great authority to the celebratory burning of straw men. This is the same illogic Askolnick and Dr. Shore applied to the politics of AGW: the existence of anti-AGW kooks and cranks on the political right validates AGW because it is a tenet of the left, notwithstanding the existence of kooks and cranks on the pro-AGW side. This weak justification for AGW is symptomatic of the model failing as a matter of science.

    • Trop sec, les M&Ms.
      ==========

      • The oceans will buffer the entire anthropogenic aliquot, biologically probably. Has the bottle been shaken first? I think not.
        ==============

      • Except for this bit of 19th century lore

        “If a dynamic equilibrium is disturbed by changing the conditions, the position of equilibrium moves to partially reverse the change.”

        curtesy of Le Chatelier

  17.  … from my text Thermodynamics of Atmospheres and Oceans…

    In the context of planets and climate the key word here is “Oceans”—i.e., they are why we are here and why everything is different here than anywhere else.

    Don J. Easterbrook: “The Pacific Ocean has a warm temperature mode and a cool temperature mode, and in the past century, has switched back forth between these two modes every 25-30 years (known as the Pacific Decadal Oscillation or PDO). In 1977 the Pacific abruptly shifted from its cool mode (where it had been since about 1945) into its warm mode, and this initiated global warming from 1977 to 1998. The correlation between the PDO and global climate is well established. The announcement by NASA’s Jet Propulsion Laboratory that the Pacific Decadal Oscillation (PDO) had shifted to its cool phase is right on schedule as predicted by past climate and PDO changes (Easterbrook, 2001, 2006, 2007). The PDO typically lasts 25-30 years and assures North America of cool, wetter climates during its cool phases and warmer, drier climates during its warm phases. The establishment of the cool PDO, together with similar cooling of the North Atlantic Oscillation (NAO), virtually assures several decades of global cooling and the end of the past 30-year warm phase.”

    • If Easterbrook were 100% correct about this his conclusion about future cooling would be sound. If however he’s only 33% correct, which is more likely, then his conclusion goes out the window.

      • Chief Hydrologist

        It is a little bit wrong – the effect is Pacific wide rahter than limitied to the north east Pacific.

        Verdon and Franks (2006) used ‘proxy climate records derived from paleoclimate data to investigate the long-term behaviour of the PDO and ENSO. During the past 400 years, climate shifts associated with changes in the PDO are shown to have occurred with a similar frequency to those documented in the 20th Century. Importantly, phase changes in the PDO have a propensity to coincide with changes in the relative frequency of ENSO events, where the positive phase of the PDO is associated with an enhanced frequency of El Niño events, while the negative phase is shown to be more favourable for the development of La Niña events.’

        The cool PDO and intense La Nina – lead to intensified drought in North and South America – but enhanced rainfalls elsewhere. Floods in Austraia – greening of the Sahel – enhanced ndian monsoon – etc.

        The NAO seems to be solar driven – e.g. http://iopscience.iop.org/1748-9326/5/2/024001

        There is a similar southern influence – which is linked to the Pacific Ocean states.

        The planet is cooling and for how long even Kim don’t know.

      • The planet is cooling and for how long even Kim don’t know.

        Right. On a geological time scale, Earth will exhaust its carbon based fuels in the blink of an eye and can then get back to the business of varying its temperature the same seemingly capricious way it’s been doing for hundreds of millions of years. I imagine CO2 will be back to 250 ppmv or so by 2150 at the very latest. That’s a hundred times closer even than the Younger Dryas. I’m confident our descendants will find some way to avoid getting that cold, if only by emitting a little methane (less hazardous to the ocean than CO2 perhaps), assuming some of them are still around.

        By way of even more random projections, in the meantime CO2 will continue to drive up the temperature, with the downswing of the PDO and AMO (Atlantic Multidecadal Oscillation) no longer able to offset it as effectively as during 1940-1960. Those two oscillations were interfering constructively in 1925, but by 2035 they will be interfering destructively and anthropogenic influences will no longer be the main factor in climate change, they will become the only factor. El Nino events and solar cycles will still come and go, but they’re too short-lived to be relevant to secular climate.

        Oh, and there’ll be thunderstorms all this week on the US southeastern seaboard. (Have to do something to maintain my forecasting credibility in the short term. ;) )

  18. Show me a description that talks about energy, mass, specific heat and conduction, convection and radiation in relative portions and I will start to believe.

    To me, G&T, Postma, Johnson, and Nordell make much more sense. Maybe they’re not perfect, but they make sense.

    As far as I can figure, the presence of an atmosphere on any planet has more to do with gravity and density that IR absorption/emission. I dare anyone to propose and conduct an experiment to solve this problem.

  19. Facts are facts and everything else is dogma. “Alternating climatic warming and cooling has occurred about every 27 years since about 1470 AD… In 2008, NASA satellite imagery confirmed that the Pacific Ocean had switched from the warm mode it had been in since 1977 to its cool mode, similar to that of the 1945-1977 global cooling period. The shift strongly suggests that the next several decades will be cooler, not warmer as predicted by the IPCC.” (Easterbrook)

    • steven mosher

      you realize that the satellite imagery is CREATED by applying the physics of RTE. That is, to sense the SST ( the temperature at the surface), the radiance at the sensor has to be ADJUSTED to account for the transmission of IR through the atmosphere. Simply, the sensor data is run through a physics model. That physics model take in the radiance at the sensor and CALCULATES the temperature at the surface. That calculation contains physics that the Slayers deny. So If you believe in that satellite data you have tacitly accepted the VALIDITY of the physics used to calculate it.

      Do you believe in satillite imagery? then the Slayers are wrong. It’s brutally simple. No point in talking past this one.

      • +1 for Mr. Mosher. Willis and Science of Doom were my first two accessible sources for understanding these basic issues. It’s only a little bit tricky, really–you have to get the system boundaries clear but then everything kind of falls out.

      • you will find that they usually leave after I point this out to them

      • So If you believe in that satellite data you have tacitly accepted the VALIDITY of the physics used to calculate it.

        So Steven, what are you advocating here? That we reject the validity of physics? All of it? Or just those parts that don’t fit in with your personal world view?

      • Vaughan,
        My reading of Steven is the opposite. He’s pointing out that many skeptics are illogical, when they accept and use results calculated based on known physics to attack the same physics.

      • Random firing killed allies.

      • Guilty as charged (blush). :(

      • I am not your allied though, hahaha.

      • yes. They are illogical. I have no issue with a skeptic who claims no knowledge. But one they use data ( like satillite data) they are bound by the force of logic to accpt the physics that created that data product.

        This is an unrecognized point in these debates.

      • I’m claiming this. Everytime a skeptic cites a piece of ‘evidence’ or data, they are logically bound to accept the physics that produced that data. Simply, if you cite cloud data from a sensor platform in space, you tacitly accept the radiative physics code that is run on the raw sensor output to create the final data product. In short, you accept RTE every time you quote or cite satillite data products

      • A. C. Osborn

        Now I know why I have so much trouble beleiving the Satellite data, it so often conflicts with the observable world. Like last month the SH was supposed to be hotter than average. That is not what the southern hemisphere reported.

      • Do you believe a weather radar when it tells you how far away a storm is?

        Do you belive the physics that allow us to shoot down a missile?

        did we land on the moon

      • Do you use a GPS?

      • steven,
        That is understood.
        But what does this have to do with climate doom?

  20. Judy, you state:

    “If radiative transfer is the only process occurring in a planetary atmosphere, the surface temperatures of the planets would be determined solely by the net radiation at the top of the atmosphere. Thus, the equatorial regions would be warmest and the poles would be extremely cold. .. There are two fundamental time scales that determine how a planetary atmosphere transfers heat. The radiative timescale, trad, is the time it would take for an atmosphere above some pressure level p0 to reduce its temperature by 1/e of its initial value via radiative cooling if solar radiation were turned off. The dynamic timescale, tdyn, is the time required to move a parcel over a characteristic distance in the atmosphere and, in so doing, transport heat from one location to another.”

    I don’t think anyone could quarrel with this in general, but as a small point, I would add that horizontal gradients at the surface are also determined by convective adjustments to radiative transfer that in most regions operate to mitigate some of the surface temperature rise that would be dictated by radiative transfer alone. It would then be the radiative/convective imbalances between regions that determine the rate of heat transfer. The convective adjustments themselves operate over relatively short timescales (days to weeks). This is much slower than the radiative timescales for an atmosphere considered only in isolation from the surface, and which lead to atmospheric adjustments that are almost immediate. However, further atmospheric changes result from the warming of the surface due to back radiation from the atmosphere (“downwelling longwave radiation” in the more formal lexicon). Feedbacks add a further element of complexity.

    • Convection, a vertical phenomenon, is of course also part of the horizontal air mass transport via its role in the creation of pressure gradients and wind.

  21. Where would a person learn the answers to:
    * How much energy is absorbed by the atmosphere directly from incoming solar radiance, and at what frequencies does this occur?
    * What frequencies are absorbed by GHG from outgoing radiation from the earth’s surface, and what percentage of total outgoing radiated energy does this represent?
    * What frequencies are re-radiated by the GHG molecules that have been excited by outgoing radiation?
    * What is the duration of excitement in an excited GHG molecule, and what determines this duration? How many levels of excitement can a GHG molecule maintain, and is the radiated frequency for each level of excitement the same as for all levels of excitement? Simply put, does it change color as it gains/loses energy? Limit responses to atmospheric energy levels, not what is possible in the presence of nuclear fusion.
    * What is the opacity of the earth’s atmosphere at the frequencies being absorbed?
    * What is the effective impedance of the atmosphere to those outgoing frequencies whose transmission the atmosphere is impeding?
    * A number of ways have been offered to describe the back radiation of energy by excited GHG molecules. In a simple form this works like a spherical mirror. Perhaps those familiar with radar cross section analysis recognize the pattern. Anyway – is there anything about a GHG molecule that would generate a non-spherical pattern of radiation and if so is there anything about a GHG molecule that would cause a non-random alignment of non-spherical radiation patterns?
    * A certain amount of energy is radiated back into space from the surface and this happens at the speed of light. Of all the energy that reaches the earth’s atmosphere or surface, how much is immediately reflected unimpeded back to space and lost forever?

    Several of these obviously are exploring the opacity of the atmosphere and the critical frequencies that are most affected by that opacity, and the percentage of total outgoing energy that are contained in these critical frequencies.

    • Many of your questions about the energy budget can be found here. See figure 1 (on the last page) for a quick overview.

    • Or in simpler terms try http://www.nasa.gov/pdf/62319main_ICS_Energy.pdf for percentages.

      You have a couple of interesting questions. “Anyway – is there anything about a GHG molecule that would generate a non-spherical pattern of radiation and if so is there anything about a GHG molecule that would cause a non-random alignment of non-spherical radiation patterns?” The short answer is no, but longer is yes because the GHG molecule is effected by its environment. At the surface collision dominates energy transfer and high in the atmosphere nearly pure radiative energy transfer dominates. So propagation of the infrared is easier out than in. Water vapor decreases dramatically with altitude. Since water vapor and carbon dioxide have overlapping portions of their emission/absorption spectrum a relatively small portion of the CO2 spectrum centered around 15 microns produces most of the down welling radiation which has a more opaque window down that up due to the wings of the water vapor spectrum around 15 microns. So while CO2 radiation is isotropic, its impact is not predominately isotropic below the tropopause due to interactions.

      That’s my story and I’m sticking to it.

    • dp
      see also Miskolczi’s graphs of quantitative radiation heat transfer.

  22. Judy’s text looks really good from the Look inside feature at Amazon. From the look inside I already realized one of my mistakes that GHE has 2 acronyms – Global Heat Engine and Green House Effect doh!

    At over $100 it’s a bit pricey, maybe it will be put on creative commons someday(hint hint).

  23. It’s over already. And, so is global warming. Look at the phenomena of global warming as it relates to the future of humanity on the planet. You will see that the only human-related factor of any consequence to the survival of Western society is the growing culture of self-defeating nihilism.

    “The ramifications of the global cooling cycle for the next 30 years are far reaching―e.g., failure of crops in critical agricultural areas (it’s already happening this year), increasing energy demands, transportation difficulties, and habitat change. All this during which global population will increase from six billion to about nine billion. The real danger in spending trillions of dollars trying to reduce atmospheric CO2 is that little will be left to deal with the very real problems engendered by global cooling.” (Easterbrook)

  24. John Vetterling

    Judith,

    I’m not saying I believe this, but just to play devil’s advocate, there is an alternative explanation that I don’t believe your textbook addresses.

    If we model the atmostspere as an ideal gas, i.e. PV = nRT, the the force of gravity imposses a pressure equal to G= g x Ma x Me/Re^2 where G is the force of gravity, g is the gravitational constant, Ma is the mas of the atmosphere above the point we are interested in, Me is the mass of the earth, and Re is the radius of the earth.

    Playing with the numbers gives us the adiabatic lapse rate: -alpha =dT/dz, which tells us the temperature of the atmosphere decreases with height, being coulder at elevation and warmer at the surface. Of course we know this is true of the troposhere. Hence the difference between the blackbody temperature (Te) and the actual temperature at the surface (To) could possibly be explained simply by the fact that most of the outgoing radiation is actually occuring at an elevation of about 5km if I’m not mistaken.

    Is there anything I’m missing?

    • John,

      Gravity determines the density profile of the atmosphere, when the effect of varying temperature is also taken into account. It doesn’t determine by itself the temperature profile. If it did, the stratosphere could not have a totally different temperature profile. The temperature profile is a combined effect of the gravity, heating of the surface by radiation, including the effect of greenhouse gases, and emission of energy from the top of troposphere. The effect of all these factors on the temperature profile is controlled by the convection and the cooling of air, when it rises in many areas, and warming of air in other areas, when it descends.

      Very small part of radiation is leaving Earth from an altitude close to 5 km. Almost all is leaving either from significantly higher (close to 10 km) or lower including the surface. 5 km is only some kind of effective average.

      • John Vetterling

        Pekka

        I didn’t say gravity accounts for everything. But if you compress a column of gas it does heat up. As for the stratospere, there obvioulsy is an elevation where other factors exceed the effect of gravity due to the inverse square nature.

        Since temperature itself is not an actual physical quality, merely an average, I don’t think that disturbs the concept of an “effective height” being merely an average. For that matter the blackbody temp is merely an average.

      • John Vetterling

        BTW: nothing in my alternative would neccesarily upset the concept of global warming warming becasue it still doesn’t address the cause of the effective radiation point being at a height of 5km or what might change that.

        At least I don’t think so without further analysis.

      • John,

        You are perfectly right, when you refer to heating of gas, when it’s compressed. That’s exactly the mechanism that determines the adiabatic lapse rate as soon as we have a driving power that keeps convection alive. That driving power is formed by the flow of heat through the troposphere, when the surface is heated by radiation and the top cooled by emission of radiation to space.

    • I think this question gets to the heart of a lot of confusion in these recent threads. That is, would an atmosphere like the Earth’s with no IR-absorbing (i.e., ‘greenhouse’) gases be isothermal or does a temperature stratification arise solely from gravity, the laws of thermodynamics and the ideal gas law?

      My own intuition on this was cloudy (no pun intended) and I asked the question in one of the earlier threads. The consensus (pun intended this time) of the experts seemed to be that such an atmosphere would, in fact, be isothermal or very nearly so. The lapse rate tells us about the convective stability criterion in an atmosphere but it’s all predicated on that atmosphere having energy pumped into it from the bottom. My understanding is that the conduction at the surface layer isn’t efficient enough to do very much and so most of the energy absorbed at the surface would be radiated directly to space through the optically thin atmosphere and would be pretty close to a 255K blackbody. Leaving us pretty darn cold. Another way of looking at this is that the tropopause moves down to the ground and the entire atmosphere would be one big stratosphere.

      Judith, et al., have I got this pretty much right?

      • FiveString,

        You got it right at least, if you ask me.

      • John Vetterling

        FiveString,

        My understanding of it is that an atmosphere with no LW-absorbing gasses would not be iso-thermic. Gravity would still impose an adiabatic lapse rate. However, without LW-absorbing gases the earth would be radiating at the surface instead of some other effective height.

        But we must remember that water vapor is the dominant LW-absorbing gas in the atmospshere. So any planet with abundant H2O will have significant LW-absorbing gases in its atmospshere.

      • Gravity leads to the barymetric formula for the density profile (i.e. approximately exponential decrease with altitude). It doesn’t lead to adiabatic lapse rate in absence of convection and convection needs a continuous driving energy flux.

        (I know this is an issue that some people contest including some, who agree with me on most other points.)

      • John Vetterling

        I’m not sure I agree with you on this point. The lapse rate is a requirement of conservation of energy. The potential energy of molecules at higher elevations is converted to kinetic energy (temperature) at lower elevations. That is the driver of convection, not the other way around.

        Nonetheless, the two phenomena must coexist regarless of which is cause and which is effect. I think we both can agree on that.

      • John,

        You are not alone in making that error in thinking, but it is an error. The way to figure out, what really happens is to do quantitative calculations based on the well known equations of thermodynamics. That proofs directly that your explanation fails. It cannot satisfy the first and second law of thermodynamics and other very fundamental laws of physics.

        You can find the correct calculations on any text book that includes atmospheric physics. The textbooks don’t usually waste space in explaining, why erroneous ideas are wrong. Therefore it may be difficult to find that in print, but it should suffice that the correct answer is easy to find.

      • John Vetterling

        Pekka,

        You and I are speaking at cross purposes. I am confining myself to the adiabatic (no energy added or subtracted) processes driven by gravity. You are dicussing non-adiabatic process. Those are certainly an important element of atmosperic physics, but not for the point I am making.

        My point was to show that we get the same result (a warming of the planet due to absobtion of LWR) without having to invoke back-radiation or some of the other elements of RHT that seem to be so diffcult for some to grasp.

        I think the term “greenhouse effect” is a horrible misnomer, but I have no dispute with the physics.

      • John,

        Such processes do not exist. The word “adiabatic” refers to, what happens to the air parcel (i.e. it’s assumed that no energy flows across it’s boundaries to neighboring air), while it’s rising, but it doesn’t mean that no energy would be needed to drive the convection. That’s needed continuously at the bottom, and energy must be taken off at the top.

        The atmosphere is not in a thermodynamic equilibrium, it’s in a stationary state, where flows continue driven by the radiative processes at the bottom and at the top.

      • (Just found this part of the discussion now, very sorry I missed it earlier.)

        John, Pekka is quite right about the isothermal solution in the simple situation it assumes, with no convection. However it is possible to arrive at this solution without any equations.

        If you model the atmosphere as a single molecule at the altitude of the real atmosphere’s tropopause, about 13 km, it will drop with acceleration g until it hits the ground at sqrt(2*9.8*13000) ~ 500 m/s. This is the process of converting potential energy to kinetic energy.

        The main reason this doesn’t happen with our atmosphere’s 5140 teratonnes of air molecules is that it is so hot that the molecules are in a permanent state of dashing around at the same 500 m/s calculated above and colliding with each other every 140 picoseconds. This creates a pressure that expands the atmosphere and raises the tropopause to its 13 km average altitude.

        Nevertheless the same principle as for the one-molecule case is still in effect, but now masked by the statistics of colliding molecules. Every molecule wants to fall to the ground eventually, but it is competing with all the others wanting to do the same thing.

        A requirement for equilibrium in this scenario is that all molecules have the same RMS velocity, equivalently the same average energy. Imagine to the contrary that the population is divided into hot and cold molecules, a binary division rather than the gradient associated with nonzero lapse rate. This violates adiabaticity, and the cold molecules will then preferentially fall downwards, displacing the hot molecules but gaining kinetic energy in the process until they themselves become hot molecules.

        As the easy calculation above showed, the kinetic energy gained by a molecule starting at rest, i.e. close to absolute zero temperature, and falling from the tropopause to the ground equals that of the molecules at sea level in our normal atmosphere.

        The tropopause is the natural height to which molecules at STP at the surface are flung when traveling vertically and miraculously avoid all collision.

        The stratosphere exists because the Maxwell-Boltzmann distribution includes more energetic high fliers that are naturally carried above the tropopause. The stratosphere’s inverted temperature profile results from anti-greenhouse gases, in particular ozone.

        Much higher still, the solar wind heats the thermosphere to 2300 K or more in the day, cooling to 800 K or so at night.

      • If you model the atmosphere as a single molecule at the altitude of the real atmosphere’s tropopause, about 13 km, it will drop with acceleration g until it hits the ground at sqrt(2*9.8*13000) ~ 500 m/s. This is the process of converting potential energy to kinetic energy.

        I have three reasons for retracting this claimed connection with the tropopause: (i) Arthur Smith thought it was just a coincidence; (ii) I estimate the corresponding altitude on Venus at 23.5 km which is nowhere near the tropopause of Venus; (iii) real tropopauses needn’t have much to do with this idealized situation of a column with no energy flux in or out.

        I think except for this coincidental connection with Earth’s tropopause, the rest of what I wrote should be ok. Just replace every occurrence of “tropopause” with “at altitude 13 km,” and remove any tautologies that result like “13 km is at 13 km.”

        I promise to keep saying something nutty from time to time, though I can’t predict how frequently.

      • Alexander Harvey

        John,

        The lapse rate is a dynamic phenomenon, it is driven. One might argue the the convection does not supply significant sensible heat, although it does, but the existence of rain tells us the latent flux from the surface into the atmosphere is significant.

        Because of the tropopause, energy cannot be escaping the atmosphere by either conduction or convection. It escapes by radiation.

        The driver is the net radiative flux out of the atmosphere, without it the lapse rate would diminish and the atmosphere would tend to stratify and the convection would tend to decrease progressively.

        The radiative flux is determined by the temperature and the GHGs present, it is not caused by the convection. The convection is a response to the radiative energy deficit.

        Alex

      • Would the atmosphere be isothermal if there were no greenhouse gases.?

        Lets look at the planet if this came to pass namely that gaseous H2O and CO2 did not radiate in the IR.
        All other things being left the same.

        Postma has calculated that on the Sun facing side of Earth there will be temperatures around 60C at the equator and overall average of 30C.
        There are underwater volcanoes.

        So it appears that we will have clouds.
        Clouds composed of condensing water are in fact BETTER radiators than the greenhouse gases.
        Radiation from the Earth surface between the clouds will also be possible.

        So the adiabatic lapse rate conditions will be unaltered.
        Therefor there is little reason to think that the climate would be isothermal.
        Perhaps even it might be not all that different from the present.

      • Nullius in Verba

        “That is, would an atmosphere like the Earth’s with no IR-absorbing (i.e., ‘greenhouse’) gases be isothermal or does a temperature stratification arise solely from gravity, the laws of thermodynamics and the ideal gas law?”

        In the absence of convection, the atmosphere would be isothermal. Adiabatic lapse rate stratification cannot arise purely from gravity – work must be done by the external environment to maintain it. If air is moved upwards it will expand and cool, and if it is moved downwards it will be compressed and will warm up, but it requires another force to move it – it won’t move on its own. And if nothing moves it around, then conduction will eventually equalise temperatures.

        Think of it as like a refrigerator. Pumping a fluid round a circuit of compression and expansion will maintain a temperature difference, but you need an external power supply for the pump. In the atmosphere, this comes from insolation being uneven, and the resulting temperature differences provide the entropy to drive the combined heat engine/heat pump. Turn the external power to the pump off, and the refrigerator stops.

        “The lapse rate tells us about the convective stability criterion in an atmosphere but it’s all predicated on that atmosphere having energy pumped into it from the bottom.”

        Convection can also occur because of the equator to pole differential, and the day to night side differential. Heat supplied at the equator causes air to rise, circulate, and settle near the poles where it transfers to the surface and radiates to space. (Hadley cells.) Similarly, heat supplied at the equator causes ocean water to rise, spread out towards the poles, where it cools and sinks, returning at depth to the equator. The driving force behind deep ocean currents is also convective, although no energy is being pumped in at the bottom. The claim that convection requires energy to be pumped in at the bottom and radiated from the top before it will occur is false.

      • In the absence of convection, the atmosphere would be isothermal.

        Convection can help to maintain an adiabat, but the temperature profile of an atmosphere without convection would be determined by a multiplicity of factors and can’t be described without knowing what else is going on. As an example, In the stratosphere, convection is inhibited, and the result is a temperature inversion.

      • Nullius in Verba

        The stratosphere has a temperature inversion because it contains a greenhouse gas – ozone.

      • Mostly because ozone absorbs short wavelength energy. You also need to have the right vertical structure of the solar absorber.

      • But your counterexamples all begin with a heated atmosphere – in the absence of any radiative opacity in the atmosphere the only heat transfer mechanism is conduction at the ground itself. My understanding (perhaps wrong) is that conduction is so inefficient compared to the radiation straight to space that there will be very little heat to transfer via convection.

        I found the Lacis et al. (2010) paper interesting in this regard, although they only model the removal of the noncondensing GHGs and aerosols, which leaves lots of H20-based opacity. Even so, it got pretty cold pretty fast.

      • Nullius in Verba

        Insolation approximately balances radiation at about 255 K, so the surface will contain plenty of heat to be conducted to the air. Without convection, the air would slowly heat until it was the same temperature as the surface it is in contact with. No matter how slow the transfer, such an equalisation is inevitable. But having air at different temperatures at the same altitude in different parts of the world is mechanically unstable. The differing densities lead to pressure differences which drive air flow. Convection results.

        It might take a larger equator-pole temperature differential to drive a given amount of circulation, but the atmosphere couldn’t be static.

        While air isn’t very conductive, it’s conductive enough that they use a vaccuum in thermos flasks rather than air – despite the expense and the risk of breakage.

      • Point taken. I’m enjoying these thought experiments – thanks for the thoughtful responses.

      • Nullius in Verba

        You are very welcome. I enjoy it too – the process of trying to explain these points clarifies my own thoughts as well.

        It was a good question.

      • Adiabatic lapse rate stratification cannot arise purely from gravity – work must be done by the external environment to maintain it.

        No I don’t think this is right. It can. Molecules at the bottom of the atmosphere have the same energy as at the top of it. Except that at the bottom it’s kinetic energy, which means a higher temperature, and at the top its lower kinetic energy and higher potential energy.

        It’s all caused by gravity.

      • Tempterrain,

        You have mistaken. This has been discussed in so many messages during the last couple of days (and before) that I leave to you to find the sources. Naturally you can use also any textbook or other main stream source to find the arguments.

      • Well if I’m mistaken then Wikipedia have it wrong too. They say ” While most often applied to Earth’s atmosphere the concept can be extended to any gravitationally supported ball of gas.”

        I’d agree with that. Certainly convection and moisture content does complicate the picture in the Earth’s atmosphere leading to the possibility of temperature inversions etc; but, this doesn’t change the basic principle that any column on gas subject to a garvitational field will have a adiabatic temperature gradient.

      • Tempterrain,

        Your quote from Wikipedia doesn’t say anything on the mechanism that produces the lapse rate, your links tell only that there is a lapse rate.

        The San Diego presentation is actually terrible. Indicating that there would be some radiative surface at 5000 m is so misleading that it’s not surprising that skeptics continue by creating their own non-physical theories. The text tells that it’s an apparent temperature “surface”, but this reservation doesn’t remove the fact that both the picture and the main text are grossly misleading. The other picture shows more correctly that the surface is one important source of radiation, but as a whole that text is simplified in a way that I find unacceptable.

        The Earth doesn’t radiate like a blackbody of 0F. It radiates about that much in total, but with a different spectrum and from all altitudes from surface to top of atmosphere. Simplifying that as the UCSD link does is counterproductive in my view.

      • Nullius in Verba

        Why do you suppose molecules at the bottom have to have the same energy as those at the top? Why could you not have molecules with more energy piled on top of those with less?

      • The mechanism is that the distribution mixes to maximize entropy. The specific decrease in atmospheric pressure with altitude can be demonstrated by applying the maximum entropy principle. The basic constraints on the system are an average energy on the system as a whole. The essential derivation is very simple, but a more detailed explanation is given in the paper “On Maximum Entropy Profiles”, W.T.M. VERKLEY and T. GERKEMA.

        What I find interesting in all this discussion is that the Stefan-Boltzmann law comes directly out of statistical mechanics, which essentially defines the amount of disorder in a black-body system. Yet the skeptics somehow conflate what this is all about when they claim that the S-B energy balance does not work out and the numbers come out too low. But the capture of various radiation wavelengths by gases will surely effect the statistical mechanics of this distribution with altitude. The system will maximize entropy and if this requires a smearing of temperature in stratified layers, nature will surely accommodate this effect.

        I just don’t get it. The skeptics assume disorder exists, otherwise the S-B law is wrong, but then claim that constraints on the system, ala Maximum Entropy constraints, can not have any effect. I use entropy arguments all the time on various physical phenomena and don’t see why someone could not come out with some simple MaxEnt explanations to explain the energy balance more elegantly.

      • The maximum entropy solution for an atmosphere in thermal equlibrium (as contrasted with a stationary atmosphere with a continuous upwards energy flux through it) is isothermal with a exponentially decreasing density.

        The adiabatic lapse rate is a property of the stationary state, when convection maintains a steady upwards energy flux.

      • The maximum entropy solution for an atmosphere in thermal equlibrium (as contrasted with a stationary atmosphere with a continuous upwards energy flux through it) is isothermal with a exponentially decreasing density.

        That is the basic solution, agreed.

        The next step is to apply another layer of disorder to the system, or maximum entropy on another parameter to derive what is happening. For example, the upwards energy flux could be MaxEnt variable as well. This approach is also referred to as the technique of superstatistics, which is just catching on in the last few years. I am fishing a little bit here because I am still getting up to speed on the dynamics.

      • Part 2:
        The Verkley and Gerkema paper I mentioned does include convection. The general idea is that they use MaxEnt subject to the mass and energy constraints (getting the conventional isothermal result) and then extend it with the constraints of a constant integrated potential temperature. This accounts for a steady-state convection.

        You have to really look at the V&G article and the excellent agreement they get with the data for the standard atmosphere to get an appreciation for this work.

        The only caveat is that they say about the additional constraint

        this choice is of course open for debate. In our view, this particular constraint still lacks a solid physical basis; yet, the above results give reason to expect that the construction of such a basis may be possible because the three constraints … together lead to a temperature profile that corresponds remarkably well to the tropospheric part of the Standard Atmosphere.

        One of the peculiar aspects of maximum entropy is that it ends up working well in spite of people having a hard time explaining why.
        Gian-Carlo Rota a few years ago said this:

        The maximum entropy principle, which may be gleaned from the preceding examples. states that, in the absence of any further information, the best guess of a random variable satisfying given conditions is the random variable of maximum entropy that satisfies those conditions. Among all mathematical recipes. this is to the best of my knowledge the one that has found the most striking applications in engineering practice. The best techniques of image reconstruction known to date rely on the maximum entropy principle. I have myself been witness to police cases where the correct number plate of an automobile was reconstructed by maximum entropy from a photograph which to the naked eye looked like chaos. Even the solution of overdetermined systems of equations is at present best carried out by maximum entropy computations.
        In view of such a variety of lucrative applications, the complete and utter lack of justification of the maximum entropy principle is nothing short of a scandal. On learning that a normally distributed random variable of finite variance has maximum entropy, it is natural to ask for an intuitive proof of the central limit theorem that
        relies on maximum entropy; such a proof has never been given, to the best of my knowledge. although several mathematicians attempted it, among them Linnik and Rényi.

      • I could have mentioned in my earlier answer that I know the paper of Verkley and Gerkema and I have referred to it in my earlier discussions on the temperature profiles of gas columns. That discussion started on this site, but continued on my own site, when it started to be too much out of topic.

        I stated in some earlier comment here that statistics should be basis of any extension of the maximum entropy principle to non-equilibrium systems based on the same basic argument that leads to the equilibrium results. I.e., one must start by determining the proper phase space density, and then analyze the macroscopic physics through the calculation of the most likely set of macroscopic variables. That set of macroscopic variables has the property that the volume of the microscopic phase space that corresponds to it must be large, but also such that they are not excessively unlike. Deriving the equilibrium thermodynamics in that way is surprisingly easy, but the task is much more complicated for non-equilibrium systems.

        Quantum mechanics might actually make the approach easier, because the phase space density is simpler to understand in quantum mechanics. The entropy is also very easy to introduce in the quantum mechanical derivation. (I have used that approach in a course of thermodynamics that I lectured years ago for students, who had had a course of QM.)

      • What I also find interesting about the Verkley and Gerkema paper is in how recent it is, dated 2004. It really demonstrates that one can always find some new simplifying views based on combining different physical principles. Their approach generates a surface temperature of 287, an equivalent isothermal temperature of 261, and excellent agreement to the edge of the troposphere.
        Isn’t this everything you need to understand a steady-state lapse rate?

      • What I also find interesting about the Verkley and Gerkema paper is in how recent it is, dated 2004. It really demonstrates that one can always find some new simplifying views based on combining different physical principles. Their approach generates a surface temperature of 287, an equivalent isothermal temperature of 261, and excellent agreement to the edge of the troposphere.
        Isn’t this everything you need to understand a steady-state lapse rate?

      • The role of the lapse rate and it’s value has been known for much longer. As far as I know the Manabe – Wethrerald paper of 1967 is given most credit on this point. They considered in their paper also the moist atmosphere making it more realistic. Manabe had an earlier paper, where the moisture was not taken into account with Strickler in 1964, but I haven’t looked at that.

        Thus I don’t think that there’s anything new in the quantitative results of the Verkley – Gerkema paper. It’s content is more in the way the derivations have been done and the theory discussed. I cannot say, how much really new the the paper has even from that point of view. I haven’t seen exactly the same content elsewhere, and the paper was accepted for publication, but I’m not a research level specialist on these issues, and wouldn’t know even, if everything could be found in earlier papers, and possibly in papers published long time earlier.

      • My comment was a bit beside the point concerning the fit with the U.S. Standard Atmosphere with the intermediary constraints, but I would expect the standard atmosphere be affected by moisture. My impression is that the paper doesn’t include moisture, making the value of the fit be of little significance, more a coincidence that anything else.

      • Alexander Harvey

        WHT,

        Regarding the dating: I am aware that these approaches are still active.

        I have glanced at the paper, Abstract and Conculsions, trusting them to get the calculations right.

        These results are tantalising, that strikes me as common with MEP approaches.

        They recognise that the choice of constraints is not clear from physical principles:

        “The question then arises what these constraints should be. Here, we have taken, following Ball (1956) and Bohren and Albrecht (1998), constancy of the integrated potential temperature as a single additional constraint 3, but this choice is of course open for debate. In our view, this particular constraint still lacks a solid physical basis; yet, the above results give reason to expect that the construction of such a basis may be possible because the three constraints 1, 29, and 3 together lead to a temperature profile that corresponds remarkably well to the tropospheric part of the Standard Atmosphere.”

        but are hopeful of a fruitful conclusion.

        I hope I am being fair.

        Plausible constraints lead to credible results, this is trying to tell us something but it is not clear as to what that is.

        It is my impression that MEP gets rapidly to intersting and potentially insightful results but then further progress becomes much more difficult.

        I wish them well, it is a minority persuit but these issues are important and the more the merry in my book.

        Alex

      • They recognise that the choice of constraints is not clear from physical principles:

        They are applying equilibrium type methods to non-equilibrium thermodynamics. It’s not surprising that the approach is not well defined.

      • It is my impression that MEP gets rapidly to intersting and potentially insightful results but then further progress becomes much more difficult.

        That is true insofar as doing the variational approach with more constraints, the complexity quickly increases.

        One of the most interesting domains recently has been in quantifying ecological biodiversity. It works so well that some researchers have suggested giving up detailed modeling of individual species (the explanatory) and simply use the statistical which doesn’t purport to explain succession or niches.This is a state space approach and the species fill up the state space consistent with maximizing entropy.

      • WHT – For the record, I should state that the conclusion that an isothermal profile is the maximum entropy state for a radiatively inert atmosphere is not “agreed”, although Pekka has made a strong argument for this conclusion, with references, which I’ve read. I don’t say this with a wish to repeat the arguments here, but simply to point out that when an atmosphere is coupled to a planetary surface, considerations may apply that don’t apply to an atmosphere in isolation. Pekka may be right, but I remain agnostic on this issue.

        I would be happy to discuss this further somewhere, perhaps even by email, but probably not in this blog, because it doesn’t apply to our atmosphere and climate.

      • Fred,
        The extended maximum entropy approach variationally includes both isentropic (adiabatic) and isothermal profiles. That’s the way that nature works in filling up the state space.

      • Use of the variational principle tells a good reason for the fact that the extended maximum entropy method works in appropriate situations. Based on the method, it’s also possible to rationalize, when it’s likely to work and when not. It’s, however, not based on a well understood theoretical understanding in the same way as thermodynamics can be deduced from statistical mechanics.

        When the overall dynamics is considered rather than details within a system, whose overall dynamics is postulated (which can be based on empirical evidence), the lack of a solid theoretical basis means that specifying the right constraints may remain ambiguous. As long as the constraints are ambiguous, the result that relate to the overall behavior cannot be deduced from the principle. That seems to be the problem in the Verkley – Gerkema paper.

      • the lack of a solid theoretical basis means that specifying the right constraints may remain ambiguous.

        As I said when I quoted Rota, MaxEnt is still not well understood but it can be amazingly useful. Somebody may yet figure it out one of these days.

      • MaxEnt is still not well understood but it can be amazingly useful.

        Right, it’s hard to see why entropy should increase with time. It seems to me the converse is more understandable: time increases with entropy.

        Picture the universe as a transition system organized as a metric space whose points are states and whose metric d(x,y) = 1/p(x,y) where p(x,y) is the probability of a transition from x to y, with p(x,y) = p(y,x). Suppose further that most states are high entropy, just as most strings of 10 letters are random words. Random motion will then tend to increase entropy simply because there are more high entropy states than low. (If this seems contrary to p(x,y) = p(y,x), bear in mind that these probabilities are really tiny, and only amount to something when you consider the probability of a transition to one of a quadrillion of nearby states, most of which are high entropy.)

        Time is merely an illusion created by this random motion. The past is absence of change, by which we are able to remember things, and we say that what we remember is that which is in the past. The future is change, and the most likely transitions are those most likely to increase entropy, making the future somewhat of a mystery.

        But not a total mystery, because some things can be seen to be changing more slowly than others, and it is therefore reasonable to expect to find the slowly changing things still in our future. Planetary orbits change very slowly, making them predictable much further into the future than the trajectories of air molecules, which at sea level change every 150 picoseconds or so (do the math: air molecules move at 500 m/s and enjoy a mean free path of 70 nm).

        Just another of my crackpot theories to spice up Judy’s blog. ;) Probably not even mine either, nothing new under the Sun and all that. Back to more serious stuff.

      • Vaughan…emergent spacetime. Have you seen Erik Verlinde’s paper?

        http://arxiv.org/abs/1001.0785

      • Picture the universe as a transition system organized as a metric space whose points are states and whose metric d(x,y) = 1/p(x,y) where p(x,y) is the probability of a transition from x to y, with p(x,y) = p(y,x)

        My simple theory of why many fat-tail probability events occur has to do with many observables being reciprocals of the measures that show either exponential or normal distributions. It has nothing to do with critical phenomena, but just that you get fat-tails mathematical when you take the reciprocal of a PDF.

        Oh, and Verlinde’s paper is the one where he claims gravity is entropy, right?

      • David N, thanks very much for the pointer to Verlinde’s paper. His account goes far further than my “short story,” addressing space, mass, and energy whereas I only considered time. It is also much more quantitatively detailed, reflecting his much stronger physics background.

        It should be obvious I’m very sympatico with this statistics-based “emergent” point of view. I’m therefore looking forward to absorbing Verlinde’s ideas and reconciling them with some of my own state-and-duality-based ideas (under the rubric of Chu spaces) about the statistical origin of Planck’s constant as a quantity whose reciprocal can be taken as an indication of the size of the universe. For Verlinde h is just a convenient but arbitrary quantity, whereas in my account it is not at all arbitrary.

        My tastes run to the least sophisticated mechanisms that do the job, and I’m hoping for statistical accounts of these sorts of phenomena that approach the elegance of the (nonstatistical) derivation of special relativity. Not on the top of my stack right now, but hopefully it will bubble up in due course.

    • John – You addressed your question to Judith, but if she is busy elsewhere, I’ll respond in the meantime. The lapse rate is not an “alternative” to the greenhouse effect (GHE), but is an essential ingredient of the effect. Without the lapse rate, the GHE could not exist. Te is determined by the need of the climate system to emit energy at the same rate it is absorbed. Without the GHE, that would occur at the surface. However, the IR opacity of the atmosphere requires the atmosphere to emit that energy at a higher altitude, where greenhouse gases are sparse enough to permit the energy to escape. Because that altitude is colder than the surface (via the gas laws and gravity), the lapse rate requires the surface to warm up above Te.

      • John Vetterling

        Fred,

        That was my inuitive sense of it. What I like, at this point, is that the adiabatic lapse rate explanation does not seem to require the “backradiation” argument that seems to be such a sticking point.

        As I understand the debate, “backradiation” has become a straw man argument.

      • The argument against back radiation has become a strawman argument, or at least a totally false argument.

      • 7/10ths of the surface is ocean. This is heated by Solar shortwave radiation only.Back radiation welling downwards from the atmosphere has only a negligible effect on the energy contained in the ocean, and the rate of it’s loss to space. This is because the net flow of longwave radiation is cooling the ocean not warming it. There is no way the back radiation can significantly warm the ocean directly by conduction or turbulent convection, so its role is limited to affecting the temperature of the air and thus the differential between ocean surface and air temperature which affects convective processes such as evaporation.

        According to NASA’s empirical figures rather than Trenberth’s fictitious energy balance figures, radiation only accounts for around 25% of the loss of heat from the ocean, and the ocean itself is in control of the temperature and humidity of the atmosphere above it. Water vapour is nearly all of the ‘greenhouse effect’ and the convective transport of energy is the greater anyway. Therefore the degree to which a change in the net radiative long wave flux caused by a change in co2 levels can affect the rate of cooling of the ocean is small.

        This leads me to believe that although the greenhouse effect is real, the variation of co2 is nowhere near as important as most of the mainstream climatologists think it is.

      • TB – The errors in that statement have been pointed out to you many times. Back radiation accounts for more thermal energy in the ocean than solar irradiation for all the reasons previously described, including those described in detail in the original Dragon Slayer thread.

        In addition, you have again confused net IR effects with back radiation. The oceans absorb back radiation at an enormous rate (about 330 W/m^2). Because they emit even more (from the energy received by both solar and back radiated photons), they have a net IR deficit, but this does not mean that they didn’t absorb the energy in the first place.

        If you reread the original discussions, or let other readers review them, the significance of back radiation will become apparent. There’s nothing wrong with doing that, but repeating the same fallacies without calling attention to the previous discussions is not a helpful way for anyone to understand what is happening.

      • Fred, That is kinda apples and oranges. Most of the solar absorbed by the oceans is between 30N and 30S where the percentage of land is less than 30%. So while the total solar absorbed by land and ocean is 51%, the portion absorbed by the oceans should be closer to 80% than 70%. I theorize that is why Earth can recover from glacial periods.

      • Pardon, that should be hypothesize since I am too lazy to pursue it.

      • The total absorbed from solar is closer to 33 percent, including the oceans. The total absorbed from back radiation accounts for the rest, and is also dependent on latitude, with more absorbed in the tropics than the poles (although the difference is less than with solar). Back radiation depends on air temperature, which is why it will vary from one region to another.

      • Of the solar input, Trenberth has 49% and NASA has 51% absorbed by land and ocean. Without that solar input there would be no LW out to create the LW in. Because of the ratio of land mass to ocean in the area of the equator, the ocean absorbs more of the 49% or 51% than the 70.8% ocean area would indicate. If you wish to consider albedo, the most reflective surfaces are at the poles which also have a higher percentage of land surface but still emit at around 255K.

        I think this is kinda important to remember because a 1% increase of cloud cover in the tropics will have a greater global impact than in the temperate latitudes than many can determine from the cartoons.

      • Dallas – We’re talking about different entities. I was referring to the fraction of absorbed energy by the surface that is solar, and you to the fraction of solar that is absorbed at the surface. Even for the oceans, however, it won’t be much higher than 60% after you subtract albedo and solar irradiance absorbed by the atmosphere. See, for example, TFK Table 2b.

      • The estimate of Trenberth, Fasullo and Kiehl for oceans is that the absorbed solar SW is 167.8 W/m^2 and back radiation 343.3 W/m^2. Thus the solar is 33% of the total incoming radiation according to that estimate.

      • Fred, Yes, 33 % from solar with a net down of 1.3 W/m^2 (no net down over land?). Is that the part of the missing heat? This paper also has one of my favorite statements, “There is a TOA imbalance of 6.4 W/m^2 from CERES data… The TOA can probably be most accurately determined from climate models and is estimated to be 0.85 +/- 0.15W/m^2 by Hansen et. al. (2005)…”

        The point I was making is that the majority of absorbed solar energy is in the oceans between 30N and 30S, That heat is transported poleward by atmospheric and ocean currents and a significant amount is stored between 10 and 100 meters below the surface. Outgoing IR and incoming IR are doing a battle as usual, but a significant amount of solar energy is stored longer term at depths and transported to land via precipitation. So it is kinda apples and oranges if you neglect the stored solar energy.

      • Fred, don’t you understand that it is the sun that warms the oceans not ‘back radiation’? The ocean’s mass is almost 2500 times greater than that of the atmosphere and its specific heat is about 4 times greater so your statements is utterly unphysical. Try heating up a cold bath of water with a pocket torch to get the picture. So please drop the back radiation BS.

      • John – Thanks for your comment. Solar radiation and back radiation both contribute to the heat content of the ocean, with back radiation contributing about twice as much as the solar contribution.

      • Nullius in Verba

        Consider a slice through the ocean a centimetre below the surface. The water below the plane radiates upwards into the water above, and the water above the plane radiates downwards into the water below the plane. The water below the slice is more than 100,000 times the mass of the top 1 cm, and the same thermal capacity. But if the top centimetre didn’t radiate downwards, then every layer of the ocean would be radiating upwards with nothing to balance it so fast that the oceans would freeze top to bottom in a matter of days.

        The balancing downward radiation coming from throughout the bulk of the oceans can be considered “back-radiation”, although the distinction is somewhat artificial, (and it doesn’t explain the controlling mechanism of the greenhouse effect).

      • Fred, I think you are missing the point that a lot of the back radiation consists of energy the oceans already emitted, thus cooling. The contribution of ‘new’ longwave is in heating the air such that the differential between air temperature and ocean surface temperature is reduced, thus slowing the cooling of the ocean. But this is a smaller effect than the one you propose.

      • TB – The net IR flux is upward- i. e., in a cooling direction, as you state, which is why ocean temperature drops at night. However, there is substantial back radiation absorbed, day and night, and the back radiation in combination with a smaller quantity of solar irradiation during the day results in rising ocean temperatures averaged over the daytime hours .

      • Fred, The back radiation is absorbed in the first few microns of the ocean surface. The next millimetre or so is at a higher temperature, so the heat raised by back radiation is unable to propagate downwards by conduction.
        In the open ocean, turbulent convection works much deeper down and doesn’t create surface eddies which create any significant vorticity which could ‘mix down’ back radiation heated water molecules.

        Nearly all back radiation is re-radiated from the surface along with the IR derived from solar radiation heated water rising from the deep.

        This means that the only way back radiation slows the cooling of the ocean is by reducing the air/water temperature differential. The fact that air temperature changes lag sst changes globally by 3 months suggests that this is very much a secondary effect compared to the larger convective processes (66W/m^2 net IR compared to ~80 latent heat and ~30 thermal conduction).

        It’s not enough to explain the warming of the ocean 1980-1998. The reduction in tropical low cloud cover empirically measured by ICCP using weather satellite data is more likely responsible for that.

        The Sun warms the ocean, the ocean warms the air, the back radiation keeps the air a bit warmer for a bit longer before the air loses heat to space.

      • Hi tallbloke,

        I don’t believe this idiotic argument is still going. You are absolutely correct of course. I would be pretty sure the oceans are cooling at night.and warming during the day.

        I am even willing to stipulate that if extra CO2 is added to the atmosphere the cooling is less.

        This is one of the very simple ideas that eludes Fred. Deconstructing the passage below – it is all logic and linear language without any leavening of imagining

        Perhaps if we talked about forward radiation? Or call it surface radiation? Being more than back radiative (as stupid as that idea is)? I don’t know – I gave up long ago.

        Cheers

      • TB – The fallacies in your argument have been detailed in the original SkyDragon thread, which makes clear why back radiation contributes more to ocean thermal energy than direct solar radiation. The fallacy of “non-penetration” is a fallacy regardless of the multiple mechanisms by which the absorbed surface IR is translated into an increase in thermal energy at depth – this involves the diurnal convective cycle, turbulent mixing, and inhibited conduction and convection in general. Some of these mechanisms involve physical bulk mixing (which we know is substantial from the shallow mixed layer gradient), and some do not, but they all contribute to the thermal energy within the entire mixed layer and below. This entails actual warming during the daytime (via combined solar/back radiation effects) and reduced net cooling at night, even though absorbed back radiation remains substantial at night. I’m not sure why a mechanism that has been so well discussed earlier, and well documented by the measured fluxes and gradients, needs to be regurgitated here, but anyone interested can see my own contributions to the discussion in the original thread by searching for my name, and can also find links to relevant references that I won’t repeat here..

        In a general sense, this is a non-issue, although in theory, some discussion would be justified as to how close the substantial mixing of solar and IR effects comes to 100 percent.

      • Another way to see the same thing is to consider the heating potency of the solar irradiance absorbed by the ocean at depths, which is in the neighborhood of 160 W/m^2. Those 160 W can only escape via the surface, but imagine, contrary to fact, that some mechanism existed that allowed 160 W to escape from a layer below and independent of a surface temperature of, say, 288 K (15 C) associated with surface IR radiation of about 390 W/m^2. What would the average temperature be at this depth if unaffected by the surface temperature? The SB equation tells us it would average about 230 K (-43 C, or 58 deg C colder than the surface). In fact, it tends to average only a few degrees cooler, depending on depth. From this we can conclude that radiation absorbed at the surface contributes an enormous amount of thermal energy to the deeper layers, and that this is independent of the mechanism by which this occurs. I can’t say whether this actual gradient might be even less if the mixing of surface and deeper energy were total (referring here to energy mixing and not to the physical mixing of water masses), but it seems clear that the two energy sources are well combined to cause the deeper waters to be as warm as they are despite the inability of sunlight alone to warm them to anything near the observed temperatures.

      • Chief Hydrologist

        The quantity of upward surface radiation quite exceeds that ‘back rasdiation’ – although at a quantum level neither terms makes any sense.

        Cloud cover changes caused most of ‘recent warming’ in both the oceans and atmosphere. Indeed there was relative cooling in the IR band.

      • John – I said above that the surface must warm up above Te, but that requires a mechanism. Back radiation to the surface from above is part of that mechanism. The entire mechanism involves the partial redirection of energy downward by virtue of the fact the IR photons impeded from escaping to space by increased GHG concentrations are absorbed by the additional GHG molecules. Some of that energy is subsequently emitted downward, leading to atmospheric warming and also surface warming. The mathematics are described by the radiative transfer equations applied to GHG concentrations and the relevant IR wavelengths. Without the redirected energy, the surface would have no way of knowing that energy was having any trouble escaping at higher altitudes.

      • John Vetterling

        Fred,

        That is the standard explanation, but I don’t think backradiation is required to explain a warmer surface. I think everyone can agree the dT/dt (rate of temperature change) =K x delta T (man I have got to learn LaTex). In other words the rate of temperature change is porportional to the difference in temperature. If the temp of the atmospshere is warmer near the surface then the surface will cool at a slower rate. This is regardless of what causes the atmosphere to be warmer – GhG or adiabatic lapse rate.

      • The surface almost everywhere is warmer than the air just above it. How the surface warms as a result of increased GHGs may be complex, but back radiation is ultimately necessary for that warming to occur or be maintained. Without back radiation, the surface would inevitably suffer from an energy deficit (more out than in) unless its temperature cooled far below what it actually is. This is simply a matter of balancing the energy budget, which can’t be done from only absorbed solar radiation, but requires back radiation to account for observed surface temperatures.

        This was where the Postma argument was fatally flawed independent of other considerations.

      • Fred, you are misrepresenting what Postma actually proves. He showed that the same energy output is achieved by modeling the earth with day and night and abandoning the fudged GHE equations whereby linear and non linear averages are incorrectly mixed.
        He proves that by applying the same equations and adding adiabatic physics and the precise insolation offered by the inertia of night and day you get the same energy outputs with or without a GHE.
        Thus:
        Predicted earth emission without a greenhouse effect: 239 W/m²
        Actual earth emission WITH a (supposed) greenhouse effect: 239 W/m²

      • John – What I say next is intended sincerely and not sarcastically. Postma’s arguments have been shown to be completely untenable in the previous thread, which readers can visit for the details. It is no favor to him for his name and opinions to be publicized further, because what he needs to do now is engage in some introspection, and then decide how to move beyond an effort that has failed and toward other contributions he can make to science. You are impeding that process by bringing him up in a new thread.

      • …And yet the earth’s surface is at a temperature of 15 C and is emitting 390 W/m^2. So, how is it that we only see 239 W/m^2 emitted into space? The only possible answer is that elements in the atmosphere are absorbing (or reflecting) some of the terrestrial radiation. They are what allow the earth’s surface to be at a higher temperature than -18 C (for which the surface itself would be emitting 239 W/m^2).

        Your claim that the emission back out into space is always 239 W/m^2 is correct…but it’s a red herring. (It is simply what is required by energy balance.) That is what makes it sophistry rather than science.

      • The only thing that the adiabatic lapse rate does is set a stability limit on the lapse rate…i.e., if the lapse rate exceeds this, convection sets in and causes the lapse rate to drop back down to the adiabatic lapse rate.

        Saying the adiabatic lapse rate causes the surface to be warmer is akin to saying that the centripetal force causes the earth to go in a circle around the sun. It is not really a causal explanation…It is more correct to say that the gravitational force is what causes the centripetal acceleration and thus the earth’s circular motion.

      • Back radiation is a nonsensical concept – http://upload.wikimedia.org/wikipedia/commons/e/e6/57911main_Earth_Energy_Budget.jpg

        Net flux is all that matters from an energy perspective. If the atmosphere warms the net flux from surface to atmosphere declines and the surface warms.

        You should try to get this right.

      • Robert – If you check the source of that diagram, you will find that it does not dispute the existence of substantial back radiation, but rather diagrams other elements of the energy budget. Back radiation is not merely a theoretical necessity to explain the Earth’s surface temperature, but has been measured by way of confirmation.

      • Chief Hydrologist

        Oh Freddy – if you understood the diagram as approximating the flux of energy through the Earth system – you could move on to more rewarding areas of discussion. Only net energy matters.

        The surface warms from the sun and loses energy in the IR to the atmosphere. It can’t be any other way. Your egregious errors on this have been patiently explained to you over months by better people than me and yet you continue ignore the essential nature of IR radiative flux in the Earth system – in that it moves in random directions within the atmosphere over a mean free photon path of about 50m – but on average out of the system.

        I am sure this is not too difficult a concept for you – but as for now I must move on from an inconclusive dialogue that frankly I gave up on quite some time ago. Others have decided already on this – and you are very much in the minority.

      • What specific error did you have in mind, Robert? Please quote exactly a specific assertion I’ve made – don’t paraphrase – and then, if your patience holds out, explain what you find erroneous.

        Back radiation (more formally “downwelling longwave radiation, or DLR) is a critical phenomenon in dictating the energy content and energy changes in the surface (land and ocean). It has been measured, and is a large quantity. Without the energy absorbed from back radiation, the planet would be much colder. The net IR flow is upward (in a cooling direction – approximately 60 W/m^2), as is necessary for balance, because solar irradiance is downward.

        I believe you’ll find all of these points from previous comments I’ve made, including the very real existence of back radaition. If you think any one of the points is wrong, please tell us what you think is wrong with it, and what the right answer is.

      • Fred Moolten 8/22/11, 9:54 am, Planetary energy …

        FM: Please quote exactly a specific assertion I’ve made – don’t paraphrase – and then, if your patience holds out, explain what you find erroneous. … Please tell us what you think is wrong with it, and what the right answer is.

        That may be magnanimous and open minded, but it also sounds hauntingly like If I’ve left anything out, just ask me specific questions and I’ll be glad to answer them one by one, except for the part about answering.

        This is a warning for Robert, aka Chief Hydrologist, specifically challenged by Fred Moolten in defense of the latter’s belief that back radiation is thermal radiation. Dr. Moolten doesn’t reveal what he is going to do with your response, but I have a test case.

        On the “Towards sane policies etc.” thread, I quoted Dr. Moolten’s writing that ocean acidification [is] a very significant threat [due to] unabated CO2 emissions . I gave a lengthy, detailed response as to why that wasn’t true, how it came to be postulated, and how it is rooted in fundamental assumptions essential for AGW. 6/7/11, 12:36 pm. His response was,

        FM: Jeff – I can see the medication isn’t working. 6/7/11, 12:58 pm.

        At the core of Dr. Moolten’s misconceptions about back radiation and acidification is his belief in AGW (meaning, as always, a measurable human warming effect), and that colors all his writings. IPCC has had to discard, replace, and ignore physics and principles of science to fabricate its AGW story. It is an elaborate fiction built on a tissue of errors. Its dutiful supporters like Dr. Moolten inherit those errors. Those supporters don’t engage in rational debate because their position is a matter of belief, not science.

        But Dr. Moolten is not unique. Here are a couple of recent examples.

        On the Planetary Energy Balance thread, I gave a detailed critique on main points of the George Tech course EAS 8803 which Dr. Curry had recommended for background on radiative transfer. 8/20/11, 2:47 pm. Dr. Curry’s dismissive reply was,

        No time to argue each of your points, but nearly all of them are incorrect. The Beer Lambert law applies only to monochromatic radiation, and hence the subscript lambda. You do not apparently understand the difference between a differential equation and its integrated form. Etc.

        Her only point was wrong, as I detailed in subsequent posts. I gave a direct response on 8/20/11, 9:56 pm. I amplified it on the heels of Dr. Pirilä’s dutiful intervention to repeat the same, false AGW story sans Beer-Lambert, on 8/21/11, 10:46 am.

        A dialog appeared on the Slaying the Greenhouse Dragon Part IV thread in which Pekka Pirilä relied on a automatic balancing in the atmosphere. Beginning 8/15/11, 4:44 pm. It culminated on 8/16/11, 10:42 am, with my extensive quotations from Pekka Pirilä’s writings with counterpoints to argue that equilibrium and balance do not exist in the atmosphere, so require an assumption from physics. He turned the dialog off:

        I have often difficulties in following your reasoning. This time I fail completely. I really don’t find anything relevant from your message. That much I understand that you try to argue against me, but that’s as far as I get. I don’t know, why you accuse me of those faults in argumentation that you list. 8/16/11, 10:48 am.

        A pattern exists in these sample responses from AGWers that no one need characterize. It has the effect when the going gets tough of quashing scientific inquiry. This behavior prevents dialog from converging on knowledge.

      • An interesting post, Jeff.

        To a large extent, I agree with the sentiment that you express. However, I also find your selection of examples to be rather ironic (note to the Chief – from a perspective of a descriptivist, irony need not be intentional – have you gotten the difference between descriptivism and prescriptivism straightened out yet?):

        A pattern exists in these sample responses from AGWers that no one need characterize.

        Now you haven’t ruled out the possibility that the characteristics that you point out are exclusive to the responses of “AGWers,” but I will “infer” that is what you were thinking (heh!). I will further “infer” that you see some causal relationship between being an “AGWer” and a proclivity for writing responses of the character you describe.

        Could you please confirm whether my inference is correct? Because if it is, I would like you to re-evaluate. I can see a slew of responses from non-“AGWers” that equally fit the description you have outlined in each and every post since I have arrived at this website. In fact, I could find many in each and every thread from non-“AGWers” that are arguably far more illustrative of the characteristic you describe.

      • Jeff – please see my incorrectly nested post (1:06) immediately below.

      • Freddy – I think it is quite clear the direction of net energy flux through Earth systems. You are specifically wrong in making a distinction between upwelling and downwelling IR radiation. It is incorrect to make this distinction – and then to ignore the upwelling component is to compound the error.

        The physical interpretation is that IR radiation moves in all directions with a mean free photon path of about 50m – with a net upwelling for reasons that involve the 2nd law of thermodynamics.

        Now this has been said repeatedly to you – I said it yet again as simply as I could. Yet you insist that that I need to be more specific? The surface warms with sunlight and cools predominantly by radiating in the IR band. I have stipulated that the cooling is less with a warmer atmosphere.

        It is simply a more correct physical interpretation of radiative flux. I don’t intend to continue this discourse – as it has been done to death and if you cannot understand yet the distinction – so be it.

      • The physical interpretation is that IR radiation moves in all directions with a mean free photon path of about 50m

        CH, how are you weighting this: by number of photons or by their energy? And up to what altitude (since it’s easily shown to be far greater than 50m if you don’t set an altitude limit)? And are you assuming dry air, 2% water vapor, or something in between?

        (Only asking because it would be a waste of time fact-checking this without the relevant parameters.)

      • Robert (Chief Hydrologist) – I think readers who review these recent exchanges will conclude that you have so far ducked the questions I asked. One of the reasons I asked you to quote me verbatim rather than trying to paraphrase.me is that I was (and am) confident that all the points I’ve made are correct, and that only by paraphrasing them in a way that misrepresented them would make them appear incorrect.
        ,
        I’ll repeat my invitation, however. If you can quote verbatim – without paraphrasing – assertions I’ve made that you think wrong. and indicate exactly what you find incorrect about them, I hope you’ll do so, to permit readers to judge these issues. Otherwise, I am content to let all my points stand on the importance of back radiation to the Earth’s energy budget.

        Back radiation is an important concept, and so it’s worthwhile for us to get it right, just as it’s important to understand IR radiation as a two-way process in which upward IR exceeds downward IR.

      • Vaughan,

        The mean free photon path for IR obviously varies from centimetres to kilometres. I read a number somewhere of the average length being 48.8m

        If you have a better number – let me know.

        Cheer

      • Freddy,

        I don’t know how much more specific I can get. Back radiation is a non-physical concept – only net radiative counts in energy transfer. You need to include the statistics of flux to make any physical sense of energy fluxes through the system.

        It is just the way it is – you have had this discussion with many here and further dissembling with me will not assist.

      • Robert – You say you don’t know how you can be more specific. I’ve explained how several times, but I’ll repeat it here. If you believe you’ve identified an error in my statements, please quote verbatim, without paraphrasing, a statement you disagree with, explain what you believe the error is, and describe what you believe is the correct statement.

        I think readers will notice that you have repeatedly declined to do this. I conclude, at least tentatively, that you realize you are wrong, but refuse to acknowledge it. In fairness, though, I want to keep my invitation open for you to quote me verbatim and then discuss what you perceive to be errors, so that readers can judge the issue.

        Back radiation, which is substantial and has been measured, is an important variable in climate dynamics because of its contribution to the thermal energy of land and oceans. Upward IR radiation is of course equally important, as is the difference between them – the net IR radiation, which is upward at about 60 W/m^2, and thereby helps to balance incoming solar radiation. Merely knowing the latter 60 W figure, however, is inadequate for a full understanding, because it is equally important to realize that it’s the difference between two large fluxes with different sources and destinations, and to know how each flux originates and the temperature with which it’s associated.

      • Chief Hydrologist

        Freddy,

        You keep banging on about back radiation – for months now with various people and I can’t see that it progresses at all. I only persist because you mislead others with this persistent error.

        Net IR is outward – although IR is emitted in all directions over a realtively short distsance – and cannot possibly be heating anything on the surface as net IR is negative at the surface. All of the heating at the surface is in the SW. Adding CO2 to the atmosphere slows down cooling and energy and heat builds up in the total system.

        If you cannot see for whatever reason that this is better physical description of the system – after some months at this – I cannot help. Your specific error is that back radiation is an irrelevant and unphysical understanding of radiative flux. You simply continue to insist that it is an important consideration. Allow me to disagree.

        Moreover – you have ‘tentatively’ raised the question of bad faith – I don’t appreciate that at all.

      • To once more invoke a phrase, Robert, that I’ve felt was appropriate on other occasions, you don’t lose gracefully.

      • CH, you have to admit upwelling and downwelling IR have completely different and distinct spectra, and are therefore the result of different emission sources. Specifically at the surface upwelling is dominated by ground emission, downwelling is dominated by sky emission (with no ground signal). Net is a useful concept, but certainly not the basic one.

      • Chief Hydrologist

        Freddy – I haven’t lost anything at all. If you will insist on nonsense I will call it such – despite your pompous insistence on getting the last word in any discussion. You are wrong and can never admit to anything of the sort.

      • Chief Hydrologist

        Jim the nestijng is getting too confusing here – http://judithcurry.com/2011/08/22/can-we-make-good-decisions-under-ignorance/#comment-103501

      • The mean free photon path for IR obviously varies from centimetres to kilometres. I read a number somewhere of the average length being 48.8m. If you have a better number – let me know.

        Absent any obvious choice for the wide range of very influential parameters, I’d say there wasn’t a single good number. All there is is a complicated function of all those parameters, and a mfp given to three decimal places is wishful thinking.

        One interesting datapoint might be the average when weighted by energy (as opposed to photon count) for horizontally directed radiation at sea level assuming STP and 400 ppmv CO2 in completely dry air. But even in the horizontal case one must truncate, otherwise the photons that escape to space will drive the average to infinity.

        What one can meaningfully ask is the mean free path of a photon at sea level within 0.07 /cm of the strongest CO2 line. I calculate this at about 4 cm. At ground level on Venus (there is no sea) it would be way less than that.

      • I have to say I’m on Jim D’s side on this Moolten-CH argument:

        CH, you have to admit upwelling and downwelling IR have completely different and distinct spectra,

        While my Dragonslayer-iv post may have given the impression I was on CH’s side, my goal there was really just to point out that the DLR/back-radiation view of things could be overstated a la Alistair Fraser. Judging by the strong reactions it was clear that this point needed making.

        CH’s net-radiation view is valuable for insight into the global energy budget. Absent clouds, for sites near sea level, net upwards flux oscillates between 100 W/m2 and 60 W/m2 from day to night. With clouds it drops pretty much to zero, but rarely goes significantly negative. These are important insights when reasoning about the global energy budget.

        Fred’s focus on DLR and ULR separately is important in understanding the behavior of ULR – DLR (net upward flux). In a dry clear sky, the 40 W/m2 swing of net flux between day and night is due largely to the diurnal temperature variation of the surface, with the DLR remaining relatively constant. Clouds change this by increasing DLR dramatically while reducing the diurnal swing of ground temperature, with the result that ULR and DLR stay close together, thereby minimizing net upwards flux

        What I find really interesting is how ULR hardly ever drops below DLR. It is as though the net upward flux was fixated on preserving the second law of thermodynamics. Buried in the pandemonium of trash talk in the G&T paper is the important point that heat flows from the surface to the atmosphere, not vice versa. Judging by the net flux data they at least got that right.

        The bottom line for me is that I see no future in the continued debate between Fred and CH. Both have sound logic on their side, and as long as each refuses to acknowledge the soundness of the other’s logic that debate cannot end.

        I wish I could say the same for Arfur Bryant, who needs to tone down his 23rd century logic if he expects those of us still working with old-fashioned 20th century logic to follow his reasoning, which I for one find nothing short of magical.

      • One thing I forgot to say in my account of the separate variation of ULR and DLR is that, while it shows the value of considering them separately as well as together (as their difference), it does not in any way support the statement that the atmosphere heats the ocean twice as strongly as the Sun. I took strong objection to Alistair Fraser’s statement to that effect in my recent post on that subject (which Judy renamed to “Dragonslayer-iv”), and Fred is no better on that score. (Not just one but two people wrong on the Internet, what is the world coming to?)

        The problem, Fred, is that you are completely ignoring the fact that the ocean radiates heat upwards to the atmosphere. Hence to say that the atmosphere adds twice the heat the Sun does to the ocean is as illogical as saying that so many people walk into Macy’s every day that it will eventually explode under the pressure. Would you base your net worth on four decades of accumulated salary while ignoring expenses?

        One has to look at these things from an equilibrium perspective, and ask whether the equilibrium is being disturbed, and at what rate. Absolute numbers considered in only one direction are meaningless.

      • Vaughan – The basis for the statement that the DLR contributes approximately twice as much as solar to ocean temperature can be found in the TFK energy budget diagram plus the comments I’ve made to Tallbloke elsewhere in the thread. The fact that the ocean radiates IR upward and that the net IR flux is upward in not really relevant.

        The basis for the conclusion resides in the magnitude of the solar flux vs DLR combined with the evidence that the upward IR flux is a response to both the solar and DLR component, and cannot be explained as a response to the DLR component alone. There may be a slight uncertainty as to whether the solar and DLR thermal energy are 100% mixed in contributing to upward IR, or less than 100%, but the ocean thermal gradients indicate that the energy mixing is very substantial. As I mentioned to Tallbloke, this will occur regardless of whether there is bulk mixing of water masses between the surface and lower layers, although there is good evidence for a bulk mixing role – particularly involving the diurnal convective cycle.

      • Fred Moolten is completely wrong about IR energy being mixed into the ocean. See my post here for the correct explanation of th eGHE:
        http://judithcurry.com/2011/08/19/planetary-energy-balance/#comment-104533

      • Joshua 8/22/11, 12:53 pm, Planetary energy …

        J: Now you haven’t ruled out the possibility that the characteristics that you point out are exclusive to the responses of “AGWers,” but I will “infer” that is what you were thinking (heh!).

        One of the reasons I didn’t actually point out those characteristics was to avoid any implication that they were exclusive to responses from AGWers. Maybe I was wrong, though, when I said “no one need characterize”. Some of the characteristics I had in mind were the overlapping qualities of arrogance, dismissiveness, evasion, and personal attacks (ad hominems).

        J: I will further “infer” that you see some causal relationship between being an “AGWer” and a proclivity for writing responses of the character you describe.

        My intention was to be explicit about that causal relationship. Applying just a little science, AGW becomes indefensible, yet the AGWers are committed to the movement, and ultimately to defending its errors. If you get too specific with them, they bail.

        Those characteristics are certainly found in posts from others. The AGWer pattern is just a spectrum, not a lock. Other posts have a background of white noise in which no part of the spectrum is empty. Much of the white noise comes from the posts that qualify as tweets. Others yet also have coherent noise, as when dialog pursues the mechanisms by which back radiation warms the surface; or how close to an adiabat the atmosphere is, must be, or tends to be; or how to get radiative transfer just right; or what name to call the greenhouse effect.

        The spectrum of the others often mimics that of the AGWers, probably intentionally. But the post fails on arrogance when a little check shows the poster had no standing to be arrogant. Dr. Curry’s policy, much to her credit and doubly so, being unique among AGWers, is to moderate almost negligibly. Let a million flowers bloom. She gives hope if not convergence. The result is much like the Internet. Caveat emptor.

      • In a dry clear sky, the 40 W/m2 swing of net flux between day and night is due largely to the diurnal temperature variation of the surface, with the DLR remaining relatively constant.

        I’d better quantify “relatively constant” in the case of a clear sky or I’ll be in trouble. During the day DLR increases along with ULR but only by half as much, on the order of 30 vs. 60 W/m2 variation respectively, whence net upwards flux variation is approximately equal to that of DLR, namely 30-40 W/m2. Throughout the night DLR and ULR decline at roughly the same rate, so that net upward flux remains constant.

        The ambient flux at the surface on a clear day, defined say as the average of DLR and ULR at the surface is way bigger than the net upwards flux, 350-370 W/m2 vs 70 W/m2. Clouds have negligible impact on ambient surface flux but bring the net upwards flux down to near zero. One might expect lapse rate to knock 20 °C off a cloud bottom at 2 km, but this is offset to some extent by the warming effect of condensation.

      • And btw, Jeff,

        In case you haven’t yet “inferred” my “implications” behind my last post – let me make my implications more concrete (so there will be no mistake due to incorrect inferences).

        The selectivity of your criteria for choosing your examples helps to explain the basic reasoning for why I couldn’t see validity in the conspiracy between widely diverse entities dispersed widely over the globe, across disciplines, that spanned widely divergent stakeholders, and among people for whom there is no logistically realistic infrastructural connections – as you outlined in our previous exchange of ideas.

      • Very interesting claim. I have made a copy of this link.

        Could you possibly be that naive? Or . . . . .

      • Here is the reason Joshua,
        Psa 2:1 Why are the nations so angry? Why do they waste their time with futile plans?

        Psa 2:2 The kings of the earth prepare for battle; the rulers plot together against the LORD and against his anointed one.

        Psa 2:3 “Let us break their chains,” they cry, “and free ourselves from slavery to God.”

        Psa 2:4 But the one who rules in heaven laughs. The Lord scoffs at them.

      • Joshua 8/22/11, 1:06 pm, Planetary energy …

        J: The selectivity of your criteria for choosing your examples helps to explain the basic reasoning for why I couldn’t see validity in the conspiracy between widely diverse entities dispersed widely over the globe, across disciplines, that spanned widely divergent stakeholders, and among people for whom there is no logistically realistic infrastructural connections – as you outlined in our previous exchange of ideas.

        Case 1: Fred Moolten solicited specifics from Chief Hydrologist, as if Dr. Moolten would answer specifics. But evidence shows (a) Dr. Moolten doesn’t answer specifics, and (b) that he is not unique among AGWers posting on Climate Etc. in not answering specifics. My examples were not selected as biased evidence for some conclusion or other. Each example, one from Dr. Curry and one from Dr. Pirilä, was sufficient proof that Dr. Moolten was not unique. 8/22/11, 12:36 pm, Planetary energy … thread.

        Case 2: Askolnick cited Muir Russell’s Report as evidence that Mann et al. had been cleared of conspiracy, and repeating what Dr. Shore had urged, that skepticism about AGW was impliedly a conspiracy of the right wing. The points I tried to make were (a) Mann et al. provided evidence of a conspiracy to defend his Hockey Stick reconstruction and to bias peer-review, a conspiracy that was successful. Moreover, the Hockey Stick was but on of a collection of small conspiracies embodied in IPCC reports (details on request). (b) The right wing is a collection of small, diverse movements sharing but one tenet – small central government, which is per force anathema to the AGW movement, now an open tenet of the left to expand the role of US Federal Government while restricting its autonomy to resist through international agreements. 8/19/11, 10:08 pm, 12:27 pm, Slaying … thread.

        I don’t see how (1) illuminates (2) in any way, and specifically not in biasing data through selectivity.

      • Chief Hydrologist
      • The fact that the ocean radiates IR upward and that the net IR flux is upward in not really relevant.

        Mathematically speaking, you’re quite right. Physically speaking, the mathematical fact you allude to has no sensible physical meaning.

        If it did, a similar argument would allow you to prove that diffusion transports a hundred times more air than a 5 m/s breeze passing through a hula hoop from left to right. This is because air molecules flow through the hoop at 500 m/s. If you only look at the air molecules that flow in the same direction as the breeze, it is clear that 100 times as much air is being transported by diffusion than by the breeze.

        Now from your perspective this factor of 100 is in fact the case, the way you’ve defined things. You choose to neglect the molecules that flow the other direction, and you end up with an amazing conclusion.

        Replace air molecules by photons and you have precisely your argument that the atmosphere heats the ocean twice as strongly the Sun. You arrive at this remarkable fact by refusing to include the photons rising up from the ocean in your calculation.

        Your math is just fine, your physics lacks a certain intuition about what’s really going on there.

      • Fred,

        Yes I think you’re right about the lapse rate not being an ‘alternative’ to the GHE but at the same time it doesn’t mean there is no back-radiation either.

        Isn’t it just a case of looking at a problem in two slightly different ways and coming up with the same answer in each case?

      • Mr. Fred Moolten, How would you feel about GHA or “the greenhouse affect”?

    • Hence the difference between the blackbody temperature (Te) and the actual temperature at the surface (To) could possibly be explained simply by the fact that most of the outgoing radiation is actually occuring at an elevation of about 5km if I’m not mistaken.

      Sorry, John, I’m not following your arithmetic here. A lapse rate of 10 °C/km and a nominal surface temperature of 288 K implies a temperature of 288 – 5*10 = 238 K at 5 km, about 17 degrees below Te. How do you reconcile your arithmetic with mine?

  25. The mystery to me in the Postma discussion why is there the preference and vitriolic defence for the ultra-simple zero-D model the earth with uniform insolation and emission. Until this week, I had no idea such a simple model was in use beyond he first day of academic study and the first chapter of a text book. What is the purpose of the zero-D model’s except to show that a GHE must exist?

    I thank Postma and Judith for posting it for discussion for bring this to the fore. We are dealing with a very non-linear system. Postma’s model is as simple as we should consider: to me that is clear as night and day. Postma’s conclusion that GHE is unnecessary is not unsupported,

    I discussion of a relatively simple model of a rotating “buckyball” with a day and a night might be a more reasonable simple model of energy balance. You I think you have only 5 unique surface orientations, rotated in 12, 18, or 24 discreet time periods on an axis through opposite vertexes , with water, land, or ice/snow as the surface. It is certainly more complicated than the zero-d model, but this is easily done in one small Excel worksheet.

    I would be surprised the buckyball model has not been done before. Regardless, discussing energy balance in this non-linear model would be more productive than the arguments that solar insolation should be divided by 4.

    • Stephen,

      Who has defended that model vitriocally?

      Everybody has told that it’s not a valid model of the atmosphere, not even close to a valid model.

      It’s only a toy educational model to explain with minimal prerequisites, what the GHE is.

      • The Postma thread established for me that the zero-D model is too much of a toy to explain “what the GHE is.” In the nearly 1200 comments, a large proportion of them maintain the “toy” is good enough. Why such an effort to defend the toy?

      • Whether it’s enough or not, is really a matter of taste. That model is not a model of the atmosphere, but it describes what happens in a system that is possible to build. (A structure built from the back pane and a window placed in empty space outside the atmosphere and illuminated by solar radiation. It’s a correct model for such a structure.)

      • There you go with another defense of the toy. My skepticism grows with each attempt.

      • The Masque of the CAGW Death.
        ==========

      • My skepticism grows with each attempt.

        Stephen – Maybe this sounds arrogant, and maybe it is, but those of us who understand climate physics principles are already aware that the greenhouse effect and back radiation are inescapable realities, and we also understand that a small number of individuals will refuse to accept those realities no matter what we say. You may be among those individuals. In that case, we have no hope of convincing you, but neither do we care very much. Your level of skepticism is your problem, not ours.

        There are others who call themselves skeptical who raise legitimate questions about our current understanding of climate change. We are generally happy to engage with them so as to explain why we arrive at the conclusions we hold. They are a different bunch, and deserve a different level of attention..

      • Fred, you are right… it does sound arrogant.

      • But also true.

      • Indeed. ¨This problem which takes most people 10 years to learn doesn´t make sense to me, and someone can´t explain it good enough on a blog; I am skeptical!!!¨ sums up most people´s ¨skepticism.¨

      • Fred – I’m going to have to break this up as the spam-catcher got hungry. If you’re interested – this will take a few posts in segments:

        You may be among those individuals. In that case, we have no hope of convincing you, but neither do we care very much.

        Here’s what I find interesting about that. It would seem to me that if you really didn’t care, you wouldn’t spend so much time interacting with folks as you did on the Postma thread and the Dragon Slayer threads. I think that there’s more there than meets the eye (although I don’t have a handle on what that might be).

        I am fascinated that folks like you and maxwell and Pekka and Judith will say that the ideas of someone like Postma are complete non-starters – yet you spend a lot of time engaging with them in a very complex dialogue.

        What’s also interesting is that in not being able to evaluate the veracity of the different technical arguments, all I can do is try to get the “gestalt” of the various opinions. Usually, when doing that in technical areas I’m not equipped to understand, I can usually get a feel for when someone delivers a knockout punch. (As an aside, it is almost never when someone says to someone else that their ideas are idiotic or some such characterization.) There’s usually a “tell” of some sort in someone’s response if their interlocutor has dealt a death blow. I think that the closest I could find was when Postma wouldn’t answer Judith’s challenge to test his hypothesis by applying it to the climatic dynamics of other planets.

      • Part II:

        So, not that my opinion really matters, but for what it’s worth, I find that dismissing someone else’s knowledge base – when from all I can tell they seem to display a grasp of a variety of relevant concepts (of course, admitting that I am susceptible to being fooled by someone who talks a good game but presents only shells of arguments – as was asserted about Postma) – backfires for me as an observer. Given that there are more than likely only two kinds of “participants”: expert combatants on each side who demean the expertise of the experts on the other side, and observers who don’t have the requisite expertise, based on my reaction, criticizing someone else’s expertise as an explicit discourse does not have any benefit:

        1. The “criticizer” already feels that the “criticized” lacks expertise. Saying that they do will change nothing.

        2. The “criticized” will simply reject the criticism of the “criticizer.”

        3. If they’re like me, seeing one expert criticize someone else who (at least to my ability to understand) seems to possess a fair level of expertise, only reduces the credibility of the “criticizer.”

        Sorry – for that rambling mess. When I started the comment, I didn’t expect it to end up where it did.

      • Part II:

        So in the end, all I’m left with is trying, somehow, to assess the credibility of the players in the game. Also, I can hedge my bets by looking at which side seems to have accumulated the preponderance of expertise.

        Given that there are more than likely only two kinds of “participants”: expert combatants on each side who demean the expertise of the experts on the other side, and observers who don’t have the requisite expertise, based on my reaction, criticizing someone else’s expertise as an explicit discourse does not have any benefit:

        1. The “criticizer” already feels that the “criticized” lacks expertise. Saying that they do will change nothing.

        2. The “criticized” will simply reject the criticism of the “criticizer.”

        3. If they’re like me, seeing one expert criticize someone else who (at least to my ability to understand) seems to possess a fair level of expertise, only reduces the credibility of the “criticizer.”

        Sorry – for that rambling mess. When I started the comment, I didn’t expect it to end up where it did.

      • I do certainly wonder every now and then, whether such discussions are of any value, or why I continue those. Sometimes they help in finding most direct ways of looking at the issues being discussed. Sometimes there’s the thought that some readers might learn something from that. What has become clear is that the most stubborn people never admit anything. They usually repeat their arguments as before independently on the number of times they have been proven totally wrong and the number of people, who have done that.

      • Joshua,

        A lot of us who argue on the blogs may do so to try to educate people, while some just do it because it may be a fun thing to do in spare time, or are argumentative, or simply hate when they see wrong things on the internet. Ever see this image?
        http://blog.pauked.com/wp-content/uploads/2008/02/duty-calls.jpg

        No one is going to pretend that people who know the science better are necessarily much better off in choosing their battles. I’m terrible at it. The idea sometimes get lost too that there are thousands of scientists you never heard of, who don’t comment on blogs, who don’t read the blogs, etc who are out there publishing. This is why lists with a couple dozen names on it of “people skeptical of AGW” sometimes look impressive.

        That said, the arguments we get into on blogs generally do not at all reflect what scientists actually debate about. Sometimes perspective is lost on where the science is very solid, and where research needs to be done, since the topics discussed in the literature are generally a complete 180 from those discussed here and elsewhere (Isaac Held’s blog may be a good example of getting a feel for where the “up to date” questions lie).

      • Chris – yes I have seen that cartoon. Unfortunately, so has my girlfriend.

        I think that your point about mistaking the blogosphere for some scaled-down version of reality is on point. In reality, the blogosphere is a very distorted little world (thank god). It’s a mistake to think that we can extrapolate, proportionally, from the blogosphere to the larger reality.

        Held’s blog is way too technical for me.

      • Hi Joshua – I think the answer to your question, for me at least, is that when I address Stephen (or some other individual), I have a much larger audience in mind, including other participants in these threads as well as bystanders. Even if I can’t convince a particular individual of a certain conclusion, I can make clear that I find enough evidence to have been cited for open-minded readers to come to the same conclusion themselves – either because they visit the evidence or respect my ability to interpret it.

        Regarding the “numbers game” as to how many skeptics believe the GHE to be non-existent vs those with some other view, I’ll confine the topic to people who are familiar with the description of the GHE, excluding lay persons who are merely vaguely familiar with the concept of AGW. I don’t know the answer, but counting blogosphere responders may not be informative. Anyone can compile a list of those who volunteer their opinions, but what does that mean?

        I’m reminded of the Dover, PA trial a few years ago involving the teaching of creationism/intelligent design. The ID proponents circulated a list of a few hundred scientists purported to endorse ID. In turn, the defenders of evolutionary theory produced a list of scientists who supported evolution. At the time, it was a few hundred but has since grown to more than a thousand.

        One interesting feature of that list, though, was that it didn’t refer to scientists in general. The list consisted only of scientists whose first name was Steve (in honor of Stephen Jay Gould).

        Google “Project Steve” for more on this.

      • But Fred – what I was saying was that as a bystander – denigrating the expertise of someone who, for all I can tell, seems to possess expertise, is ineffective. Of course, I may not be a typical bystander – but unless I’m already inclined for one reason or another to trust your expertise over someone else’s, when you denigrate someone else’s expertise, it is ineffective. And if I’m already inclined to trust your expertise over someone else’s, then I’m not an objective bystander, and all I’m doing is looking to confirm my bias – something I’ll do whether you criticize your interlocutor’s level of expertise or not. I see no equation where you come out ahead via that level of discourse.

        As opposed to reading you denigrating the expertise of your interlocutor, I find it more effective when you do as you just describe – lay out your argument, provide resources for those interested in researching the questions in more depth, and explain that based on your level of expertise, your combatant’s argument isn’t consistent with the facts, logical, etc. I’m thinking of the ocean acidification thread in particular, when you stated your reasoning, offered your opinions, and let Willis et. al. hand-wave about hand-waving.

      • Joshua – You make good points. I think it’s worthwhile for each of us to try to establish our credibility through reasoned arguments. When we read the same fallacies offered over and over again, we face a dilemma. At what point do we stop repeating the evidence and either dismiss the claim or simply ignore it. Since we don’t really have much feedback on the effects of our choices, it’s hard to know, but your comments are helpful.

        My sense is that we (or at least I) will sometimes err on the side of dismissal when we should merely have ignored the comment or linked back to earlier discussions of the evidence.

      • I think that in the case of “debates” such as this to prepare a really strong canned response which addresses the standard contra arguments and just repost it each time a non original objection is raised.

        For original “Galileo” thinking you might have to think a bit…

      • Brandon Shollenberger

        Rattus Norvegicus, your answer here is the same answer I would offer. Indeed, that’s why I encourage people to examine Postma’s paper in detail and come up with a clearly defined and explained refutation. If you can find a contradiction within Postma’s paper which invalidates his conclusion, you have your canned response.

        It obviously won’t convince everyone, but it will convince most people.

      • Brandon,

        I think most people already have been convinced that Postma has nothing to offer, and so far I’ve seen no objection to my SkS counter-arguments. You claimed it was not good enough, but I showed while his whole geometrical reasoning was both a strawman and wrong. I don’t know if there is a step you are missing from his paper, to my response, to you, but there’s very little left to say about it. It wasn’t even wrong in the sense that serious scientists would sit down and think about it, as is the case for some of the “better” skeptics like Lindzen, where it actually takes a few years to re-work the results, check the data, etc; in this case, the issues are just on the level of an undergrad getting his homework wrong and the professor needing to correct him. In this case, the “professors” have been a good amount of people, and the student refuses to listen. At that point, there is no need to bother with it. A rebuttal was unnecessary in the first place.

        You might not like the answers anyone gives you, or even understand them, but there’s a lot of interesting things to be skeptical about that all involve accepting that a greenhouse effect exists.

      • Brandon Shollenberger

        Chris Colose, I now see I misread one of your comments, and thus my response was off-base. I basically misread your comment as saying you spent half of your piece showing how the correct calculations are done. Naturally, my response makes no sense because you actually said you spent half your piece providing clarity on the fact Postma’s entire geometric reasoning was wrong. I apologize for misunderstanding you.

        However, you are still wrong. In your section about geometrical reasoning, you barely even discussed his position. Your second paragraph is the only one which actually says his reasoning is wrong (and it was only two sentences). Your next two paragraphs were far more lengthy, but they contribute little. Basically all they say is Postma’s argument is an argument against a simple model when more detailed models exist.

        The problem with your piece is you never tie your criticisms to any conclusion. You say he’s wrong about certain things, and then you move on. You don’t discuss what the impact of your claim is. You don’t show any mistakes you mention actually invalidate his conclusions. If you want to refute someone, you have to show the impact of the error or errors you found actually invalidate his or her claim. You didn’t do that.

      • Brandon,

        I find it interesting that you fail to account for the fact that in his 40+ page manifesto that Postma ran off the railis in his analyis around page 10 where he attempted to justify doubling the average TOA insolation. Once I got to this point I just couldn’t continue, pencil in hand to check his math or not. This error is so severe that continuing, IMHO, was not worth my time. Please justify for me why this assertion (and assertion is all it is when dealing with a model which deals with averages) is correct.

      • Brandon Shollenberger

        Rattus Norvegicus, I’m not clear on just what assertion you want me to justify. Could you be a little more specific? Also, could you make sure whatever assertion this is is one I actually support?

      • Hey Josh, read Chris Colose’s and the rodent’s responses above. See what I mean?
        The non-linearity in the effect of heating water means Postma has a point, but his thesis is rejected in it’s entirety because They don’t understand how non-linearity makes averaging a blunt instrument.

      • Tallbloke –

        I concede that you may have a point, but may is the operative word there. Not having a technical background, I can’t tell if a fatal flaw has been identified as “They” claim.

        I’m left to weighing your claims of better understanding of the physics against “Their” claims of better understanding of the physics. And that’s where your aforementioned facile conclusion (I could list others, if you’d like – think Lord Monckton and Nazism) doesn’t improve your standing.

        What I continue to find interesting is that usually in these kinds of situations I can “grock” a debate well-enough to get a sense for who’s blowing smoke. As I said above, usually there’s a “tell,” or some basic structural logical flaw that is visible. The online equivalent to a change in breathing pattern or a falsely confident smile. I’m having a hard time doing that with the Postma thread and a few others – except in a few obvious spin-off convos like when Claes obviously mischaracterizes something Judith says or accuses everyone and their uncle of only agreeing with Judith because they must be her students.

        I think in part it’s because this material is so complex and so esoteric and involves so many unknown variables that it is easily possible for smart and knowledgeable people to full believe in completely contradictory viewpoints. But I think that more likely the biggest influence is the politicized nature of the debate – or to go into the political aspect more deeply in a psychological sense, the whole component of “motivated reasoning,” which can include the personal and/or psychological drivers that lay beneath the political ones. Thus, reasonable and knowledgeable people can successfully pull off what looks like a viable argument because they really do believe it at a conscious level – with their unconscious is working in the background to adjust their reasoning so as to overcome logical or factual disconnects in their arguments – because their unconscious is really driving motivations and goals.

        So that’s why I go back to looking at how people’s perspective is grounded in their larger sociological constellation. That’s how I can get a better sense of their deeper driving motivations. When I see “anti-consensus” combatants arguing against a theory that would refute the effects of GHGs – that’s to their credit because they’re going against what would be, at least in some ways, their sociological motivations. But even below sociological motivations lie the psychological motivations – for example, more important than “tribal” motivations could be the personal motivation to be the “smartest” skeptic – leading someone to reject other skeptical theories even though the sociological goal of attacking AGW is shared.

        But then, in that regard, another “tell” for me is when people flat-out deny the overwhelming role of “motivated reasoning” for everyone involved in the debate. That’s where Judith’s position looses weight in my book. I am very sympathetic to her focus on quantifying uncertainty – but when she then turns around and expresses absolute certainty about the role of politics in the debate, or when she fails to accurately describe the arguments of combatants on one side of the debate or the other, I can’t accept it when she says that the ideas of someone like Postma are a non-starter because of their poor grounding in foundational physics.

        More of a response than you expected from your short comment to me, I’m sure – but you should always be careful what you ask for, eh?

      • Maybe my response was so long I should just summarize:

        Whenever I see someone ranting about the conspiracy amongst the “climate establishment,” I can’t take anything they say about basic physics at face value – in that they demonstrated that they, themselves, believe in something that defies the basic physics of how the political world works.

      • Josh, presumably you feel the same about people who rant about scientists being ‘paid shill’s of Big Oil’ etc?

      • Postma ran off the railis in his analyis around page 10 where he attempted to justify doubling the average TOA insolation.

        I got to the same point and had to work it out as a sanity check. The deal is that in his mindset, you have to think about the non-linearities in the effect and the instantaneous nature of the path to a steady-state.

        Say that the daily temperature, T, goes as T=T0*(1+k*sin(wt)) oscillates about some value T0 with a daily period. This models a day-to-day heating cycle. The SB law goes as T^4 so that if we average this cyclic value over one day, then the radiation is increased to first order by 3*k^2 from the stable T0 value (easy to do the integration). So according to Postma’s theory and what parameters he says are required, the T0 temperature is about 300K and the upper excursion is about 360K, so that k=0.2 and the radiation increase is 3*(0.2)^2=0.12 from the average.

        He then says that these temperature excursions aren’t actually measurable but that they exist in some state obscured by going into latent heat, evaporation, and convection in the macro case, and intramolecular degrees of freedom in the micro case, in his words.

        So this is the non-linear averaging of some instantaneous variation of the Stefan-Boltzmann law. The higher temperature excursions produce much higher radiation so that an overall higher equilibrium temperature is achieved.

        So I think I understand the rationale for his reasoning but his round-about way of showing this does not jive with a calculation. The temperature excursions required are just way too large. The oceans show virtually no change from day-to-day, so that the 3*k^2 would be completely buried in the noise.

      • Yep, very arrogant.

      • The Toy (Pekka word, not mine): (A structure built from the back pane and a window placed in empty space outside the atmosphere and illuminated by solar radiation. It’s a correct model for such a structure.) and with such a model with known solar insulation, average ground temperatures and average emission, you CAN prove a GHE is necessary.

        Ok, but is such a uniform temperature Toy model an adequate stand in for the Earth: a rotating planet, illuminated on one side, with oceans, with large amounts of water existing in 3 phases (with heat of fusion, heat of vaporization), heat sinks and buffering of oceans and land, and N2,O2, Ar atmosphere of specific sea level pressure and a surface temperature range from -89C to 58C? I’m doubtful. Based upon my doubt, the Toy does not prove a GHE on Earth.

        Mind you I believe the Earth does have a GHE; the Toy just isn’t proof. Postma says, “Look, with my model, I can make liquid water without any GHE. I even need a mechanism to cool the dayside. I didn’t prove how the water stays liquid overnight.” Is that so? Hmm. Is the need for a GHE reduced or increased? So how much GHE effect do we need with this model to keep most of the water between 1 and 6 deg C 24/365? Is it the same as the Toy? How much of the GHE is adiabatic and how much is back radiation as a function of GHG (which is mostly H2O)?

      • It’s not supposed to be a proof of anything, it’s most definitely not supposed to be a model of real world systems. You are not supposed to complain, when it’s not something, it’s not supposed to be.

        It’s just a extremely simplified model that may or may not help people to understand, what GHE is. If it doesn’t help you, please ignore it totally.

      • In my case, the Toy model did not help. When I was reading Postma, I was thinking “that’s a ridculous strawman.” The hundreds of comments made clear it is a real, simple model that people were defending. I would not trust any “enlightenment” coming from the Toy.

        The most simple model that should ever be taught is one that has night and day. A coin that flips every 12 hours. No, that concentrates energy, too much. How about an octahedron with a vertex pointed at the sun? Or the buckyball.

      • I don’t really understand your point. The toy model was not discussed much in the argumentation. There was some discussion, where Postma and his defenders tried to maintain that same position that you present here claiming that main stream supporters would give credit to the model. All the main stream supporters told that they really don’t and that the model’s unrealistic nature is always accepted and presented. This was just an perpetuated strawman argument by Postma and his defenders.

        Everyone agrees that the model is not a valid model of any real planet (on that point Postma and all others agree). The only question, where opinions vary is, whether it’s in spite of that a useful educational example. It can explain one phenomenon, but it’s obvious that some people may take it erroneously as an approximate description of the real Earth GHE even, when they are told that it’s not that. Thus it has a good point and a bad point. Educators should use it with care, if they decide to use it.

        Most of the discussion was on the other part of the Postma paper that claimed that the GHE is not important but that the temperature of the surface can be explained miraculously by the difference of day and night. Others pointed out that Postma did his calculations in reality as if the night would not be there at all. Of course he didn’t accept that, but that was really the point leading to an error by a factor of two. That was one of the two major errors, the other one was essentially imagining that sun warms as effectively everywhere as it does at equator (that amounts to an error of pi/2=1.57). Most of the discussion was trying to explain the errors in different ways.

        Postma made his error in calculating average power densities. Thus much effort was used looking at the calculation of average power densities.

        The other approach that I took for much of the discussion was looking at the total energy balance of the Earth surface, because it’s in some way more basic to look at the total energy fluxes than to look at the average densities of the energy fluxes. Of course the approaches are equivalent, but one may be easier to understand than the other. I preferred my approach, because it makes it easy to tell numbers for all main factors. The numbers were totally devastating for the Postma case. I asked him several times to present an explanation of that observation. First he just pretended not to notice my request although it was repeated in places, where he was participating in the discussion. Then I told that I consider the question a test of honesty. That made him answer, but of course not to the question itself, but something totally irrelevant for the question. How else could he have done, when he cannot have any valid answer to support a “theory” based of crude errors of factor 2 and factor pi/2. Nothing but mumble jumble can be presented by someone caught so terribly wrong, but not willing to admit that.

        Then we have still the people, who declare that they cannot follow the discussion, but use their ignorance to maintain the position that all arguments are of equal value and that the argument is not decided. Some of them feel even free to formulate that opinion as insults on those, who have explained in detail and as clearly as they can, how terribly wrong the Postma paper is. Pathetic.

      • Mind you I believe the Earth does have a GHE;

        This comment helps clear things up for me. If there is something else you are skeptical about, then we may not disagree. However, if you believe there is validity to Postma’s argument that the climate can be explained without the GHE, then that is still an untenable position. His argument can clearly be seen to have no merit, as has been exhaustively explained in the previous thread, independent of the simplified model under discussion.

      • I am skeptical of almost anything except the concept that “We do NOT live in a BINARY world.” It isn’t “Is” “Is Not”. It is how big and what is the range of uncertainty.

        It is not whether a GreenHouse Effect exists, it is rather how powerful it is. By the way, what are the units that specify the strength or power of a GHE in any given configuration?

        It is not whether CO2 contributes to the GHE, but what is quantitative difference in the GHE when CO2 concentration changes from 350 ppm to 400 ppm.

        What drives me most crazy about climate science that I have seen so far is the tendency to express uncertainty as ” >75% probability that A is greater X”. I don’t think that way. My industry doesn’t think that way. We think the P90-P50-P10 of A is X90, X50, X10. Capture and communicate the range.

        When it comes to quantifying the GHE, I would like more clarity on the different components of GHE. How much of it is adiabatic? How much of it is phase change of water (heat of fusion, heat of vaporization)? How much is back radiation by each gas and its concentration?

        Do these questions meet your threshold of legitimacy?

      • The questions are certainly legitimate, and although 100% certainty is unachievable, they have been addressed in detail throughout the climatology/geophysics literature. GHE strength is typically expressed in terms of “climate sensitivity” (deg C rise in temperature per CO2 doubling), and these values have been assigned pdfs, with the ranges debated thoroughly (including a previous post here on the topic) – a typical 90% confidence interval encompasses the range between 2 and 4.5 C., with much discussion of reports citing values outside that range. The logarithmic relationship between CO2 concentration and W/m^2 forcing (with consequent effects on temperature that are linear within the restricted range under consideration) has been quantified and explained, with the delta flux equal to about 5.35 ln of the ratio of the changed CO2 to the previous concentration. The respective roles of the dry adiabat, moist adiabat, and convective adjustments to the adiabat have been estimated. The back radiation as a function of the concentration and radiative properties of CO2, water vapor, clouds, ozone, methane, and other constituents has been analyzed, and the flux measured at various sites, including wavelength-specific measurements that distinguish between different molecular sources. The negative lapse rate feedback due to latent heat release has been evaluated and compared with the positive feedback from the heat-trapping properties of the same quantities of water vapor, with the difference between them, which is positive, found to be less variable than either quantity alone..

        Obviously, those questions can’t be answered in a few comments, but I think it is to Dr.Curry’s credit that one can go back through all the posts and threads and find much of this material even without the need to refer to cited papers or texts.

        Rather than continue to be driven “crazy”, you should probably go back and visit this material here, and if you want more detail, read the textbooks and start following the literature as well as the blogosphere.

      • Fred, GHE is far more than sensitivity to CO2 concentration changes, isn’t it?

        Let me back up to first principles: There are heat trapping mechanisms in the daily cycle of the earth’s heat flow. I have been lumping them all as GHE.

        GHE certainly includs
        1 — back radiation from GHGs, of which CO2 is only one and not the most important one. Agreed?

        The following heat trapping mechanisms are also in play.
        2 — Heat Capacity of water and air in the ocean and atmospher.
        3 — Heat of Fusion as water turns to ice.
        4 — Heat of Vaporization as water vapor condenses into water.
        5 — Adiabatic physics of the atmosphere.
        6 — Thermal conductivity of the air and water in the ocean.

        I will not include convection here. all the above are valid in a model with changing temperatures and without currents.

        Here I make an observation that I invite your comment:
        The Toy model is a static, single temperature model, and as such the contribution of 2 through 6 are zero. The whole answer is in 1, the GHGs.

        But in Postma’s model, which I find far more realistic than the Toy, Temperature MUST vary by lat, Long, h, t. Heat is trapped by all mechanism 1 through 6. and as a result, the contribution of the GHG to GHE might be smaller that implied by the toy model.

        Here is my crucial set of questions:
        Does GHE include A) 1, the back radiation? or B) all heat trapping mechanisms 1 through 6.

        If A), what then do we call 2, 3, 4, 5, and 6?
        If B), then what are the units or dimensions of GHE to capture its strength?

      • Stephen – We’re running into problems with column width. If you want to start a new comment, that would avoid the problem. You asked about units to measure GHE strength, and climate sensitivity addresses that.

        Regarding some of your other points, GHGs are heat trapping molecules, but I wouldn’t call the other phenomena trapping mechanisms, although all are important for the rate and location of heat changes. Of the GHGs, I would list CO2 as the most important as a climate driver, with methane second, followed in no particular order by nitrous oxide, halocarbon compounds, tropospheric ozone, and a variety of others. With the exception of water derived from methane oxidation, water doesn’t belong on the list because its short atmospheric lifetime prevents it from having a role in determining temperatures except as a feedback mechanism.

        I didn’t pay too much attention to the “toy model”, but because the GHE requires a lapse rate, the adiabatic element is important in any description of the GHE.

        I don’t know whether I would characterize Postma’s description as a “model”, but I do know that it was wrong, because no matter how tweaked, it couldn’t account for the surface temperature of our planet. That was documented in adequate detail, I thought, in the previous thread, including a contribution I made, based on much of the earlier discussion, in which I dissected the various fluxes in ways that would allow anyone to compute them and refute the Postma conclusions. See Comment 101095.

      • I in your comment 101095, you were using a constant temperature argument, Yes? In a constant temperature model, only Back radiation is available to provide the needed warming or heat retention. But in a variable temperature day-night model, your have at least five other mechanisms (my 2 thru 6) that will also help retain heat. I think the variable temperature aspect with all the heat storage factors is crucial to to understanding the energy balance.

        I will continue this line of thought in a first level comment with a summary and link back to here, but I’m out of time for the night. I’ll sleep on it.

      • Stephen – Constant only in the sense of averaging. Not constant either regionally or diurnally. If you take all individual temperatures at different locations and different times of day, computed the emission rates for each in W/m^2, added them up, and then computed a temperature that would yield the same emission rate (about 390 watts) averaged over every square meter of the surface, you would arrive at about 288 K, based on observed values. Postma agrees with this part of the exercise. There are probably a few ways this could be tweaked (as I suggested in the linked comment and you could try out examples), but they aren’t going to come out with anything even remotely close to what Postma’s attribution of surface temperature to solar influx alone would give us – that would make the planet an extremely frigid place to live.

        (In fact, if you created an absurdly unrealistic scenario designed to maximize his argument, you could do the following. Consider the Earth as a flat disk rather than a sphere, with one side always facing the sun at right angles to the irradiance (no slanting), and then have it instantaneously switch sides every 12 hours (without any slanted intervals). Subtract the albedo (about 30 percent) and another fraction for solar radiation absorbed in the atmosphere (about 23 percent, with no downward emissions but allow some of this energy to be conducted to the surface from lower altitudes – e.g., 10 percent of the irradiance), Here, the average value is clearly one half of the total irradiance (not 1/4 as in a realistic scenario), and every illuminated region will receive the same irradiance.

        When you do this, which in effect doubles the energy the Earth actually receives, the average emissions will turn out to be about 411 W/m^2, which is only slightly higher than the 390 based on the observational data, with only a minimally higher surface temperature. Thus, even doubling the Earth’s rate of energy supply barely allows one to reach an average temperature comparable to the one we enjoy. This illustrates the gap between a solar source alone without back radiation and what is needed to keep us comfortable.)

      • I like the coin-flip model. I discarded that as being too hot. But If I can’t show enough heat retention in that model, I’ll conceed the issue. The test is whether the energy received in the daylight can be stored in non-temp radiating means (2,4,maybe 5), and bleed back at night to keep H2O liquid. Later.

      • So since you do not dispute the existence of the greenhouse effect, and ask a legitimate question about it’s strength, here is a quick answer. Currently the GHE is about 155 w/m2, but it varies with latitude ranging from about 250 w/m2 in the tropics to around 100 w/m2 at the poles, mostly because of changes in water vapor.

        So the 64 dollar question is how will this change with increases in anthropogenic CO2? Just the increase in CO2 will strengthen it some, but the feedback uncertainties are large. It is almost certain that it will increase with increasing CO2. Is is also 100% certain that it will stay at an elevated level for thousands of years, which is scale at which geologic removal of CO2 from the biogeochemical works. The canonical estimate of the value of S (sensitivity for 2X CO2) which is 2C to 4.5C provides a range of values. There has been a steady trickle of papers which dispute this on the low side, but none seem to have gained traction.

      • Rattus, thank you very much for the reply. Thank you for the Range in GHE values and confirming that water vapor is a large component of the range. Do you have a link to the source of these numbers so that I can follow up?

        I noticed you use W/m2 for GHE strength, which makes sense, but then shifted to deg C per doubling for CO2. Ever seen the CO2 part expressed in W/m2?

        Excuse my absence from the discussion these past 3 days. I haven’t given up the subject, just pre-occupied with end-of-summer driving from Colorado to Houston. I thought much about this thread while looking at towering thunderstorms in the rear-view mirror in CO and observing whether or not I could see UHI effects passing through northern Texas towns at 104-110 deg F. But those thoughts will be left for a level 1 post, not here.

      • Peka,
        You and I know what you say is false. Postma (and the Slayers) cited 60 examples where universities from Standford to MIT are teaching students the zero-D model and no other. The Slayers have hunted high and low for a more complex model but none exists in academia. If it doesn’t exist in academia where is it? Last year I asked Judith Curry to provide me with a more complex model that is taught at Georgia Tech than the K&T zero-D version and she couldn’t offer me one.

      • John, read my post, including the wikipedia link i provided and the discussion of radiative transfer models. I have dozens of models that are used for different applications, that are used for teaching and research at Georgia Tech. There is just no point in going further until you accept the basic physics behind infrared radiative transfer.

        If you want a complicated model, here is one example, you can download the code and read all of the the documentation. Once you’ve done that, then lets talk.
        http://www.cesm.ucar.edu/

      • Dear Curry,

        the models you have linked to have all FAILED to predict the current temperature trends and yet you continue to insist that these models provide validity for the so called “greenhouse effect” hypothesis.

        That is an untenable position.

        These models have failed to predict the current reality because they are based on a failed hypothesis, false assumptions, and incorrect input parameters.

        One specific and easily identifiable example of this is “convective parameterisation”. It is simply impossible to correctly estimate the total convection taking place on a planetary scale. If you overestimate convection you get cooler estimated temperatures than the observed.

        If you underestimate it, your model produces exactly what occurs inside a greenhouse.

        GI=GO.

      • Shouldn’t it be: (GI+GO)=0

      • Pekka, if you had read Postma’s paper with any care you’d have seen 60 cited examples of universities teaching the zero-D model and no other. I asked Dr Curry to advise me if Georgia Tech teaches only from the zero-D model.I have never received an answer.

      • John, I replied to you at length already on the blog, including the links to sokolik’s text that a number of people have commented on. This describes what Georgia Tech students learn about radiative transfer.

    • [this is a rewrite of my third paragraph above]
      A relatively simple model of a rotating “buckyball” with a day and a night might be a more reasonable simple model of energy balance. I think you have only 5 unique surface orientations, convolved with 4 basic surface (jungle, desert, water, snow/ice), rotated in (12, 18, 24..) discrete time periods of the day on an axis through opposite vertexes. It is certainly more complicated than the zero-d model, but this is easily done in one small Excel worksheet as a homework assignment.

  26. Alexander Harvey

    When creating a mental model of the GHE there has always been two approaches, two points of view, looking up and looking down.

    Most here will be familiar with the looking up POV, the one with DLWR and many will not like it.

    The alternative perspective, the looking down POV, has at least as long a history and I know it was once taught at the graduate level.

    The alternative, looking down model explains exactly the same GHE, only the POV is changed.

    A model that explains the 33C discepancy by way of the lapse rate and a significant height (~5km) but fails to mention how this height is determined is just the GHE in another guise. It is the downward POV of the GHE but without mentioning the GHGs on which it relies.

    There is nothing novel in an explanation based on a lapse rate mechanism, it is well trodden ground, and it relies on just the same GHE. Only the POV has been changed.

    Anyone who has rejected the upward POV (DLWR and all that) and accepted the lapse rate mechanism as an explanation, on the basis that it makes the GHE go away, has been sold a pup, and should ask for their money back.

    Alex

  27. Western academics must stop tilting at windmills. Does anyone believe Western civilization should continue to put promoters of wacky-science on the public dole?

    “Hence, the big question: if in the Middle Holocene, when dung and firewood were the only fuels used by Mankind and the world population was at least two orders of magnitude smaller than today’s, there were temperatures and sea levels considerably higher than the present ones; if the atmospheric warming at the end of the Younger Dryas 11,600 years ago and the sea level rise between 18,000 and 6,000 years ago were much faster than the observed since the 19th century; so, where are the evidences that would allow us to point to the “human fingerprint” in the small variations of the latest 140 years, against the background of the much wider and faster natural oscillations of the historical and geological past?

    “The answer is: there aren’t any (for evidences I mean, obviously, hard facts observed in the physical world, not mathematical concoctions extracted from the climate models).”

    ~Geraldo Luís Lino

  28. Judith,

    I like your chapter…though, Equation 14.13 does not appear correct,

    it should be To = [(n+1)^1/4]Te*

    • Yes you are correct, i copied the text from my word version of the chapter, that is in the errata (which i forgot to check). thx for pointing this out.

  29. Dr Curry writes “If you want to disprove the existence of the greenhouse effect, you need some sort of alternative explanation for a whole lot of fundamental physics that explains the infrared emission spectra as measured at the Earth’s surface and high above the emitting portion of the atmosphere (see Pierrehumbert). ”

    I am not interested in disproving the greenhouse effect. The majority of skeptics agree that AGW is real. The problem we have is with catastrophic AGW (CAGW).

    It is impossible to prove that the hypothesis of CAGW exists. I write “impossible”; I mean “impossible”. There is no science, no physics, no observed data that shows that CAGW happens. Such little observed data as we have, tends to show that climate sensitivity is significantly less than 1 C for a doubling of CO2. How much less, no-one has any idea. The effect could be completely negligible. There is simply too much noise in the data on GMTs. Noise, by definition, averages it’s signal to zero if there is enough time to do the integration. However, we have very little data on what constitutes natural noise; what it’s amplitude is, and what it’s time constant is. There are indications that some significant noise could have a time constant measured in centuries. In which case, it is simply impractical to ever have a long enough data base for this noise to get averaged to zero.

    So I suggest, Dr. Curry, you forget about trying to convince skeptics to stop trying to prove that CAGW is scientific nonsense.

    • I suppose it is impossible to prove a concept with no scientific definition…

      • Chris,
        That is an annoying bit of ignorance on your part.
        Of course CAGW has a scientific definition. Hansen, et al have been selling just that for over 20 years.
        To dodge that makes you look cowardly.

      • Mr. Chris Colose, Please let Mr. Ben Bernanke (our current money scientist from Princetown), know this fact before he discovers that ‘gold IS money’.

        PS Please hurry. Thank you.

    • The majority of skeptics agree that AGW is real

      Really?

      How do you quantify that? I read some “skeptics” say that is so – but when I look at these here threads and those over at WUWT, I see many a post from “skeptics” who doubt, even, the GHE. I see, over and over, statements that the GHE was an “invention” for the purpose of justifying political goals.

      Perhaps you’re right. What’s your evidence?

      • Indeed Joshua, there are some skeptics who think AGW is real (that is, significant) but benign, but they are a relatively small minority. Pat Michaels for example, who argues that future AGW will be like the past, less than a degree on winter nights, etc. Perhaps the so-called lukewarmers fall into this camp, I really don’t know that much about them.

        But the vast majority of skeptics are skeptical of AGW for one reason or another. Of those there are two camps, those who think the jury is still out and those who are convinced that AGW is either insignificant, false or phony. Note that as the latter camp falls into three sub-camps we have a taxonomy going. One could also subdivide the population according to the reasons they give.

      • David –

        I guess my interest is in breaking down that last group and getting a sense of the relative size of the sub-categories. Jim above, and often Judith, say that the “false or phony” group is relatively insignificant in size; my sense is that it isn’t. I often see the size of the “jury still out” group downplayed. And I think that depending on the particular thread or context, some folks move back and forth between groups. And hunter says that the dragon-slayer contingent is a majority or skeptics, although looking at the threads, that doesn’t seem to hold up.

        At any rate, I think that some “skeptics” are too quick to diminish the size of those sub-groups, and I’m “skeptical” about their reasons for doing so.

      • Joshua, false or phony is two different groups, and each is substantial. So are the lukewarmers. Dragon slayers are too new but gaining ground. It is an empirical question how big each is and the polls do not properly discriminate them. Moreover, as you point out below, a skeptic may participate in different groups, depending on the argument at hand.

      • Lukewarmer is easy.

        1. we accept radiative physics. GHGs warm the planet, they do not cool it.
        2. We accept the range of sensitivity cited by the IPCC.
        3. If you give us an over under bet on sensitivity at 3C, we take the under
        4. Free the code, free the data, open the debate.

        Or rather there is a debate in climate science, but you dont get to join that debate until you accept #1. You can debate #1 if you like, but dont be shocked when Ray P doesnt show up to waste his time. (no disrespect to the guys who do show up to attempt a discussion)

      • Mosh, what you describe sounds like accepting AGW but not CAGW, which sounds right to me. However if you agree with the assessment that AGW becomes dangerous at 2 degrees then your lukewarmer might accept DAGW (dangerous but not catastrophic). Of course every group of skeptics has a full range of shades therein. Given the range and complexity of the arguments there are few if any sharp divisions here.

      • “1. we accept radiative physics. GHGs warm the planet, they do not cool it.”

        Radiation is a cooling process, not a warming process.

        An object or substance, is heated by the energy it receives and cooled by the energy it looses, RADIATES.

        Perhaps this confusion is the source of your relentless ignorance.

      • TimTheToolMan

        GHGs slow the rate of cooing. There is a lot of scope for that effect to be much slower than the IPCC believes will happen. Ultimately the warming will probably win though. I wouldn’t rule out “ultimately” to be measured in millennia.

      • Any substance in between a body (such as the Earth) with a temperature and space with 0K temperature (empty space) slows down cooling of the body (the Earth). If there are multiple substances (such as air) in between, then those have less radiation effects (such as N2 and O2), will have less effect of cooling the body. Those have more effective radiation effects (such as H2O and CO2) will help those less effective substances (N2 and O2) cooling faster due to moleculer collisions in the exchange of energy.

      • TimTheToolMan

        I’d like to add to that Mosh, lthough it may not be one of your personal points…

        5. We dont believe that today’s temperatures are necessarily optimal for life on earth and dont discount the fact that that higher temperatures may be on the whole beneficial.

      • Joshua, Please slash the A in AGW… ok? Time will tell the rest.

        Strong men believe in cause & effect.
        R.W.E.

        See a simple agenda, laid out…

        http://www.un.org/esa/dsd/agenda21/

        Hope, it helps.

      • Joshua, Please slash the A in AGW… ok? Time will tell the rest.

        Now that, to me, seems like a more accurate way to go. I see reason to believe that yes, many “skeptics” accept GW but not AGW.

        But even there, I find claims about disproportionality to likely be overstated.

        I read people say “Most skeptics don’t deny that the climate is changing, we just don’t think that the change is anthropogenic,” right before they talk about how much snowpack there was this year in the High Sierras, or how some seals uncharacteristically showed up in Boston Harbor, or how the measurements that show warming were corrupted by the UHI or deliberately manipulated by fraudulent scientists bent on creating a socialist “One World Government.”

        In other words, I read people saying that they don’t doubt GW and then turn around and present arguments that suggest that they doubt GW. It happens quite a bit. I don’t know how to quantify the proportionality – but from what I’ve seen, many folks want to portray the proportions in ways that don’t fit the evidence.

      • Nullius in Verba

        We don’t doubt present GW, or that the climate has always changed, but we do doubt that it has been quantified as accurately as is claimed.

        Making points about levels of snowpack are a form of weather-is-climate argument, and not valid whether used for or against. I only use them as counter to the pro-AGW weather-is-climate stories, but I suppose some people may have genuinely got the impression that weather is climate. The mainstream has certainly worked hard to give that impression.

      • We don’t doubt present GW,

        What do you mean “we,” Kemo Sabe?

        How many times have we read on this blog about “global cooling?”

        In fact, Tallbloke showed up right on cue.

      • In fact, the plot shows a strong global averageness.

      • Joshua, you are once again, making remarkable progress… Now that we agree man has nothing to do with the weather let’s move on to Agenda 21.
        Here is an agreed to plan that is well laid out for all to read. Turn left and you are here. Nuts…

      • You noticed that too.
        You will find that very few skeptics have a well thought through position about what they believe. If you even try to get to a common ground about whether it is warming or not, some will resist. Until you question the existence of the LIA. Then, they will believe in warming and thermometers.
        Tactically they dont need a consistent world view. But I cant live that way

      • It’s warming AND cooling, depending on timescale, magnitude and significance.

        Regarding thermometers, it’s really not that complicated. One can believe that the trend at the particular time scale is warming for example, but one can still disagree on the magnitude of that warming.

      • Mr. Steven Mosher,
        “You will find that very few skeptics have a well thought through position about what they believe. If you even try to get to a common ground about whether it is warming or not, some will resist… Tactically they dont need a consistent world view. But I cant live that way”—
        Yes we will & it should scare the hell out of us. Lots of U.S. regular folks, believe in this End Mode…II Peter 3:9-13. And yes, man does have ‘something’ to do with it all. Have you read the seventy books of the Bible, & do you believe in the plan of God? Perhaps, there is a ‘stumbling block’ in the way of science? I know the Bible, is not peer reviewed but it has been very accurate over six-thousand years (or six days; please read II Peter 3:8). Please,…Do not resist:)

    • people should be skeptical of CAGW; skepticism about the existence of the greenhouse effect is not defensible, this is what i am trying to convince people of. The existence of the greenhouse effect is not equivalent to CAGW.

      • i.e. the degree of its effect – from helpful to insignificant to carbongeddon, no?

      • Claes Johnson

        Judy, you do not answer my question about the greenhouse effect you are speaking about: Is it based on a simple radiation model without thermodynamics?

        The discussion is about CAGW, or a potentially catastrophical greenhouse effect. If you agree that the greenhouse effect,
        whatever it is, is so small that it cannot be observed, then there is little to discuss. Is this what you are saying? That there is a greenhouse effect but that it is so small that we can forget it? If so, tell your readers and much time and effort will be saved. If not, you seem to believe in CAGW which you say one should be skeptical of.

        Your readers expect you to speak out.

      • yes, I am referring to the so called tyndall gas effect, the infrared radiative transfer. The response of the atmosphere to the heating is both thermodynamic and dynamic. Lets settle on the radiative transfer part first. We have had these discussions on the earlier greenhouse threads. the word greenhouse is bad terminology.

      • Judith, please can you help me understand something. Do proponents of AGW believe that the ‘back radiation’ is ‘additional energy’ when it hits the surface? Surely the surface cooled by the amount of longwave it emitted before it ‘re-warmed’ when the ‘back radiation’ bounced down again.

        I can see that sunlight absorbed in the atmosphere and propogated forward to the surface as longwave is ‘additional energy hitting the surface’ (not that it does much IMO). Surely scientific people differentiate between these components?

      • the earth received radiant energy from infrared emission by gases such as CO2 and H2O and also clouds. The earth’s surface temperature is determined by a balance of fluxes for shortwave and longwave radiation, sensible and latent heat fluxes, freezing or melting of water, and thermal conduction in the ground or mixing of the upper ocean.

      • Well, yes, but with respect, I can’t tell from your reply whether scientists differentiate between ‘new’ longwave and ‘recycled’ longwave. I can see that if fluxes are properly considered that should cover it, but a lot of scientists seem to talk about ‘Downwelling longwave’ as if it can be considered as an energy flow in its own right, without regard to the fact that most of it was previously emitted by the surface thereby cooling it, before ~30% of it returned to haunt us.

        There’s a lot of double accounting going on.

      • There is an upwelling and a downwelling flux. Upwelling-downwelling = net IR flux. You also need to account for evaporation (surface loss) and sensible heating (usually surface loss, not always). Why is this so difficult?

      • Hi Chris, as I see it, there’s only one flux, and it doesn’t just contain radiant photons going up or down, but sideways, at 45 degrees etc etc.

        My point is that we can understand more about the degree to which the flux is slowing the cooling of the ocean if we differentiate between the quantities of radiation which are contributing to new effects, and the quantities which were already emitted before they were re-absorbed.

      • tallbloke, Chris is right. These two fluxes can be treated independently. It is wrong to think of it like reflection. They are separate emissions in each direction.

      • Jim, where did I mention reflection? Back radiation was emitted by the ocean which consequently cooled before it came back down to the surface along with ‘new’ longwave derived from theabsorption of solar radiation in the atmosphere. True or false?

      • “back” radiation exists because the atmosphere is warm. Whether it is warm due to radiation or convection is beside the point, but actually it is convection that keeps the atmosphere warm.

      • Jim D,

        “… actually it is convection that keeps the atmosphere warm.”

        Congratulations you have evolved from the CO2 ivory tower.

      • Convection keeps the atmosphere warm, when the back radiation from the atmosphere keeps the surface warm to feed energy for convection. These go together. Without the convection the surface would be even more warm and so would the lowest levels of the atmosphere, but the higher altitudes would be colder.

      • LOL, I love this! With 2/3 of the radiation to the surface from the atmosphere, then the back radiation could very well be considered the up welling. It is like who have more angels on their pin. What I think is lost is that nearly 20% of the atmospheric warming is caused by direct absorption of solar which is “new” energy warming the surface, the rest is just hot potato energy being passed back and forth which reduces the rate of cooling.

      • SamNC, most heat enters the atmosphere via the surface. Why should that be controversial? AGW needs this fact to explain the tropospheric lapse rate, which is a critical part of the argument in climate sensitivity.

      • Jim D | August 20, 2011 at 11:40 am

        Jim, not sure if that reply was to me, but if so, it doesn’t answer the question I asked, which was:

        “Back radiation was emitted by the ocean which consequently cooled before it came back down to the surface along with ‘new’ longwave derived from theabsorption of solar radiation in the atmosphere. True or false?”

        Looks like Dallas gets it anyway.

      • Pekka Pirilä | August 20, 2011 at 12:00 pm |
        “Convection keeps the atmosphere warm, when the back radiation from the atmosphere keeps the surface warm to feed energy for convection. These go together. Without the convection the surface would be even more warm and so would the lowest levels of the atmosphere, but the higher altitudes would be colder.”

        How the back radiation feed energy for convection? You keep yourself confused with convection from the ground surface and convection from air to air. Only the warmer air below keeps convection goes on with cold air above. Energy is fed from air below at warmer temperature. Not from the same temperature at the same altitude level’s back radiation.

        Jim D,

        I found your lapse rate confusing, how do you define lapse rate here?

      • Jim D | August 20, 2011 at 12:20 pm |
        “SamNC, most heat enters the atmosphere via the surface. Why should that be controversial?”

        No controversial at all as heat is convected and conducted to the atmosphere from the Earth’s surfaces. Controversial is that the Earth surface LW radiation heat does not heat up the atmosphere at all but AGW believers insisted that the radiation part do most of the heat transfer from the Earth surfaces to the atmosphere.

        ” AGW needs this fact to explain the tropospheric lapse rate, which is a critical part of the argument in climate sensitivity.”

        Define your lapse rate here and how to effect the backradiation heat up air.

      • tallbloke, yes, my 11:40 pm answer was to you. “back” radiation does not require upward radiation to be doing much. It only requires a warm atmosphere, and convection is mostly what keeps the atmosphere warm. So I can only give your true or false a ‘partially true’. As I say elsewhere, the net effect of IR is to cool the atmosphere, which helps convection.

      • SamNC, no IR radiation is not a major source of heat to the atmosphere when you consider that its net effect is cooling. The tropospheric lapse rate comes from convection and the properties of water condensation in it. So the air temperature is mostly determined by the lapse rate, and IR radiation from the atmosphere is determined by its temperature and GHG distribution.

      • Jim D | August 20, 2011 at 1:23 pm |
        “SamNC, no IR radiation is not a major source of heat to the atmosphere when you consider that its net effect is cooling.”
        There is no dispute here that IR is cooling as air IR energy is radiated towards the space.

        ” The tropospheric lapse rate comes from convection and the properties of water condensation in it.”
        Water condensation in the atmosphere is due to loss of its latent heat to the space via radiation or thru convection to nearby colder air. I have no problem with that.

        ” So the air temperature is mostly determined by the lapse rate,”
        So your lapse rate is defined as the loss of heat content in the air due to convection to lower temperature air nearby and and IR radiation to the space. I have no problem with that.

        “and IR radiation from the atmosphere is determined by its temperature and GHG distribution.” I have no problem with the 1st part, i.e. ‘IR radiation from the atmosphere is determined by its temperature’, but how do you get the GHG distribution into the picture. GHG distribution is only a small composition of the air. Its lapse rate is minimal, heat content is minimal.

      • SamNC, you and others, like Arfur Bryant who just posted, always say it is only 2.54% so how can it do anything? It can, and you can work out W/m2 of radiative fluxes from them, and they are significant and they are measured to be the expected values at the expected wavelengths. What else can be needed to convince someone? Even if you have no hope of understanding the theory, you should understand the measurements.

      • Jim D | August 20, 2011 at 1:56 pm |
        “SamNC, you and others, like Arfur Bryant who just posted, always say it is only 2.54% so how can it do anything?”
        No, wrong with CO2, it has only maximum of 0.04% of radiation absorption effect when all CO2 in the atmosphere assumed to have 100% effect of intercepting of the Earth’s IR radiation. 390ppm effectively in empty space can maximim absorb 390/1,000,000 th of CO2 absorption wave bands if the CO2 has 100% absorption efficiency. I have been extremely generous to give credit to CO2’s absorption of IR radiation from the Earth’s surface IR radiation.

        ” It can, and you can work out W/m2 of radiative fluxes from them, and they are significant and they are measured to be the expected values at the expected wavelengths.” Show me your calculations.

        “What else can be needed to convince someone? Even if you have no hope of understanding the theory,.” When you are facing the reality of the thermodynamics and radiation laws, you always resort to these negative comments that will not help you to understand energy contents of air and CO2.

        ” … you should understand the measurements” I have done measurements and calibration to standards for years. Do you understand measurements?

        There is no hope for you to understand that GHE is violating radaition and thermodynamics that you prefer to stay inside the CO2 ivory tower.

      • SamNC, I don’t think Arfur was wrong when he said 2.54% because he was including H2O which is also a GHG. You can take it up with him. You say you know measurements, so when you have a measurement of 300 W/m2 downward IR from clear sky, how do you interpret that? Science has its explanation down to the specral details, but maybe you have an alternative. Can you explain why the spectra correspond to CO2 and H2O lines?

      • Jim D | August 20, 2011 at 3:09 pm |
        “SamNC, I don’t think Arfur was wrong when he said 2.54% because he was including H2O which is also a GHG. You can take it up with him. You say you know measurements, so when you have a measurement of 300 W/m2 downward IR from clear sky, how do you interpret that?”
        I don’t do IR measurements but I can tell you that 300W/m2 if measured is the air’s total radiation measurement at its temperature of radiation at the point of measurement which has almost nothing to do with GHGs which CO2 is only a trace gas contributed minimal quatity of IR heat content. Major radiation of heat comes from the air. Even though the sky is clear, the measurements contain refractions which has energy contents greater than CO2’s and if the moon is above, reflection from the moon is greater than CO2’s radiation and if you include the terrestrial stars radiations, they are most probably greater than CO2’s radiation contents.

        ” Science has its explanation down to the specral details, but maybe you have an alternative. Can you explain why the spectra correspond to CO2 and H2O lines?”

        Science is limited so far on radiation measurements of O2 and N2, especially N2. You can do an experiment by heating N2 in a glass container to say 100C to see if the N2 in the glass container radiates energy and measure the cooling rate inside. You can also do that with pure CO2 gas in an identical glass container and do the same measurements. You can also do the same with air in glass container and measure the cooling rate of the air inside the glass. You can compared with the results and see if the N2 can cool by its radiation. You can also place ice inside a glass container near the N2 glass container and another ice in another galss container which is completely blocked radiation from the N2 container to verify if N2 raidates IR waves. These tests should be done in a laboratory with controlled environment.

      • SamNC, your hard-thought theory, which I am sure you prize as a major academic achievement falls down when you try to explain (or ignore in this case) the spectral lines in that IR radiation. Especially if you see an emission peak at 15 microns. How does your theory explain that one? A long-lost O2 line?

      • Jim D,

        “A long-lost O2 line?”

        There are plenty of O2 lines, which one you like?

        ~700nm and ~1300nm.

      • SamNC, you probably did not notice that your wavelengths are not in the part of the IR that matters for temperature, and those O2 lines would be several orders weaker than the CO2 lines.

      • Jim D,

        No, I said CO2’s 15um is not significant compared with whole spectrum of IR radiation. CO2 dwarf miserably compared with H2O in the air and H2O dwarf miserably compared with the Earth’s surface radiation. Heat content is proportional to their compositions with their respective specific heats.

        N2 radiation spectral content cannot be determined by the present technology does not mean it cannot radiate IR wavelengths. Its just the science is not capable of measuring them at the moment. You can do the experiments to determine if N2 has the IR radiation effect as suggested by me a few threads above.

      • Jim D,

        O2 has shorter wavelengths bands. In general, shorter wavelengths means higher energy intensity as compared with CO2’s 15um. O2 has 3 orders of magnitude more than CO2. Its interception of the Earth’s IR radiation will dwarf CO2’s interception of 15um.

      • 3.7wm-2 forcing per doubling isn’t it for CO2? Hardly insignificant.

        I think we can safely assume scientists have measured the radiative properties of N2 and found it lacking.

      • SamNC, if you were able to find a spectrum of downwelling longwave radiation, particularly in dry environments you would not say 15 microns is insignificant. It would be a dominant feature. Even more so from a satellite looking down anywhere on earth.

      • Jim D,

        Are you sure satellite radiation measurements indicated all or mostly 15um? If not what is the proportion of radiation energy (or heat flux or whatever you call it) at 15um, other wavelengths. I thought spectral lines present to identify atoms of different materials. Can you confirm with NASA that their radiation measuring equipment not biased with a band of particular wavelengths? Or is the instrument just measured total radiation that are detectable to their instrument design and unbiased/neglect others? Has the calibration done against all wavelengths all intensities?

      • SamNC, the instruments measure all IR wavelengths equally well, and the 15 micron band would be a prominent dip from a satellite due to emission by CO2 at high levels. I could answer the fraction of the amount by using MODTRAN, but I won’t.

      • Jim D,

        Judging from your upset about my comments on the MODTRAN, my apologies if your family living depends on your MODTRAN.

      • SamNC, not upset. I think you are quite comical, and not at all serious.

      • Jim D,

        Not that MODTRAN again! Any climate models are manipulted to give the modeler desired results. You are correct, I have zero faith in them.

      • Yes, I know radiative transfer is not your area of knowledge as you keep proving. It is therefore a waste of time arguing about the radiative models with you because those are the tools of the trade.

      • Jim D.

        HaHaHa. You did it again. I told you your negative comments get you nowhere except inside the CO2 ivory tower. You cannot escape from it. Thats your life.

      • Claes Johnson

        Ok Judy, so greenhouse effect is bad terminology and back radiation as well, and one should be skeptical to CAGW. How much then remains of the climate alarmism you seem to be defending? Or have you converted
        to skepticism to alarmism?

      • I am skeptical in the true scientific sense. Read my previous posts on uncertainty, also attribution. I am not skeptical about the tyndall gas effect or our ability to simulate clear sky radiative transfer in the the atmosphere to within our capabilities of measuring basic atmospheric state and spectral radiances.

      • You said yesterday that satellites can measure the amount of longwave being emitted from the actual ocean surface. how do the satellite instrument people know how much of what they are measuring is coming from the contiguous surface of the ocean, and how much is coming from water vapour immediately above the ocean surface?

      • Read the validation reports. As I have explained many times to reconstruct the surface temperature from the sensor data, they apply RTE. All you have to do is visit any one of the data product pages and walk through the software descriptions. They lay out every step and radiation codes they use.

        heres one. The ground truth comes from Buoys.

        http://www.star.nesdis.noaa.gov/sod/mecb/goes_validation/test/background.php

        http://www.star.nesdis.noaa.gov/sod/mecb/goes_validation/test/sst_validation.php

        http://www.star.nesdis.noaa.gov/sod/mecb/goes_validation/test/val_coast.php

      • Hi Mosh. OK, they can validate temperature retrievals with buoys. How do they validate longwave emission? In situ measurements have the same problem.

      • What do you mean by “validate” long wave emission?
        You have a sensor(s) that receives in various bands, some of then tuned to IR or tuned to say C02 bands ( transmission windows)

        Are you talking about the calibration? That’s done on the ground.

        Or are you talking about experiments where you have a source of known illumination and you do a retrieval of that? Like an IR beacon on the ground? Did that ages ago for star wars using a ground laser that operated in the correct band. This engineering CANNOT WORK if the dragon Slayers are correct.

        http://www.aero.org/publications/crosslink/summer2000/03.html

        or are you talking about sub orbital validation
        http://www.google.com/url?sa=t&source=web&cd=7&ved=0CE4QFjAG&url=http%3A%2F%2Fairbornescience.nasa.gov%2Fsites%2Fdefault%2Ffiles%2Fdocuments%2FSPP_Rqrmnts_v6.pdf&ei=XVpPToivO6niiALrptiRAQ&usg=AFQjCNGEt26Fexf0u6yXbf2sNucPvuF8uA&sig2=2TwTh_ccXgEyihcHNFZ9hA

        IR or any other band its the same problem. The physics of radiation transfer refers to the ENTIRE spectrum. I shine Xband at a target. I know my output. I measure the signal at the reciever. I can predict how that radiation will travel through the atmosphere. how different molecules will react. I can predict how that wave will hit an aircraft. How much will reach the aircraft, how much will be backscattered, absorbed, forward scattered, hit a millimeter crack and bounce back. Hit a rotating fan in an engine duct, scatter in the duct and come back out. the theory is well known and tested. I can do the same thing with IR. whether its the earth emitting IR or a jet aircraft flying at 600 KEAS. That sucker emits a tell tale signature in multiple bands ( hot gases, hot metal, aeroheating) and we know how that radiation propgates. Thats how we build missiles to shoot stuff down. And we know how bright to make a flare to fool that missile or how to build a device that sends out IR to blind the missile. I build a cell tower. I emit radiation. How far does that signal go? Depends upon the atmosphere and the radiated power. The atmosphere attentuates the signal.
        These physics work. Because they work you won’t find engineers who have worked in this area ( like jeffId for example) who will deny RTE. why? cause we liked getting paid and we liked building shit that worked.

      • Mosh, all very cool, but I still don’t see an answer to my question:

        “how do the satellite instrument people know how much of what they are measuring is coming from the contiguous surface of the ocean, and how much is coming from water vapour immediately above the ocean surface”

        Also, light scattering in clouds is a bit of a problem area in theory. An experiment was done with planes carrying measuring gear above and below clouds simultaneosly. They used a stack of five planes I believe. The empirical result showed that the clouds absorbed 30W/m^2 more solar energy than theory predicted. This leaves a bit of a hole in the theoretical energy budget. The models are given ad hoc adjustments to try to deal with this, or so a modeler says.

        Strong focussed signals are one thing. Measuring power associated with diffuse weak signals is another. An IR thermometer will tell you that radiation coming from the Sun is at ~5000k whether you are stood on the moon or pluto. The amount of power arriving is very different. Signal does not equal power.

        Cheers

      • Dear Curry,

        The confusion surrounding the “Tydall gas effect” as you call it, is the direct result of John Tyndall’s inability to distinguish between absorption, scattering and reflection, all of which are forms of absorption and re-emission.

        At the time of his experiments, which are written up in his memoirs “Contributions to Molecular Physics in the domain of Radiant Heat”, Tyndall was completely ignorant of the fact that absorption does not equate to warming.

        This level of understanding comes from quantum physics which was developed long after Tyndall’s demise.

        In-fact in none of his experiments, does Tyndall ever attempt to measure an increase in the temperature of the gas under observation. He merely detected a decrease in the energy arriving at the pile, from the source and made the assumption that this would result in warming, but Tyndall was wrong.

        Reflection and scattering are forms of absorption and re-emission. They do not have to result in warming. The potential for warming is determined entirely by thermodynamics processes which are governed by the 1st, 2nd, and 3rd laws. All three of which, “AGW” hypothesis and the “greenhouse effect” hypothesis are undeniably in complete violation of.

      • Judith – I hope that you’re congnizant of the degree to which “skeptics” are not aligned with the position you just stated. You seem intent on downplaying the # of skeptics who reject that basics of the GHE that you consider to be established fact. Read back over this thread, and perhaps you’ll reconsider?

      • Judith, you say ‘skepticism about the existence of the greenhouse effect is not defensible.’ But it is when discussing it in the form presented by the standard model. Postma has demonstrated that by the addition of calculations for night and day insolation, adding adiabatic physics and by scrupulously not mixing averages from the linear and non linear aspects of the equations the sum of the energy budget is the same WITH or WITHOUT a GHE. Thus:
        Predicted earth emission without a greenhouse effect: 239 W/m²
        Actual earth emission WITH a (supposed) greenhouse effect: 239 W/m²

    • Jim,

      How do you define CAGW? Are you saying that 3 or 4 degs of warming would be catastrophic?

  30. “Since I am no longer affiliated with any organization nor receiving any funding, I can speak quite frankly….As a scientist I remain skeptical…The main basis of the claim that man’s release of greenhouse gases is the cause of the warming is based almost entirely upon climate models. We all know the frailty of models concerning the air-surface system.”

    ~Dr. Joanne Simpson

  31. Claes Johnson

    Judy, you write a post

    ” to untangle the Skydragons’ misunderstanding about the greenhouse effect and the planetary energy balance”

    and when I ask you a question directly related to my “misunderstanding about the greenhouse effect”‘, then you refuse to answer.

    Is this how a serious scientific debate is to be performed? Why are you not answering my question? It is a serious question about the basics of climate alarmism and I expect a serious answer. If you are nor willing to answer you should stop criticizing my work in public.

    • Claes,

      Sorry, but you have now earned the right to be ignored. We have had a whole thread (and overlap with several others) devoted to your nonsense, which hardly qualifies as a ¨scientific debate.¨ You have proceeded to distort Judith Curry´s word on your own blog. Sorry.

      • Chris, I have been advised that you were a student of Dr. Curry’s. Is this correct and does it color your comments towards Claes?

      • It is not correct. Also, I have disagreed with Judith Curry on numerous occasions but she is absolutely correct on this issue.

      • Dr Curry has recently conceded that back radiation doesnt exist in any thermodynamics textbooks,nor in the laws of physics. It is merely a hypothetical construct of the IPCC and she has affirmed she will no longer use the term. I’m pleased she’s slowly learning. When are you going to renounce such myths?

      • Wrong, i stated that the term back radiation is misleading terminology, because many people (like you) seem to misunderstand it. infrared radiation emission by molecules such as CO2 and H2O definitely exists. THe IPCC is correct in this regard.

      • The entire ridiculous and unsubstantiated 200 year old nonsense concept of the so called “greenhouse effect” is entirely based on thoroughly misleading terminology.

        More commonly referred to as sophistry, fallacious arguments with the intent to deceive.

      • Judith, I disagree. It’s people like you who extol the work of Tyndall who are so off base. Tyndall was completely ignorant of the fact that absorption does not equate to warming. The American Meteorological Society refuted Tyndall’s ideas when they published their debunk of the GHE in 1951. Radiation is not the same as heat. Don’t you know these things?

      • John – Hopefully, we are all students of Dr. Curry’s.

      • Fred, so that’s your wish to see the blind following the blind?

      • Even if this were the case (and as Chris points out, it isn’t) it couldn’t possibly eclipse your blind defense of Dr. Johnson. Your ridiculous insinuation just points out, yet again, how little you people understand about how real scientists conduct themselves. Scientists argue – often, and strenuously. Yet they generally engage on a level playing field and discuss the issues using a common language and relying upon an implicitly agreed-upon starting point; they don’t just blather on about their new theory that nobody else is smart enough to comprehend.

        Disagreement, and the desire to prove the other person wrong, is how science progresses. Almost every single published paper contains some flaw, but the overall progression is always towards the correct answer and often pays societal dividends along the way. Goodness knows there are roadblocks, and egos intrude – oh yes, they surely do. But there are no Galileos anymore, nor are there Dragon Slayers. But good luck with that.

      • I admire Dr. Johnson because he’s thinking deeply and trying to understand and clarify our understanding of radiative physics…taking a fresh approach and revisiting our assumptions rather than regurgitating formulas memorized from a textbook. Every formula is an approximation and a tool for describing an underlying reality. The formula is not the reality. Don’t believe what you are told…examine the data and assumptions and think things through.

      • I’m sorry Ken, but anyone who says that the equation P = sigma(To^4 – T1^4) represents something different that P = sigmaTo^4 – sigmaT1^4 is talking bullocks and is not thinking clearly.

      • Rattus,
        It is you who is not thinking clearly. The P/4 argument postulated by Postma is entirely valid because it demonstrates that the standard GHE model, which omits the existence of night and day is utterly flawed. Postma proves that going back to first principles and factoring in the energy inertia and insolation of night and day plus adiabatic physics explains the missing 33K of heat that GHE religionists say is accounted for by their mythical GHE.
        Postma’s numbers prove that abandoning P/4 and the mess it creates by mixing linear and non linear numbers helps us find that ‘missing’ 33K such that we no longer need to factor in any GHE to still get the same total energy budget. Thus we see the following:
        Predicted earth emission without a greenhouse effect: 239 W/m²
        Actual earth emission WITH a (supposed) greenhouse effect: 239 W/m²

        Thus the GHE is a tautology premised on the back radiation meme which can be now be discarded by application of the principles of Occam’s Razor i.e. if you can explain reality without it then abandon it.

  32. Let all ye who are ideologically pure reconcile the following:

     “The only current ‘evidence’ for blaming carbon emissions are scientific models… Historically, science has not progressed by calculations and models, but by repeatable observations. Some theories held by science authorities have turned out to be spectacularly wrong… For excellent reasons, we have much more confidence in observations by several independent parties than in models produced by a small set of related parties!” ~David Evans

     “To give an idea of how strong this enhanced cooling mechanism is, if it was operating on global warming, it would reduce estimates of future warming by over 75 percent. At least 80 percent of the Earth’s natural greenhouse effect is due to water vapor and clouds, and those are largely under the control of precipitation systems. Until we understand how precipitation systems change with warming, I don’t believe we can know how much of our current warming is manmade. Without that knowledge, we can’t predict future climate change with any degree of certainty.” ~Dr. Roy Spencer

     “All temperature and weather observations indicate that the earth isn’t like a greenhouse and that there is in reality no ‘natural greenhouse effect’ which could warm up the earth by its own emitted energy and cause by re-emission a ‘global warming effect’. With or without atmosphere every body looses heat, gets inevitably colder. This natural fact, formulated by Sir Isaac Newton in his ‘cooling law’, led Sir James Dewar to the construction of the ‘Dewar flask’ to minimize heat losses from a vessel. But the most perfect thermos flask can’t avoid that the hot coffee really gets cold. The hypothesis of a natural and a man-made ‘greenhouse effect’, like eugenics, belongs to the category ‘scientific errors,” ~Dr. Wolfgang Thuene

  33. I have a question, most probably very stupid, but I’ve tried a couple of times, without a real answer. I’ve seen it asked at WUWT, also with no real answer.

    Two planets. Both are pretty much like Earth, same position, same albedo, same day and night speed, everything equal, but without atmosphere. One of them magically maintains oceans, like Earth. Maybe it has a plastic cover over them, or something. The other one is a rock planet, without water.

    The question is, would we expect them to have the same mean temperature, or to see a significant difference. In the later case, why?

    Sorry if it’s been answered and I lost it. I have read quite a few comments at the many Skydragon’s threads, but far from all. Not even all in this thread.

    • Alexander Harvey

      plazaeme,

      I can’t guarantee that I could answer but first could you indicate the motivation. Are you concerned about heat transport which the existence of the ocean may permit, or are you concerned about IR emissivity which is likely to be higher for the ocean than for a rocky surface, or something else?

      Alex

    • Nullius in Verba

      Differential heating between equator and poles would drive ocean circulation, which would spread heat more efficiently around the globe, and thereby cause it to radiate less efficiently. (The T^4 radiation law means that hot spots can increase outward radiation far more than the corresponding cold spots reduce it.) The planet with oceans would be slightly warmer on average.

      Although given that the average surface temperature would be -20 C, you would need some more magic to stop it freezing solid and turning into your rocky body solution.

    • Thanks a lot for the answers.

      I was beginning with a liquid oceans planet, and then letting it go its way. And I thought there was a difference from the fact that the sea cools quite less at night, so it has more temperature when the sun comes next morning.

      If greenhouse effect is open doors to incoming photons, and semi closed doors to outgoing ones, I sea a lot of it happening at the oceans.

    • plazaeme,

      It´s not stupid; I like thinking about these hypothetical situations. It lets you understand climate a lot better, and has some practical significance for understanding the growing field of extrasolar planetary atmospheres, and the myriad configurations of climates that can arise.

      An experiment comparing aquaplanets and dry planets similar to the one you are describing has been done in a few studies, one of which is Manoj Joshi. Astrobiology. June 2003, 3(2): 415-427 Climate Model Studies of Synchronously Rotating Planets¨, although the aquaplanets here have evaporation and precipitation, and thus oceans interact with the atmosphere. The authors also had in mind slowly-rotating planets around M-stars
      http://www.liebertonline.com/doi/abs/10.1089/153110703769016488

      I don´t think the mean temperature would deviate too much, but the absence of any oceans leads to higher day–night temperature differences, and could get incredibly hot at the sub-solar point, and incredibly cold on the nightside. With rotation (or high eccentricity) you have to consider the length of day (year) relative to the thermal response time of the ocean.

  34. Judith
    Please check equation 12.1a
    The exponent “2” appears to be missing from “r” on the right side.
    i.e. area of a sphere = 4 * Pi * R^2.
    As is shown correctly at the end of the following paragraph.

  35. Ignore him Claes, your question wasn’t addressed to him anyway. :)

  36. As Claes Johnson has appeared, it’s perhaps worth examining that he even claims that radiometers don’t measure radiation correctly (though he calls them “IR cameras”.He says:

    “The instrument reading of 360 W/m2, is thus incorrectly interpreted as a reading of a real physical flow of heat energy of 360 W/m2, by letting the instrument calculate the flow of energy from measuring the temperature, using a physically incorrect interpretation of SB
    IR detectors or sensors used in IR cameras are sensitive to the frequency of the incoming radiation, from which the emission temperature is calculated (using Wien’s displacement law) and from the emission temperature the emitted radiation is calculated (by SB)”
    http://claesjohnson.blogspot.com/2011/01/correct-interpretation-of-stefan.html

    Wiens Law connects three factors – wavelength, radiative flux, and temperature.. To calculate any one, values for the other two are needed. In fact of course, radiometers don’t need either SB or Wien’s Law to “calculate” radiative flux, as they measure it directly, using photo-electric sensors, just as light meters do. Apart from the fact that measuring the frequency of light is extremely difficult, it would be useless on its own to input to an equation with three variables.,

    He’s claiming that radiometers (he never uses the word) don’t measure the radiation from surface (in the example in his post), or atmosphere to surface correctly because the reading is calculated incorrectly from frequency via temperature, using Wien’s and Stefan Boltzmann’s laws. He seems .quite happy to accept values for solar flux and TOA flux to space however, though these are measured using fundamentally similar radiometers on satellites.

    I know how radiometers work – I’ve used them. They only measure one thing, and that’s radiative flux. They certainly don’t measure frequency, and they don’t need to calculate temperature either. Precision radiometers as used on satellites and in ground-based measurements are calibrated continually using one or more reference black-body sources at known temperature, which therefore radiate a known amount. His claims about Wiens Law and SB being used are designed to “blind with science”.

    • Hey Tony that is very interesting. It seems to me they show only visual(human eye spectrum/frequency) objects like clouds or other solid objects.

      Why are thermal images not blocked or clouded by the air and green house gases? Hopefully this is not too off topic. Just curious

      • Most thermal imagers (at least the ones anyone not richer than Warren Buffet can afford) work in what is called the near IR which, extends from about 800 nm to about 3-4 microns (3000-4000 nm). Absorption by greenhouse gases is very weak in those regions until you get to the hydrogen stretches in water vapor and emission from the surface @ 300K peaks about 10 microns and longer.

        It’s a feature, not a bug

  37. Most skeptics do not agee with the dragon slayer book, so we can be in agreement with the believers on this one.
    Since the topic has now become more of kicking a dead dragon than actually discussing new ideas, I wonder why the many posts on it?

    • Most skeptics do not agee with the dragon slayer book…

      hunter – do you have some way of quantifying this? I see claims being made to that effect, and I didn’t follow the original thread, but when I read through the Postma post, if you factor out the “warmists,” I don’t see a clear majority of “skeptics” that rejects Postma’s analysis. In fact, I’d say that if you exclude the “warmists,” the # of “skeptics” that seem receptive to Postma’s thesis is higher than the # of “skeptics” that reject it.

      I see those “skeptics” who reject such theories proclaiming that they represent the majority of “skeptics,” – but I don’t see evidence that clearly backs such an assertion.

      If you could provide some evidence, I’d appreciate it.

      • Personally, I found Postma’s thesis stronger in some aspects than others. For a sceptic, parts of a thesis might contain useful concepts applicable elsewhere. For a ‘supporter of the consensus’, a single fault gives grounds for wholesale rejection.

        Differing priorities lead to different perceptions of the usefulness or otherwise of the contributions put forward.

      • I’m highly dubious of black hat/white hat portrayals of the different camps in the debate (no matter which side they emanate from).

        You characterize the anyone who is a “supporter of the consensus” in a completely negative light, and anyone who is a “skeptic” in a completely positive light.

        Sorry, tallbloke – but I can’t look at such an opinion and not conclude you’re confirming your biases.

        To be more specific – a single fault, depending on the severity of that fault, can be sufficient reason for an entire structure to collapse. Further, I’m not sure that you’ve fully defined what a “consensus supporter is,” as some of the major contributors to the Postma thread are non-skeptics who align to varying degrees with “the consensus” (another unspecified term),.Finally, there were non-skeptics in that thread who found multiple lines of (in their opinion fatal) flaws in Postma’s analysis.

        And I gotta say, tallbloke – sometimes your analysis seems cogent to me – but when you throw up such facile conclusions, I can’t help but question how thorough you are in your reasoning process.

      • Well Josh, don’t read too much into a couple of hastily constructed sentences. I just got tired of writing theses on that sort of stuff.

        Solving the climate puzzle is far more fun.

      • Lets be clear TB.

        You have a “theory” of how the sun controls the climate and is the main driver of everything that happens. More precisely you have a theory that argues that the planetary alignments drive sunspots and sunspots drive the climate.

        To maintain this theory you are required to:

        1. Accept CGR theory before it has been established
        2. Discount the role of GHGs in any way shape or form.
        3. rely on the uncertainty inherent in the TSI record.

        That’s clear to me from our discussions. You lean heavily on some of the uncertainties (#3) hope for the unproven (#1) and disregard the well established (#2) in order to keep your “theory” alive.

        I mean no dis respect but that is how I see it. I know you love your work and your pet theory. we all have them. But that, on my view, really drives your evaluation of things.. even to the point of accepting things like reanalysis data that rely’s on #2.

        Anyway, just a view from outside. Hope all is well

        ps. the hacker showed up at CA today. ya. he. did.

      • TSI is not a measurement of actual amounts of bright sunshine reaching the surface of the earth.

        Bright sunshine is up by more than enough to account for any warming.

      • To maintain this theory you are required to:

        1. Accept CGR theory before it has been established
        2. Discount the role of GHGs in any way shape or form.
        3. rely on the uncertainty inherent in the TSI record.

        Mosh, I know what dependencies my own climate theories have. You have it wrong.

        1) There are other mechanisms which contribute to the solar activity level – albedo link which is evident in the historical data apart from a potential GCR link.

        2) I think the GHG’s are very important, especially the one which constitutes the vast majority of the effect.

        3) TSI is only part of the story.

      • 機関士:何者かによって、爆発物が仕掛けられたようです。

      • Maybe you can answer. Why does Postma’s gas only radiate upwards?

      • Jim, ‘Postma’s gas’ as you call it radiates in all directions but the transport of energy (heat) is always from hot to cold. Do you see the difference? It is why Postma’s calculations show the movement of heat outward to space as per the laws of entropy.

      • OK, I see his Fig. 1 now, and he does have that downward arrow to the ground with Ta. You would call this “back” radiation, I suppose. The curious thing about his budget compared to the real one by Kiehl and Trenberth, is that he neglects anything like heat fluxes from the surface.

    • Nullius in Verba

      To ensure that everybody understands why.

      Analysing fallacies and paradoxes is a worthwhile activity, because it exposes common misunderstandings, and motivates the refinement of explanations into ever simpler, more elegant, and more precise statements. A complicated technical mathematical proof may prove the point, but developing a simpler, more intuitive explanation can give deeper insight into the physics. I suspect many of the best insights into thermodynamics were motivated by debunking the many plausible-sounding proposals for perpetual motion machines.

    • Hunter argues first from opinion and then wonders why the facts don’t agree with him. A more rational person would start from the fact that over 5,000 comments about the Slayer science appears on this blog alone making it by far the most popular single topic. Clearly, there are far more people interested in addressing this issue than he is comfortable with. I wonder why?

    • Hunter,

      If you are in agreement with the ‘believers’ , then you must be a ‘believer’ too!

      So if you ‘believe’ in a natural 33 deg C of warming, what’s so hard about ‘believing’ that doubling CO2 levels will add another 3 deg C?

  38. Jeff Glassman said: “Back radiation is not heat…”

    This statement is inconsistent under the definitions of ‘thermal radiation’ and heat’ given in the Wikipedia. The Wikipedia specifically gives thermal radiation as a form of heat.

    Since we can’t make progress in clearing up the misunderstandings about the greenhouse effect if we don’t define consistent terms that must form the very basis of any true understanding, what are you trying to accomplish?

    On the other hand, I understand and fully support what Dr. Curry is trying to accomplish.

    • George Crews |
      Wait till Joel shore returns.

      He will explain why radiation from a colder to a hotter surface cannot be called heat.

      “The Wikipedia specifically gives thermal radiation as a form of heat.”

      This is why unfortunately, Wikipedia is not a reliable source.

      • Guys,
        THERMAL radiation is indeed radiation that is transporting HEAT but radiation per se does not necessarily provide heat otherwise my TV satellite dish would get far hotter.

      • “…..radiation per se does not necessarily provide heat otherwise my TV satellite dish would get far hotter.”

        The signal level into your TV satellite receiver will be something like -90dBm ! Yes, it will produce heat but its just such a tiny amount that I wouldn’t hold your breath waiting for a potato to cook, should you try putting one in the focal position of the dish! However you might just survive long enough if you put one into a microwave oven.

        Incidentally, this reminds me of one of the more amusing sceptical theories on why the Earth has warmed:

        http://globalmicrowave.org/

    • “Thermodynamically, energy can only be transferred by heat between objects, or regions within an object, with different temperatures…”
      http://en.wikipedia.org/wiki/Heat

      It takes two for heat.

      • Being precise with the definition of heat was important when all thermodynamics was classical thermodynamics. The present day thinking is commonly based on statistical thermodynamics and the microscopic processes are also often considered. That has led to the situation, where thermal energy can be described as something having a absolute value and a minimum of zero, when nothing moves (at the temperature of 0K, and I’m purposefully dismissing the special properties of fermions). Now it’s not anymore essential to say that heat is something that is only defined through its transfer.

        As we have now the other alternative, and as that other alternative tells more and is also a more natural way of thinking, there’s not much reason to be dogmatic with the usage of the word heat. It’s now common that people, who know thermodynamics very well and who use it regularly in their work, use the word heat caring little about the dogmatic requirements based on long gone history.

      • I’m not dogmatic about that. I also use for example “heat content” or similar. But regarding heat transfer, it’s always the NET radiative transfer between bodies (or across system boundaries) that should be balanced. One-way thermal radiation is not heat.

      • The issue has two sides. One is the one you prefer.

        The other is supported by the fact that the net radiative heat transfer is very often calculated as the difference of two terms, which describe the amounts of energy that move in each direction. When our simple formula is of such a form, it’s very natural and practical to describe those terms as radiative heat transfer from A to B and from B to A. That doesn’t lead to problems or misunderstandings unless the reader or listener is dogmatic.

        I know that a lot of confusion has been created in net discussions from that, but a major part of that is done by purpose to perpetuate such confusion.

      • I see your point Pekka. IMO, confusion is created when radiative heat transfer is described as only from A to B and from B to A is overlooked. It happens. It can mess up the balance.

      • It’s indeed important to include both. Not doing that properly has led, e.g., to many erroneous claims that increasing the DLR cannot cause the ocean SST to rise. (I avoided by purpose the formulation that increased back radiation can warm the ocean, although that is used to mean the same thing.)

      • More like 3 dimensional billiards – with a mean free photon path of 50m. The net flux of IR is outward – it makes physical sense to think of it in this way.

        If there is more CO2 the mean free path decreases – more heat is retained in the atmosphere and the net flux from the surface up decreases.

      • spartacusisfree

        Correct, and ‘climate science’ has forgotten this basic axiom. ‘Back radiation’ cannot exist on its own because if you do separate the forward and back ward radiative energy fluxes [impossible in reality because the two are coupled quantum-mechanically through the respective density of states], the forward flux increases by exactly the amount supposed to be ‘back radiation’.

        Perhaps ‘climate scientists’ should learn about the Law of Equipartition of Energy and realise that when an IR photon is absorbed by a greenhouse gas molecule, another is near-simultaneously emitted and this cascade occurs right to the top of the atmosphere.

        Postma’s ‘ensemble’ is truly the whole system.

      • Yes, heating has gone postmodern. People talk about back radiation heating the ocean when the net longwave flux is actually cooling the ocean…

      • Pekka Pirilä 8/19/11, 4:50 pm

        PP: Being precise with the definition of heat was important when all thermodynamics was classical thermodynamics. The present day thinking is commonly based on statistical thermodynamics … .

        You must be a Popperist:

        They all asserted that there cannot be such a thing as the correspondence between a statement and a fact. This is their central assertion. They say that this concept is meaningless (or that it is undefinable, which, incidentally, in my opinion does not matter, since definitions do not matter) In other words, the whole problem arises because of doubts, or scepticism, concerning correspondence: whether there is such a thing as a correspondence between a statement and a fact. Popper, K., “Objective Knowledge: A Realist View of Logic, Physics, and History”, [reported in “Objective Knowledge (1972), Clarendon Press] 1966, Section 4: Realism in Logic, p. 24.

        And that’s part of the reason why scientific theories must have a falsification clause.

      • Concerning my views on the philosophical approached that I have looked at with any care I have not met any philosopher with no valid thoughts. That includes many pairs of philosophers, who disagree with each other. Popper has many valid thoughts, but I’m certainly not a Popperist. Many of his statements on the significance of falsification are too restrictive at least, when they are interpreted as they commonly are. With such interpretation Popper offers fuel for anti-science skeptics in web discussions. They use Popper to support their view that some valid and valuable science were not science at all.

      • Pekka Pirilä 8/20/11, 4:22 am, Planetary

        I called you a Popperist because your argument wasn’t based firmly in definitions.

        Popper’s first step out of the box went wrong when he modeled scientific models as: For all x{F(x)}. This, if it were true, would be the reason the counter examples run to swans, ravens, and crows. Falsification would be required, an experiment to show that: Not for some x{not F(x)}. Science doesn’t declare all swans will be white. Instead, the definition of that genus is white color. If it’s not white, it’s not a swan, or not that kind of swan. Popper said, “Definitions do not matter.” Wrong again!

        Scientific models are better modeled as: For all x {H(x) -> C(x)}. H(x) being the hypothesis or observed initial conditions, the domain, and C(x) the conclusion to be observed, the prediction with a range. C(x) has an accuracy prescribed, a probability distribution, and when we observe H(x_0) and not C(x_0), H(x) is modified objectively to exclude x_0 or we widen the probability distribution.

      • Science is of course based on precisely defined concepts, but some concepts that were essential in the 19th century are less essential now.

        What is “heat” isn’t hot anymore.

      • Pekka says
        ……”What is “heat” isn’t hot anymore.”…….

        What is your motivation for encouraging people to be “sloppy” with respect to the definition of heat?
        Is this fudging completely necessary to support the greenhouse theory?
        Certainly his type of reasoning is very common with supporters of that theory.

        Your argument seems to follow this line.
        For classical thermodynamics a strict deffinition of heat is required.
        From a statistical mechanical viewpoint the definitions don’t matter.

        Why is it that thermodynamics textbook writers like Zemansky, Bauman and Feynman say exactly the opposite.
        They say you need to be very careful and consistent with these fundamental definitions.
        Perhaps you could illustrate your reasoning by giving us your own personal definition of heat.

      • There’s nothing wrong in using precise language, but making arguments on sloppier statements that people have presented, is sophistry.

        It’s reality nowadays that the word “heat” is used in sloppy fashion by layman and professionals alike. When a word is used so, it’s not right to interpret the sentences as if they would use the precise definition of the 19th century thermodynamics for “heat”.

        Sophistry is one of the common forms of fallacies of argumentation on this site. Applying sophistry to the use of word “heat” is a common case of that.

      • Pekka says

        ……”Sophistry is one of the common forms of fallacies of argumentation on this site. Applying sophistry to the use of word “heat” is a common case of that.”……

        Perhaps you don’t realise it but you are the one practicing sophistry here.
        Encouraging people to use definitions in an ambiguous way is a classic example of sophistry.

        Multiple contradictory meanings for the same word.

        Since you seem unable to supply your own definition then heat can mean anything or nothing.
        The more interesting question then must be answered.
        Is it essential to fudge the meaning of heat for belief in the greenhouse theory

      • My encouragement doesn’t have much effect on the way people use words.

      • Pekka Pirilä 8/20/11, 4:55 am, Planetary energy …

        PP: My encouragement doesn’t have much effect on the way people use words.

        Nor your discouragement. But don’t feel like the Lone Ranger. We’re all riding on Silver.

        You say you are not a Popperist, and then launch into a defense for not relying on definitions. Bryan is right, you are a Sophist, as well as a Popperist.

        You let common misuse of language change your science. That violates the principles of science embodied in the scientific method.

      • Glassman said:

        C(x) has an accuracy prescribed, a probability distribution, …

        Which is what statistical mechanics is all about. People rely on classical thermo because it doesn’t require you to think about probability distributions. Pekka is just saying that using real statistical arguments is a more fundamental way of thinking and can get you much further along. I almost always think about entropy as in information theory, as the integral of a function of probability.

      • WebHubTelescope 8/21/11, 11:31 am, Planetary energy …

        WHT: Which is what statistical mechanics is all about. People rely on classical thermo because it doesn’t require you to think about probability distributions. Pekka is just saying that using real statistical arguments is a more fundamental way of thinking and can get you much further along. I almost always think about entropy as in information theory, as the integral of a function of probability.

        I wouldn’t want to suggest that predictions include a probability distribution is in any way unique to statistical mechanics. It applies to the most elementary models in electricity and all its analogs in first year physics.

        I particularly liked this quotation:

        During the 20th century, the laws of mechanics have been [were] profoundly modified by two major revolutions in our understanding of natural sciences: quantum theory and relativity. The laws of thermodynamics, instead, have survived both revolutions, unaltered and strengthened. The deepness and generality of the principles of thermodynamics, and the importance and conceptual difficulty of its enigmas have enflamed the minds of all principal developers of modern physics. For these reasons thermodynamics is certainly not a dead subject, but one of the most lively scientific disciplines: a perpetual generator of scientific thinking and technological progress.

        While the importance and the empirical successes of the laws of thermodynamics and their applications have never been questioned, their profound physical significance and domain of validity have been and still are at the center of several debates and controversies among the different schools of thought.

        http://www.quantumthermodynamics.org/

        The miracle of statistical thermo is not that it replaced the classical, but that it still agrees with it. If analysts would respect scale, their modeling would be greatly simplified. They rarely visit classical thermodynamics and its simplifying and linearizing properties, all because its domain consists of macroparameter state variables.

      • The miracle of statistical thermo is not that it replaced the classical, but that it still agrees with it.

        I would say it is exactly the opposite:
        The miracle of classical thermo is that it substantiates the statistical physics. It does this because of the properties of the Boltzmann distribution and the mathematical properties of the exponential & natural logarithm. The work of Jaynes is useful to understand this.

      • ” there’s not much reason to be dogmatic with the usage of the word heat.”

        There was always dogma in this area for a specific reason. That reason was so as to be careful as to not confuse cause and effect. Something to which we must all remain vigilant lest we become lead astray.

        Radiation is the cause, heat is often, but not necessarily the effect.

    • George Crews 8/19/11, 3:51 pm

      GC: Jeff Glassman said: “Back radiation is not heat…”

      supported by a thought experiment for why the same back radiation seems to heat one thing while cooling another. I also provided definitions for what IS relevant heat for Fred Moolten (8/13/11, 4:47 pm, Slaying etc, Part IV thread):

      The gain or loss of internal energy, equal to the difference between the energy of the thermal radiation which is absorbed and that which is radiated, is called heat. Bold added, Zemansky, M. W., Heat and Thermodynamics: An Intermediate Textbook for Students of Physics, Chemistry, and Engineering McGraw-Hill, Fourth Ed., 1957, p. 105.

      We have for the heat transferred by radiation between a body at the temperature θ and walls at θ_W, is Q_dot = A*α•σ*(θ^4 – θ_W^4), where α refers to the temperature θ. Id., p. 106. Note: the radiation from the walls of the assumed enclosure is the back radiation.

      • spartacusisfree

        The use of the S-B equation as written by Zemansky to determine radiative heat transfer rate is perfectly correct. This is because the IR emitting density of states in the hotter body is linked quantum mechanically to that in the colder body and the equation gives the net energy transmission.

        Whilst it is easy on paper to split the equation into the hot and cold components, with the latter being ‘back radiation’, this is physically incorrect.

        (1) a colder body cannot transfer energy to a hotter body, a corollary of the 2nd Law of Thermodynamics: it’s all about entropy.

        (2) in practical terms, by separating the two terms, the radiative energy from the hot body has increased exactly by the amount of ‘back radiation’, so the latter is automatically cancelled out.

        The fact is, greenhouse gases cannot store energy. They simply increase the IR optical path length/optical depth. There may be some thermalisation, but that isn’t guaranteed at all places in the atmosphere.

        In practical terms adding CO2 to the atmosphere is likely to have a net cooling effect because self-absorption will give a negative emissivity. ‘back radiation’ becoming ‘forward radiation’.

        ‘Climate science’ doesn’t have enough basic physics so it’s got to unlearn the rubbish and rebuild.

    • George, you are confusing radiant energy with heat. All objects above 0K radiate but that does not mean they emit heat. My satellite dish receives radiation but it does not heat because it not receiving thermal radiation. THERMAL radiation is not ‘back radiation’ because THERMAL radiation, by definition is radiation that DOES have temperature. Please don’t mix apples and oranges .

      • spartacusisfree

        All absorbed photons heat the absorbing matter.This applies no matter what the frequency and no matter what the mechanism of absorption..

      • spartacus, how much does the ice in my freezer ‘heat’ the fridge? The laws of entropy tell us you only need to address the end output. Dispense with the sophistry please.

      • The freezer transports heat away from the ice – which is why it remains frozen. But in some equilibrium state it is mostly heat leakage into freezer. The freezer stops the ice from melting of course.

        In outer space the ice block would continue to cool to absolute zero – minus 273 degrees C. Entropy at work.

  39. The boffins of Japan understand that Western scientists who believe in the ‘greenhouse effect’ are the “astrologers” of climate theory and that they all deserve an “F” in Climate History: “climate change theory is still dominated by anthropogenic greenhouse gas causation; the IPCC 4th Evaluation Report’s conclusion that from now on atmospheric temperatures are likely to continuously, monotonously increase, should be perceived as an unprovable hypothesis.” (Kanya Kusano)

  40. I’m curious about what Dr. Curry means by this statement…

    “If radiative transfer is the only process occurring in a planetary atmosphere, the surface temperatures of the planets would be determined solely by the net radiation at the top of the atmosphere.”

    Does this mean we can estimate the surface temperature based on the measured TOA radiation? I am hoping JC is not saying the surface temperature is controlled by the TOA radiation.

    • no, read the word “If” and then read the rest of my argument.

      • Sorry, but I still don’t understand what this sentence means…and I think my confusion comes from alternate meanings of the word ‘determines’.

        Is there a synonym that could be used to clarify which sense you mean?

        Possible synonyms from the Bing dictionary:

        find out, verify, ascertain, clarify, uncover, establish, decide, settle, conclude, resolve, agree, finalize, influence, affect, shape, form, mold, control, regulate, govern, fix, limit, define

      • As Sherwood Thoele observed, If atmospheric CO2/H2O absorbed infrared radiation from the Earth and did not let it pass through—thus creating a ‘greenhouse effect’—then, atmospheric CO2/H2O would act in the same manner whereby infrared radiation from the Sun would not get through to the Earth either.

      • Wagathon,
        The GHE religionists would have us believe that CO2 ‘knows’ the difference between inward and outward energy and only ‘traps’ the energy going outwards to create their beloved GHE.

      • Nullius in Verba

        I think your confusion comes from the meaning of the word “If”.

        If the Earth’s surface temperature was -100 C then the oceans would freeze. That’s a true statement. But does that mean that the Earth’s oceans have frozen? Is it some confusion about the meaning of the word “freeze”, perhaps? What do you think?

      • You might possibly find out that the Earth’s surface was frozen but you would have to verify and ascertain with some clarify as to whether the oceans were likewise frozen and then uncover and establish what the temperature of the frozen ocean actually was before you could decide, settle and conclude that you had resolved whether it was the surface of the Earth that influenced the ocean or perhaps agree that it might be possible that the temperature of the ocean influenced the surface of the Earth before you can finalize the matter as to which influence had what affect and by what manner of shape or form of the mechanism of molding or control or regulating might govern, fix, or limit the influence according to some definitioin, right?

      • Nullius in Verba

        :-)

        You would quite possibly face people telling you that the oceans are not frozen, so you must be lying when you say that IF the surface was at -100 C the oceans would freeze, and of course the oceans would stay liquid at -100 C because they couldn’t radiate the heat away, being opaque to IR.

      • Assuming that ice has heat conductivity same as typical rock, at 30C/km gradient and -100C surface, ocean would be unfrozen already 3 – 3.5 km beneath surface. Since average depth of the ocean is around 5 km, most of the ocean would exist in ‘european’, undersurface but still liquid state.

      • Ken, Dr. Curry recently disavowed herself of ever using again the term ‘back radiation’ in serious scientific discussion. Perhaps there will soon be other words that need semantic clarification in the Curry Dictionary of Climate Fudge.

      • John, would you take an educated guess of a synonym to clarify what she’s saying? Anyone else? I’m genuinely curious.

      • Nullius in Verba

        She said “If radiative transfer is the only process occurring in a planetary atmosphere”, but radiative transfer is obviously not the only process occurring in a planetary atmosphere, so you can ignore everthing after that point from the point of view of actual facts.

        It’s a thought experiment, a hypothetical mental model, to help one understand the physics by considering it one piece at a time. Interpreting the thought experiment as a statement on reality when the statement itself makes clear that it isn’t is perverse.

      • But we agree–for a planet racing through nearly-empty space, there is nothing but radiation. Radiation in and radiation out (ignoring internal heat sources). I hope you’re not telling me this is one of those wonderful thought experiments like Pierrehumbert’s 800,000K temperature. Please don’t tell me that’s what she’s saying. Please tell me she means “determined” like “ascertained”. Please.

      • I hope you are not guilty of overinterpretation.

      • Nullius in Verba

        For a planet racing through near empty space, there are many different temperatures. The temperature at the top of the atmosphere, the temperature at the average altitude of emission to space, the temperature at the surface, near the poles, near the equator, the temperature deep below the surface. For most of these, it is not true that there is nothing but radiation. The relationship of one to another depends on far more than just radiation.

        Trying to understand everything at once leads to confusion. The climate system is tremendously complex. If she tries to explain it all, people misunderstand, and if she tries to explain it one bit at a time, people object that the model is unrealistic because of all the things she omits.

        In this case, she tried to explain that she was omitting most of the details, so as to explain this one aspect of the physics, and you still took it as purporting to be some sort of complete explanation. I don’t see what else she could have done.

      • NiV, is it safe to say you interpret her sentence as so:

        If radiative transfer is the only process occurring in a planetary atmosphere, the surface temperatures of the planets would be established solely by the net radiation at the top of the atmosphere.

      • Nullius in Verba

        I would say if radiative transfer is the only process occurring in a planetary atmosphere and the optical depth of the surface is considered fixed, the surface temperatures of the planets would be a function of the net radiation at the top of the atmosphere.

        The implicit assumption is that other factors are held fixed, but it could be made more explicit which such factors are important, and this way of phrasing it leaves out considerations of what is cause and what is effect. Does that help?

      • Hmmm…we’re getting closer, but I’m trying to figure out if the thought experiment suggests:

        A) radiation controls the surface temperature

        -or-

        B) surface temperature controls the radiation.

        This touches on Claes’ comments about the form of the equations. You can manipulate equations and move variables around, but the physical process is not so flexible.

      • Nullius in Verba

        “but I’m trying to figure out if the thought experiment suggests: A) radiation controls the surface temperature -or- B) surface temperature controls the radiation.”

        Ah! A much clearer question. Unfortunately, there isn’t a clear answer – it goes both ways.

        If you read on, the point she was trying to make was that at the top of the atmosphere the equator receives more radiation than the poles, and if the only way it can get away is by radiation, then approximate energy balance at the top of the atmosphere requires the surface temperature to be hotter at equator than poles to supply it. So incoming radiation causes initial atmospheric/surface temperature causes outgoing radiation causes net balance at top of atmosphere causes increase or decrease of temperature causes a feedback tendency towards equilibrium which cycles forever to keep temperature and radiation tracking any perturbations together. (It’s analogous to the question of a mass bouncing on a spring – does the motion of the mass determine the spring’s stretch, or does the force applied by the stretched spring determine the mass’s motion?) For explanation of that particular point, I suppose, it might have been better if she’d said it was a function of the incoming radiation rather than the net, although it would require more caveats to make it strictly true.

        I doubt Judith thought about it that carefully. All she wanted to note was that in thin atmospheres the surface at the equator is hotter than the poles, but in thick atmospheres the surface is the same temperature everywhere, because non-radiative stuff is spreading it around horizontally.

        Again, from the point of view of explanation, it might have been better for Judith to avoid begging the question and call the times t_vertical and t_horizontal, rather than t_rad and t_dyn. That would then be true even with non-radiative vertical processes included, and would be justifiably applicable to real planetary atmosphere like Venus or Mars.

        The point she is trying to make in answer to Postma is that for Mars, assuming t_vertical is purely t_rad works well enough, and she acknowledges at the end it doesn’t work well for Earth because the non-radiative bits are more significant. I can see what she’s trying to do, but I don’t think the argument convinced.

      • Thank you, NiV, I appreciate the explanation. I caution people in the soft sciences to avoid thought constructs like this:

        If is true, then will happen.

        An infinite number of these kinds of undisciplined statements can be made. They contribute no useful information and do not lead to productive thinking. Of the hundred or so engineering and technical books on my bookshelf, 0 PPM of them have statements in that form intended to be taken seriously (the caveat is that you’ll find a Dilbert or Lewis Carroll quote here and there for humor’s sake).

        You’re welcome.

      • WP did not like my text in brackets. Interesting.

        What I intended to say is:

        If something ‘open-bracket’ nonphysical and impossible ‘close-bracket’ is true, then something ‘open-bracket’ absurd ‘close-bracket’ will happen.

      • Nullius in Verba

        “If something ‘open-bracket’ nonphysical and impossible ‘close-bracket’ is true, then something ‘open-bracket’ absurd ‘close-bracket’ will happen.”

        Well, yes. In this case, the oceans would explode.

        But if you pursue it far enough, that’s true of all our understanding of the world. We are not capable of containing a faithful mental model of the universe. So it doesn’t leave much for us to talk about.

      • Ken, I fear Dr. Curry is probably “too busy” to address this as she often seems to be when something inconvenient to her arguments crops up. As to what she means, your guess is as good as mine.

      • John,

        ‘I fear Dr. Curry is probably “too busy” to address this as she often seems to be when something inconvenient to her arguments crops up.’

        how can you say that with a straight face?

        Two dozen different contributors here with physical science credentials that supersede both yours and Ken’s have pointed out as many flaws with both Johnson and Postma’s work and you’ve done nothing but evade, ignore and pretend that such criticism are part of another reality.

        That you should think it wise to call others out for what you see as doing the same (despite the fact that everyone else can see you don’t even know what’s what) is further proof that you’re along the way to marginalizing yourself as a PR person more than a scientist of even publisher.

        Good riddance as far as I’m concerned.

    • We could theoretically calculate a change in Earth system heat content from TOA radiative flux. But it is not quite that simple because the absolute values of the flux is not measurable with much accuracy at all. Change (anomalies) in flux is much more easily measured and provides reasonable indications of trend – ie. relatively warming or relatively cooling.

      This one for instance – http://isccp.giss.nasa.gov/zFD/an2020_LWup_toa.gif – shows large relative cooling in the IR band in both ERBS and ISCCP-FD data in the tropics between the 80’s and 90’s.

  41. Alexander Harvey

    Hi all,

    In the case of the oceans during the daytime with no inversion layer, do people think that the troposphere:

    absorbes more energy/sec in the form of IR radiation from the surface than it emits through the tropopause plus any that it emits and is absorbed at the surface,

    absorbes less energy/sec in the form of IR radiation from the surface than it emits through the tropopause plus any that it emits and is absorbed at the surface,

    or the same amount?

    Alex

    • A study of the Earth’s albedo (project “Earthshine”) shows that the amount of reflected sunlight does not vary with increases in greenhouse gases. The “Earthshine” data shows that the Earth’s albedo fell up to 1997 and rose after 2001.

      What was learned is that climate change is related to albedo, as a result of the change in the amount of energy from the sun that is absorbed by the Earth. For example, fewer clouds means less reflectivity which results in a warmer Earth. And, this happened through about 1998. Conversely, more clouds means greater reflectivity which results in a cooler Earth. And this happened after 1998.

      It is logical to presume that changes in Earth’s albedo are due to increases and decreases in low cloud cover, which in turn is related to the climate change that we have observed during the 20th Century, including the present global cooling. However, we see that climate variability over the same period is not related to changes in atmospheric greenhouse gases.

      Obviously, the amount of `climate forcing’ that may be due to changes in atmospheric greenhouse gases is either overstated or countervailing forces are at work that GCMs simply ignore. GCMs fail to account for changes in the Earth’s albedo. Accordingly, GCMs do not account for the effect that the Earth’s albedo has on the amount of solar energy that is absorbed by the Earth.

      • Alexander Harvey

        Hi Wagathon,

        If that be an answer it would be to a different question, I ask specifically about IR radiation.

        Alex

    • Alex – In general, the OLR at the tropopause will be about 239 W/m^2, slightly higher during the day than at night, The net IR reaching the atmosphere from the surface (up minus down) will be about 60 W/m^2. Both of those figures should be corrected for upwelling IR that escapes to space without absorption, so the net effect shouldn’t be altered. This is less than the energy transported from surface to atmosphere via latent heat transfer (about 100 W/m^2). A small quantity is also transferred by conduction.

      Did you have something more subtle in mind?

      • Alexander Harvey

        Fred,

        See below with my reply to Jim, this is not about specific numbers just inequalities which I think are more fundamental. The existence of the lapse rate down to a temperature minima at the tropopause and basic thermodynamics tells us that the energy we know is entering the atmosphere e.g. latent heat for we know that it rains, cannot be exiting by conductance or convection but is not continuously building up in the troposhere, this implies that the radiative flux is divergent there, more out than into the troposhere. This implies either a low emissivity surface or downwelling radiation or an effective emissivity greater than 1 which is not permitted thermodynamics.

        I do believe that simple application of just a small subset of the physics and just those laws that are best proved and most favoured leads to the correct result qualitively and can establish a proof of the principle.

        It seems that any attempt, that stays within the laws of thermodynamics, to explain the 33C discrepancy is the GHE whether declared by that name or not. I have read the relevant part of Joseph Postma’s explanation, the lapse rate effect, and it is one of the essentials of the GHE, I can say nothing against that but urge that the train of thought is continued and the whole effect revealed.

        Alex

      • Alexander,

        I can’t find where you explain that, but if I am thinking correctly, that should be a better way of explaining the Earth system. Back radiation to the surface to some, creates two thirds of the energy impact, but that back radiation is largely due to latent heat and solar absorbed by the atmosphere. The radiation balance at the troposphere better represents the planet as a whole with the Tyndal gas effect explaining the difference between the surface and the troposphere. It has the same results, but puts the gas effect in better perspective.

      • To understand the whole effect it’s not enough to know that the adiabatic lapse rate is known and that the troposphere is about 10 km thick. It’s necessary to understand, why that’s the height of the tropopause. That’s a much more complex issue than anything discussed here recently, but that’s really the crux, and that’s very strongly related to the GHE. These issues cannot be understood separately, they must be considered as a whole for real understanding.

        Even the thick book of Pierrehumbert gives only partial answers to this essential issue.

        For this reason I don’t agree fully on the idea that the GHE shall be explained through the adiabatic lapse rate and a 10 km troposphere.

      • Pekka, I agree with a caveat. With a larger percentage of the 16% solar absorbed, all the 3% absorbed by clouds and the 10% transferred to the atmosphere trough latent, you have roughly 30% of the energy transferred to the atmosphere at or below the tropopause by means other than the gas effect due to up welling IR. The gas effect amplifies the impact. All you are doing is changing your frame of reference.

        Without the gas effect, the tropopause would be say 1 kilometer on average and the gas effect increases that to say 10 kilometer. While comparing Earth to other planets is fine, Earth is a water world with the water located mainly in the high solar intensity region. Separating out only the gas effect of water to outgoing long wave produces a more realistic model of Earth without a gas effect.

        You could say that without the gas effect the tropopause would be at zero elevation because all the water would become ice etc. etc., but no model seems to be adequate to explain the situation to the masses, a more top down model could be more generally acceptable.

      • Alexander Harvey

        Dallas,

        I haven’t explained it as yet, but I do think that some things follow from just a view principles and can be shown to by taking small steps and relying on inequalities that will hold for planets with certain plausible characteristics.

        I think that there are arguments that progress qualitatively and they suffice to show that the laws have consequences.

        These arguments are perhaps a little tortuous particularly if one is not to appeal to things that although true may not be instantly intuitive. Also somethings that it would be nice to rely upon have subtle wrinkles and in a world where one is not permitted the slightest error by failure of inclusion these wrinkles would have to be gone through.

        For instance, I could appeal to the isotropy of spontaneous emissions of radiation of molecules which give the genesis of the downwelling radiation but I would have to be careful for the entirety of all the emissions is non-isotropic due to the stimulated emissions whenever there the radiation field is non-isotropic. Should I fail to disclose this, it would be soon found out and just that one flaw used to discredit the whole.

        There are some principles such as the laws of thermodynamics that seem to have a near universal level of acceptance and I think that a qualitative description of much of the GHE can be deduced and importantly it can be seen why, where and when the GHE would produce warming.

        Throughout all these discussions I am hampered by my imagination, e.g. that I can conceive of world’s with certain exotic properties where the GHE (the results of adding and atmosphere with IR absorbing/emitting atoms and molecules) would result in a cooling of the surface. I know certain arguments if stated pedantically and insisted upon do rely on a narrow view of the of the weatlth of all imaginable planetary configuration, in turns of outcomes even though the reasoning was essentially sound.

        I think that it is possible to clarify the reasoning and attempt to categorise the implied outcomes and show when, why, and where the effects are in a certain direction and when in the opposite.

        I do believe that the physics is decidable but not in the universality of some implied outcome. A lot of stuff really does depend on niceties and I think that a lot of sane and rational people sense both that and that in many simplified explanations there is a misdirection of the eye.

        Mostly I am pondering whether a journey through a explanation that is more qualitative, more subtle and more all encompassing is a worthwhile pursuit.

        I suspect that almost everyone would hate it. Should I suggest that the GHE (in terms of the radiative physics) does not universally lead to warmer surfaces I will meet opposition from one side, and should I connect the simplest of dots and show that the laws of thermodynamics lead to downwelling radation in systems with certain asymmetries I will meet with different opponents. So in a world where nothing please everybody perhaps there is a moment when one should expose a view that pleases almost nobody.

        I think that somethings could be shown, and importantly which bits of the picture are due to which parts of our physics.

        There is some personal risk involved, that is the challenge and part of the motivation. Can one walk the highwire when people seek your falling.

        I am sorry if this is not satisfying, but I have not yet begun, I would need to go step by step and I suspect that this may not be possible on a blog.

        There is a narrative element in the sense that things need to be in sequence and the web abhors narrative.

        In a seemingly post-narrative world I do wonder what could still be achieved but I am willing. However I am slowing down and am no longer sure if I am up to it, particularly if it turns to being shouted down from all sides.

        Of one thing I am certain and that is I have a unconventional point of view and I seldom argue against things in the whole when they contain insight in some details. I also lack status, I am a commoner and I think that is a strength that compensates for other failings. I also lack a point to prove I bring only a mystery to be elucidated.

        Alex

      • Alexander said, “I suspect that almost everyone would hate it.”

        I learned a long time ago you can’t make everyone satisfied, you can make most equally dissatisfied. You may have an unconventional point of view which I believe can either be creativity or insanity. I tend to rationalize that it is creativity. In any case, a planetary imbalance energy budget at the troposphere may be more educational than an energy budget at the surface that requires model input with an accuracy of 0.85 +/- 0.15 instead of direct measurements with a accuracy of +/- 7.0 W/m^2.

      • Dallas,

        The energy balance presented by Trenberth, Fasullo and Kiehl (TFK) is not a way of learning about the net warming term. As you indicate, the accuracy is not nearly sufficient. What TFK tell is an overview of different fluxes. The net balance is known to be small. Even, if the estimate 0.85 +/- 0.15 were not correct, the correct value must be small, because the Earth has not been warming or cooling rapidly. Thus the smallness of the net value helps in determining the most inaccurate of the other numbers.

        The balance at TOA is simpler. Thus we can more easily fix the values, but we are not really interested on, what happens 10 km above us, we are interested in the future of the Earth as whole, and specifically what happens, where we live. Finally everything comes together, and nothing is really understood until all parts are understood. That leads to the complexities that Alex described, whether one agrees in detail on his assessment or not.

      • True, the accuracies and uncertainties are a distraction from the principles. A tropopause energy budget imbalance would not be easy, it would still be controversial, but I think it would be educational. It would be easier for the more radical skeptics to see that a good portion of the down welling radiation is due to basic thermodynamics and they may gain a better appreciation of the “real” gas effect.

        As far as I know, there has not been a published tropopause energy balance, though all models should have it included.

      • Alexander Harvey

        Dallas,

        I wouldn’t rule out my insanity, and I do mean I!

        I can guess why my thinking would be unpopular with those that believe the physical explanations.

        I suspect that there are many who genuinely doubt the physics of it all, and that they rely on there reasoning. This is a ray of hope. If rather than tell them how wrong they are, one might risk allowing them to expand on their thoughts until they can stumble over the contradictions. This is not without risks but it does deny them many regions of the field of play. It denies them the tortured Gallileo role and puts lie to the charge of argument from authority and the arrogance that entails. One might never know if they have been given reason to doubt their position but the bystander might see them gently tie themselves in knots.

        I think that many of the “Dragon Slayers” and their adherents are influential and persuasive. Most commonly when they explain the GHE but under another guise that protects the innocence of CO2. I think that this opens and avenue of approach that is not well used and closes another approach which is to say that it is all nonsense for in amongst it all is the GHE and one risks arguing against oneself.

        Sadly this thread has grown too long and too problematic for my computer equipment and I may not be posting more here.

        Alex

    • Alex, this is related to our previous discussion, where I say it is the latter, which is that radiative flux divergence leads to a net loss of energy by the troposphere, the balance being made up by surface heat fluxes including latent heat.

      • Alexander Harvey

        Jim,

        Good of you to pop by.

        Yes, and without the divergence ones view would be deeply problematical. We come to another place were people sometimes claim that some effect is disallowed by a thermodynamic law when it is required by just that same law.

        The existence of a globally universal lapse rates and tropopause, plus little more than a knowledge of basic thermodynamics gives rise to the divergence. Once there, except in some exotic set ups, such as world’s with a very low emissivity surface, which we haven’t got, the divergence implies downward path thermal emissions, without needing to appeal to their isotropic genesis for a proof, else we confront a system with an emissivity greater than 1 and that is forbidden by basic thermodynamics.

        I thought I would ask for people’s views, but few were given.

        Alex

      • I see the discussion about the lapse rate above, and yes it is critical to why the IR effect is a net cooling in the troposphere. The warming factors I forgot in my initial reply are also direct solar absorption by the atmosphere and clouds. Since the other factors are all warming, it is necessary for IR to be net cooling. There are other temperature and GHG profiles that might permit IR to be a net warming effect in the atmosphere, but not on earth. The lapse rate is governed by convection and horizontal heat transport by weather systems outside the low latitudes. But I think a big driver for the global temperature profile is the tropical convection lapse rate.which is set by the surface temperature there. The surface temperature at low latitudes is key because that is where most of the solar energy enters the system.

      • Alexander Harvey

        Jim,

        Yes there are definitely arguments on those grounds.

        In your opinion is it worth my arguing that one can proceed gently from the mere existence of a globally universal lapse rate to its implications. To make an appeal based on things that most people hold to be true, and in doing so show that they have consequences, as I should hope is possible.

        I was recently reminded of this quote:

        “Who is man? Is he a rational animal? If he is, then the goals can ultimately be achieved. If he is not, then there is little point in making the effort.”

        There is an inference of despair that I shun. I do think that many of the difficulties are more due to inate rationality than a lack of it.

        Can we proceed by discussion embued with tollerance in a dialoque of ideas? If we can, McNamara’s question can be answered in the affirmative. I do hope so.

        Alex

      • Alex, I have to confess I am not completely sure what you are driving at, but the idea that downwelling radiation which diverges over the troposphere dominates over the upwelling radiation convergence comes naturally out of radiative transfer models, and so the IR cooling can be explained in these terms. This of course depends on the lapse rate and GHGs, but as I say, it is generally true with profiles on this planet. Having resolved what radiative transfer does, the next question would be why is the lapse rate as it is?, and that answer involves convective-radiative equilibrium.

      • Jim D,

        I would like to see Alex nail down his Idea as well since it appears to be similar to mine. The tropopause is the transition layer between mixed thermodynamic and nearly pure radiative energy transfer. I think of the tropopause as the heat sink for the planet because it has the strongest overall thermodynamic impact on the surface. Neat stuff happens in the tropopause where temperatures can drop to -90 plus degrees C. It is a perfect point to pick for a frame of reference.

      • I see the tropopause as a regime boundary. Below it the air is more in touch with the surface, most of it having been at the surface within the recent days. Above it I imagine air that has not been near the surface for much longer time periods, perhaps months. Therefore the tropopause marks the highest level that convection can reach. It is there because of ozone in the stratosphere providing a permanent and global capping warm layer to prevent convection from getting any further, The independence of these two layers makes studying the troposphere easier, especially by looking at the tropopause energy balance.

      • The tropopause would be close to it’s present altitude even without the warming from the UV absorption of stratospheric ozone.

        The level of tropopause is determined by the requirement that the net flux of energy up from the layer is equal to the net flux from below even without convection. At that level the driving force for the convection is not present any more.

      • Pekka, it is true that the tropopause is past the driving point of convection, but convection can penetrate the tropopause. There should be a major increase in radiative cooling when that happens. It is an interesting place.

      • For those, who have the book of Pierrehumbert, the most directly relevant chapter is 4.8 Tropopause height for real gas atmospheres.

      • Alexander Harvey

        Dallas,

        I don’t promise much but I think that it is possible to demonstrate somethings, even if those with a more extensive knowledge think them insignificant.

        For instance, there is the seed of a puzzle in that energy from the surface and the troposhphere can be transfered away at all. For in between is the stratosphere a region of much higher temperatures. It seems that one must consider that the energy either passes through a region with a higher temperature or that the flow of energy is not continuous, somehow disappears to reappear somewhere else. I think that this may be a useful level of dialogue to approach basic mechanisms when many participating may be exceedingly doubtful.

        Energy passing up a temperature gradient does go against much commonly held perception of what is physically possible. Conduction, diffusion, and convection simply don’t work like that. There is something different about how the Earth transfers energy away. It does imply that we should think carefully about the essence of the thermodynamic laws and how they should be applied. Radiation does obey those laws but in a way that does cause much argument and may be best deduced with reference to things that the majority do hold to be the case.

        Alex

      • Dallas,

        The balance between the energy transfer excluding convection from below to the layer and up from the layer changes slowly. Thus all kind of disturbances may allow convection to continue higher for a while or stop at lower altitude. It’s not a precise and perfectly constant level.

      • Alex,
        The energy fluxes of the stratosphere are relatively small. They do certainly affect the tropopause at top of troposphere significantly, but not enough to change the general picture. Most of the radiation that enters the stratosphere from below goes through it without being absorbed, but certainly not all. Similarly the LW radiation down from the stratosphere heats the layers below to some extent.

      • Pekka, we could speculate what would happen without ozone, and maybe it is as you say, but the ozone is the reason for the stratosphere being so stably stratified in its lower part. I think without ozone the stratosphere would be more or less isothermal up to the mesopause and determined by the radiative properties of CO2 mostly. I am not sure what isothermal temperature that would be, however.

      • There’s no doubt that ozone has a major effect on the stratosphere. Through that it has some effect also on the tropopause and the troposphere immediately below tropopause, but those effects are limited by the relatively small energy fluxes. There is now a small net energy flux from stratosphere down. In the alternative case that net flux would be zero. In both cases I exclude the radiation that traverses the stratosphere unabsorbed.

        (Without the absorption in the stratosphere the solar radiation reaching the troposphere would in addition be significantly larger, but that change is handled at a different step of the considerations.)

      • Alexander Harvey

        Pekka,

        I know many things some complex some simple. I have come to doubt whether others understand the simple perspectives.

        Your answer, no matter how appropriate in terms of physics, did not in my view obtain much insight from the puzzle, it gave a mechanistic explanation.

        There is a principle here, one you may think too trivial to address. A principle that some may consider that energy can be transfered up a temperature gradient is unphysical. Both from a commonsense knowledge and through a belief that it violates the laws of thermodynamics. This certainly seems to be an issue in the downwards direction. I am not sure that some would even accept that it can depart in that direction.

        I am merely attempting to use something that no one seems to dispute to show that energy flux can travel up a temperature gradient. It may be a small step but have you considered how crazy all these discussions must seem to someone lacking such a insight.

        If people do agree that energy can at least set out in the direction of higher temperatures I can get the photon to the surface and then people are left to consider whether it is then transmitted, reflected or absorbed and the consequences of each.

        This is science at its most conceptual and least quantative but I think that there are people who doubt it and others that may prey upon those doubts. To make an argument that downwelling radiation is unphysical is easy and countering it by the use of a more sophisticated understanding counter-productive, if the original argument appealled to a commonsense view of how the world works.

        It may be necessary to show that the commonsense view leads to commonsense contradictions.

        Somewhere on this thread I said that such an approach would be generally unwelcome for it may suit nobody. I still think this be the case. What I had not considered was that it would not be comprehended.

        Alex

      • Alex, you are re-arguing against the DLR deniers, but should realize that they are a small but vocal minority. I would not bother trying to formulate an argument to convince them because the standard arguments and measurements of DLR are sufficient for any reasonable person.
        My other way to formulate the upgradient heat transfer by radiation is this. GHGs emit according to their temperature and GHG concentration. This happens in all directions regardless of their surroundings. These radiated photons fall as they may on the surface or whatever else is in their emission direction which is truly random. It is as simple as that as far as explaining why GHG photons come to the surface.

      • Alex,

        I’m again for the legitimacy of different ways of looking at the same physics. It’s pointless to argue, whether a component of the energy balance may go from lower temperature to higher, because both answers are right, when appropriate caveats are stated.

        One way of looking at the issue is more acceptable to some people, and the other to other people, while some people prefer one for some phenomena and the other for the rest.

        Putting physics in words can really be done in many ways. The equations can also be written in many ways. It’s, however, essential that for well understood issues only those alternatives are valid that give the right answer for every question they can answer and those alternatives are better that give that right answer to a wider set of questions.

        Simplifying idealizations are alright as long as they describe a well defined idealization without internal contradictions.

      • The lack to reply thingies is frustrating, maybe the tropopause will get its own thread some day. While Pekka is absolutely right, an average height of the tropopause energy balance should be very educational for a good percentage of the audience.

        First, it may better explain the impact of the warmth of the atmosphere with a starting point where radiative heat transfer dominates the scene. The energy balance at an imaginary stable tropopause would have several windows for energy flow in and out. A little complicated, but assuming a stable average height, it is doable.

        Then a second atmospheric energy balance at a layer of the atmosphere where radiative heat transfer is mixed with conductive and convective heat transfer. The windows would be different so the balance would be different. Solar that passed through the tropopause and was absorbed by the atmosphere and clouds would have their impact up and down, infrared from the surface absorbed by the atmosphere would have its impact. The clear window for IR from the surface would be the same and the clear window for surface reflection would be the same. The conductive and latent heat from the surface begin to transition to more radiative flows both up and down.

        It would be like the budget cartoon on steroids giving more detail and a better accounting of the heat flows by source. I am sure that this has been done in text, but I have never seen separate diagrams for the different levels.

      • Alexander Harvey

        Jim,

        I am trying to consider just the essentials, just the most bare of concepts, sticking to just what people hold to me true.

        It is my observation that any number, any formulae, any value arrived at by calculation is open to challenge and that many a defense of these relies on external authority.

        There are things that people do respect, in particular the essential laws of thermodynamics. I suspect just these are enough to imply the existence of a downwelling energy flux in world’s similar to Earth.

        This is argument from the existence of phenomena irrespective of their cause. How else might one proceed hopefully when for many the cause seems irrational and unphysical. Many view the downwelling radiation to be prohibited on thermodynamic grounds whereas I think it is directly implied by them. And I mean directly without recourse to the details of the mechanism, without recourse to what has already been rejected as irrational.

        Perhaps I am wrong and if I am then there is little point in making the effort.

        Alex

      • Alex, yours is a philosophical view in the direction of Cartesian doubt. I think that things can’t be inferred regardless of scientific knowledge of such things as radiatively active gases, why they are radiatively active, and how much so. Where would you draw the line of what speaks for itself and what has to be proven? I am a believer in scientific building blocks, and that everything has to be firmly rooted in well proven and tested theories that are the foundation blocks or maybe cornerstones. Those building blocks go several levels below radiative transer models, and even further below AGW which sits on a pyramid from multiple foundation blocks at its base.

      • Alexander Harvey

        Jim,

        Not without any scientific knowledge but only those bits that are most universally held true. That we should at least categorise the building blocks to see which are truly essential. Introducing the inessential without warrant just increases the scope for doubt.

        I see little advantage in arguing from a point that is commonly rejected as unphysical no matter how well founded that points is.

        It may come down to an opinion on whether the disagreement is due to lack of reason or lack of insight.

        It is my disposition that it is at least as much that we all see different things than that we reason differently.

        I may not hope or even wish for people to think how I think, rather to see what I see. I think there are insights that can challenge our conception of what is real.

        Alex

      • Alex, I don’t know your scientific level of education or what building blocks you were given by it, but really this is where everyone differs. There is no shortcut to understanding radiative transfer, and while it can be done without first understanding quantum mechanics, it might leave you unsatisfied. At some level, everyone even with a high level of scientific education takes a building block as a given. I learned quantum mechanics in my education, but that is far from it being understood at a fundamental level, though it has proven itself as a valuable tool in explanations, so I can take it as a building block.

      • Alexander Harvey

        Jim,

        My level of physical understanding may or may not be relevant. I do however believe that superior knowledge alone is insufficient to convey simple concepts and that it is in the realm of simple concepts that much of the disagreement lies. Perhaps I simply more capable of thinking things through at an inferior level, of arriving at the destination with less knowledge.

        It is my prejudice that I do think more like the common person than many who are more fluent with there knowledge. Perhaps I do see many of the issues that give rise to the disagreement. Maybe not but I do not see many that show more promising signs.

        Perhaps you do think that we would be unable to understand the GHE even qualitatively without our understanding radiative physics with or without quantum mechanics. Historically it was a close run thing and perhaps no 20th century science was involved.

        If you do think that an understanding is reliant on a university education in the physical sciences that excludes the many. I do not think this is the case. I think that much of it can be appreciated by the many providing care is taken to comprehend which bits conflict with such education and learning that we all have in common.

        It is not just that, it surely must require an ability to see how things look when you deprive oneself of certain benefits. Of asking how one would reason about all this if one where a contempory of Stefan or Arrhenius or Newton.

        Alex

      • Alex, to me as someone with a scientific background, as for others here who may have similar backgrounds, this is about the fundamental issue of communicating science to the majority, who don’t have this background, in such a way that the public “get it”. Not just to impress their friends with, but in a fundamental believing way that really has to be based on faith when they don’t have what I called the building blocks. As in other areas, people wrestle with faith all the time. Do you believe these preachers, or those people with placards outside the church? I can see that people need it to be a fundamental part of their knowledge rather than just a fact like a list of state capitals. How do we get there without educating everyone in quantum mechanics? That is the question.

      • Alexander Harvey

        Jim,

        I thought that this would be problematic.

        I am saying quite quite the opposite to what you seem to hear, that many people do have the ability to “get it” as you put it but without having to rely on faith. Many of these will be people with technical backgrounds, which covers a broad range, many just sentient beings with no particular science bias. That they do have the necessary capabilities. They may not have your building blocks but their knowledge, their world view, is not baseless.

        I “get it” as you put it but that does not rely on sophisticated knowledge, by “get it” I mean find it plausible and likely. Now I may aslo have some level of sophistication which may be useful in the decidability of specifics but I think it neither necessary nor particularly reliable. I see many who from time to time let their sophistication get the better of them.

        I seems fashionable to suggest that the common person cannot reason to the level required to “get it”. It is not a fashion I follow.

        If this issue were to be decided on the basis of who has a science degree then the majority falls to those that do not. If you think that, then perhaps you should prepare for the consequences.

        It is suggested that doubt in this matter is associated with technical training as opposed to pure science training. On occassion I can see how this may come about. There are some parts of the arguments that do not jibe well. There are also many issues concerning how people treat each other based on quite narrow perspectives of each other.

        I see a lot of superior knowledge being bandied about but little attempt to understand the divergence of views and how these come about and why they are justifiable.

        When you ask yourself why people do not share your views does it inspire you to find common ground and seek the points from which the divergence commences?

        I do and I reason about why I think as I do.

        I think that the “back-radiation” is a case in question for I can see just how problematic it can be. Perhaps others cannot. I could rehearse all the arguments that might support such a doubt. Were I that inclined I would need a killer insight not a killer fact. An insight that meant I would have to choose between abandoning some other dearly held notion or accepting down path thermal emissions.

        I see much in the way of the communication of facts and superior knowledge but rarely anything insightful.

        Alex

      • Alex, I find what you say very encouraging, because the impression from these blogs, which may not be typical, is that people are not willing to accept the science until they fully understand it at the most basic level. This is why the questions here get so technical, because there are a lot of those people. I think reasonable people should accept things that refer to text-book level conclusions without having to know how to derive every equation. Nobody does that, including the scientists. You take it that the text book has it right or someone would have corrected it by now, and there would have been a major scientific retraction to boot. Some skeptics question the text books, and these are neither typical nor viable lines of questioning.Better lines refer to uncertainties that the scientists themselves agree with. The basics of AGW go back over a century without any major reversal of thought. It is a straight line with just refinements as we get more data. Current climate and a lot of paleoclimate is understood. The uncertainties surround future projections. Those same text books tell us how we can make a reasonable estimate, and that is what is done.

  42. Skydragons, if you want to know what Georgia Tech students learn about radiative transfer and which models they use, go to this website
    http://irina.eas.gatech.edu/EAS8803_Fall2009/Default.aspx

    The lectures are all online at the above links, and computer modeling labs are found here
    http://irina.eas.gatech.edu/ATOC5560_2002/LABATOC5560_2002.htm

    Once you’ve mastered this stuff, and if you still think you have an argument, come back and tell us about it.

    • John Carpenter

      I came in from the wilderness
      a creature void of form
      ‘come in’ she said ‘I’ll give ya’
      shelter from the storm

    • This is a useful resource, brought me back to a lot of what we covered in my radiation class a couple years ago. I’d recommend people look at it.

      • If the skydragons actually spent some time looking at this, they might realize the lare gap between what they are talking about and the fundamental physics that is widely uderstood and empirically verified. And why it is so difficult to take them seriously and motivate to countering their arguments.

    • I think you should repost this as a main topic for easy reference.

    • Dr. Curry you cynically employ the tired old warmist strategy of telling us to go away and read through mountains of weblinks or books rather than just summarizing for us here and now what we are asking of you.
      I’d like to know PRECISELY from you does Georgia Tech teach just the zero-D climate model or not. If not, then please summarize in a few sentences what parameters your other models possess. Hand waving us away just makes you look like you’ve got something to hide.

    • curryja 8/19/11, 8:17 pm, Planetary

      Perhaps Dr. Sokolik’s course, EAS 8803, is intended only for Georgia Tech Earth Sciences students. It wouldn’t be suitable for engineering or physics.

      Seeing the course begin with some basics and then prominently the Beer-Lambert Law was a pleasant surprise. Not so pleasant was the immediate formulation of the Law with the subscript λ for most of the parameters. This is unnecessarily restrictive, and it leads to misunderstandings about the development and application of the law. The derivation in Wikipedia under Beer-Lambert law does not employ wavelength subscripts, and leads to the reasonable conclusion that the law would hold for any constant mixture of gases with parameters peculiar to the mixture.

      Also not so pleasing was the EAS 8803 formulation of the Law as the change of intensity along of a path, presumably I_out – I_in, instead of the conventional development relating to the ratio or fraction of intensities, I_out/I_in. (For this little course critique, I have substituted in and out for λ without changing any meanings.) That ratio is called the transmittance. See for example the introduction here from a chemistry department:

      http://www.chemistry.wustl.edu/~courses/genchem/Labs/Dyes/Extinction.htm

      Unfortunately, Lecture 5 defines transmittance, T_ν, unconventionally as exp(-τ_ν), and then fails, perhaps because of a typographical error, to define τ_ν. It is likely the unnumbered equation above equation 5.10 at the top of page 16. More importantly, the course then defines absorbance, A_ν as the complement of T_ν, i.e., A_ν = 1 – T_ν. Conventionally, the harder sciences define the Beer-Lambert Law in terms of an absorbance which is the negative of the logarithm of transmittance, A = -ln(T), which is fundamental to the physics and mathematics.

      The course also defines dI_in as the product of the volume extinction coefficient, β, the differential thickness, ds, and the incoming intensity. As a result the output is equivalent to I_in * (1 – βds). This was an opportunity the course missed to introduce some elementary and relevant mathematics relating to exponential functions.

      In the hard sciences, the term 1 – βds would be called an attenuation. EAS8803 says extinction = attenuation, which might imply unconventionally that β is an attenuation coefficient.

      EAS8803 says uncertainly and unconventionally in Lecture 11,

      Radiative processes may affect the dynamics and thermodynamics of an atmosphere through the generation of radiative heating/cooling rates.

      however

      Thermodynamics does not attempt to deal with any problem involving the rate at which a process takes place. Zemansky, Heat and Thermodynamics 4th, p. 25.

      As a result, the course doesn’t portray heat consistent with thermodynamics.

      Lecture 4 says ambiguously that atmospheric CO2 concentration is 0.0365% (increasing ~0.4% per year). Table 4.1, p. 6. Taking percentages of percentages is always hazardous. The next year is not likely to be 0.4365% by volume. The course material would be far more understandable if units were added to every equation.

      Lecture 26 says,

      Determination of GHG forcing is well-posed problem in radiative transfer (because IR absorption is well quantified for all GHG gases). P. 8.

      And earlier in Lecture 4,

      Temperature profiles (top), water and ozone density (bottom) of the standard atmospheric models often used in radiative transfer calculations. “Standard U.S. 1976 atmosphere” is representative of the global mean atmospheric conditions; … . P. 5.

      While Table 4.1 lists water vapor and CO2 as variable gases.

      A radiance algorithm has been used to treat the vertically inhomogeneous atmospheres resulting in substantially improved accuracy, and the model is directly applicable to longwave cooling rate calculations. P. 8.

      The course discusses problems with characterizing scattering and the composition of the atmosphere, but gives students no information about the accuracy of radiative transfer in answering the well-posed problem relating to calculating radiative forcing.

      The course gives the IPCC definition of forcing, which is a change in the <em<radiative energy budget of the Earth's climate systems. One cause included in the citation is secular changes in the concentrations of radiatively active species (e.g., CO2, aerosols), … . Bold added. For the baseline budge, the course reproduces the diagram from Kiehl and Trenberth (1997). That reference and IPCC together explain how those baseline budget numbers represent global averages taken over diurnal and seasonal cycles. The baseline greenhouse effect is allow only 66 W/m^2 of longwave radiation to escape to space. The well-posed problem is whether radiative transfer calculations produce that number, and if so, to what accuracy.

      Earth is never in the condition of a Standard Atmosphere, and even if it were, it varies seasonally. Lecture 4, Figure 4.1, p. 5. It varies diurnally. It varies with the mixtures of water vapor and CO2 over the globe, with emissions, and most important of all, with surface temperature. Since radiative transfer is not linear, the average global radiative forcing of GHGs is not the radiative transfer of any particular average or typical atmosphere. The course outline gives no hint how well the radiative transfer algorithms might suit the task.

      The course says,

      Radiation equilibrium at the top of the atmosphere (TOA) represents the fundamental mode of the climate system.

      Planetary radiative equilibrium (over the entire planet and long time interval):

      TOA incoming radiation ≡ TOA outgoing IR radiation Lecture 25, p. 1.

      This is not true. To repair it, the word mode might be changed to assumption, and the word averaged inserted before over the entire planed and long time interval.

      The Lecture somewhat remedies the problem later where it says,

      In a real atmosphere, solar heating rates do not equal to IR cooling rates. This imbalance is the key driver of atmospheric dynamics. Id., p. 8.

      The first sentence is good enough, but it would be better cast as a statement about Earth’s climate instead of just the atmosphere. The second sentence is false. It would be improved somewhat if it had the phrase “in IPCC’s AGW model” at the end.

      The key driver of the climate is the Sun, coupled into the variable transfer function of the Earth’s climate system. That transfer function is affected by Earth’s rotation, the resulting ocean and air currents, the hydrological and carbon cycles, and especially the surface and cloud albedos.

      This course is like a physics course for non-physics majors. It is not adequate to inform the discussions on Climate Etc.

      • Jeff,

        Unfortunately for your case the Beer-Lambert law is valid only, if the absorption coefficient is the same for all wavelengths present, and that requirement is not satisfied in any general case. How serious the error is depends on the case, but it may be very large, as it indeed is in the Earth atmosphere.

        That the Wikipedia is sloppy is not so rare.

      • No time to argue each of your points, but nearly all of them are incorrect. The Beer Lambert law applies only to monochromatic radiation, and hence the subscript lambda. You do not apparently understand the difference between a differential equation and its integrated form. Etc.

      • curryja 8/20/11, 3:15 pm, Planetary …

        JC: No time to argue each of your points, but nearly all of them are incorrect. The Beer Lambert law applies only to monochromatic radiation, and hence the subscript lambda. You do not apparently understand the difference between a differential equation and its integrated form. Etc.

        You and Pekka Pirilä at 3:00 pm are wrong. First, if you look at the Law you’ll see that it passes algebraic tests for a source spectrum in which all the line components have the same molar absorptivity, not wavelength. The restriction to monochrome is unnecessarily restrictive on its face, violating a principle of science.

        Secondly, the earliest work and demonstrations by Beer and Lambert established the law without even access to monochromatic light. Indeed, pure monochromatic light never exists. Even if the first formulation of the Law were for monochromatic light, it could today be generalized.

        Third, the Law is an empirical law, not an algebraic identity. A random spectrum of light will follow the Law with its own peculiar molar absorptivity. If the light source is (approximately) monochrome at wavelength λ, then the appropriate molar absorptivity would have the λ subscript. It is sufficient, but not necessary.

        Here’s a web site where you can experiment for yourself by changing the center wavelength and the width of the non-chromatic source:

        http://www.chem.uoa.gr/applets/AppletBeerLaw/Appl_Beer2.html

        You will find that over a wide range for this hypothetical (I assume) absorption spectral density, the absorbance seems approximately (it is unquantified) linear. That is sufficient to show that the Beer-Lambert Law can be valid without the monochromatic assumption, and by using a molar absorptivity appropriate, as always, for the light source.

        Molecular absorptivity is always determined by experiment. Besides, the Law begins to break down at high concentrations, meaning that its accuracy is not uniform in the first place.

        Fourth, you have failed to justify your monochromatic requirement by error analysis. How close in molecular absorptivity can two bundles of light be to be treated as a single bundle with a single molecular absorptivity? You need a criterion to answer that question, such as getting radiative transfer “right”, as if that were possible. You should always expect a law of physics to have an associated probability distribution. This is especially true for the Beer-Lambert Law which at the outset can be cast as a theory about the probability of absorption for photons passing through a medium.

        I would never baldly assert that nearly all of [someone’s points] are incorrect under any circumstances or with any excuse. I believe a scientist has a duty to be specific and to support his claims, or to say nothing.

      • Jeff,

        Mathematics is indisputable. The sum of two or more exponentials with different rates is never an exponential. Your view leads to direct conflict with mathematics. You are wrong.

      • Pekka Pirilä 8/20/11, 3:48 am, Planetary energy …

        PP: Mathematics is indisputable. The sum of two or more exponentials with different rates is never an exponential. Your view leads to direct conflict with mathematics. You are wrong.

        Mathematics is indisputable when applied correctly, but you have misstated the problem. You and Curry have claimed that the Beer-Lambert Law only applied to monochrome light. My argument is that monochrome is sufficient but not necessary, and that the restriction to monochrome is poor physics. Now you defend your position NOT for monochrome light but for what we might call “mono-rate” light, or perhaps for “mono-molecular absorptivity”, ε. In other words, for your argument, you do not rely on the wavelength being the same, but instead rely on the molecular absorptivity being the same.

        (1) If ε_λ_1 = ε_λ_2, then light comprising those two wavelengths, that is non-monochrome light, will obey even your monochrome Beer-Lambert Law. It can be extended to any number of λ_i. That demonstrates your error on the algebra.

        (2) The Beer-Lambert Law is statistical, based on the probability of absorption by a photon. It is not a perfect relationship like the power 2 in the square law spreading of light. The molecular absorptivity is an empirical number, determined in laboratory experiments, which always have an associated error.

        Now, if ε_2 = ε_1 + δε, then A_1 = A_2 + ΔA, where for some δε, ΔA is as small as you wish. A here is the Beer-Lambert absorbance. You cannot say that the ε_effective is any less accurate for some arbitrary colored light than is the line-by-line sum of A_λ_i. This demonstrates your error on the basis of accuracy.

        (3) That an effective molecular absorptivity exists under some circumstances is demonstrated by the Applet at the link I gave previously. That demonstrates your error on the basis of the algorithm used in that Applet simulation.

        Applying (1) to bands, you can see that if a light source comprises bands all with the same ε_effective, then the sum will as well, and the Beer-Lambert Law will be as accurate for the sum of the bands as it was for each band.

        That the Beer-Lambert Law is only monochrome is an abuse of physics created solely for climatology, apparently for the purpose of making radiative forcing logarithmic in gas concentration. Logarithmic dependence means that IPCC does not have to determine initial conditions on saturation curves for CO2 or water vapor. Logarithmic dependence also validates a constant climate sensitivity. What appears to have happened next is that the radiative transfer investigators back-filled the absorption spectrum, eliminating any windows, regions of zero absorptivity. (The initial condition for radiative forcing, the K&T97 budget, had a window.) This backfilling eliminated RT saturation.

        However, IPCC never mentions the Beer-Lambert Law, and instead relies on radiative transfer arguments for the proposition that radiative forcing increases logarithmically with CO2 concentration, a physical impossibility except as an approximation, and a strict violation of the Beer-Lambert Law. It is akin to IPCC inventing a new process for CO2 uptake that violates Henry’s Law, and never mentioning Henry’s Law. Climatology physics is physics for non-physics majors.

  43. I think it is time for American citizens to close ranks with the Australians [1,2] in demanding an end to tyranny and a return of the basic human right for citizens to control their government!

    With kind regards,
    Oliver K. Manuel
    Former NASA Principal
    Investigator for Apollo

    1. JoNova’s blog:
    http://joannenova.com.au/2011/08/gillard-has-a-problem-convoy-on-the-way-powers-through-north-queensland/

    2. “No Confidence Rally”, The International News Magazine
    http://www.international.to/index.php?option=com_content&view=article&id=2035:gillard-convoy-of-no-confidence-rally

  44. There is a term missing in the simple energy balance equation which states that;

    Ein = Eout

    Energy in and energy out are not constant and never equal when the planet is warming or cooling.

    The missing term is the change of heat content (HC) in oceans, atmosphere and enthalpy in a period of interest. If we subtract energy out from energy in over a period – we get the change in the Earth system heat content – delta (HC)

    Ein – Eout = delta (HC)

    By the 1st law of thermodynamcis energy is conserved. We can put it in a 1st order differential form:

    Ein/s – Eout/s = d(HC)/dt

    We can substitute back in the solar constant expression – with geometric and albedo factors – for energy in and blackbody radiation for energy out. But it doesn’t help because both the average Earth temperature and albedo are variable. Average temperature because of changes in HC and albedo because it just does – mostly as the result of clouds.

    One thing that emerges from this is that – although surface temperatures are of course spatially and temporally variable – changes in global heat content depend only on the energy balance at TOA.

  45. Richard S Courtney

    Dr Curry:

    You say:
    “The planetary energy balance equation has its greatest utility in the context of comparative planetology.”

    OK, in that case I think it necessary to include consideration of the Jelbring Hyothesis which has not been mentioned anywhere in this thread so far. And, to be clear, I do not know if the Jelbring Hypothesis is right or wrong, but I do think it needs inclusion as part of the discussion in this thread.

    The hypothesis was presented by Hans Jelbring and says the atmosphere of any planet with an atmosphere adjusts such that it acquires a lapse rate which determines the mean surface temperature of the planet. The hypothesis is that the adiabatic temperature lapse rate has to be –g/cp with cp being the heat capacity of the gas which is the planet’s atmosphere.

    Simply, the hypothesis is that all the radiative, convective and conductive effects within a planet’s atmosphere adjust to approximately fulfil this lapse rate whatever the chemical composition of a planet’s atmosphere.

    Clearly, if the Jelbring Hypothesis is true then it has direct implications for assessments of global temperature changes resulting from alterations to GH gas concentrations in the atmosphere of the Earth.

    In the solar system all the planets with an atmosphere except Mars seem to fulfil the hypothesis. And it could be expected that Mars could not fulfil the hypothesis because its atmosphere is mostly CO2 that freezes on its winter pole and sublimes in the Spring so Mars never has a stable atmosphere which could adjust to fulfil the hypothesis.

    Several people, notably Hans Erren, have attempted to rebut the Jelbring Hypothesis but the rebutals provided to date are all flawed. And, in my opinion, that alone is sufficient reason to include consideration of the Jelbring Hypothesis when discussing comparative climate planetology.

    Richard

  46. I think skeptics would be able to explain their case much better if they went to text books and explained which parts they don’t agree with and why exactly. That way we know what we are arguing about. Call it a crazy idea.

    • Brandon Shollenberger

      I think people attempting to refute Postma’s paper would be able to explain their case much better if they went to his paper and explained which parts they don’t agree with and why exactly.

      In over 1,000 comments in the last thread, I don’t remember ever seeing someone say something like, “I think equation number XX in Postma’s paper is wrong because….” I don’t remember seeing anyone make reference to specific page numbers or text from his paper either. I agree with your proposal for greater clarity from “skeptics,” but I think it’s one lots of people need to work on.

      For what it’s worth, I haven’t seen anything in these discussions which I’d considered a refutation (of Postma’s work or of the greenhouse effect). There is lots of disputation and rebutting, but the lack of an organized and clear response means they fall short of proving anyone is wrong.

      • It is very clear Postma’s paper is not from first principles because he hasn’t derived where he gets his 5 km from. He only says it is by observation. Unless he can derive 5 km, he has an incomplete model with a circular argument to show it matches observation.

      • Brandon Shollenberger

        If anything is clear, it’s you can’t be trusted as a source of what is in Postma’s paper. He never says that height is by observation. In fact, his paper makes it clear one couldn’t use an observation to establish that height. He does refer to an observation on this issue, but not in any way you describe. For a specific reference, anyone can look at equation 34 in Postma’s paper. That is where he derives the height, and the text immediately after is where he says what I claimed he says.

        Not only are you wrong, you are so wrong your claims are completely backwards. Added on top of this, your claims are so easy to check all one needs to do is perform a search for “5km” in Postma’s paper. . Anyone, anyone at all, could see your claims are nonsense even without having the slightest idea of what Postma’s paper is about. That’s how ridiculously wrong your claims are. I don’t think you could have made a more bogus argument against Postma’s paper than the one you just offered.

        On the upside, you did just do a great job of showing why my call for clarity is important.

      • Postma says this “This altitude is found at about 5km in height above the ground surface by observation.” Maybe you are reading the wrong paper. This one is called “Understanding the thermodynamic atmospheric effect”. So, as I say, he is getting his primary parameter by observation. I could go into how over-simplified the concept of a radiating altitude is too, so there is actually nothing special occurring at this altitude, because it represents a weighted mean of radiating altitudes. It is like saying the middle of the US is Kansas, so let’s take Kansas to represent the US.

      • This should be interesting.

      • Brandon Shollenberger

        Indeed. I find the fact I have to tell someone to read and discuss the paper Judith Curry made a thread about rather than some other paper quite interesting.

      • Brandon – you don’t “have” to tell anyone anything. You can shut down your computer anytime you wish.

        You certainly don’t “have” to tell people that they couldn’t possibly make a more bogus argument than they did. It just simply isn’t necessary – and it will never be optimally productive in an exchange of viewpoints.

      • Brandon Shollenberger

        Jim D, that’s not the paper we’ve been discussing for the last few days. The paper we’ve been discussing is this one, as you can easily see if you check the thread made specifically for it. I have no idea why you are talking about a different paper, one apparently published some four months before the one this discussion is about. All you’ve done is show a flaw in a older paper which Postma corrected in the paper Judith Curry made a thread about.

        So can we now agree you need to go read the paper we’re supposed to be discussing?

      • Just about as interesting as I expected.

        Next time, Brandon, instead of going off like a loose canon, and “tacitly” insulting people – you might consider just asking for clarification.

        It’s generally a much more effective way to exchange viewpoints on a topic – although I have to say that in this case Jim D didn’t seem particularly taken aback.

      • Here – I’ll give you an example.

        “Jim – I’m confused about how you derived the opinion that Postma said that height could be obtained by observation. I’ve looked at the paper discussed in the previous thread and found that he said that one couldn’t use observation to establish that height.”

        I dunno – but somehow I think it might be more effective than telling someone that they couldn’t make a more bogus argument.

        But hey – I guess that’s just me.

      • Brandon, he doesn’t run off the rails in any of his equations (at least up until page 10), he runs off the rails around page ten when he makes this statement:

        Dividing the solar flux by a factor of four and thus spreading it instantaneously over the entire surface of the Earth as an input flux amounts to the denial of the existence of day-time and night-time, and violates the application on the Stefan-Boltzmann Law which deals only with instantaneous radiative flux.

        I really couldn’t read much beyond that. The problem here is that he ignores the effect of the atmosphere and the ocean in spreading the energy gained from insolation over the globe in justifying his use of a factor of two for averaging insolation rather than 4. There were a couple of other howlers of interpretation prior to that, but this is the first really, really bad one. Up to this point, it is not his equations which are wrong (they aren’t), it is his rhetoric.

      • Brandon Shollenberger

        Rattus Norvergicus, the “problem” with this type of comment is the same as with Chris Colose’s rebuttal at Skeptical Science. It is just a declaration of error. Postma undoubtedly made errors in his paper. However, to refute his paper, it would have to be shown some error in his paper negates his conclusion. Most people don’t even refer to his paper, much less show where some fatal flaw lies.

        Mind you, this isn’t a criticism of your comment. There is no obligation for every person to try to refute Postma’s ideas, and I can certainly understand it being difficult to find the motivation to read through the entirety of Postma’s paper. The problem is if nobody sits down and does it, no refutation will ever get made.

        Knowing something is wrong isn’t enough. Proof is needed.

      • Brandon, in this case it is the decision by Postma to ignore some key characteristics of the system under analysis If you can’t see the difference between the moon and the earth, then you will never understand that Postma is ignoring exactly what he claims the toy model is ignoring in his analysis.

      • Brandon Shollenberger

        Rattus Norvergicus, I don’t understand your response here. I never disputed the point you raised, yet your response here seems to act as though I did. You claimed Postma did something, and I explained that claim didn’t disprove his paper as the claim stands. I then explained there is a need to elaborate on that sort of claim in order to show it’s impact.

        I have no idea why you talked about if I “can’t see the difference between the moon and the earth.” Whether or not I can see the difference has no bearing on what I said.

      • Brandon, to be Shakespearean, this reeks of the scent of the emanations of the southern end of a steer headed north.

        Postma had defended this assertion strongly in the previous thread and it’s predecessors. It is a fundamental error and once a fundamental error in the input of the model of this magnitude has been made, the outputs of the subsequent analysis are suspect, to say the least. In his favor, at least he didn’t get it wrong by 10 orders of magnitude as some of the “denizens” arguing (not here, but elsewhere) for a strong greenhouse effect from O2 and N2. He only gets it wrong by a factor of two because he ignores the influence of the ocean and the atmosphere, not to mention (which I didn’t, sorry) the radiation from the unlit side of the earth. This is, to say the least, a catastrophic misunderstanding of the system under analysis.

      • Brandon,

        The assertion he made was highlighted a couple of comments up *in this very thread*. See here.

      • Brandon Shollenberger

        Rattus Norvegicus, obviously if you find “a fundamental error,” it makes the results of the paper seem questionable. However, people are not saying Postma’s conclusions are questionable. They’re saying his conclusions are wrong. Simply finding “a fundamental error” doesn’t prove that.

        For example, imagine you were reading a 100 page paper. On the 15th page, you saw an error which caused the input to an equation be off by a factor of ten. You decide this error is too severe, so you throw the paper away. A week later, you’re told about a stunning new conclusion which has just been proven. That conclusion is the same conclusion as the paper you threw away. Upon talking to people, you find out there was a second mistake on page 30, and that mistake canceled out the first mistake you found. When the mistakes were noticed, both got corrected, and the conclusions didn’t change.

        As you can see in this example, simply pointing out the existence of an error is not enough to invalidate the conclusions of a paper. If the error does invalidate the conclusions, you need to demonstrate such. Failing to do so means you haven’t refuted the paper; you’ve just called its conclusions into question.

      • Brandon Shollenberger

        Rattus Norvegicus, there is a reason I asked you to make sure the assertion you wanted me to justify was one I actually asserted. I’ve never said that assertion by Postma is correct, so I can’t see why I would be expected to justify it now.

      • Brandon –

        For example, imagine you were reading a 100 page paper. On the 15th page, you saw an error which caused the input to an equation be off by a factor of ten. You decide this error is too severe, so you throw the paper away. A week later, you’re told about a stunning new conclusion which has just been proven. That conclusion is the same conclusion as the paper you threw away. Upon talking to people, you find out there was a second mistake on page 30, and that mistake canceled out the first mistake you found.

        What do you suppose the chances might be that you’re be reading a paper and find that an equation important to the conclusion of the paper was off by a factor of 10, and thus decide to stop reading the paper, only to later find that another equation that a second mistake was off by a similar degree so as to correct for the first error perfectly and allow the conclusion to stand as valid?

        Now compare that likelihood to reading a paper and finding an important equation important to the conclusion to be off by a factor of 10, and to later find that a large number of other experts in the field addressed by the paper found that the paper’s conclusion was invalid?

        Personally, I would judge the later scenario to be much, much more probable than the first scenario (which seems to me to be extremely unlikely), and I would consider it highly illogical to use the first scenario as a kind of guide for how I should approach my treatment of papers that I found, early on in the reading, to contain fundamental theoretical errors.

      • I just find the “error which makes no difference” line of argument ironic coming from Brandon.

      • Brandon Shollenberger

        Is there some particular reason for that?

      • Is there some particular reason for that?

        That’s funny, Brandon. You really expect that anyone following this exchange can’t figure out why RN finds your line of argumentation here ironic?

        You’re really trying to say that you don’t know?

        You’re joking, right?

      • So now instead of using the observed 5 km to derive 15 C at the surface, he used the observed 15 C at the surface to derive 5 km. He just puts in 15 C, and has no way to predict that value that I could see. He seems to play with a lot of numbers, and does a speculative atmospheric budget ignoring things like heat flux from the surface, and comes up with a few temperatures. I don’t see the big picture here. What did he explain? He admits the atmosphere radiates IR up and down and that the outgoing longwave has to be equivalent to 255 K. He does need to explain why heat flux doesn’t contribute to his atmospheric budget because we know convection exists and is important to the tropospheric temperature. In fact it helps to determine his precious 6.5 K/km lapse rate. Maybe by assuming this lapse rate, he thinks he can ignore surface fluxes, but he hasn’t explained how latent heating fits in at all with his budgets. It is very confusing, which is why he needs to look at some text books and outline his differences with them.

      • Brandon Shollenberger

        Jim D, is there a reason you didn’t acknowledge the fact you had made a (very serious) mistake? I understand wanting to talk about the issue(s) you raise here, but it’s peculiar to try to discuss one person’s mistakes while not admitting your own. Not only is it peculiar, but it can easily shut down a discussion if the other person decides to do the same thing. A quick, “My bad, sorry about that” seems like it might go a ways in establishing mutual respect between parties.

        I know if somebody kept talking about me being wrong while not admitting their own mistakes, I wouldn’t feel inclined to talk to them.

      • Brandon, is it a mistake to refer to someone’s paper that he hasn’t even retracted? If his view changed so much in 4 months he himself would have emphasized that in the July paper. The differences are subtle at best,and what is to say his next paper doesn’t change as much again. I don’t think his views are stable yet because he hasn’t considered heat flux and a few other terms in his balance.

      • Brandon Shollenberger

        Jim D, your question completely misses my point. Obviously my answer to it is no. There is no mistake with referring to a different paper of his. Indeed, it can be very useful to highlight the differences between his papers.

        However, that does not change the fact you made untrue claims about his (more recent) paper. Somehow you thought the discussion was about a different paper than it was really about. You messed up. I can’t see any reason to justify not admitting this.

        Now then, if you admit the mistake, this doesn’t mean the point you’re discussing somehow vanishes. You still have a point to discuss, but now it is clarified. Now it is a real point, not one based upon misrepresentations.

        But nobody can be expected to discuss your real point if you won’t retract your false point. If you won’t admit what is an obvious mistake (one you don’t even deny), how can anyone expect to hold a meaningful conversation with you?

      • If you won’t admit what is an obvious mistake (one you don’t even deny), how can anyone expect to hold a meaningful conversation with you?

        If I may, I would suggest something like:

        “Ok – well, I think you should have simply acknowledged that you were referring to the wrong paper – and I can admit that I should have simply asked you how you got that opinion instead of saying that you can’t be trusted about his paper and couldn’t have made a more bogus argument – but getting past all of that, let’s move on to discussing the points of substance that you were making about Postma’s analysis of energy balance.

        Now in my view…….”

        See – it really wouldn’t be that hard. I came up with that pretty much right off the top of my head.

      • OK, Brandon, my first remark made the mistake of referring to a slightly different Postma paper than the one linked on the Postma thread, but that paper is linked at the end of his July paper. I have also added more thoughts on the Postma paper at the end of the Postma thread.

      • My bad if I am asking a stupid question, but isn’t the dry adiabatic lapse rate on earth 9.8oC/km? It would seem that Postma has just solved the following equation:

        a = (15 – (-18)))/5

        and given an approximate answer. This doesn’t square well with the actual lapse rate.

      • RN, he chose a lapse rate of 6.5 K/km which is an approximate lower tropospheric moist adiabatic lapse rate and either chose 5 km or 15 C to make this equation come out, depending which of his papers you look at. As I mentioned in various places above, this is not a derivation of anything, just a passing remark of some sort.

      • Brandon Shollenberger

        Jim D, your mistake wasn’t simply to refer to the wrong paper. You made claims which were false due to your incorrect reference. What you just said doesn’t address the points I’ve been responding to you with throughout this entire exchange. What you just said in no way retracts or corrects anything you said in the comment which started this exchange.

        Your latest two responses have been very non-responsive.

      • Ah, well that’s what I get for giving up after he decides that the proper way to estimate insolation was to only use half the globe in his average.

        Of course the moist adiabat holds only in limited conditions and so seems an invalid starting point to base broader claims on.

      • Brandon, at 11:46pm yesterday I expressed an opinion on a Postma paper. This was my own opinion, based on my reading. You trashed my statement that I later was able to attributed to Postma by quoting that paper. I think this has upset you, and you need to search for your own reason why. I don’t need to apologize for my opinion that his argument was made circular by putting in an answer from observations. Maybe you have your own definition of a circular argument, or didn’t understand what I was complaining about. There are other reasons for not liking the Postma paper, as I have posted elsewhere.

      • Brandon Shollenberger

        Jim D, nothing you’ve done has upset me, and I don’t know why you would think otherwise. I’ve been a bit baffled by you, but that’s it. Unfortunately, your latest response is as baffling to me as any of your others. For example, you say:

        Brandon, at 11:46pm yesterday I expressed an opinion on a Postma paper.

        That’s true. However, you leave out the fact the paper you were expressing an opinion on was not the paper being discussed. Naturally, leaving out this fact causes the rest of your response to be quite peculiar. For example, you say:

        I don’t need to apologize for my opinion that his argument was made circular by putting in an answer from observations. Maybe you have your own definition of a circular argument, or didn’t understand what I was complaining about.

        This makes no sense at all. I haven’t said a word about whether or not Postma’s argument is circular. Indeed, I haven’t said a word about whether or not Postma’s argument is correct. You say you “don’t need to apologize for my opinion that his argument was made circular by putting in an answer from observations.” I agree! That’s why I’ve never asked you to apologize for it.

        All I am saying is you ought to admit the criticisms you raised in this comment were incorrect. That those criticisms may be able to be applied to a different paper doesn’t change the fact they were wrong in regards to the paper being discussed.

      • Brandon, that opinion I expressed looks completely innocuous. It was an opinion on a paper by Postma that had relevance to the Postma thread. This is not far out of bounds, and does not merit a complete overreaction. You don’t even know for sure that Postma doesn’t still stand by his earlier words, and the way I see it, they are equivalent to his later words making them fair game. That is my opinion. You don’t have to agree.

      • Brandon Shollenberger

        Jim D, once again your response to me is non-responsive. You say your opinion “was an opinion on a paper by Postma that had relevance to the Postma thread.” Nobody has said otherwise. All I have expected you to do is admit the criticisms you raised about the wrong paper don’t apply to the right paper, and thus you were wrong to raise them as criticisms about the paper we were actually discussing. What I’ve effectively managed to drag out of you so far is, “These criticisms were based on paper B, not paper A.” That’s true, but it leaves off the part, “They don’t apply to A.”

        In addition to the non-responsiveness of your comment, you make several peculiar remarks. For example, you say your comment does not merit a complete overreaction.” As a matter of definition, nothing can merit an overreaction, so your comment is true. However, it is irrelevant unless you mean to suggest I have overreacted. This implication is untrue as my reaction has been, if anything, an underreaction.

        You also say I “don’t even know for sure that Postma doesn’t still stand by his earlier words.” This is as true as your earlier remark. Namely, it is true, but meaningless. Whether or not Postma stands by the contents of one paper has no relevance to whether or not criticisms which apply to one paper can be applied to a different paper.

        All in all, I must say I find i am baffled at how much effort you’ve put into addressing issues nobody has raised. That amount of effort far exceeds the amount of effort it would take to address my concerns, concerns which you have still never addressed.

      • Brandon, believe me, I am baffled too. My reply at 2:00am 20th on the correct paper showed that I thought it had the same problem. It doesn’t matter which observation he puts into his theory, it is incomplete by putting an observation in, and not predicting that observation. Maybe my 2am reply did not make that clear enough.

      • Brandon Shollenberger

        Jim D, to an extent your reply wasn’t clear enough, but now that you’ve pointed me to that comment of yours, I can definitely see I’ve been overly critical of you. That comment never said anything about you having been wrong, and as I made my subsequent comments, that’s what was stuck in my memory the most. Your next response didn’t address any of the issues at hand, and it was only in the one after that you admitted any sort of mistake. Because of the sidetracking and difficulty of getting you to make the simple admission of error, I didn’t think much more on the details of the comment you now reference. Because of this, all I saw later was you admitting a part of your error with seeming reluctance while not saying anything about another part.

        I now see if I take what you said in that comment and combine it with what you said in others, you most certainly did address the full extent of your error (and for what it’s worth, I agree with you that the same basic issue exists in both papers). You could have done it with far more clarity and less difficulty, but I in turn should have done a better job of keeping track of the whole conversation.

        There are a number of factors which contributed to this, and I think without them I wouldn’t have missed the significance of that comment, but ultimately, I messed up. I still think you should have been more upfront about your mistake (and I still have no idea how you made it), but I have unfairly criticized you, and for that, I apologize.

      • Brandon, I was thinking you were being unreasonable, but this explains that we assume too much about what the other has taken in or is basing their remarks on. I hope we can both do better in the future.

      • Bravo, gentlemen.

      • Jim – My comment to follow is not snark but an accurate description of my opinion. I think something has happened to Brandon to sour his outlook, and it shows in the nastiness of his recent comments of the past few days. They are also inaccurate, but that is less troublesome than their tone. I hope he takes a step backwards and recalibrates the way he comments here for the sake of general civility as well as accuracy.

      • Brandon Shollenberger

        Fred Moolten, Jim D is apparently limiting his discussions to some paper other than the one Judith Curry made a thread about, and that ridiculous limitation is the basis for claims.

        Maybe you should step back for a minute and think about that. I discussed the paper our host posted to discuss. Jim D discussed a different paper. You called my comments inaccurate.

      • Jim D,

        Where Postma gets the 5K and why is bugging the heck out of me. I asked him was he was considering the lower atmosphere as part of the solid Earth and he said no and his math indicated he was using the surface area of the solid Earth, but using an integral of a larger area/volume. He as pretty cryptic in his responses and mentioned he was off to do more calculations.

        If he was considering the earth plus 5 Kilometers of atmosphere for an energy input radius and the solid Earth as receiving the energy, he would be missing a step. The Earth/atmosphere center would receive the normal 1/4 of the input allowing for diurnal and geometry and the doughnut around would receive about 20% with about 50% of that transferred to the center by changing your frame of reference to 5K above Earth. Then there would be no greenhouse effect per se because it would be internal to his Earth/atmosphere system. That may have some meaning to an astrophysicist studying a distant planet but no real meaning for a greenhouse discussion of this planet.

    • Jim,
      That is PRECISELY what Joe Postma’s paper did. He examined in great detail 60 standard references of the GHE in the first half of his paper. In the second half he dissected the standard model and exposed the flaws. The worst flaw of all is that the standard model treats the Earth as if it were a flat disk receiving an average of solar radiation over 24 hours (i.e. no night and day).

      Postma then showed that the equations used by climatologists (P/4 based) to convert the numbers from this flat disk into a rough approximation of a sphere ended up losing 33k of energy. This is because the GHE equations crassly mix non linear functions with linear functions, like mixing apples and oranges.

      Climatologists then said the missing 33k must be accounted for by their GHE, something Postma shows is not required if we go back to first principles and redo the equations by treating the Earth as a sphere, add in adiabatic physics and the insolation and entropy that entails. As such he proves that the ‘missing’ 33K of heat isn’t missing at all.
      Thus he shows Earth energy budget balances with or without a GHE and we see:
      Predicted earth emission without a greenhouse effect: 239 W/m²
      Actual earth emission WITH a (supposed) greenhouse effect: 239 W/m²

      • “Thus he shows Earth energy budget balances with or without a GHE and we see:
        Predicted earth emission without a greenhouse effect: 239 W/m²
        Actual earth emission WITH a (supposed) greenhouse effect: 239 W/m²”

        But what about the surface flux? It’s not hard to balance one part of the system, in this case emission out with energy in. But it’s another thing to balance all parts of the system.

        In your understanding how much energy is the surface emitting? and how much is it absorbing?

      • Of course, John O’Sullivan’s description of what Postma did is complete fantasy. What he actually did was this:

        (1) Considered the simplest possible model for the greenhouse effect, a model that is useful only to illustrate the basic principles, and used it as a “strawman”, pretending that it was the “standard model” greenhouse effect.

        (2) Set out to confuse people about averages and refused to engage in a simple discussion of the total energy into and out of the earth system. The reason for this refusal is obvious: With averages, he can work his sophistry but if one computes total energy or power, one immediately dispels his nonsense.

        (3) Failed utterly to explain how the earth’s surface could be at a temperature of ~15 C where it is emitting ~390 W/m^2 when the earth as viewed from space is only emitting ~239 W/m^2. The only possible explanation for this is that the atmosphere is absorbing (or reflecting) some of the radiation from the earth’s surface. In doing so, it is allowing the surface of the earth to be warmer than it could possibly be if none of its radiation was absorbed. This is, of course, the greenhouse effect.

        What is amazing to me is that anybody could read these threads and still believe that Postma is anything but very embarrassing nonsense. I would be willing to contribute money if Postma’s work would be broadcast to the scientific community as an example of the best that the “skeptic” side on AGW has to offer! Postma serves as a useful benchmark: Anybody who does not admit that it is nonsense has shown that they have absolutely positively zero ability to distinguish actual science from nonsense.

      • Joel,

        “(3) Failed utterly to explain how the earth’s surface could be at a temperature of ~15 C where it is emitting ~390 W/m^2 when the earth as viewed from space is only emitting ~239 W/m^2. The only possible explanation for this is that the atmosphere is absorbing (or reflecting) some of the radiation from the earth’s surface. In doing so, it is allowing the surface of the earth to be warmer than it could possibly be if none of its radiation was absorbed. This is, of course, the greenhouse effect.”

        The measurement of 390W/m2 is presumably at ground and the 239W/m2 is measured at the TOA. It is basically radiation law inversely proportinal to R^2. Not absorbed by atmosphere, get it!

      • Sam: Nice try, if a bit desperate. Alas, the difference in radius between the earth’s surface and any sensible definition of the top-of-the-atmosphere is way too small to make that sort of difference. (Also, as a matter of principle, they may correct numbers to do the TOA measurement per unit area of the earth’s surface…I am not sure…But even if not it would be a much smaller difference than the difference between 390 W/m^2 and 239 W/m^2.)

      • Joel Shore might have a point if Postma was committed to the idea of a global average surface temperature of 15C.
        I think that the whole logic of his approach is that this average is bogus.
        G&T and a wide body of opinion both sceptic and non sceptic think that the concept of a 15C average has no merit.

        Postma used 15C in someof his calculations without any real comment.
        His main points were
        1. There is an effective radiation level of about 5Km for the Earth/atmosphere ensomble.
        2.Thermodynamic calculation of the dry adiabatic lapse rate of – g/Cp.
        3.Work out the radiation absorption/emission consequences separately for day and night.

        However he mentions throughout the text of the benefits of treating the local actual surface conditions.
        So G&T and Postma would see a multitude of local lapse rates throughout the planet surface.
        The lapse rates would differ according to local conditions.
        There would be a similar thermodynamic link between the local troposphere and the local planet surface temperature.
        There is no need to account for a 33C greenhouse effect

      • Bryan,

        What you write is, as usual, not correct. First of all, there is no reason why one cannot define an average temperature for the surface. But, if you don’t want to do that, you can certainly define a total power radiated by the earth’s surface and divide by 4*pi*R^2 to get the average intensity in W/m^2…and you will get a number around 390 W/m^2.

        Second of all, while there may be a benefit to treating the local actual surface conditions, that benefit would only accrue if they were treated correctly. Calculating a local temperature on the basis of the local amount of radiation absorbed using the S-B Equation is not at all correct because it neglects the fact that such radiative balance does not have to be satisfied locally because of other energy transfer mechanisms and the possibility of storing energy too.

        The radiative balance can only usefully be applied on a global scale, since the only interaction between the earth system and space is through radiation and energy storage terms become basically negligible over the whole planet and on a reasonable timescale (of, say, a year).

  47. Brandon – I believe many of your recent comments have been inaccurate, but what is troubling is your attitude. I didn’t revisit the paper in regard to the 5 Km height, and so my point was not about that specific item. I hope you will take my response seriously.

    • Brandon Shollenberger

      Fred Moolten, I do take your response seriously. I just think it’s biased and wrong. You say “many” of my “recent comments have been inaccurate,” but you don’t point to any. I think it’s safe to guess you would include my response to you in the other thread, but that’s the one I can guess at. Even if I stipulate that response was wrong, that’s one comment out of about thirty.

      You keep claiming “many” of my comments are wrong, yet you haven’t ever responded to me to say how they are. You haven’t made any effort to show I am wrong, and you haven’t made any effort to help me get things right. Instead of doing any of that, you just now randomly claim I’ve been wrong many times. You don’t even have the decency to point out where I’m supposed to be wrong. You’ve made a serious claim, but instead of dealing with it, you just hand-wave your way through so you can talk about my “attitude.”

      I understand why I have the attitude I have perfectly. I’m happy to discuss it if people actually care. However, I’m not going to have a discussion based upon you making claims about me while providing absolutely nothing to back them up, or even just to explain what you’re actually meaning.

      • Brandon – You have a point. I disagree with a number of your recent comments, including, as you surmised, the one regarding my energy budget analysis, but the word “many” was careless. Even where I disagreed with comments you made, others have already responded and so there’s no need for me to revisit comments now buried in the middle of long threads. I do hope future disagreements will have a moderate tone to them that promotes discourse and discourages defensiveness.

      • Brandon Shollenberger

        Fred Moolten, this response is useless. You say “others have already responded” to my comments, and thus you don’t to revisit the exchanges. This couldn’t be farther from the truth. Take for example my exchanges with Pekka Pirilä. About one third of my comments on the Postma thread were in response to him. Pekka Pirilä made certain claims, and I disputed them. You say you have no need to revisit them.

        However, the disputes between Pekka Pirilä and I were never resolved. I explained, time and time again, how Pekka Pirilä was saying things which were completely made up. He never refuted what I said (in fact, he didn’t even dispute most of it). Despite this, your comments have implied Pekka Pirilä was right, and I was wrong. Without revisiting the exchange, you cannot possibly support this.

        Put bluntly, you’ve made claims which impinge my credibility. You’ve provided no evidence for those claims, and you’ve now effectively refused to even address your claims. As it stands, I flatly state what you said about me is wrong, both on the correctness of my comments and the tone of them. Moreover, I find the fact you’ve made these claims while being unwilling to discuss them highly offensive. You’ve effectively done nothing more than insult me.

        I am willing to discuss the accuracy of my remarks. I am willing to discuss the tone of my comments. If you are going to make comments about either, you should be willing to do the same.

      • Brandon,

        You are really annoying. You are repeating continuously unjustified and false claims about my participation in the discussion. I entered the discussion with you with this comment

        http://judithcurry.com/2011/08/16/postma-on-the-greenhouse-effect/#comment-101121

        It was an answer to your comment that I felt as insulting towards people, who understood, how seriously the Postma paper was in error, but I don’t think that my own answer would be classified as insulting by anyone else than you. Your next answer brought the word “insulting” to the discussion.

        That’s the place, where it started. Anyone, who wishes to judge the behavior of me and you, can check the origins from there.

      • Brandon Shollenberger

        Pekka Pirilä, you say:

        I don’t think that my own answer would be classified as insulting by anyone else than you. Your next answer brought the word “insulting” to the discussion.

        I did not say “insulting” as you claim it did. I said “insult.” I normally wouldn’t care much about this distinction, but since most of my claims revolved around you making things up., I find it interesting. I’ve repeatedly criticized you for failing to get basic facts straight, and here you are messing up a simple fact.

        I am happy to let people read our exchange and make their own determinations, but I will say this. Most of our disagreements are as easy to understand as the one I just posted. They require nothing more than reading and comprehending simple sentences. With just a little reading, anyone interested will be stuck with one of two conclusions:

        1) You repeatedly made things up.
        2) I repeatedly made things up.

      • When people disagree so essentially on the nature of the context (here: what has been proven about Postma’s article, and how to judge, what has been proven), it seems impossible to understand at all, how the other side rationalizes his thoughts. I really cannot follow Brandon, and he evidently really cannot follow me.

        When we both feel that not only the details, but the whole logic of the other side is wrong, we end up being quite impolite.

      • Brandon Shollenberger

        Actually, almost none of our disagreements require any context about Postma’s paper. Most of it was simply a matter of what has and has not been said.

      • That’s exactly the point, where we disagree on the logic that should be applied.

        You stated your view above. My view is that the strength of the evidence presented in the thread is totally convincing and that’s essential for the right conclusions.

      • Brandon Shollenberger

        And yet most of what we disagree about has nothing to do with “the logic that should be applied.” We mostly disagree on whether or not certain things were ever said. You claimed they were. I claimed they weren’t. There is little room for logic in that. Either the things were said, in which case anyone could quote them and prove me wrong; or they weren’t said, and nobody could quote them and prove me wrong. It’s that easy to figure out who is making things up.

        When we get past the point of one of us just making things up, then we can maybe look at disagreements about logic. Until then, there’s no point trying to be logical with someone who just pulls anything they want out of thin air.

      • I made my claims, because I trust that I understand the arguments and know they are there,

        I can even now not understand what’s your basis for your conclusions. How can you know, what they contain, when you declare that you cannot follow the arguments.

      • Brandon Shollenberger

        I find it telling in one comment you again flagrantly make something up about me and complain about the difficulty of understanding my thought process. As I told you before, if you want to understand me, read what I say. It would work better than reading your delusions or whatever it is which causes you to constantly contradict reality.

        I believe you are capable of understanding simple sentences. I hope someday you apply that capability to our exchange.

      • Hey Shollenberger, If you understand how this works and have enough sophistication with science, you tend to gravitate to people who know what they are talking about. Those are the people that will advance the dialog and allow you to come up with innovative ideas. Pekka is one of those people.
        I know that it may seem difficult because the learning comes in fits and starts but you have to be patient. I think the goal here is to come up with a simpler more canonical explanation for energy balance than is available right now. I have a feeling that one exists but it will take time to find it.

      • Pekka, you are quite correct when you say this: “When people disagree so essentially on the nature of the context (here: what has been proven about Postma’s article, and how to judge, what has been proven), it seems impossible to understand at all, how the other side rationalizes his thoughts. I really cannot follow Brandon, and he evidently really cannot follow me.”

        This is an excellent first order description of what I, following Kuhn, call “talking past one another.” The debate here is full of this phenomenon.

      • Brandon, regarding this complaint of yours to Pekka: “I believe you are capable of understanding simple sentences. I hope someday you apply that capability to our exchange.”

        You misunderstand the nature of the confusion. There are very few simple sentence arguments here. Most points made require (1) many complex sentences over (2) a series of comments and exchanges. Moreover, considerable interpretation of these comments is required, because we are talking about complex models and complex physics.

        Confusion due to complexity is the challenge, so deep patience is required. Insults are not helpful. People need to ask more questions, as well as saying “I do not understand what you just said” a lot more often.

      • It is pretty frustrating. There is a chain of knowledge transfer and this discussion is missing a few links. I prefer the net approach to radiative balance mainly because it avoids confusion for the overall audience, If using independent fluxes, more detail of the source of the fluxes. It is pretty obvious there has to be a middle ground since the status quo ain’t working too good.

      • Brandon Shollenberger

        WebHubTelescope, I have to say I don’t understand your comment here. You seem to be praising Pekka Pirilä and suggesting I could learn from him, but that’s just a guess.

        Regardless, in the current exchange there is no learning to be had. There is no discussion of points. Pekka Pirilä has simply made untrue claims which he has refused to justify. I’ve directly accused him of making things up time and time again, and so far, pretty much nobody has attempted to show I am wrong (though it would be very easy to do if I were wrong). Given that, I don’t know what your comment could apply to in the current discussion.

        By the way, the goal is not to find a simpler explanation that we have available. We already have a simple explanation that works as well as any simple one could hope to work. However, simple explanations cannot truly explain the issues so there is no point in focusing on making new ones. It might be useful to try to simplify some of our more complex explanations/models, but that’s not what any of this is about.

      • Brandon Shollenberger

        David Wojick, your response to me is completely untrue. Pekka Pirilä and I are not discussing anything technical or complex. We’re stuck on a different issue. Namely, he keeps making things up about what people have said, and I keep calling him on it. That is very simple, and until it gets resolved, there is no complex discussion to be had.

      • David –

        . People need to ask more questions, as well as saying “I do not understand what you just said” a lot more often.

        I already made the following suggestion to Brandon as a possible form of discourse:

        “Jim – I’m confused about how you derived the opinion that Postma said that height could be obtained by observation. I’ve looked at the paper discussed in the previous thread and found that he said that one couldn’t use observation to establish that height.”

        Apparently he felt my suggestion was beneath even being worthy of a response. My theory is that he’s unlikely to be receptive to your suggestion either – although perhaps coming from you he will be more receptive.

      • Brandon, I seriously doubt that Pekka would make anything up. Why would he do that? What he says is obviously his understanding of what was said. If he misunderstands then that is good evidence that what was said was not clear. This is a confusion, so the challenge is to get clear about it, not to lay blame. Blaming is often diagnostic for confusion. So I suggest you stop complaining and work out the confusion, or drop the subject.

      • WebHubTelescope, I have to say I don’t understand your comment here.

        Good. I was responding to you but my praise was really directed at Pekka. My desire is to have Pekka keep on providing valuable insight. In other words, whether or not we talk past one another makes no difference, but I do want to stay on Pekka’s wavelength because I always get some info from his perspective.

        By the way, the goal is not to find a simpler explanation that we have available.

        Yes it is.

        It might be useful to try to simplify some of our more complex explanations/models, …

        Which you seem to agree, but apparently I butted in on a personality conflict.

      • Joshua, re ID I thought I did respond. What is the link to your comment?

      • Brandon Shollenberger

        David Wojick, you ask a question I asked myself::

        Why would he do that?

        The conclusion I reached is that it wasn’t his intention. Somehow he has genuinely come to believe things which are obviously untrue. The fact it is unintentional does not change the fact he is making things up.

        If he misunderstands then that is good evidence that what was said was not clear.

        It may be evidence to suggest such, but it is not evidence to demonstrate such. If you check the things I’ve said he has made up, you’ll see they are all easily verifiable (one way or the other). You’ll also see the disagreements are quite clear.

        This is a confusion, so the challenge is to get clear about it, not to lay blame. Blaming is often diagnostic for confusion. So I suggest you stop complaining and work out the confusion, or drop the subject.

        I’d be happy to try to work out the confusion. However, my accusations have generally been met by Pekka Pirilä refusing to address them. If he doesn’t engage on any of the points, there is nothing I can do about it.

        If you genuinely think the issues I’ve raised are unclear, could you please give me an example? I really don’t see how it could be more clear than him claiming somebody said something and me saying they didn’t.

      • Brandon Shollenberger

        I don’t know what it is with me and blockquotes lately. Maybe my forward slash key is broken or something, but I seem to keep messing up the close tags.

      • David-

        Both of these threads are a hopelessly jumbled mess – but here goes:

        http://judithcurry.com/2011/08/19/week-in-review-81911/#comment-102716

      • Brandon, this is a good example of confusion. To me, and I think generally, making things up implies deliberate intention. In many cases it suggests lying. You really meant misunderstands, so I was confused when you said Pekka was making things up. In fact we agree there was a misunderstanding.

        If you are claiming that what Pekka misunderstood was too simple for that, then I doubt you are correct. People often have a hard time understanding how they are misunderstood, for obvious reasons. In fact misunderstanding is very hard subject scientifically. Just ask the people who build intelligent tutors.

      • you should just all agree with me. Studies have shown that most people who agree with me are satisfied.

      • Brandon Shollenberger

        David Wojick, I did a quick check with Google out of curiosity, and I saw the phrase used both as implying intent and as not. That’s the way I’ve heard it used my entire life, so I don’t think most people will be thrown off by my using it. However, if you can think of a clearer, simple phrase with the same meaning, I’d be happy to use it instead. Unfortunately “misunderstands” doesn’t work. It makes no sense to say, “He claimed I said X, but I never did. He misunderstands X.”

        As for the simplicity of the problem, I don’t claim the issue was too simple for Pekka Pirilä to understand it. My belief is it is too simple for any clarification to resolve his misunderstanding. Whatever the reason for his failings, I don’t believe a lack of clarity in people’s comments had anything to do with them.

        Even the simplest sentences can be misunderstood.

      • Brandon-

        I mean this most sincerely:

        ZZZZZZZZZZZZZZZZzzzzzzzzzzzzzzzzzzzzzzzzzz

      • David W. –

        You may have missed it or simply had no interest in responding – but I wrote you a response about belief in ID and “skepticism” on the other thread – and would appreciate a response if you’re so inclined.

      • (Got lost in the nesting so am re-posting here.) Joshua, re ID I thought I did respond. What is the link to your comment?

      • This is all a bit absurd. Brandon has been going around saying that people (paraphrasing) couldn’t be more wrong, absurd, are “full of it,” etc. People keep trying to bring him back to the technical discussion about Postma’s theories, and he keeps wanting to focus on whether they’ve acknowledged that when he called them “full of it,” or whatever, he was correct.

        Although Brandon says that he can’t get back to the technical discussion unless an increasingly long list of people acknowledge his “implicit” insults as factual, IMO, he could actually easily do so by simply agreeing to disagree about some elements of the discussion and moving forward to examining the technical questions.

      • Joshua has his PhD is “Mommy Mommy” studies. He should be considered a valuable resource.

        LOL

      • Hmmm. Bruce, you’re now chasing me around on different threads?

        I appreciate the attention, but in all honestly, it’s getting a little creepy.

  48. Claes Johnson

    Thanks Judy for your response. It seems that we agree: there is something called the Tyndall effect or greenhouse effect, which is not to be confused with climate sensitivity as global warming from doubled CO2, because the Tyndall effect concerns radiation alone and climate is thermodynamics with radiation. You are skeptical and say that climate sensitivity is smalller than
    the 3 C of IPCC, probably smaller than 0.3 C. We thus agree on the basic question, and should combine forces. What about a common blog since we have so much science in common?

    • Brandon Shollenberger

      I’m sure Judith Curry would have more to say on this subject, but I can’t just ignore your comment. You claim Curry agrees with you on a number of points, but she does nothing of the sort. For example, while Curry is skeptical the climate’s sensitivity is 3 C, her skepticism isn’t one sided. She thinks the uncertainty around that estimate is understated, and that means she believes the sensitivity could be higher or lower than the “consensus” estimate. She certainly doesn’t believe the climate’s sensitivity is “probably smaller than 0.3 C.”

      I don’t know how you concluded you and Curry agree on all those issues, but I can ensure you, you are wrong.

    • Claes, glad to hear that you agree that the greenhouse effect exists. WIth regards to climate sensitivity, I am on record saying that it is likely between 1C and 6 C, we really don’t know (and my likely leaves room for values lower than 1C) because of the complexities of the nonlinear thermodynamic and dynamic processes.

      My suggestion to you is to distance yourself from the skydragon group (who do not acknowledge the existence of the tyndall effect) and join the serious skeptical discussion on the subject.

      • Dr Curry,

        No. There is no glass wall (except clouds which can be considered as walls for IR radiation ) to reflect IR radiation in the Earth’s atmosphere like real greenhouse.

        390ppm CO2 in the atmosphere can absorb and emit its respective wave bands in the complete LW radiation spectrum. According to the specific heat of CO2 at around 250K, it will require the CO2 to absorb 0.8KJ/Kg of CO2 to raise 1K. Before CO2 gain enough of 0.8KJ, CO2 has long dissipated its absorbed IR heat to its neighbouring gas molecules by convection, conduction and radiation. Other so call GHGs likely wise doing the absorption and emission likewise with their respective wave bands. Most of the Earth surfaces IR radiations wave bands not related to so call GHGs just go directly to the space conform to Planck’s radiation. IR radiations from the Earth surfaces (say 30C) toward air molecules (say -50C temperature) is reduced heat transfer in accordance with Stefan-Boltzmann’s radiation. If you assume N2 and O2 are transparent to IR radiation, then the N2 and O2 cannot reflect the GHGs wave bands towards the GHGs. All IR radiations radiated towards them go directly into the space at the speed of light with reduced heat transfer from the Earth ground surfaces. GHE in the atmosphere clearly violated Planck’s and Stefan-Boltzmann’s radaition laws as well as violated thermodynamics laws.

      • Sam, as clarification, the heating in a greenhouse has nothing to do with IR…the elevated heat comes from restricted convection. The net effect of IR in a greenhouse is as an agent of cooling.

      • Ken,

        Thanks for the correction.

        I understand glass house enclosure created restricted convection like our homes we have cooler inside temperature in the house and warmer inside temperature in the evening if there is no air-conditioning.

        Yes, glasshouse is a clothing for the plants growth inside just like clothing for us in the winter, to restrict convection and conduction heat transfers to keep us warmer.

        Theres is no such GHE in the greenhouse, just pure mis-interpretation of heat transfers.

      • Dear Ken,

        You are only partially correct.

        Glass is a solid. All solids absorb and emit IR. The glass of a greenhouse emits IR to the interior.

        Sunlight is both SW and LW.

      • Oh, I get it, Will. Excellent. The interior of the greenhouse is hotter than the outside ambient air so the greenhouse glass restricts (or “back radiates”) outgoing radiation and makes the interior temperature hotter than it would have been. In this case, if we made the walls from salt which do not interfere with IR emission, the interior of the greenhouse would be measurably cooler. Finally we can verify that modulating IR radiation can do some work. I love it. This is outstanding.

        Wait? What was that?

      • The glass is a solid which is heated by the sun Ken. Did you miss that point?

        It then radiates from both surfaces (take a piece of glass and leave it in the hot sun for a couple of hours) . There is also internal refection which raises the temperature above the ambient.

        Salt is a crystal in which the molecules are aligned uniformly and so IR appears to pass straight through but really it is being internally reflected in a uniform direction.

        Glass on the other hand consisted of a random alignment of molecules formed at high temperatures and therefore the internal reflection is also random, so there is a delay in the emission from glass while is equilibrates to the ambient internal/external environment.

        This is confirmed by the first of the Robert Woods experiment where the box with a glass plate takes longer to heat up than the box with the crystal.

        However, please feel free to take these points and deliberately muddle them all up so as to imply that it is all so incomprehensible, as is your apparent predisposition in life.

        By the way Ken, was it your idea to invite Oliver K. Manuel on to the Dragon Slayer team?

        Sure glad I pulled out of that little project when I did.

        http://mominer.mst.edu/2006/08/30/dr-oliver-manuel-arrested-for-multiple-counts-of-rape-and-sodomy-of-his-children/

        Was that below the belt? I hope so son.

      • Ken Coffman | August 22, 2011 at 12:36 pm | Reply

        Was that even remotely close to anything I said above?

        It seems you are up to your usual antics of deliberately muddling perfectly reasonable points to imply that they are incomprehensible as is, it would appear your raison d’etre.

        Perhaps that is why you invited O.K.M to join the “Slayers”.

        h t t p : / / mominer.mst.edu/2006/08/30/dr-oliver-manuel-arrested-for-multiple-counts-of-rape-and-sodomy-of-his-children/

        Sure glad I pulled out of that little project when I did.

      • There is absolutely nothing unique relative to nonlinearity with respect to the physical phenomena and processes occurring in the Earth’s climate systems. All real-world phenomena and processes are governed by the same fundamental descriptions; mass, momentum, and energy conservation and balances.

        Nonlinearity is the rule. Linearity is the exception

        Linearity is generally encountered only in highly specialized cases and these cases are almost always devised by the academy so as to increase the peer-reviewed paper count in approved journals having high impact factors. Plus, H-numbers can be easily increased by self-reference to previously published papers on linear situations.

        The same is true for complexity.

        The sole confounding factor relative to weather and climate is the concept of chaos as reflected in the associated sensitivity to initial conditions. Our understanding of chaotic response is based primarily on temporal chaos exhibited by (a) simple systems of (b) autonomous (c) non-linear (d) ODEs with (e) constant coefficients. None of these characteristics are shared with the mathematical models used for numerical weather prediction or GCMs. If an explicit function of time is added into the systems, it is certain that restrictions on the nature of the function will be necessary in order for theory of chaotic response developed on autonomous systems to be applicable. If the parameters in the constant-coefficient case are allowed to vary, then being implicit functions of time, restrictions will again be necessary.

        It is interesting that the famous ODE system of Lorenz was devised as a model of the models of weather / climate in use in the 1960s. The chaotic response is due to an exceedingly careful balance between energy input to the system and the viscous losses within the system. This careful balance allows for bounded response. The momentum losses are represented by a linear term. A non-linear representation of the losses in the model of the model of the momentum balance will very likely cause the system to attain a null equilibrium state. It is also interesting that the quasi-linear transport terms in the fundamental equations become non-linear terms in the model of the models equations. Conversion of mechanical viscous dissipation into thermal energy, frequently used as a mechanism for physical realization of the mathematical model, is not included in the equations. Further, the terms in the original fundamental equations, again quasi-linear, which are responsible for physical realization of transition from laminar to turbulent flow are not sufficiently represented in the model of the models.

        I think discussions of the mathematical basis for the GCMs can be improved if the focus is on only the unique properties and characteristics of the models. Non-linearity and complexity are not among these. Oh, and coupled multi-physics with multi-scale spatial and temporal phenomena and processes are not either.

        Corrections for incorrectos will be appreciated.

      • coupling two chaotic systems (atmospheres and oceans) adds substantially to the complexity, along with the range of time and space scales involved. There isn’t any engineered system analogue to this level of complexity. complexity does not equal nonlinearity.

      • How does the con-current flow of two fluids, or two phases of a single fluid, turbulent in each fluid or phase, both occupying a single flow field and thus coupled, differ from this example. Both are governed by the exact same fundamental equations.

      • Let it be heard here first…Madam Curry thinks climate science is more complex than any engineered system. The way I read this, she’s saying climate science is more complex than marvelous feats of engineering–like putting a man on the moon or getting a commercial jet into the air. Excellent!

      • Dan Hughes 8/21/11, 10:14 am, Planetary energy …

        DH: There is absolutely nothing unique relative to nonlinearity with respect to the physical phenomena and processes occurring in the Earth’s climate systems. All real-world phenomena and processes are governed by the same fundamental descriptions; mass, momentum, and energy conservation and balances.

        Nonlinearity is the rule. Linearity is the exception. …

        Corrections for incorrectos will be appreciated.

        You need to define linearity. In system science, which applies here, it is the property that R(aX + bY) = aR(X) + bR(Y), where R is the system response and X and Y are inputs. This property exists nowhere in the Real World. The Real World has no parameters, no variables, no coordinate systems, no algebra, no measures, no units, no dimensions, no initial conditions, etc. These are all man made constructs based on his language, logic, and observations of Real World projections on his senses and instruments.

        Linearity is a property of models, not the Real World. (Chaos, too.)

        Microparameter descriptions of the greenhouse effect are non-linear. A macroparameter description of the GHE can be linear.

        Some truly complex waveforms can be linear. A system model can contain multiple orders of differentiation and integration and still be linear. I have inferred from some posters here that they believe that feedback creates nonlinearity. Not so.

        The way system science works with this concept is in a transform domain, e.g., Laplace, Fourier, z-Transform. In transform coordinates, differentiations and integrations reduce to power functions and delays to exponentials. Plus in natural space, a system responds to its inputs according to ugly convolution integrals. In transform space, convolutions turn into multiplications. This introduces the concept of a transfer function, a concept not apparent in IPCC works. These properties raise the level of the test for linearity to the transform domain.

        Many systems, including almost all the tractable ones, can be represented by a rather complex forward transfer function plus a complex feedback transfer function and still be a linear system. The class of linear systems is huge, and it is far from straight line fits.

      • Alexander Harvey

        Jeff,

        I am aware of some or much of this, in so far as it pertains to LTI systems.

        Some of this thinking is included in the study of climate but it does not seem to surface often and does not seem to be particularly popular.

        At a first pass, the response to a doubling experiment is just the response to step function curve and is represented in the literature in that way. What I consider the more useful, the equivalent response to impulse function is not represented possibly because recorded responses to step functions are too noisy to differentiate, that is just a guess.

        However the frequency domain transform of the output of model experiments, perhaps most usefully the control runs which have no variation in the forcing do yield characteristic spectra and could be used to progress to plausible functions for the response to impulse function under the change of daomain. I am not aware that this is commonly done, I mean just that I haven’t seen it.

        I think that these approaches are useful and that they work at all, tells us something about the limits of the non-linearity of the system.

        An approach that is studied that I consider to be deeply akin, is via the FDT, I think that there are a number of researchers looking at fluctuation/dissipation in both the models and the real world.

        Elsewhere a climate scientist commented on the FDT approach a little scornfully in something of the form: “This is all very elegant but …”. Well it is very elegant and it does have some theroretical underpinnings.

        Mostly I am writing this as I am not sure that many people will know much of these approaches which is a pity in my view and to encourage you to write more. Also including a few links to the basic underpinnings would be helpful.

        Your: “plus a complex feedback transfer function” is much appreciated for I think the richness of feedback transfer functions is something that gets almost attention at the level of the blog and little in the published literature.

        Finally there is a direct link from the response functions to the statistical properties of the system, I have not seen this highlighted. This is I thnk very important as I think that there is a indication of LTP in the responses and this does impact the significance tests and can be quantified to a degree just from the historic records using transforms and some fitting under certain assumptions of the forced response.

        Alex

      • Alexander Harvey 8/21/11, 12:52 am, Planetary energy …

        AH: What I consider the more useful, the equivalent response to impulse function is not represented possibly because recorded responses to step functions are too noisy to differentiate, that is just a guess.

        The responses reported by IPCC are from GCMs. Those models use the radiative forcing paradigm, so they don’t have flow variables. This rules out transfer functions, the transform domain, impulse responses, and convolutions.

        AH: However the frequency domain transform of the output of model experiments, perhaps most usefully the control runs which have no variation in the forcing do yield characteristic spectra and could be used to progress to plausible functions for the response to impulse function under the change of domain. I am not aware that this is commonly done, I mean just that I haven’t seen it.

        Lindzen and Choi (2009) tried to estimate the feedback effects on climate sensitivity using satellite data. To do that, they had to come up with a model which had a flow variable to apply a forward (G) and feedback (F) transfer functions. So far, so good, except that their breach of conformity is likely why they had difficulty getting published. Then L&C tried to fit a straight line to F. This was a severe and unnecessary restriction. I wonder if they elected to use a straight line in the erroneous belief that that was necessary for a linear system? They might have fit other curves to the scatter between input and output, or estimated the pair of transfer functions from transforms of the input and output.

        AH: Mostly I am writing this as I am not sure that many people will know much of these approaches which is a pity in my view and to encourage you to write more. Also including a few links to the basic underpinnings would be helpful.

        These investigations into climate junk are overwhelming. Check my blog for more by clicking on my name in the post header. What I would like to see is objective experts from the parent fields of physics, chemistry, and systems science become involved, whether here or elsewhere.

      • This introduces the concept of a transfer function, a concept not apparent in IPCC works.

        What a load of crock. Climate scientists use convolutions all the time when they take a forcing function and work out the response function.

        What they also realize is they have to do this if the system does not show stationary principles. If the system is not stationary in time, then the transform techniques (i.e. Laplace, Fourier, z-Transform) you mention will not work.

      • WebHubTelescope 8/21/11, 1:07 pm, Planetary energy …

        WHT: What a load of crock. Climate scientists use convolutions all the time when they take a forcing function and work out the response function.

        Wow! That must have been a load, indeed! Do you remember where you read it?

        Readers here, meaning those who rely on every word in a sentence, will realize that what I wrote in that regard was the APPEARANCE of convolutions in IPCC writings. Do you know how many times the word convolution appears in the TAR and AR4 combined? Throw in all the titles to all its hundreds to thousands of references.

        If you said “once”, that was one too many.

        And remember, those two Reports contain the entire validation of the AGW model.

        And just as convincing, if you know how to do convolution in IPCC’s radiative transfer paradigm, you ought to publish it.

        WHT: What they also realize is they have to do this if the system does not show stationary principles. If the system is not stationary in time, then the transform techniques (i.e. Laplace, Fourier, z-Transform) you mention will not work.

        Of course we all know, or should know, the Real World is neither linear nor nonlinear, chaotic nor peaceful, stable nor unstable, dependent nor independent. It lacks numbers, so has neither infinities nor infinitesimals. It has neither ratios nor rates. It is neither stationary nor nonstationary. The mere existence of the Real World has to be an axiom of science. What we know of it is a consequence of its projections on our senses and instruments, shadows on our stage. Every one of those properties is a human creation belonging to man’s models of the Real World, statistical or Cause and Effect. The challenge to a modeler is to find a model that works, making the fewest assumptions that are reasonable and sensible for his capabilities. But his tool kit is immensely larger using the transform domain.

  49. Claes Johnson

    Judy you ask me to follow the Georgia Tech course material on Radiation and Climate Lecture 25: I find there the Kiehl Trenberth energy budget with massive back radiation and an estimate of the greenhouse effect G = 155 W/m2.

    Since you do not believe in back radiation, are you not misleading your students? And G = 155 W/m2 is also misleading, right?

    How would you explain this to your students?

    • Claes, I fear you will find that Dr. Curry plays with words such that you would be well advised to be aware that for the past year several of the Slayers have asked her to clarify if Georgia Tech has anything other than a zero-D model. She has never answered.

  50. Claes, you say od Dr Curry “You are skeptical and say that climate sensitivity is smalller than the 3 C of IPCC, probably smaller than 0.3 C”

    This sentence appears to be based on the same type of logic, reasoning and evidence that you based your arguments against the existence of the GHE on. Please show any comment of Dr Curry’s where she has even implied this view.

    Your dishonesty shines like a beacon when you make up such obviously false statements and helps to show that, if that is your approach to science, you are not to be trusted.

  51. Claes Johnson

    In what way did I misrepresent what Judy said? Maybe I misunderstood what she said and I am more than willing to correct myself if I only get to understand what she said. So what did she say which I misunderstood?

    I know she said that she does not like the term back radiation, yet she teaches it to her students. Maybe you are one of them? Or if you were, how would you react to the double message from Judy?

    • Claes, where did Dr Curry say “that climate sensitivity is smalller than
      the 3 C of IPCC, probably smaller than 0.3 C”?

  52. Claes Johnson

    Ok Louise: She said she did not believe in 3 C, so I guessed she meant something different, e.g. 0.3 C. The question is what number she considers to be a best guess? Can’t you ask her if you are one of her students and then tell me. To get Judy to answer questions is not easy.
    But she invites to discussion and criticizes my work, and then I think she should be open to answer my questions. Don’t you agree with me?

    • Claes,

      I do take your point about Judith’s position. It would be much easier to understand if she did agree with your figure of 0.3degC. She has answered your question here:

      http://judithcurry.com/2011/02/26/agreeing/

      “I think we can bound this [2 x CO2 sensitivity] between 1 and 6C at a likely [66%] level, I don’t think we can justify narrowing this further.”

  53. Claes Johnson

    Between 1 and 6 C? With a likelihood of 66 %? This is not science. It is nothing. Why does she speak when she has nothing to say? Why is she criticizing my 0.3 C if she has no science behind?

    • Claes, again do your homework. go to Climate Etc. category “sensitivity and feedbacks” and read my posts, the basis for my statement is explained there.

      • Claes Johnson

        I have done my homework Judy, but the trouble is that your students doing their homework get mislead and misinformed because you feed them with the Kiehl Trenberth budget with it’s back radiation which you say you do not believe in. Is this fair to the students?

        Yes, I do not any longer belong to the core of the Slayers, since I prefer to be independent and stay free from group thinking. Are you now open to a real discussion?

      • Claes, you are once again mis-stating Dr Curry’s position when you say “back radiation which you say you do not believe in”

        She has clearly stated although the label is misleading, the effect is a true one. Perhaps you have trouble understanding this distinction but most of us here do not.

        Whether one labels an object with four legs and a seat as a chair or a stool does not alter the fact that it is an object for sitting on.

      • Claes Johnson

        No Louise, it is not about labels but about physics we are talking. When Judy says she does not accept the term backradiation this is the same as saying that DLR is fiction, and that is to remove the basic pillar of CO2 alarmism. Since you are so ardently defending Judy you must be one of her students. But as a student the most important thing is to understand that what the teacher says or what you read in a book is not necessarily true. You have to think yourself. What would happen if you did that?

      • ‘When Judy says she does not accept the term backradiation this is the same as saying that DLR is fiction, and that is to remove the basic pillar of CO2 alarmism.’

        NO! Are you trying to misunderstand on purpose?

        Think about Shakespeare’s quote: a rose would still be a rose by any other name.

        In the same sense, the physics that is happening doesn’t care what we call it. Vibrationally excited GHG molecules will still emit IR isotropically, thus decreasing the ability of the earth’s surface to cool. There is no observable physical mechanism by which those molecules could ‘know’ what we call their radiative relaxation processes or how their ‘knowing’ would affect the physics that’s happening.

        Sorry, but that’s just nonsense.

        Whether it’s called ‘backradiation’, ‘DLR’ or ‘the hokey pokey’ is an totally irrelevant point and your continuing to make it is further degrading how the ‘lurkers’ here are interpreting your understanding of the basic physical processes, let alone your overall physical insight.

      • Claes is intentionally lying now. He knows full well Judith’s position and has now misrepresented her on several occasions. You have to not understand a single word of English to legitimately get this wrong. The best way to get rid of him is to ignore him.

      • Chris,
        Stop the insults it betrays your desperation. English is not the first language in Sweden. Claes may have misunderstood Dr. Curry’s words but he is NOT a liar. Please try harder to keep to a civil debate.

      • Chris, actually Claes is the one dragonslayer that I have some time for :)

      • maxwell says “Vibrationally excited GHG molecules will still emit IR isotropically, thus decreasing the ability of the earth’s surface to cool.”

        If this was true, and we concentrated that LWIR using the mirrors of a solar cooker, then the focal point should cool even more slowly, right?

        Wrong! The cooling rate INCREASES due to this supposed power source you “physicists” claim is more than twice as powerful as the Sun!

        http://solarcooking.org/research/McGuire-Jones.mht

        http://chriscolose.wordpress.com/2010/05/08/stoat-taking-science-by-the-throat-latest-posts-archives-about-rss-contact-profile-me-my-family-and-me-more-make-sure-youre-familiar-with-the-comment-polic/#comment-2363

        http://hockeyschtick.blogspot.com/2010/06/why-conventional-greenhouse-theory.html

      • Schtick,

        It’s interesting because the one link to Colose’s blog you have discusses an explanation of the solar cooker as being determined by two totally different processes. The comment claims that the cooker cools by both radiation (not dominated by the 2nd law) and convection (which is dominated by the 2nd law). Sooo, I’m not really sure what to make of it substantively.

        What’s also interesting is that Roy Spencer made a box like a solar oven to prove that the greenhouse effect, as I have explained it, exists.

        He even explicitly states,

        ‘This reduces the rate of cooling of the Earth’s surface (and lower atmosphere) to space, and makes the surface warmer than it otherwise would be.’

        And I said,

        ‘Vibrationally excited GHG molecules will still emit IR isotropically, thus decreasing the ability of the earth’s surface to cool.’

        Basically the same thing, but with some more baseline physics involved.

        So who should I trust? An anonymous guy on the internet or a former NASA scientist and expert in radiative transfer?

        I’ll give you a hint…it’s not you.

      • Surely the solar mirror will reduce longwave radiation reaching the focal point rather than increase it.

        This is because the solar mirror is blocking infrared emitted from the ground and replacing it with infrared reflected from the sky. Sky is colder, so the presence of a solar mirror would reduce the infrared reaching the focal point.

      • Max says “The comment claims that the cooker cools by both radiation (not dominated by the 2nd law) and convection (which is dominated by the 2nd law). Sooo, I’m not really sure what to make of it substantively.”

        Uh, both radiation and convection are in fact dominated by the 2nd law.
        And the Earth cools using both, so if we don’t alter convection, but increase “backradiation” by concentrating it, we in fact find the cooling rate INCREASES.

        Spencers “Box” does not concentrate or focus “backradiation,” but even so, what does he say happens:

        “The plate gets colder at night than the ambient temperature because it “sees” the cold sky, and is insulated from heat flow from the surrounding air and ground.”

        He then claims some unmeasured totally subjective cloud effects heated the plate intermittently, fails to account for instrument noise, etc. Read the comments on that “box” thread and his other “box” threads.

        Spencer is also as confused as the rest of climate science that an IR thermometer or pyregometer measures heat flow. It does not, as explained by Claes Johnson. From comments today at WUWT:

        Bob_FJ says: August 20, 2011 at 1:38 am
        Willis,
        It disturbs me that you still do not acknowledge advice that it does not matter how much EMR* is whizzing around, unless there is a potential difference (PD) between two sources of it. Otherwise, it amounts to nothing in terms of HEAT transfer. Furthermore, unless there is a change in HEAT level in matter, there is by definition NO change in temperature.

        Hockey Schtick says:

        Exactly, and that is why the physics literature expresses heat transfer on the potential difference (Ta^4 – Te^4):

        E_net = sigma (Ta^4 – Te^4)

        rather than using the form assumed by pyregometers

        E_net = sigma Te^4 – sigma Ta^4

        which assumes massive heat transfer is occurring from both cold to hot and vice versa, in violation of the 2nd law.

        The whole DLR heating red herring is based on this false assumption of 2 way heat transfer. Doing the calculations this way leads to false answers because it does not consider the fact that if a cold body could heat a warm body, the warm body would simply emit more heat to compensate. Doing the calculation properly using only the potential difference eliminates this error. The first equation above is analogous to proper dual entry accounting, the second equation analogous to single entry accounting resulting in “missing heat.”

        http://claesjohnson.blogspot.com/2011/08/how-to-fool-world-by-measuring-masive.html

        Maxwell says:
        I’ll give you a hint…it’s not you.

        Another classy ad hom from the alarmist side of the isle. If you can’t argue the facts…

      • Lolwot says “Surely the solar mirror will reduce longwave radiation reaching the focal point rather than increase it.”

        No, you are forgetting that the mirrors are focusing the LWIR from a large area onto a small spot, concentrating it many many times to thousands of Watts per cm2. If focused “backradiation” is thousands of watts at the focal point and the unfocused blocked LWIR from the ground is 396 W/m2, then the LWIR reaching the focal point is still much much higher than it would be without the mirrors.

      • It was already going from a large area to a small spot before because the infrared radiation is diffuse. It’s being emitted from all over the ground and through all the air. From any direction you look from the focal point infrared is heading in. Putting a mirrored dish in the way just replaces some of that energy from the ground with energy from above which is less.

      • lolwot continues to dig a deeper hole.

        According to TK, “backradiation” is 333 W/m2, ground emission is 396 W/m2. Are you, lolwot, seriously suggesting that 333 W/m2 concentrated to thousands of Watts per centimeter squared is exceeded by unconcentrated 396 Watts per meter squared blocked from the ground?

        If so, you have passed the physics exam for climate scientists.

      • This is why you should never cede ground to deniers. If you sacrifice a term such as “backradiation” to try and appease them it just hands them the opportunity to falsely tell everyone that you agree with them that backradiation doesn’t exist.

      • Claes,

        I’m trying to decide it matters whether the GH effect is explained in terms of back radiation or in terms of an effective height for the radiating surface of the atmoshpere. Adding GH gases and therefore making the atmosphere more opaque raises the effective height (the 255 degK level) and therefore we can use the adiabatic lapse rate to show that the surface will warm too.

        Are these just two ways of looking at the same problem? What’s your opinion?

      • Claes Johnson

        Back radiation is fiction and following Judy should not be referred to.
        The lapse rate argument is simplistic and may tell something about reality or not tell anything. Climate is thermodynamics with radiation
        keeping thermodynamics (lapse rate in particular) constant while varying radiation does not make sense as model of reality.

      • ‘…Back radiation is fiction…’

        Yes, it’s so fictional I can go out and measure it with an IR thermometer found at your nearest Crate and Barrel. Or there is even the cosmic microwave background point I’ve made now two dozen times. Not only can we measure radiation from the atmosphere, but we can measure downwelling radiation from outer space! And even detect with room temperature detectors that, according to Claes’ theory, should not be able to detect it.

        How does Claes get around these facts?

        Ignoring reality. It’s a convenient tactic, really. Because he MUST be right.

      • Maxwell, this tired old IR thermomer argument has been debunked numerous times, not least by Professor Johnson. Of course you can measure back-radiation. My Slayer colleagues have measured it and got -9C with my IR thermometer and professional units can find an average radiating temp of about -5C. However, a radiating lower frequency/entropy/temperature body cannot heat a higher frequency/entropy/temperature body at all. Yet the GHE requires the NET transfer of heat via radiation to be 60C from the colder atmosphere to hotter earth. (60C figure from Lindzen/Spencer).
        So please spare us that old garbage, thanks.

      • So you defend someone claiming backradiation is fiction by saying you can measure backradiation…

      • lolwot, can’t you distinguish the difference between radiation and heat? They are NOT the same. Jeez. we’ve explained this to you time after time and still you don’t get it! If radiation necessarily caused transport of heat then we’d need to think about fitting cooling fans on TV satellite dishes!

      • No, GHE requires nothing of the sort. The net transfer is from the earth to the atmosphere (see the KT energy diagram). The atmosphere prevents the earth from cooling as much as, for example, the dark side of the Moon. What is wrong with that idea?

      • If you want to argue that radiation doesn’t equal heat then say something like “radiation doesn’t equal heat”.

        Not “backradiation is fiction”

      • John,

        What you wrote does not make any sense.

        You wrote that you can measure what Claes claims is ‘fiction’, but then conclude that it cannot ‘heat a higher frequency body at all’. If the earth can absorb at all frequencies given the density of states of solid, how can the ‘back-radiation’ (as you’ve called it) not be absorbed by the earth?

        If that energy is absorbed by the earth, then it must change its energy. Such a change manifests itself as an increase in temperature.

        QED.

        You’re wrong…yet again.

      • Maxwell,

        Are you a part time clown by any chance?

        Your obsession with all things circular leads me to suspect that you may have spent much time circling very large rings on very small cycles.

      • Yes or no, John:

        However, a radiating lower frequency/entropy/temperature body cannot heat a higher frequency/entropy/temperature body at all.

        A “lower frequency/entropy/temperature body” can however retard heat loss “from a higher frequency/entropy/temperature body.”

        Yes or no, John?

      • But why is it fiction?

        If a molecule of a Greenhouse gas absorbs a photon of IR radiation from the ground and then later re-emits it, surely it will just as likely be in any direction?

        So back radiation will be 50% of the total absorbed, won’t it?

      • PS to my origical comment.

        Can’t we also measure back radiation from clouds? A cloudy winters night is always noticably milder than a clear night. So if from clouds, why not from GH gases too?

    • Claes,

      I wouldn’t agree with your comment “This is not science”. Judith may well be right in her assessment. On the other hand I can see where you are coming from with this. Your 0.3 degK figure may not attract much, if any, support from anyone else, but at least your argument, that there is no need to mitigate against a build up of CO2 makes sense on the basis of a claimed very low climate sensitivity.

      Its much harder to follow Judith’s logic that while there may be a 1/6 chance of climate senistivity being as high as 6 deg K , and an even higher chance of it being more than 3 degrees, there no real need to do anything just yet.

      She used to say things like “Think of risk as the product of consequences and likelihood: what can happen and the odds of it happening.” Not anymore though. I’m not sure why. That made perfect sense to me.

  54. No,
    The difference between Te and T0 does not arise (entirely) from the emission of thermal radiation by atmospheric gases and clouds at temperatures colder than T0.

    The main cause may very well be convection/advection. How much, remains to be seen. So to judge that, we need to do the null hypothesis, which is not quite as simple as the basic black body situation. We need to have an atmosphere fopr that, which is unable to absorb and reradiate electromagnic energy? That would give a true null hypothesis
    First off, ‘heat’ energy can be transported by three means, conduction, convection (advection) and radiation. What happens if we take the IR radiation effects away for the atmosphere in the null hypothesis?

    Also, instead of using an average steady state solar radiation, we need to realize that we have the diurnal cycle with max insolation radiation at noon and no radiation incoming when the sun is below the horizon. So during daytime the earth surface warms up and much more than the according the average radiation. Equilibrium temperature at the equator in a steady state with the sun in zenith, using the full incoming 1365 w/m2 (albedo 30%) would be 360K or 87C (Stefan Boltzmann).

    So this much higher temperature of the earth surface is conducted to the lowermost boundary layer of the atmosphere. This heated air gets is less dense, and it becomes buoyant, so it rises up; Convection, the very basics of meteorology. So at daytime the atmosphere receives thermal energy of the earth. How can it lose this energy again?

    Remember we are in the null hypothesis, no radiation, no greenhouse effect, so the inert atmosphere cannot lose the energy by radiation. Now, at night time the solid Earth does not receive radiation energy from the sun but it radiates energy out and cools quickly, obviously much more quickly than with the greenhouse effect, which would have directed (“reflects”) some radiation back to earth.

    Now, the cooler earth also cools the boundary layer of the atmosphere by conduction again, however there is no negative convection as the cool air gets more dense and tends to stay put; the inversion; also very basic meteorology. So despite the cooling of the earth, the missing radiation from the atmosphere prevents it from cooling at night and the next day more conducted energy is convected into the atmosphere. Actually the main convection cell on Earth, where the sun is directly overhead, rotates around the earth and it is constantly transferring energy into the atmosphere, no matter if this is a strong or a weak effects, the main thing is that it is one way only Obviously we have an unbalance. And equilibrium can only be reached, maybe after thousands of years, when the convection at daytime is stopped due to adiabatic balances.

    Conclusion, in the null hypothesis, without greenhouse effect, the average temperature of the lower atmosphere would be clearly higher than the average Stefan Boltzman temperature for the Earth as a black body without atmosphere, consequently it can not be that difference between Te and T0 comes entirely from the emission of thermal radiation by atmospheric gases and clouds at temperatures colder than T0.

    • The main cause may very well be convection/advection.

      That’s not possible, because convection and advection act always to reduce temperature differences, not to cerate them.

      • I guess you stopped reading there. I should have added the sentence: “if you think this is nonsense, read on”

      • You are right. The sentence contained a explicit claim that led to my response.

        In what you write lower down in the message has much in common with some messages that I have written both recently and earlier on this site. I have specifically argued that the atmosphere would be fairly warm in absence of all LW radiative energy transfer, because it would have no mechanism of getting effectively rid of energy that it has received from the surface. The only mechanism for energy loss from the atmosphere would be by conduction to the surface in areas, where the surface is colder. Conduction is so weak that the balance would be obtained at a rather high temperature.

        The air near cold surface would flow towards the warmer areas, but the positive temperature gradient near the surface would be so strong that the layer participating in this circulation would be thin and the total resulting energy flux relatively small.

      • Alexander Harvey

        Andre,

        I think it is sad that here and elsewhere, argument by discourse between paragraphs by one author is no well tolerated. To argue a point by dialoque is problematic. It is understandable that people have hair-triggers and that is to be tolerated.

        Fortunately we can come to know people’s style and read on with growing interest.

        Alex

  55. Maxwell wrote on the discussion about Postma’s paper and other attacks on the basic physics:

    In my mind, the battle has been won and because of this fact we just need to move on. If someone in the future is interested in assessing the value of the work that dragons are pursuing, these threads will always be here to help.

    That’s true. There’s not much value in continuing the discussion on, how specifically some particular paper is totally wrong. Certain issues that are not understood by many can be discussed in such threads, but after a while, the worthless noise covers all the little grains of value.

    We should indeed move forward. There are lots of important issues also in the atmospheric physics. We have seen that all of us learn something from such discussion. Even those, who know the physics very well, can often see new connections and learn, how to present the state of understanding better. The leading logic should be towards new understanding, not looking back and fighting multiheaded monsters.

    • Pekkra, more heavyweight intellectual are weighing into this debate. Just read the assessment of Dr. Pierre Latour released yesterday. Here is what he concluded:
      “it seems fair for this old Skeptic to adopt Postma theory until Postma is proven wrong, to my satisfaction.”
      Below is Dr. Latour’s assessment:

      Conclusions: Postma affirms there is no such thing as a GHG. CO2 causes slight global cooling rather than vanishingly small global warming.
      Atmospheric gases do indeed “trap” or hold more energy when CO2 increases because their heat capacity increases with CO2 exchange for O2. H2O works the same way and dominates. But if global average temperature is fixed at -18C, 5 km up, the temperature-altitude profile adjusts a bit and air becomes cooler at the surface, not warmer. This procedure works for all planets of all stars.
      Whole CO2 cooling package.
      Measure and analyze solar spectrum.
      Calculate corresponding T avg from radiating surface = 5778K using Plank’s Law of Radiation (1905)
      Calculate energy emission rate, w/sq mtr from Stephan-Boltzmann law W = kT**4 (Solar Radiance is 2.01*10**7 w/sq mtr – steridian)
      Determine dispersed reception rate at Earth by geometry 150 kk km away. The solar constant is 1366 to 1370 w/sq mtr at Earth.
      NASA measured 30% is reflected. Rest is absorbed and reradiated into space; constant input = constant output.
      So remaining 959 w/sq mtr absorbed and readmitted at corresponding avg T from Stephan-Boltzmann Law again. T avg = -18C.
      (It is important to account for geometry, stationary flat disk to rotating sphere. GHG model got this wrong.)
      Actual temperature at 5 km is measured to be -18C.
      Conservation of energy = KE+PE proves atmosphere KE and hence T decreases with increasing PE and altitude, determined only by gas heat capacity, Cp.
      (GHG theory got this wrong because it imagined a glass greenhouse in the sky, back-radiating energy from cold high altitudes to warmer surface in violation of Second Law of Thermodynamics.)
      Using Cp air, average T of air at surface is 14.5C.
      Cp depends on composition. Cp of CO2 is 63% greater than Cp of N2; Cp of H2O and O2 are 23.5% greater than N2.
      Temperature measures molecular motion and it takes more energy to move a heavier molecule of CO2 than O2 the same amount.
      This means a cu mtr of CO2 contains 63% more energy than a cu mtr of N2 at the same temperature and pressure.
      If CO2 increases from 400 to 500 ppm, and T avg remains the same where did the additional energy trapped in CO2 instead of O2 come from? Below! Cools.
      T avg at surface decreases 0.675C to 13.825C. GHG theory says it goes up about +0.005C, maybe.
      Whole temperature vs altitude profile pivots a bit at 5 km to accommodate higher CO2 according to these laws of physics.

      • This is what I could find about Dr. Pierre Latour is a simple google search.

        From the blog Sowell’s Law Blog, a post points out the opinion of Dr. Latour on the ability of CO2 to affect climate. It states as the quote of Dr. Latour,

        “CO2 only absorbs and emits specific spectral wavelengths (14.77 microns) that constitute a tiny fraction of solar radiation energy in Earth’s atmosphere. The first 50 ppm of CO2 absorbs about half of this tiny energy, each additional 50 ppm absorbs half of the remaining tiny fraction, so at the current 380 ppm there are almost no absorbable photons left. CO2 could triple to 1000 ppm with no additional discernable absorption-emission.”

        So Dr. Latour is of the opinion that CO2 cannot affect climate because the solar spectrum does not overlap with the absorption spectrum of CO2.

        WHAT?!

        The greenhouse effect occurs because of THE EMISSION OF RADIATION BY THE SURFACE OF THE EARTH! NOT SUNLIGHT!!!

        Then he throws in the whole ‘saturation’ argument, which has been thoroughly debunked by a great many skeptics at this point.

        It seems to me that Postma would want to distance himself as much as possible from this guy because his understanding of the basic physics involved is even more flawed.

        So again John, major fail on the PR front.

        Not surprising though.

      • Maxwell, are you actually saying that Latour’s assessment of Postma is invalid because of Latour’s prior beliefs? Surely we all have the right to finesse our views and form a newer understanding. Isn’t that how science advances?
        All you’re doing, Maxwell, is attacking the man not the argument which is typical desperation tactics from warmists. Sadly for you the strength of interest in the arguments presented by Postma has drawn in big hitters like Latour and many other top specialists from various disciplines who are most condemnatory of the slapdash standards of those generalists (climatologists) who pontificate across specialisms in which they have no training. I do believe Dr. Curry is a case in point having admitted she tried to study advanced thermodynamics but gave up.

      • John,

        ‘Sadly for you the strength of interest in the arguments presented by Postma has drawn in big hitters like Latour…’

        I’m sorry, what’s Dr. Latour’s record of publication on radiative transfer, molecular spectroscopy or the thermodynamics of atmospheric processes?

        Oh wait, I know what it is because I just did a search on Web of Science and guess what? He has no such publications!

        And the only evidence that we can find as to his expertise on the topic of CO2’s effects on climate are a quote pointing out that CO2 doesn’t absorb that much sunlight.

        Well, great! So if he’s now changed his mind he should be up to the comprehension level of a high school sophomore. Exactly the type of fellow anyone would want to ‘throw some weight’ behind a defunct scientific thory.

        Here’s a tip. Just because you can say the same thing over and over again (the correctness of the work of Johnson and Postma) doesn’t make it true. I know that’s hard for you to believe, but it’s the truth.

        You should look into it.

      • John,

        this is even better,

        ‘I do believe Dr. Curry is a case in point having admitted she tried to study advanced thermodynamics but gave up.’

        …right, she gave up just in time to write a text book cited above.

        What’s your personal expertise with thermodynamics again? Do you have an advanced degree in physics, chemistry or another physical science?

        Oh yeah, I forgot. You’re just a science poser.

      • …oh, and Dr. Pierre Latour worked for 40 years as a chemical engineering in petroleum processing. Coincidence?

      • Again, Maxwell, you put on your pointy foil hat and whine ‘conspiracy’ when you can’t think of a valid scientific argument. Why don’t you accuse Phil Jones and everyone at CRU of being in the pay of big oil, because Jones and others involved in the Climategate scandal have been massively funded by Shell and BP since the 1980’s or didn’t you know that? By your paranoid reasoning they are discredited too!

      • I don’t trust Jones anymore than I trust you. That’s the confusion you have. You think that if anyone points out your flaws, it’s because he/she is part of the ‘other side’. It’s the same mistake Ken made with me over 6 months ago.

        I just care about sound science. And everything that you’ve put your name behind is about as unsound as promoted science gets these days.

        So yes, I can point out that Latour worked for oil companies for many years and this fact may or may not play into his interpretation of the work by Postma. I can also simultaneously say that the way that the CRU handled their data and processing was less than stellar. It certainly raised my eyebrows.

        So there is no dichotomy in my case. The science you promote is just as flawed and problematic no matter how many ‘heavy hitters’ support it as is the processing supported Jones and his ilk. To me, the two of your camps are in the same boat. Just on opposite ends floating off to a fantasy land…

      • Yes, I made a mistake which cost me a hundred bucks. Score one for Maxwell. I don’t mind the money so much, people pay for their education all the time, that’s fair, but I suppose I’ll be hounded by the doctors w/o borders group with their junk mail for the rest of my life. Blah.

      • Your endless efforts in trying to discredit the messenger rather than the message tells me all I need to know about you Maxwell.

      • Is that really what you’ve taken from my consistent criticisms of the theories of both Postma and Johnson? That I’m being critical of the messenger?

        Is that what you thought when I pointed out that Claes’ theory violates the first law of thermodynamics, the conversation of energy, by claiming that, in the presence of a cooler blackbody, a warmer blackbody will spontaneously emit MORE low frequency radiation? Or is that what you thought when I pointed out that I could calculate the entire optical response of the atmosphere showing explicitly that it emits radiation down toward earth that the earth HAS to absorb based on basic quantum mechanics and the Maxwell-Bloch equations? Is it what you thought when I ALREADY pointed all of this out to you 6 months ago?

        Or when I showed Postma that he was confusing linear and nonlinear response to sunshine due to the fact that sunlight is no where intense enough to access the higher order terms in the susceptibility of the material on earth? The night turning to day is not a ‘step function’ in the physical response of the earth in terms of climate. There is nothing nonlinear about the absorption of sunlight on earth.

        Even in the case of Latour, the very first point I made was that he could not even identify the pertinent physical concepts involved in understanding the role of CO2 in climate. The fact that he works for the petroleum industry is just a sidebar to that little gem. He proved that he had the comprehension of a 7th grader with respect to the greenhouse effect.

        And with all of that technical criticism you’re going to claim that I am going after the messenger?

        I wish I could say I don’t believe it, but at this point, with nothing else for you to really claim, it’s not that surprising to me.

  56. Of all the things we know, we know this; heat on the surface of the Earth does not travel through the Earth to the other side.
    Now, armed with this piece of knowledge, let use prepare two models of the temperature distribution of the Earth.

    We take a pair small soccer ball sized spheres, into which we have arrayed 60 icosahedrally positioned temperature sensors. The two balls have a non-conductive pole running through the center, with the sensor cables running through them. This allows the two balls to be rotated.
    Next we paint them. We are going to cover the surface with a mixture of carbon black and titanium oxide. The pole regions have higher TiO2 and the equator, more carbon black; The two spheres have identical changes in albedo, from pole to equator. All we are trying to do is match a ‘model’ Earth.
    Now the actual point of the experiment; does a temperature driven solid/gel transition change change the properties of radiative transfer, making averaging meaningless.
    The matrix we use to adhere the carbon black and TiO2 to the two spheres consists of two different grades of silicone oil. The first has a transition temperature of >50 degrees and the latter 0 degrees (273K).

    Now all we do is place the two spheres in a black painted NASA vacuum chamber. We make the two spheres rotate at 1 rpd at a 23 degree angle, As well as monitoring the temperatures of the sensors we also have a bank of IR sensors measuring the efflux from the spheres.

    Now we illuminate on face, and allow the two systems to come to steady state. During the ‘day time’ they will absorb light, matched to that of the Earths own incoming light flux , during the ‘night time’ they will radiate IR.
    Here is a small bet, $1,000. If you use such a system, you will find out that :-
    The kinetics of temperature rise on the ‘day’ face and drop on the ‘night’ face are different in the phase changeable silicone model vs. the non-changing.
    The Tmin and Tmax are different.
    The thermal distribution across the spheres will be different.

    Moreover, in my models, we can look at changes in day length, angle, seasonal cycles and the amount of heat required to produce a phase change.

    This sort of model would not only be cheaper than the various climate models beloved of climate scientists, they would actually give you real data.

    • Using scale models is a well known and useful tool in engineering. The models agree never fully on the real system, but in many cases it’s possible to construct models that behave in a very similar way. That requires always that several different properties are matched. In fluid dynamics Reynold’s number is usually the most important parameter that must have the same value in the model as in the real system. Prandl’s number is another common important parameter.

      In the example described by DocMartyn some critical numbers must also be matched to get results of any significance. Without going to the details, it appears totally clear that the small balls must be made to turn much faster than the Earth turns. I cannot tell, what would be the most appropriate length of day, but it might well be closer to 1 min than to 24 hours.

      Such an experiment tells practically nothing unless it’s supported by a careful theoretical analysis that tells the right combination of parameters.

  57. What if…?
    II Peter 3:8 But you must not forget this one thing, dear friends: A day is like a thousand years to the Lord, and a thousand years is like a day.

    1000 years, times 360 days= 360,000 night-day cycles (NDC)
    Divided by 24 heavenly hours= 15,000 NDC per HH
    Divided by 60 heavenly minutes=250 NDC per HM
    Divided by 60 heavenly seconds=4.167 NDC; as revolutions per second… for your model?

  58. A. C. Osborn

    I find it very interesting that all the “Scientists” on this thread arguing the case for AGW can’t explain with their Science why it isn’t working as described.
    Every day new Peer Reviewed papers are being published showing that the CONCENSUS is wrong and the Earth is not behaving the way that their science describes that it should, especially the models.
    Yet they won’t admit that anything is wrong.

    It is a fascinating Forum and very enlightening.
    Thank You Dr Curry.

    • A. C. Osborn, I am in complete agreement with your observation. When the scientists ignore the creator’s model… It must be that there is no ‘peer review’ or it is to simple.

      • A. C. Osborn

        Tom, the same applies to the NASA Scientists studying the Sun, they have been proved wrong time & time again over the last 2 decades and yet they pronounce their theories with such conviction. Some years later they have to have new theories because the old ones were proved wrong by the Sun.
        The way the Conveyor system works is a classic example.

  59. It’s never ending…

  60. “A. C. Osborn
    I find it very interesting that all the “Scientists” on this thread arguing the case for AGW can’t explain with their Science why it isn’t working as described.”
    Pekka Pirilä’s dismissal to the suggestion of an actual experiment, designed to explore one facet of the Earth, phase transition, says it all.
    Climate science is not a field of science; we have keyboard monkeys whose models show that clouds increase the surface temperature and never test it by taking a stroll.
    Although Judy tries, this field is dead as far as data collection and experiment method is concerned.
    Pity really, because it is rather interesting.

  61. Boy it’s hot outside, I must be getting more than my share of 240 W/m^2 :)

    Since the reply things have run out for the more interesting stuff, I going to put some comments here:

    Someone said that latent heat warms the atmosphere. From the NASA energy budget cartoon (yes, it just uses nets) 36% is due to latent heat, 23% is due to net radiation flux, 25% due to absorbed incoming solar, 5% due to clouds absorbing solar, and 11% by conduction and rising air. These are just the percentages of the atmospheric warming. These numbers disagree with the KT energy budget a little, but are pretty close. For the KT the net energy absorbed by the atmosphere from above and below is 198. So the atmosphere (air and clouds) absorb 39 percent instead of 30, latent is 40% instead of 36%, net radiation flux is 11% instead of 25%, thermals are 8.6% instead of 11%. So latent and solar are major players in both budgets for atmospheric warming, with 61% in the NASA and 79% in the KT budgets. It looks like the biggest reason for the difference is because KT has the surface absorbing 4% less than NASA. That 4% is going to the atmosphere where it results in about 8% more solar absorbed and about 3% less thermals. I have no Idea how KT came up with the only 11% absorb by outgoing radiation.

    Tallbloke mentioned “new” energy as in not recycle infrared. The 30% NASA or 39% KT would be brand new energy from the sun absorbed by the atmosphere which tends to warm things. The 36% latent NASA or 40% KT would be slightly used which most folks will agree helps to keep things warmer, the rest would scratch and dent specials that people can haggle over. So 66% NASA or 79% KT would be in the new or near mint condition aisle for full price.

  62. Perhaps a bit late, but about the nonsense Claes has been peddling, here is an article from 1916 where the thermal radiation from the atmosphere was detected essentially for the first time using a device sensitive to heat. Quibble away but RTFR

    • Nice! According to Angstrom Earth doesn’t receive more radiation from Atmosphere than from the sun.

      “Did you ever realize that we receive almost as much heat in a year through radiation from the atmosphere as from the sunlight itself?”

      • Edim,
        Angstrom’s got the tail wagging the dog on that one. All Earth’s radiation comes from the sun – gases do not generate ANY heat, they are heat transporters.

      • Do gases generate radiation though?

        Yes I saw what you did there.

      • Read Postma’s paper properly. Energy in = energy out in his budget and he does it without the need to include a greenhouse gas effect. Conservation of energy demands that heat cannot be created so that there is no way that gases can add any extra energy to the equation.

      • John, Most of the heat in the atmosphere is generated directly by the sun and latent heat. The gases aren’t generating heat, they are just inefficiently transporting heat kinda like the post office does mail. If you look at the top of the troposphere, the total energy in is 312 W/m^2, so a lot of that energy from the sky is week late next day mail.

      • sorry, 305 W/m^2 using the 19% estimate for clouds and atmosphere by NASA.

      • Buy that set and you are going to admit that the surface is not generating heat, it is just inefficiently transporting heat kinda like the post office does the mail.

  63. Thanks to Dr. Curry for the link. I was hoping to find something like Trenberth’s budget diagram with indicators for quantities as directly measured, derived or theoretical. It will simplify deciding if something is debatable or not. There’s no point in the Postma debate about areas if the incoming solar flux has a direct measurement.

  64. I read some of this. If there is another thread on “Slaying the Skydragons” I’ll skip it entire.

  65. For those who believe an atmosphere with no greenhouse gases would be isothermal, propose for me an experiment to demonstrate this phenomenon.

    If the surface is warmed by solar radiation, and outer space is a vaccuum with a very low temperature, how can the atmosphere with no GHGs possibly be isothermal? At what temperature will this isothermal atmosphere be??

    Anyone who thinks Tyndalls experiments demonstrate heat trapping need to properly read tyndall and have a think about where scattered energy might go in Tyndall’s gas tube.

    • BLouis79, this is climate ‘science’, you shouldn’t do experiments in case you get the wrongs answer.

    • It doesn’t matter how “cold” outer space is. Energy can only be lost from the atmosphere into space by radiation and if the atmosphere is just nitrogen and oxygen, hows that going to happen. Surely in that case the Earth can only lose energy by emitting it from the surface. I don’t know if the atmosphere would be isothermal in that case, but the surface certainly would be a lot cooler.

      • Wrong. Energy can be radiated directly from the Earth surface to space without interception with any GHGs which are not in the energy paths. Space between molecules are huge compared with their molecular sizes.

      • Sam NC,

        Lolwot isn’t say that. You’ve misunderstood.

        Yes energy can be radiated directly into space but because the atmosphere is not totally IR transparent, some ground radiation, not all, is absorbed by GH gas molecules and then re-radiated.

      • tempterrain,

        I don’t know why you defend lolwot. Worth it? His insulting comments were deleted by Dr Curry.

        According to warmists GHE theory, GHE can only absorbed their respective spectral lines, presumably all non GHG spectral lines will be radiated directly to space from the Earth surface. Not all GHG spectral lines will be absorbed by the GHGs as the space occupied by these GHG molecules is minimal and 99% of these spectral lines also go to space directly. Only very minimal portion of the Earth surface radiated IR spectrum absorbed by the GHGs assuming they are 100% effective absorbing their respective spectral lines. OK, here I was very generous towards GHGs absorption portions 1% of the total Earth surface LW radiation.

      • This is kind of amusing. The wavelength of IR light from the surface ranges from about 5 microns to about 50 microns. The space between molecules at atmospheric pressure is about a tenth of a micron. The size of a molecule is about .0002 microns.

        Of course you have to wonder how the small molecule absorbs the huge photon, which it does. . .

      • Eli,

        Nice catching. I thought all AGWers’s are blind. Obviously you are an exception. Anyway, thank you for correcting.

      • No I don’t understand what an IR photon is, and there is probably nobody with this understanding, not even WebHubTelecope if you seriously ask yourself and not just repeat a mantra.

      • Claes Johnson

        To explain IR by photons does not make sense. If you read my treatise on blackbody radiation available under upcoming books on my blog, then you will find IR as a collective wave phenomenon involving many atoms which may give a better picture of the physics than mysterious giant photons being captured by miniscule atoms.

      • The photoelectric evidence anyone? The workings of an infrared laser?

        I seriously want to understand the way your mind thinks because it does not seem to have a collective filter. Good theories do not cherry pick the evidence.

      • What evidence am I cherry picking?

      • You can’t ignore the photoelectron effect (by which the energy of photoelectrons emitted by a surface is independent of the amount of light falling on the surface). That makes light a particle known as a photon in particular cases.

        But the broader point is that it doesn’t matter whether light is a wave or a particle. A CO2 molecule (as a collection of three atoms and their electrons) will absorb and emit light. Quantum optics, on which all these assertions are built, predicts behavior which we observe in the lab. Therefore we accept it.

        You’re ignoring both of these facts in order to promote your theory.

        Your theory also predicts that a 300 K radar dish detector won’t absorb cosmic microwave background radiation, which it clearly does. So, to me, that’s enough to know it’s wrong.

      • Do you even understand what a photon is? The idea is that energy is quantized. So if you wanted to spectrally excite an electron from one discrete energy level to another in a material, but if the energy level of that photon was not great enough, then it wouldn’t matter how intense the radiation source is — it wouldn’t budge the electron from the grounded state. This works across the spectrum, from infrared on up.

        I am willing to be embarrassed if this is not the case and your theory holds true. It would be worth a scientific prize or two.

        One other thing, it takes someone with crackpot theories of his own to root out other people’s. The difference is that mine are correct.

      • Maxwell: A radar dish can sense IR by amplification of the signal and thus is not a simple black body sensing little.

        The photoelectric effect can probably be explained by wave mechanics, see my book.

        Yes, it matters if IR is waves or particles, because believing it is particles makes you believe in back radiation which is fiction and not physics. IR photons is fiction without physical reality. You must make a distinction between fiction and physics.

      • Claes,

        you’re missing the point.

        Your theory clearly states that a warmer blackbody cannot absorb ‘low frequency’ radiation from a colder blackbody.

        A radar dish and its detector at 300 K clearly emit a spectrum of radiation based on their thermal energy that is peaked at a higher frequency than outer space, which is between 2 and 4 K. So while I agree that such a dish and detector are not ‘blackbodies’ in the sense that nothing in the real world is a ‘blackbody’, the spectrum of thermal radiation they emit is well described by the blackbody spectrum and thus it should not absorb radiation from space according to your theory.

        Furthermore, the fact that such an apparatus would use an amplifier is neither here nor there because the equations you derive do not include some parameters for amplification of the radiation. The fact that ANY radiation is absorbed to be amplified in the first place is a observation in violation with your predictions.

        As for the photoelectric effect, there is no wave explanation, but let’s assume there is.

        How then do you explain cavity QED, the Lamb shift, resonance fluorescence and dressed states in atoms without photons? All of those theories have been tested to many more significant digits than your own and have been proven correct time after time. Each and every one of them necessitates second quantization of the modes of the vacuum field that allow for the existence of light. And the quantization itself leads to photons!

        So not only is your own theory incorrect, you’re pointing at theories proven correct by 4 decades of experiments and calling them incorrect.

        I just don’t think we’re living in the same world anymore…

      • I am guessing that your blind spot is perhaps of never understanding the concept of statistical physics, and the role of probabilities in ensemble particle systems. That is what your “generic emergent phenomenon” is all about; disorder in the state space is a pretty generic phenomena and it is described by Bose-Einstein statistics effectively.

      • Yes, it is true that I do not understand statistical physics, but I am in good company. Can you name a person who claims to really understand? You?

      • I understand it very well. I had a great instructor in grad school who introduced us to the classic textbook by Reif, “Fundamentals of Statistical and Thermal Physics”.

      • Congratulations! A good instructor can teach anything.

      • Claes,

        Thank you for the link. I always think photons are within visible spectrums only and do not exist at any other wavelengths. However, people here at Climate Community always refer to IR radiation as always have photons that extend beyond the visible spectrum as in the old school learning. Is there a definition of photons generally accepted by scientists, physicist, engineers, hydrologists …

      • A photon is a discrete package of light energy. Almost like a particle. The amount of energy it contains is related to its frequency. That is, higher frequency, higher energy.

        Photons are correct way of understanding optical physical in any energy region of the EM spectrum, not just visible light.

    • Heat trapping is a very poor term that that I rarely hear. Delay is much more descriptive and pretty easy to see if you compare the dry and wet lapse rates. More water vapor in the tropics means the energy has to travel further to escape.

      Comparing Earth to a no greenhouse planet is close to an exercise in futility because of all the water. Mars with a lot more water would be a different world. Venus’ atmosphere would still be isothermal is it was water instead of CO2.

  66. BLouis79,

    An atmosphere with no GH gases wouldn’t be isothermal. You’d have to remove gravity for that to happen.

    I’ve just posted this link in a reply to Pekka but I think it is worth duplicating here:

    http://earthguide.ucsd.edu/virtualmuseum/climatechange1/02_1.shtml

    • Tempterrain,

      You are still wrong. I answered to your previous post.

      You are not totally alone in your view as you can see, if you check the discussion I continued on my own site

      http://pirila.fi/energy/random-topics/#comment-52

      but I don’t think that the opposing side in that discussion shared your simplistic view of the situation either.

      Removing gravity would remove the atmosphere. The gravity does not lead to the adiabatic lapse rate without convection and convection dies out without radiative energy transfer in the atmosphere. When it has died out the temperature profile approaches slowly the isothermal state for almost all of the atmosphere. differences in surface insolation maintain low altitude circulation for ever, but that cannot penetrate far and the rest is isothermal in equilibrium.

      How do you explain the stratosphere?

      • Pekka
        I asked you this question recently but your answer here seems a bit different.
        On a dry day with clear sky and very still conditions (little or no convection) what would be the temperature profile of the troposphere.
        Consider a vertical section which is heated from surface and cools by radiation at the top.

      • A column of dry air heated by warm surface from below and radiating from the top has the temperature profile determined by the adiabatic lapse rate, up to the altitude where the convection stops. That altitude is determined by the radiative energy balance. Convection is present, when the radiative energy balance is negative, but stops, when the radiative energy balance goes to zero.

        Above that altitude the temperature profile doesn’t follow the adiabatic lapse rate, but is determined by the radiative energy balance alone. The lapse rate based on the radiative energy balance alone is smaller than the adiabatic and approaches gradually zero with increasing altitude. The gradient is not exactly zero, because the lower and warmer atmosphere warms by radiative energy transfer the layers the more the smaller the distance to the troposphere.

        (In the Earth stratosphere the absorption of solar radiation reverses the temperature gradient to positive).

      • In meteorology theory the departures from the dry adiabatic lapse rate(DALR) are explained by latent heat considerations.
        You seem to agree by saying that in such circumstances
        …”Convection is present, when the radiative energy balance is negative, but stops, when the radiative energy balance goes to zero. “…..
        I have tried to fit the radiative slab theory on top of the DALR but it seems redundant.
        Since CO2 is well distributed in the atmosphere, its not easy to understand conditions where “radiative energy balance goes to zero.”

      • Pekka,

        I agree that the real atmosphere, including the stratosphere, is a complex system, I wouldn’t wish to oversimplify the problem and claim that a gravitational field can fully explain each and every observable.

        But before we discuss what can’t be explained by the Earth’s gravitational field, we need to establish what can be.

        So if we consider the case of a vertical column of dry gas in a gravitational field and subject to no localised heating, does or does it not exhibit an adiabatic temperature gradient?

      • If the column is not exchanging energy at all, it will become isothermal. That point is discussed in numerous messages in this thread, and in many other threads that discuss atmospheric physics. If that would not be the case, it would be possible to build a perpetum mobile of the second kind (i.e. a machine that makes mechanical energy from a single source of heat).

        The natural intuition that the temperature must be warmer at the bottom, because molecules gain energy, when falling, is wrong. It can be wrong, because it’s exactly compensated by another phenomenon: Those molecules that have little energy cannot go up as well as those with more energy.

        Imagining that we could count separately the molecules of each speed in the equilibrium column, we would notice that for every speed separately, the density decreases exponentially with altitude and with the same exponential rate. Therefore the ratio of fast moving molecules to slow moving molecules is the same at every altitude. For the isolated column the only result of gravity is that the density decreases exponentially with the altitude. The distribution of velocities and the temperature are as uniform as they would be without the gravity (if the gas is kept inside a closure and cannot escape as it would without the gravity and closure).

      • Pekka,

        Thanks for your comments. I first began thinking about this problem just over a year ago now and my initial thoughts on the problem were very much in line with what you are saying. I had made the comment on a contrarian website that falling temperatures with increased altitude was evidence of a GH effect and was, of course, howled down!

        I came across the San Diego university site and at the time they had some contradictory information on there. I emailed Gavin Schmidt who was kind enough to suggest I should contact Jeff Severinghaus at San Diego Uni. They were both of the opinion that a non greenhouse atmosphere would not be isothermal, and the San Diego website was altered to remove the contradiction, but if what you are saying is correct, maybe in the wrong direction.

        I think this is an important point which should be resolved. I’d be happy to forward the relevant emails if you’d like to get in touch.

        Regards
        peter_martin_2001@hotmail.com

      • Tempterrain,

        Higher up in this thread WebHubTelescope, Fred, Alex and I discuss the issue based in part on a paper by Verkley and Gerkema. Another paper related to the issue is Berberan-Santos, Bodunov, and Pogliani: On the barometric formula, Am. J. Phys. 65 ~5!, May 1997 that I have discussed on my site. This paper explains in some more detail, why the exponential barometric formula and the exponential in the Maxwell-Boltzmann distribution indeed cancel each other. I have extended that argument in some of my comments here and on my site. In our earlier discussion Quondam brought up the simple argument that really proves that anything else than isothermal allows building a perpetum mobile of the second kind.

        Several of the most knowledgeable participants in the discussion on this site have also mentioned in passing that they know that the result is isothermal. That’s not most significant, as they have done it in passing.

        I don’t really see that there’s any uncertainty left for the isolated and insulated column of gas in gravitational field. The atmosphere is, however, not a simple column even, if there’s absolutely no radiative energy transfer, as the Earth surface doesn’t have a uniform temperature. I have argued that even so almost all of the atmosphere would be isothermal stratosphere and that the circulation maintained by the temperature differentials of different parts of the surface would be restricted to a thin layer close to the surface.

        I cannot tell, how thick this thin layer would be, perhaps a few hundred meters, but that’s only a guess. Calculating that should not be particularly difficult, but still a too large effort for me to consider worth the effort at least for the moment. The answer would be dependent on the assumed/estimated albedo of the Earth surface. It would be different for a snow ball earth and for an earth where either the equatorial regions all the whole surface would be free of ice and snow. The heat conductivity of air is one of the most important parameters in the determination of the thickness of the circulating layer, as the positive temperature gradient over cold surface is the larger the lower the conductivity. A large gradient means that the layer is thin and the slopes of the surfaces of equal temperature small. From that follows further that the circulation maintained by the density difference of warm and cold air weak.

        One of these surfaces of constant temperature touches the ground on the warm side near the maximal surface temperature and has the maximum altitude in the dark side. I expect that the temperature is essentially constant as soon as we have gone above that surface. That sloping surface would be essentially the lower limit of stratosphere.

      • Alexander Harvey

        Pekka,

        This is a subject that I brought up years ago (2007) over at RealClimate and I was a bit of a lone voice in the wilderness. I thought it did show that people do have very fixed ideas about many things, in this case the regular posters, (I do not think the RealClimate board owners got involved).

        I did meet rejection on the everything radiates basis (maybe technically true but a very thin argument), and because the equatorial/pole gradient would remain. At the surface yes, but with a seemingly unavoidable inversion at the cold end, so in the atmosphere I must agree with your view as stated above.

        I am not sure I did all that well and perhaps I was suspected of some devilish anti-AGW plot. I was surprised how closed the thinking was on this issue.

        Alex

      • Alexander Harvey

        Pekka,

        I forgot to add that it occurs to me that people don’t see the role that radiative transfer has in convection and surface/gas energy exhange in general. It certainly used to be true that at a practical level in something such as heating and ventilating calculations it was not considered but in practice this equated to an assumption that the IR field was largely homogeneous both indoors and out.

        Alex

      • Alex,

        Continuing on your comment:

        In case of the atmosphere one of the most important roles of the LW radiation is to create the thermal connection between the surface and the lowest atmosphere. It’s exactly the lack of this connection that allows the positive temperature gradient to be very large for the atmosphere without radiative energy transfer.

        The second of the most important roles of radiative heat transfer is the connection at the top, where the LW is almost totally out-going.

        The third important role is in energy transfer either directly to from surface to the space or at least through most of the atmosphere without intervening absorption.

      • OK I’ll need to think about this some more. You could be right , but I would say that there no established scientific consensus that you are.
        It is important there should be IMO. Climate models should give the right answer for reduced as well as elevated GH gas levels and I’m just wondering if they do.
        Maybe you’d like to raise the issue at Realclimate? I’d be interested to follow the debate.

  67. Alexander Harvey

    For any who are that interested in the emission/absoprtion spectra of gases, notably H2O, CO2, O2, N2, etc. one can reference them here:

    http://vpl.astro.washington.edu/spectra/allmoleculeslist.htm

    I believe this database to be well recognised and upto date as of 2004 at least.

    I am no expert but I have learned that some care needs to be taken in their interpretation.

    The PNNL data are the measured cross-sections at particular temperatures and pressures. They are stated on a per molecule basis. When viewed as absorption spectra the units cm^2/moleclue give the apparent “size” of one molecule at each particular frequency (wavenumber cycles/cm) or wavelength. This “size” is a measure of apparent cross-sectional area, hence cm^2/molecule. This is not necessarily an intuitive measure but it is the way it is done and it makes for a lot of practical sense. Personally I would prefer a radius or diameter as otherwise it makes it sound like a lot of little discs and not a lot of little spheres.

    One could interprete the data as implying that these spheres of absorption appear to be large at the centre of the absorption lines and contract to nothing where there is no absorption e.g. where the gas is transparent there are no spheres to see and where it is opaque the spheres “look” large to incoming radiation of that frequency.

    The PNNL data is a true spectrum (compare with HITRAN below) it shows the line shape not just a peak but the wings and any overlap between lines. These are accurate for the conditions of the measurement but would change if they changed, e.g. less pressure would reduce the pressure broadening of lines making them taller and narrower. The scales vary enormously, perhaps by 10^12 or more between molecules and between frequency bands.

    The HITRAN data is quite different in appearance in that it shows just thin vertical lines at each frequency. The units are different too (cm/molecule). This data are integrated line strengths they could be considered as the inherent intensity of the line. If one were to take a single isolated line shape from the PNNL data and measure the area under the curve one would have the integrated absorption. The HITRAN data is just these integrated values. These observed values are ultimately determined by the quantum nature of the interaction. The advantage of the HITRAN data is that one can calculate normalised lineshapes due to the line broadening mechanisms for various temperature and pressures from formulae and multiply by the HITRAN value for that line to get an observable line spectrum. I think that people actually do this stuff and calculate all the absorption from the surface up through the column.

    It might be tempting to equate the visual appearance the HITRAN data as suggesting the the true spectrum consists of razor thin lines separated by big gaps, i.e. mostly transparent, this would be an error, the true spectra will look like the PNNL data.

    For those awaiting the evidence for N2, it is in the database, it has some weak lines between 2100 and 2600 /cm, by weak I mean around one billionth (9 orders of magnitude) less than a strong line for CO2.

    I am not sure that I could find a use for this data beyond making visual comparisons of the number and height of the lines and the frequency of the bands between molecules. But it reassuring to know that they exist.

    All errors are mine and mine alone. If you want an expert’s opinion be kind to one or read a book.

    Enjoy

    Alex

    • Alex,

      Thanks for the info. So N2 has radiation within IR spectrum that warmists conveniently ignored them. So N2 obeys radiation laws which overflown all GHE theory that N2 has no effect in absorbing and emitting LW radiation.

      • Sam, look at the y-axis.

      • Rattus,

        Not just y-axis, whole lot of all molecules, their specific energies and total energy involved. Spectral lines are just signatories identifying specific gases. y-axis is comparing single atoms wavelengths. There could be lines that the present equipment is incapable of detecting N2 spectral lines with IR spectrum.

      • Alexander Harvey

        Sam,

        You have been given that for which you seemed to wish. Produced along side many many other spectra. I think if you wish to make a case for undetected lines you must try something much more imaginative. If you wanted the spectrum you have got it.

        FWIW the lines of N2 or rather the lack of them makes it a cheap and sensible gas with which to dilute gases that are active in the IR when they are put in a spectrometer. I must wonder whether due to this it maybe the most spectrally observed gas of all time.

        Alex

      • Alex,

        “You have been given that for which you seemed to wish. Produced along side many many other spectra. I think if you wish to make a case for undetected lines you must try something much more imaginative. If you wanted the spectrum you have got it.”

        You maybe right. The spectroscopy is probably incapable to produce more spectral lines for N2. But for the N2 has a temperature, it must emit radiation to absolute zero space temperature or it can never be cooled violating radiation laws.

      • Alexander Harvey

        Sam,

        Give this a fair reading, just in case we are in agreement on some points.

        To the degree that N2 & O2 do have sepctral lines in the IR, they will absorb and emit EM raditation. A body consisting of N2 or O2 would radiate energy if it had a temperature > oºK. It would do so at a rate determined by several factors of which its temperature is an important one. OK so far?

        One of those other factors is the number of frequencies through which it can radiate, the strength of the emission lines at those frequencies, and a characteristic curve for emissions at that temperature (whether the lines are at frequencies that well matched to the temperature).

        Some gases seem to be better at emitting IR radiation than others. Some are referred to by some as GHGs but that distinction is arbitrary, it is not a matter of either/or but one of strength.

        A body of gas would cool down to an equilibrium temperature (or up) at a rate determined by its mass, specific heat, and the strength of its ability to emit IR (if that was its only method of exhanging energy with its environment). This forms a sort constant for the body of gas but it might not be a linear effect.

        I think you are saying that if H2O or CO2 are more efficient at emitting IR energy than say N2, O2, etc. The small amounts of CO2 would not change the specific heat significantly but would change the ability to emit IR radiation, hence if the body of gas was at a temperature whereby it was not in radaitive equilibrium with its environment it would approach that equilibrium more quickly (all other things being equal). If it was unsustainably hot (in terms of radiative balance) the addition of CO2, if it were a stronger emitter of IR at that temperature than N2, O2, would increase the rate of cooling.

        If you agree with the above (and I have not made and error), I agree with you as far as this posting goes. I haven’t read all that you wrote high above but in part you seemed to be saying something similar to what I have written here.

        Alex

      • Alex,

        “To the degree that N2 & O2 do have sepctral lines in the IR, they will absorb and emit EM raditation. A body consisting of N2 or O2 would radiate energy if it had a temperature > oºK. It would do so at a rate determined by several factors of which its temperature is an important one. OK so far?” OK.

        “One of those other factors is the number of frequencies through which it can radiate, the strength of the emission lines at those frequencies, and a characteristic curve for emissions at that temperature (whether the lines are at frequencies that well matched to the temperature).” I agree and also you seem know better than me.

        “Some gases seem to be better at emitting IR radiation than others. Some are referred to by some as GHGs but that distinction is arbitrary, it is not a matter of either/or but one of strength.” I agree.

        “A body of gas would cool down to an equilibrium temperature (or up) at a rate determined by its mass, specific heat, and the strength of its ability to emit IR (if that was its only method of exhanging energy with its environment). This forms a sort constant for the body of gas but it might not be a linear effect.” I agree but in reality it also exchange energy thru conduction and convection (at a very much slower pace than IR radiation).

        “I think you are saying that if H2O or CO2 are more efficient at emitting IR energy than say N2, O2, etc.” I don’t know if H20 and CO2 are more efficient emitting IR energy than N2 and O2 myself. I have just assumed that the warmists science is correct in this aspect. Basically I agree with you as I don’t have any knowledge to disapprove H2O and CO2 radiated are more efficient than N2 and O2.

        ” The small amounts of CO2 would not change the specific heat (of the air) significantly but would change the ability to emit IR radiation,” I don’t know as there is no mechanism or theory that I know of that CO2 can change the air’s ability to emit IR radiation significantly better or just tinyly better. It is a possibility but that will rely on real scientists to do some research to prove. However, due to small quantity of CO2, the heat content is very small. With huge mass, volume and heat content of air (mainly N2 and O2), it can effect a tiny amount of air heat content to loss to the CO2 due to now air is at a higher temperature (since CO2 has already radiated out some of its energy is at a lower temperature) if assuming that the air radiate to space at a less efficient manner. But convection and conduction will also give up heat to the colder CO2 to establish equilibrium at a very slow pace compared with the CO2’s IR radiation.

        “hence if the body of gas was at a temperature whereby it was not in radaitive equilibrium with its environment it would approach that equilibrium more quickly (all other things being equal).” Agreed and with CO2 present, air cools tinyly quicker and not much more quickly.

        ” If it was unsustainably hot (in terms of radiative balance) the addition of CO2, if it were a stronger emitter of IR at that temperature than N2, O2, would increase the rate of cooling.” Yes, tiny though.

        “If you agree with the above (and I have not made and error), I agree with you as far as this posting goes. I haven’t read all that you wrote high above but in part you seemed to be saying something similar to what I have written here.” I agreed except you wrote better and express clearer than me.

      • Alexander Harvey

        Sam,

        Thanks for taking the time to reply in detail.

        As I see it, from what you say, one aspect in particular is difficult to guess or have some intuition about.

        That is the rate at which energy can flow from the main body of the gas to those molecules that are best at emitting in the IR. That is how quickly can the deficit of energy in those molecules be “recharged” from a swarm of molecules around it that may not emit so readily.

        I, like you, may see this in the same way. It is not obvious how fast that rate should be. Once a molecule has released come engery how long is it before it so knocked about by the other molecules that it once again enters a state where it can emit some more energy.

        It does seem obvious that something of this sort, or at least something with that effect must occur. The emitting molecules are a sort of “drain” on the rest of the gas, maybe creating a small local imbalance, and the final rate of emission is determined by how quickly this drives the recharge.

        Now I do not know the figures of hand and it seems likely that it may vary according to the temperature (which effects the velocities) and the density (which effects the distance between collisions). If that is the case, then it seem likely to me that for low densities like perhaps the upper atmosphere the effect will be that the emission rate is “choked off” by the inability to “recharge” the emitting molecules quickly enough. I am guessing as to if this is the case, but if you see this as the issue then it is intuitively correct to both see it and worry about it.

        So the big question is whether in the main body of the atmosphere the “recharge” rate is high enough so one can put it in the category of small effects and assume that the actual “recharge” rate is so much faster than the emission rate that it makes little difference to the result.

        This is a concern and I believe that it is held that at least below some level in the atmosphere the “recharge” rate is so high that it can be ignored.

        It may be called something like the thermalisation rate, how quickly the deficiency (after an emission) or surplus (after an absorption) is restored to the equilibrium level; the process involved being called thermalisation.

        I think that this rate must be some function of the collision rate but I could not calculate it but I suspect that the function is understood by those that can.

        It is this rate, or rather some sort of ratio of this rate to the emission rate, that you need to find out and perhaps someone else knows the answer or at least whether or where in our atmosphere it is save to say that it so quick (much faster than the emission rate) that it matters little.

        You would be right to raise this as a question, perhaps ask it in isolation, as a one liner. Hopefully some one can give you some answer that is not just a vague guess. It is not just the thermalisation rate (perhaps best to use that expression not “recharge” rate) that they need to explain but how fast that is compared to the emission rates for a weakly emitting mixture of gasses or a strongly emitting mixture of gasses that would be necessary for teh effect to me significant in terms of the climate. This second part knowing both the thermalisation rate and the emission rate is not such a simple question to answer so watch out for people waving their arms about. I think that some here would be able to provide an adequate answer, and it will be interesting to see what they say.

        I hope that is OK, I will not try to think about your other concerns right now (I think the measurements of the lines is one of these) but if you cannot get an answer to the point about rates then I may not need to.

        I hope that I have understood you correctly and if I have it is a good question.

        Alex

      • Alex, I believe the collision rate is much higher than the emission rate, maybe three orders of magnitude, so thermalization is very efficient at maintaining the population in the higher vibration state (which is I think about a quarter of the atoms at typical temperatures). That vibration state is the important one for 15 micron photons as the molecule transitions to the ground state.

      • Alexander Harvey

        Jim,

        I am hopeful that Sam will ask and you and others will have that opportunity then, I expect that he will read your comment here.

        Until he replies I cannot know if I have correctly interpreted one of his principle concerns, if I have, I would expect he will canvas the field, or even ask elsewhere. I cannot know how detailed a response he would like nor in what terms. Hopefully time will tell, for now thanks for replying.

        Alex

      • Alex, I have had previous discussions with SamNC and they have not ended well. I predict he won’t be convinced, but good luck.

      • Alex,

        You explain extremely well and I learn from it and corrected some of my thinking, in particular, the introduction of CO2 enhance atmospheric air cooling rather than heating or heat trapping, just opposite to what AGW believer’s thinking, thank you.

        Your explanation of thermalization (or recharging) of CO2 and H2O is good and thought provoking. You indicated IR radiation from these gases is a function of temperature and density, highest IR radiation at sea level at their highest temperature and density and lowest at TOA. What I am still not clear is the magnitudes of these IR radiation and to compare with the surface IR radiations. Theoretically, if you measure IR radiation at 10m above the ground has higher IR radiation than at sea level because the 2 IR radiations together but due to inversely square of distance, the radiation measured is actually less. Can we assume CO2 and H2O IR radiation from the air minimal?

      • Alexander Harvey

        Jim,

        I have no intention of convincing Sam of anything, but I have tried to aid his questioning, and may continue should he so wish. I should not wish for him to think my thoughts. What then would be left for me?

        I see far too much lust for a dominance of ideas by which people should become victor and vanquished. One may convince without one single change of one single idea, purely by strength of arm. Convincement may be declared by forced submission and commonly is. My usage may be archaic yet strangely fitting here.

        Alex

      • Alex, OK, I understand your view. I have to realize that many denizens aren’t in this to take sides, but to aid their own understanding, and help formulating their own views, not anyone else’s. Fair enough.

      • Alex,

        Jim D said “Alex, I believe the collision rate is much higher than the emission rate, maybe three orders of magnitude, so thermalization is very efficient at maintaining the population in the higher vibration state (which is I think about a quarter of the atoms at typical temperatures). That vibration state is the important one for 15 micron photons as the molecule transitions to the ground state.”

        Alex, I have no idea of what is the thermolization rate at near sea levels nor any idea (->0?) at the near TOA levels, nor any levels at all. Jim’s guesses, such as, ‘three orders of magnitude’, ‘ a quarter of the atoms at typical temperatures’, are perhaps right but could be any wild guesses as any one. If I ask him to elaborate/clarify as in past, he was not interested at all. If you know these thermalization rates, please enlighten us.

        “That vibration state is the important one for 15 micron photons as the molecule transitions to the ground state.” I have no idea what is Jim D talking about, if you have a guess, let us know, thanks.

        My old knowledge about e-m emission involves atomic level electrons jumping from their electron shell levels. That jumping of electron shell levels emit their characteristic spectral lines. I don’t know if that theory has been superceded. Correct me if I am wrong, collison of atoms involve exchange of energy between the atoms at the vibrational level, will not excitate electron jump of at their shell levels.

        BTW, do you have any comment about the 10m above ground level IR radiation measurement. Do you think CO2 and H2O IR radiation part is minimal as compared with the ground IR emission part in the measurement?

      • Alexander Harvey

        Sam,

        I have seen your questions below and I am trying to put something together. When I have I will add to a new posting below.

        Alex

    • Alex,

      I said “You explain extremely well and I learn from it and corrected some of my thinking, in particular, the introduction of CO2 enhance atmospheric air cooling rather than heating or heat trapping, just opposite to what AGW believer’s thinking”. You did not repond. I guess I drew conclusion too soon. Will you lecture?

  68. No Sam, look at the y-axis. 10 orders of magnitude ain’t peanuts.

    • Alexander Harvey

      Rattus,

      Can you confirm that it is not just a matter for spectroscopy as we know where the lines must be from first principles when it comes to the diatomics, if not precisely where at least know the quantum numbers of each band and roughly where to look.

      Alex

    • Where is the y-axis you were referring to? I did not see the log scale you referred. Can you provide a link?

    • Rattus,

      Dispite of y-axis which I have not seen your graph, the heat content of N2 is huge which makes CO2 heat contents peanuts. For radiation to space, N2 must be huge in accordance with thermodynamics. If N2 cannot radiate at IR spectrum, then N2 keeps the Earth atmosphere warm, not CO2, not H2O, non of the GHGs can keep the Earth warm. This N2 heat content falsified GHE completely.

      • Sam NC, You have to remember where the nitrogen got its heat. Only about 1 billionth of the heat the nitrogen receives is from electromagnetic radiation. It only gets its heat really by collisions and it gives off almost all its heat the same way. The higher the altitude, the fewer molecules to hit the fewer collisions. As far as gases go, N2 and O2 are pretty middle of the road thermal conductors. So double glazed widows purged with nitrogen or oxygen are only so so insulators. Argon or Krypton are better with a vacuum best. So for thermal transfer, N2 is a so so conductor and a crappy radiation absorber/emitter.

        At the surface nitrogen does help keep the atmosphere warm. Partially by colliding with the surface, but also by colliding with water vapor and to a smaller extent CO2, the overall impact is small compared to water vapor, which dominates at the surface. Two thirds of the thermal energy in the atmosphere is due to incoming solar absorbed by the air and clouds plus latent heat transfer by water from the surface. Only about 7% is due to conduction, where nitrogen and oxygen contribute.

      • Dallas,

        Thanks for the reply and in general I agree with you all. However, you may like to calrify the following:

        1. “Only about 1 billionth of the heat the nitrogen receives is from electromagnetic radiation” Please elaborate.

        2. ” Two thirds of the thermal energy in the atmosphere is due to incoming solar absorbed by the air and clouds plus latent heat transfer by water from the surface.” Seems contradicted with 1. above. Please elaborate 2/3 is due to solar absorbed by the air and clouds. Where is the other 1/3 coming from?

        3. How did you get that 7% by conduction? I guess this contribute to the 1/3, where are the remaining heat (26%?) come from?

      • Sam NC,

        The 1 billionth is an estimate, I didn’t feel like integrating the entire spectrum, but N2 is on the order of 10^-28 while water is on the order of 10^-20, that 8 orders of magnitude with N2 having a smaller spectrum. Thermal conductivity of N2 is around 0.025, so it has a much bigger role.

        The two thirds is from the NASA energy budget, the KT budget is actually higher at 79% or so. I did the percentages as a total of the energy into the atmosphere from above and below.

        The 7% is an estimate of N2 conduction/thermal contribution of the 8% from the KT budget or the 11% by the NASA budget. It should be in the ballpark. The remaining warming of the atmosphere is due to outgoing IR being absorbed by water vapor, CO2 and all the other excitable molecules.

        I just got through posting my thoughts about the difference between the two budgets on my trash blog if you like, http://ourhydrogeneconomy.blogspot.com/2011/08/earths-energy-budget-controversy.html
        Nothing particularly earth shattering.

      • Dallas,

        Thanks for the reply.

        “The two thirds is from the NASA energy budget, the KT budget is actually higher at 79% or so.”‘ “The 7% is an estimate of N2 conduction/thermal contribution of the 8% from the KT budget or the 11% by the NASA budget”. Are these scientific or unscientific of guesses? Estimation should have bases.

      • Sam, I am still trying to figure that out. Both are based on observational data with KT or TFK tweaked with a model. They agree well in most cases, but since KFT is looking at the greenhouse effect and NASA is not there are differences and some that I think may be too large, dunno. The actual should be in the range of the two. Water seems to be the biggest bugger as usual.

      • Dallas,

        Thanks for help clarifying. If you figure them out, let s know.

        No doubt ” Water seems to be the biggest bugger as usual”. CO2 is nothing, whatever is compared with.

  69. Konrad over at WUWT on this post
    http://wattsupwiththat.com/2011/08/15/radiating-the-ocean/#comment-724536
    at Konrad says:
    August 20, 2011 at 6:05 pm
    has posted an interesting simple DWIR experiment that shows the Greenhouse effect, but only when evaporation of water is not allowed. Once evaporation takes place the greenhouse effect virtually stops.
    Perhaps Tallbloke and others are correct.

  70. Here are some results using MODTRANS:
    http://geoflop.uchicago.edu/forecast/docs/Projects/modtran.html

    The MAXIMUM no feedback increase in temperature predicted by a doubling of CO2 was 0.9C. This was clear sky at the equator. As you moved towards the poles or increased the clouds, this was reduced, to 0.55C for subarctic winter with clouds.

    This argues strongly that the “no feedback” warming from CO2 is somewhere between 0.55C and 0.9C. Breaking with tradition for “peer reviewed” Climate Science, I’ve publishing my methods (below):

    I didn’t bother trying out H2O feedback as this is by no means certain. It seems logical to me that if H2O feedback was positive as per the IPCC, than the paleo record would show run away greenhouse warming in the history of earths temperature – but it does not.

    Paleo temperatures appear stable for hundreds of millions of years at either 11C or 22C, which argues strongly that feedback is negative, as echoed in the satellite data, perhaps due to the energy step function in the phase change of water. Negative feedback would result in temperature changes smaller than those predicted by the no feedback model,

    Here is a method to use MODTRANS that appears to work at top of atmosphere.

    1. Start with defaults
    2. enter current CO2 levels – say 390 ppm
    3. Submit
    4 – result is 287.687 W @ 299.70 K

    5. Double the CO2 to 780 ppm
    6. Submit – result is 284.484 W @ 299.70 K

    The model is now out of balance as our W have gone down. This is because CO2 is blocking radiation from the surface. But over time W cannot go down as as energy in must equal energy out. So, we must adjust the Ground T offset.

    7. Change the Ground T offset to 0.9
    8. Submit
    9. – result is 287.687 W @ 300.60K

    Our model is now in balance. Our energy out balances the original energy in. What this tells me is that a doubling of CO2, clear sky in the tropics, will raise temperatures by 0.9C. The top of atmosphere radiation must remain unchanged, as the energy from the sun is unchanged.

    If we repeat this for the US standard atmosphere, and balance the model for energy in = energy out, clear sky we get at TOA:

    390 ppm CO2 – 258.673W @ 288.2K – Ground T offset 0
    780 ppm CO2 – 258.673W @ 289.07K – Ground T offset 0.87

    So, the predicted increase in temperature is 0.87C for the US standard atmosphere for a doubling of CO2, which is not very different than the 0.9C predicted for the tropics.

    When we repeat the above using 1976 US Standard atmosphere with 12.5mm/hr rain at 390 ppm CO2 we get the following:

    250.98W @ 288.2K – Ground T offset 0

    When we balance the model with 780ppm CO2 we get:

    250.98W @ 289.03K – Ground offset 0.825

    This is slightly less than our clear sky result, which says that we will get a slightly smaller temperature increase from a doubling of CO2 if it is raining.

    Repeating this in the subarctic summer, clear sky I get

    390ppm CO2 – 261.75W @ 287.20K
    780ppm CO2 – 261.75W @ 287.92K – Ground T offset 0.72

    Repeating this in the subarctic winter, clear sky I get

    390ppm CO2 – 196.564W @ 257.20K
    780ppm CO2 – 196.564W @ 257.84K – Ground T offset 0.64

    Repeating this in the subarctic winter, cumulus cloud I get

    390ppm CO2 – 186.736W @ 257.20K
    780ppm CO2 – 186.736W @ 257.75K – Ground T offset 0.55

    This tells me that the 0.9C temperature increase predicted for the clear sky tropical atmosphere model is likely the worst case situation, as the predicted increase appears less in the other situations tested.

    • Your assumption that positive feedback leads to runaway warming is wrong. It leads to an amplification factor on the original effect. Paleo evidfence and solar forcing support a distinct positive feedback with a minimal chance of no feedback at all.

      • Jim D 8/21/11, 1:27 pm, Planetary energy …

        JD: Your assumption that positive feedback leads to runaway warming is wrong. It leads to an amplification factor on the original effect. Paleo evidence and solar forcing support a distinct positive feedback with a minimal chance of no feedback at all.

        Reinforcing what you wrote, the most powerful feedback in climate is cloud albedo (including cloud fraction, i.e., not as modeled by IPCC). It is slow and negative with respect to surface temperature, mitigating warming from all causes. It is fast and positive with respect to solar variations, acting as a solar amplifier but causing no runaway. This is based on what IPCC calls the “simplest theoretical grounds”, fully supported by observations. IPCC is proving that modeling climate without the effect of cloud albedo has no predictive power.

      • It is always amazing to see such unsupported claims used to reinforce any theory that goes against AGW, simply because it goes against AGW. That clouds act as a strong negative feedback to GHG’s, but a huge positive feedback to solar variations, is just wishful thinking and hand-waving.

        To what extent can people realize that these sorts of arguments are not meant to advance science, or to improve how we understand climate; they are simply there as a distraction tool, and to cast doubt.

      • Chris Colose 8/21/11, 3:35 pm, Planetary energy …

        For motivation, read IPCC’s lament about its inability to represent cloud feedback and that clouds can produce a 100% error in its climate sensitivity on the simplest theoretical grounds.. AR4, ¶Model Clouds and Climate Sensitivity, p. 114.

        For a model of how cloud feedback works, see SGW at

        http://www.rocketscientistsjournal.com/2010/03/sgw.html

        or just click on my name in the header. The full model is at H. Albedo Dependence on Solar Radiation & Humidity. To navigate their quickly, just search there on the word positive. For more intuitive information, also read the few comments on burn-off. For the total effects, see Figure 1, showing how a simple 4 or 5 parameter transfer function on solar radiation reproduces the entire, 140 year surface temperature history with an accuracy comparable to IPCC’s unrealizable smoothed estimator. Id., Figures 1 and 6.

        For observations on the amplification (positive feedback) effect, see Stott, P. A., G. S. Jones, & J. F. B. Mitchell, Do Models Underestimate the Solar Contribution to Recent Climate Change?, J.Clim., v. 16, 4079-4093, 12/15/03; Tung, K. K., J. Zhou, & C. D. Camp, Constraining model transient climate response using independent observations of solar-cycle forcing and response, Geoph.Res.Lett., v. 35, L17707, 5 pp., 9/12/08.

        If there is anything else in climate that is bothering you, don’t hesitate to post a response here or on my blog.

      • Zhu et al (2007) found that cloud formation for ENSO and for global warming have different characteristics and are the result of different physical mechanisms. The change in low cloud cover in the 1997-1998 El Niño came mainly as a decrease in optically thick stratocumulus and stratus cloud. The decrease is negatively correlated to local SST anomalies, especially in the eastern tropical Pacific, and is associated with a change in convective activity. ‘During the 1997–1998 El Niño, observations indicate that the SST increase in the eastern tropical Pacific enhances the atmospheric convection, which shifts the upward motion to further south and breaks down low stratiform clouds, leading to a decrease in low cloud amount in this region. Taking into account the obscuring effects of high cloud, it was found that thick low clouds decreased by more than 20% in the eastern tropical Pacific… In contrast, most increase in low cloud amount due to doubled CO2 simulated by the NCAR and GFDL models occurs in the subtropical subsidence regimes associated with a strong atmospheric stability.’ Zhu, P., Hack, J., Keilh, J and Zhu, P, Bretherton, C. 2007, Climate sensitivity of tropical and subtropical marine low cloud amount to ENSO and global warming due to doubled CO2 – JGR, VOL. 112, 2007

      • Chief Hydrologist

        Oh yeah – in case you have missed the solar ENSO connection.

        One potential cause of Pacific Ocean variability is shown by Lockwood et al (2010). ‘During the descent into the recent exceptionally low solar minimum, observations have revealed a larger change in solar UV emissions than seen at the same phase of previous solar cycles. This is particularly true at wavelengths responsible for stratospheric ozone production and heating. This implies that ‘top-down’ solar modulation could be a larger factor in long-term tropospheric change than previously believed, many climate models allowing only for the ‘bottom-up’ effect of the less-variable visible and infrared solar emissions. We present evidence for long-term drift in solar UV irradiance, which is not found in its commonly used proxies.’

        Judith Lean (2008) commented that ‘ongoing studies are beginning to decipher the empirical Sun-climate connections as a combination of responses to direct solar heating of the surface and lower atmosphere, and indirect heating via solar UV irradiance impacts on the ozone layer and middle atmospheric, with subsequent communication to the surface and climate. The associated physical pathways appear to involve the modulation of existing dynamical and circulation atmosphere-ocean couplings, including the ENSO and the Quasi-Biennial Oscillation. Comparisons of the empirical results with model simulations suggest that models are deficient in accounting for these pathways.’

        It is sad to see confiirmation bias have such a profoundedly distorting effect on Chris’ knowledge base.

    • Alexander Harvey

      ferd,

      My reply is misplaced here:

      http://judithcurry.com/2011/08/19/planetary-energy-balance/#comment-102985

      Hopefully this is not misplaced too.

      Alex

  71. Alexander Harvey

    ferd,

    I hope that I am sufficiently familiar with this issue, which I recognise, to allow me to skim over your method section. I will trust that you are methodical.

    The following is my opinion and I have thought on this previously.

    It would be an error to claim that MODTRAN3 is inferior to the transfer calculation in the GCMs in terms of how it handles a defined atmospheric profile. It is my belief that MODTRAN3 has the finer resolution at least in terms of the number of frequency bands. That said it is likely inferior to the line by line approach, so in that sense it depends on which outcomes are being compared.

    However the subject of how well MODTRAN3 treats a rise in surface temperature is less clear. It seems to cheat slightly when it comes to the tropopause which can I think be revealed if one makes a big jump in the surface temperture and exmines the temperature/altitude curve in the righthand pane. There may also be issues with clouds due to their being defined at a fixed height irrespective of temperature, that is my recollection which you may check by examining the saved data file.

    The anomalous outcome of MODTRAN3 in this respect has hitherto passed with little comment. You might take this up directly with David Archer for I would be surprised if he is not aware of this.

    Alex

  72. A. C. Osborn

    Another experiment posted by Hockey Schtick says:
    August 21, 2011 at 9:11 am
    is even more interesting as it shows that with IR Solar ovens that night time downwelling Radiation does not heat it COOLS.
    Can you please explain why that should happen?

  73. Maxwell thinks that the second law of thermodynamics does not apply to radiation.
    Completely wrong!

    Clausius himself tested for this using lenses and mirrors.

    To to sum up the law in its most condensed form.
    For radiation as for conduction and convection;

    Heat will not move spontaneously from a lower to a higher temperature surface.

    Maxwell will notice that no professionally qualified scientist from the IPCC side of the discussion represented here will support his rather odd idea.

    This must come as a big embarrassment to Maxwell , as yesterday he was saying that David Wojick had no right to comment on these matters because David was not “expert” enough.

    • Bryan,

      Can you link to the post where maxwell made such a claim? I haven’t been able to find it.

  74. Alexander Harvey

    Sam,

    I am trying to find you some more data on both the thermalisation rates and emission rates. I really do not think I should attempt to try and calculate these as I could easily be quite a long way out and it does not appear to be completely straight-forward.

    When I come up with soemthing I will post down here.

    Alex

    • Alex,

      Thanks and looking forward to your data. Regards, Sam.

    • Alexander Harvey

      Sam,

      A little progress on the collision rate and photon emission rate.

      The collision rate is derived from the kinetic theory of gasses and the results are well established for air at common surface temperature and pressure. This rate is or the order of 10^9 – 10^10 collisions per second per molecule.

      That is to say that the average time between your average molecule colliding with another molecule is or the order of one billionth of a second.

      Trying to find a figure for the emission rate, i.e. how often a molecule such as H2O emits a photon at surface temperrature and pressure proved to be much more difficult. I could not find a simple reference for this figure so I resorted to using the MODTRAN3 radiation model and after a lot of head scratching came up with a figure that I wouldn’t trust so I will give it a broad error margin.

      That figure is around 0.01 and 1 photons per second per molecule.

      I wouldn’t put too much faith in that.

      I realised that I could calculate a theoretical maximum rate for photon emissions based on the size of the molecules. Simply put if a molecule emitted more than a critical amount of photon energy per second a cloud of those molecules would emit energy across its surface boundary more efficiently than a black body and that would violate the laws of thermodynamics.

      That maximum figure is equivalent to around 10^3 – 10^4 photons per second per molecule. I am more convident that this figure represents a good estimate of the maximum rate.

      This is smaller than the collision rate and the ratio of collisions per photon emission looks like it is at least 10^9/10^4 = 100000.

      So on average an a molecule would recieve on average a minimum of 10^5 collisions between each photon emission (based on my maximum feasible emission rate) for air at surface temperatures and pressures. In reality as none of the gasses approach being perfect emitters the ratio of collisions per photon emission would be higher than that.

      So it really does look like it is the case that any molecules that can radiate energy would be able to thermalise with the non-emitting molecules and hence they would be able to take energy from the main body of the air at a rate fast enough not to slow down the rate of emission.

      I hope that makes sense even if it is not what you wanted to hear.

      The situation much higher up in the atmosphere is not so clear cut as the collision rate drops with decreasing temperatures and pressures but it seems reasonable to suggest that thermalisation is sufficient to support any likely emissions in the troposhere.

      I will look at the rest of your last posts again and see if I can do more in the next day or so.

      Alex

      • Alex,

        Thanks for your hard work and tried to get me an idea of the collision rates and emission rates.

        Presumably the collision rate of 10^9~10^10 at common temperature (which I will assume 273K or 288K) and pressure at sea level, i.e. at STP or NTP. So do you have any idea what is the collision rate at TOA?

        My old school physics taught me that emission happens when a electron jump from its its excitated state shell orbital level to its normal state (I have a vague idea about that) shell orbital level, it emits its characteristic spectral line(s?), more lines if other electrons are aslo jumping within their respective shell levels, I wonder if this theory is still valid.

        There appears to have a missing link between the tansformation of kinectic energy gain during collisions into internal atomic electron jump energy between shell levels to emit e-m radiations. If you know, please enlighten us, thanks.

      • Alex,

        I thought this message did not get thru.

      • Alex,

        Thanks for the hard work.

        How do you find the 10^9~10^10 collision rate?

        “I realised that I could calculate a theoretical maximum rate for photon emissions based on the size of the molecules.” How is photon emission rate related to the size of the molecules? More electrons involved with electron orbital shell jumps in the bigger molecules?

        “I hope that makes sense even if it is not what you wanted to hear.”
        Yes, it does make sense to me. Not so ‘even if it is not what you wanted to hear’. I keep an open mind to whatever makes sense.

    • Alexander Harvey

      Sam,

      Thanks for staying with me, it is appreciated.

      I will come back to the mechanisms behind the emissions but some questions can be answered from very general considerations.

      The link between maximum emission rates and area only relies on the laws of thermodynamics and the concept of a black body radiation source.

      A black body is described as emission source with a surface emissivity of 1, i.e. a perfect radiator.

      If one was to cover such a black body with a layer of gas at the same temperature then as viewed from above a each molecule of that gas would shade a small area of the surface from the observer, there will be a small area of the surface that the observer (think single eyed) would not see.

      In that direction the observer would see the emissions from the molecule instead. If these emissions were stronger in the sense of energy emitted per second than those of the surface (which hase the same temperature) then the effect of adding that molecule is to increase the total emissions as viewed by the observer this is not possible according to the following reasoning.

      If you consider that this molecule and the surface to constitute a new system then the maximum amount by which that molecule can increase the rate of energy radiated is in proportion to the amount by which it changes the total surface area. The surface is already emitting as much as possible by definition as it is a black body, so the maximum rate of emission from the molecule is just that which a small black body the size and shape of the molecule could emit.

      There are many ways of looking at this, another is to imagine the surface as consisting of a very dense gas so dense that the molecules are nearly touching here it is clear that each molecule can not emit energy at a greater rate than an area the same as the area of a disc the same size as the molecule.

      I prefer illustrations like that whenever possible as it does not rely on complete understanding of the mechanism involved for no matter what the mechanism somethings are not possible.

      This sets an upper limit to the rate at which energy could possibly be emitted by the molecule in a system at a certain temperature. It does assume that the molecule is close to thermal equilibium, e.g. its energy is determined by the system temperature and that the laws of thermodynamics hold.

      I think it should be relatilvely easy to show that a system with an emissivity greater than 1 leads to the necessary conditions for a perpetual motion machine as soon as you bring it into proximity with another body of the same temperature as the first system. For the first system due to its emissivity being greater than 1 would transfer heat (net energy flow) to the second body and raise its temperture which is not possible.

      I think the reasoning is sound even it it doesn’t need equations which is something I welcome.

      You can boil down all of the above and derive a value for the maximum average rate photons emitted per molecule per second based on the surface area of the molecule.

      This value is just a theoretical limit it does not imply that it could be achieved. What can be achieved depsends on the details of the mechanism.

      I hope you liked that, I did.

      It has been interesting and there will be more as and when.

      Alex

      • Alex,

        Thanks for the explanation.

        “In that direction the observer would see the emissions from the molecule instead. If these emissions were stronger in the sense of energy emitted per second than those of the surface (which hase the same temperature) then the effect of adding that molecule is to increase the total emissions as viewed by the observer this is not possible according to the following reasoning.”

        Yes, I agree with the above statements when both the surface and the gas layer are at the same temperature and going in the same direction. The total energy is the surface + the gas layer as a new system, but the radiation cannot be anything bigger as the gas layer radiation cannot exceed the black body radiation. We now have more energy available within the system and can only radiate the same amount or less if the gas layer has not the same radiation effect as the black body. The new system will radiate a longer time than with the gas layer.

        Hope I am not side tracking if I said the Earth surface is warmer than the air temperature. Correct me if I am wrong if I am sitting above the gas layer, I will see the Earth surface radiation since the Earth surface is at a higher temperature and not the gas radiation at a lower temperature. Strong surface radiation over-power weak atmospheric radiation.

        “If you consider that this molecule and the surface to constitute a new system then the maximum amount by which that molecule can increase the rate of energy radiated is in proportion to the amount by which it changes the total surface area. The surface is already emitting as much as possible by definition as it is a black body, so the maximum rate of emission from the molecule is just that which a small black body the size and shape of the molecule could emit. I prefer illustrations like that whenever possible as it does not rely on complete understanding of the mechanism involved for no matter what the mechanism somethings are not possible.

        This sets an upper limit to the rate at which energy could possibly be emitted by the molecule in a system at a certain temperature. It does assume that the molecule is close to thermal equilibium, e.g. its energy is determined by the system temperature and that the laws of thermodynamics hold.

        I think it should be relatilvely easy to show that a system with an emissivity greater than 1 leads to the necessary conditions for a perpetual motion machine as soon as you bring it into proximity with another body of the same temperature as the first system. For the first system due to its emissivity being greater than 1 would transfer heat (net energy flow) to the second body and raise its temperture which is not possible.

        I think the reasoning is sound even it it doesn’t need equations which is something I welcome.” Yes, I agree and better explained without equations.

        “I will come back to the mechanisms behind the emissions” I am looking forward to them.

    • Alexander Harvey

      Hi Sam,

      We both seem to be coming up with more quesions than answers, I will try to address a couple of the most obvious issues here and now. I know there is more that we need to discuss but I am forgetful as to what, perhaps you could remind me of just a couple. I read your reply and one bit of it puzzled me:

      “The new system will radiate a longer time than with the gas layer. ”

      If I should read this as implying that the sytem would radiate less efficiently e.g. at a lower rate for the same surface temperature, that would seem right to me.

      ***

      In gasses with temperatures within the range to be found in the troposhere the dominant mechanism for emission and absorption involves transitions between molecular states with different vibrational and rotational energies. This is not the same thing as the electon transitions that are perhaps more familiar and which you seem to be considering as the mechanism.

      I have tried to think of a simple way to think about the difference between these different mechanisms.

      Perhaps the easiest is to consider that an in an electron transtion the nucleus (nuclei if we are considering molecules) play a very secondary role and it is principally the difference in the electron energy that matters. The electron may jump between levels that take it closer to or further away from the nucleus and that requires either a compensating loss or gain of energy by the emission or absorption of a photon, or some high energy impact by other particles, atoms, or molecules.

      When a molecule vibrates the nuclei, which have nearly all the mass, move with respect to each other in an oscillatory fashion. That is the distances between the nuclei are changing. The modes of vibration could be seen as stretching and bending motions when viewing the molecule as a whole.

      When a molecule rotates without vibrating the nuclei can be thought of as rotating around some axis but the distances between the nuclei do not change. Here it is ony the orientation of the molecule in space that changes.

      As I understand it, a molecule can and does both rotate and vibrate at the same time resulting in a some more complex overall motion.

      By and large the difference in the energy levels of these vibrational and rotational levels are lower than between electron transitions, and it is transitions between differing vib and rot states that dominate the IR spectra. Electronic transtions are more commonly associated with higher temperatures such as occur in flames. There is no clear cut division, I think that there are electron transitions that occur in the IR band and also the microwave band and these can be significant in certain cases (interstellar matter may be one such case).

      As far as I am aware, all of the lines listed in IR spectra of the atmospheric gases at STP result from transitions between differing vib and rot states.

      These vib and rot modes are sometimes thought of as being mechanical as opposed to electronic. This agrees with my thinking of them as relative motion of the centres of mass (nuclei). In that respect they are more like kinetic motions, the translation of the centre of mass of the molecule in space. It is tempting to start thinking of them as Newtonian motions but it must be remembered that these motions are quantised, only certain amounts of molecular vibration or rotation can occur.

      The number and tyoe vibrational and rotational modes is closely related to the overall shape of the molecule (the distribution of the nuclei in space and the electrons that surround and may pass between them). These shapes can be categorised along with their permitted vib and rot modes.

      Now shape isn’t everything, it is necessary for determining the vib and rot modes but in itself tells us litlle about photon emission and absorption. Here the electron configuration of the molecule comes back into play. In order to interact with the EM field I believe that the molecule needs to be capable of producing a changing EM field of its own. Some molecules are better at this than others. You may hear a convenient cut down classification that states that diatomic molecules neither emit not absorb in the infrared. You all ready know that this is not strictly the case but that is not the point I wish to make here. The point I wish to make is that it is not strictly speaking that it is being diatomic that is the issue, it is whether they are symmetric in their charge distribution. Molecules such as O2 and N2 and H2 have symmetric charge distributions (at least for their electronic ground states but not necessarily when excited or when ionised). Diatomic molecule such as NO and CO have asymmetric charge distributions and are fully paid up memebers of the IR active club (but mainly high up the frequency band),

      However it seems reasonable to make the case that more complex molecules, triatomic and above, and those with the more irregular shapes such as H20 as compared to say CO2, or more heterogenic, NO as compared to N2, are more capable of interacting with the EM field and entering in to vib and rot transitions through the absorption and emission pathway.

      We have looked at the relative frequency of collisions and photon absorption/emission at the molecular level but neglected describing the collision pathway at that time. To the degree that the molecule can behave mechanically in the Newtonian sense these collisions can and will result not just in changes in kinetic energy and momentum but in vibrational energy and rotational energy. This seems to be the dominant transition pathway for gasses at STP, i.e. there are many more collision induced transitions than photon mediated transitions. As noted above these collisions are not strictly Newtonian because the vib and rot states are quantised.

      Most of the above is probably a fair general statement but I expect that there are wrinkles which may be need some consideration when the gas is abundant in the atmosphere such as O2 which does have some IR activity at higher frequencies and higher temperatures and whereas it would be a small player in the troposhpere it might, and I only say might for I do not know either way, be of signifcance in the stratosphere and above.

      ***

      As part of your reply you brought up the issue of what happens if a gas is not at the same temperature as the surface.

      This is interesting and obviously needs thought. In my last response I was only looking at the very narrow quesion of whether we can at least put a limit to the photon emission rate per molecule in a gas and in that case it was necessary to consider the case where the temperatures did not differ.

      Now I understand that you see the potential for an IR active atmosphere to promote more efficient cooling of the surface atmosphere system. I think it is clearly the case that at least in certain cases this would happen. I.E. that the addition of an IR active atmosphere or the addition of IR active molecules to an atmosphere would lower the surface temperature.

      That being the case, how would one square this with the conjectured warming effect for the Earth?

      That is a good question and if it was easily answered then the a lot of web conversions would not be taking place.

      It is perhaps more easy to describe circumstances whereby increased atmospheric IR activity would lead to a cooling surface and this is going to get bound up with the notion of “efficiency” when it comes to IR emissions.

      For system receiving a net input of SW radiation but without an IR active atmosphere, or more generally a weakly acitve one, there is one clear way of increasing the surface temperature. One could reduce the IR emssivity of the surface (providing one didn’t increase the albedo by the same degree). This I think must result in a warming trend for the surface and hence a higher equilibrium temperature. I will not attempt to quantify amounts but it really does seem to be intuitively true that provided the emissivity of the surface is sufficiently low adding IR acitve molecules to the atmosphere is going to make the IR emission into space more efficient, e.g. will lower the equlibrium temperature. In the extreme case where the IR emissivity of the surface approached zero the IR activity of the atmosphere would eventually dominate the emissions. I do think this is would be the case.

      Now this does suggest that increasing the atmospheric IR activity can result in a lower surface temperature but it does not suggest that it must. We must consider the opposite case, the one you queried. What happens if the surface emissivity is 1, the atmosphere contains IR active molecules and it is cooler than the surface. It will help if it is assumed that the incoming radiation is all SW and does not warm the atmosphere, in that case the atmosphere can only be at the same temperature or cooler than the surface as it does not have a source of energy independent of its interaction with the surface.

      As an aside, I hope that throughout my writing my usage of the word “could” merely expresses some plausibility and nothing more than that, and that some other consideration could in fact ensure that some effect is not plausible. In a sense I simply mean it seems plausible “all other things being equal”.

      Also I will resist the temptation to explain the lapse rate, it is an observable, if present it becomes an established fact and one that has consequences, it implies that certain other things either could be or must be the case.

      I think that you see that in the case of a surface that is a perfect black body a cooler and IR active atmosphere would tend to result in the system as a whole having an emissivity less than 1, i.e the system would be slightly less efficient. Now if that be the case it could imply a higher temperaure for the system and if the atmosphere lacks an independent energy supply, a higher temperature for the system as a whole implies a higher surface temperature.

      I said “could imply a higher temperature”, for there I see a wrinkle. If the solid body part of the system is much smaller than the system including the atmosphere, e.g. the hard surface area is much smaller than the “effective surface” area of the IR active atmosphere, this larger gasseous “surface” neither has to be as efficient as a black body nor at the same temperature as the surface to emit the same amount of energy, it culd do so by simply being very much bigger. This is not necessarilty a case that can be realised but I certainly cannot rule it out at this level.

      I think that in the case of a black body ground surface and a “thin” atmosphere e.g. most of the Earth’s atmosphere is within a few tens of kilometres of the surface but the surface is a few thousdand kilomtres from the centre of the Earth, a reduction in the “efficiency” of IR transmission would result if the atmospheric temperature declines with altitude. This I think can be argued strongly and it is not necessary to account for some effects, (lapse rate) as we can argue from their existence and the permanence of that existence.

      For me, the case for a system with all of the following properties: a univeral (at all latitudes and longitudes and heights) lapse rate, no SW absorption by the atmosphere, a perfect black body ground surface, and a “thin” atmosphere, is a case where the addition of IR active molecules to the atmosphere would reduce the efficiency (effective emissivity) for IR emissions which would result in a higher equilibrium temperature.

      Now not one of those conditions is true in the Earth’s case. In a sense all I have managed is to suggest that a resultant increase in the surface temperature is plausible but not guarranteed. Also there is a suggestion that for systems where those properties are not at all the case, the addition of IR active molecules to the atmosphere could result in a lower surface temperature.

      That may not be particularly useful and it is certainly not a killer fact, but I think it could be a useful insight into the plausibility of certain effects and certainly insightful when it comes to thinking about why people are sceptical. Simply put, at this level of thinking, one could not rule out a rise in the equilibrium temperature but one would be cautious before accepting it as being necessarily the case.

      The phrase “at this level of thinking” is key, for what are the alternatives, one could accept the “facts” and that would be a legitimate stance, one could learn all the physics, become very rich and build your own computer models, but this is not practical, or one could try a little harder to reason from first principles. I would suggest that for many this is the only or at least the most preferred option.

      However we may have one potential killer fact that we are pretty sure about and that is the universiality of the lapse rate upto the tropopause. This is not without consequences and I think it may be possible to show several things, Perhaps that the SW absorption by the atmosphere is not sufficent to preclude a higher surface temperature for although it does produce rising temperatures in the stratoshpere and above it is obviously still leaves us with a troposheric lapse rate and from that we can perhaps show that for a black body surface (which admittedly is not the case) viewed at the level of the tropopause the surface/troprospheric system would be less efficient than that of a black body and given that the atmosphere is actually quite “thin” upto that level, and that the known surface emissivities (ocean at least) are fairly close to 1, the effect of the IR active molecules in the troposhere could have resulted in the current surface temperature being higher than it might otherwise have been.

      Not much but it does suggest that the “it can’t happen” case is not proved at this level of thinking either.

      I suppose my point, such as it is, could be summarised as an indication that higher or lower temperatures are decided by may secondary factors and not simply on the IR properties of gasses.

      Some will hold that these secondary factors have all be looked at, which is almost certainly true, and that it has been decided that the effect of additional IR activity in the atmospere will result in a higher surface temperature. The trouble is that there are quite a number of people, and the number seems to be growing, that don’t trust that judgement. I am not in any position to defend that judgement for I know I do not understand the sicence at the necessary level. But I think I may be in a position to clarify, at least in my own mind, which are the necessary and perhaps which are the sufficient conditions for rising surface temperatures (all other things being equal). As it happens my disposition is towards suspecting a rise in temperatures, but I am not totally sure why I think this, for I can imagine circumstances under which the temperatures might actually fall.

      Right now, when I have any doubts, I remind myself that “it is the lapse rate, stupid” for I find it difficult to conceive that there “has been” no “GHE”, for the combination of the lapse rate and the oh so cold tropopause is a bit of a smoking gun which I cannot ignore, it is all to do with my way of thinking that few others might share. Even still there is a difference between there “having been” a “GHE” and the relationship between additional atmospheric IR activity resulting in further increases in surface temperatures. I have a suspicion that I might be able to fathom it all out in a way that is “obvious” and relies on a minimum of little contested observations, I really am that vain. And no matter what, these are interesting questions and I think it far better to think about the questions than shout about the answers.

      Alex

      • Alexander Harvey

        Sam I have read your comment again:

        “The new system will radiate a longer time than with the gas layer. ”

        If I should read this as implying that the sytem would radiate less efficiently e.g. at a lower rate for the same surface temperature, that would seem right to me.

        On second reading I think that you probably didn’t mean what I thought above so ignore that. If it is important perhaps you could break it up and add some extra details, if it is not an issue then leave it be.

        Alex

  75. Here’s one for Kim:

    As Pekka and Fred Moolten argued about back radiation, the angels danced off the head of the pin.

  76. I have a question for those arguing that lapse rate is the mechanism underlying climate change. I can see that lapse rate can account for snow on mountain tops and a much warmer sea level. In fact it seems to me that lapse rate is the primary mechanism governing surface temperature, and hence climate. And I would disagree sharply with anyone arguing that lapse rate contributes little to the surface temperature.

    But climate is not climate change, which science attributes not to changing lapse rate but to increased GHGs and also increased aerosols at cloud altitudes. (Higher aerosols, even black soot, tend to have more of a cooling effect because the higher the soot the more readily it can radiate whatever energy it absorbs out to space, and of course reflected insolation from light-colored aerosols just goes straight back out.)

    The lapse rate may be temporarily disturbed by rapidly changing levels of emissions (we are doubling CO2 emissions every 3-4 decades), but if CO2 is held at a steady level, whether 200 ppmv or 2000, the (dry adiabatic) lapse rate should always be the same g/c_p degrees/meter. Venus is a rock-steady 97% CO2 and its lapse rate is g/c_p on the nose over the entire bottom 60 km, following a dead straight line on the temperature profile.

    If you don’t accept the human-emissions explanation of why the surface is getting warmer, and that instead lapse rate is somehow responsible for temperature change at the surface during the past four decades, what exactly is the nature of that change, what is its underlying mechanism, and why just now?

    Given how lapse rate works, I find this lapse rate theory of climate change extremely implausible.

    • Vaughan – The temperature response to a forcing imposed by increasing CO2 (e.g., a doubling) is calculated on the assumption of an unchanged lapse rate. Any subsequent lapse rate change is considered a feedback and is calculated separately as an amplification or diminution of the original response to the forcing.

      With a fixed lapse rate, increasing CO2 impedes the escape to space of IR photons emitted from the surface or from lower altitudes. A balance with incoming absorbed solar energy can only be restored if the photons are emitted on average from higher altitudes where CO2 molecules are sparser. However, because of the lapse rate, these altitude are colder, but since emission rates are a function of temperature, emissions will remain inadequate to restore the balance until the altitudes where photons are escaping warm up. This happens because the trapped energy that is failing to escaping adequately results in atmospheric and surface warming until the higher temperature once more is sufficient for adequate emissions to space at what is now a higher average radiating level.

      Without a lapse rate, a change in CO2 could not cause a change in temperature, because merely moving to a higher altitude would not be associated with a colder temperature insufficient for adequate emissions until everything else below warms up. The lapse rate is thus required for climate change due to greenhouse gas concentration changes.

      • Right, Fred, CO2 is driving the change, you and I obviously agree on that. What I wanted to know was, who was arguing otherwise on the basis of lapse rate?

        However you did say something that brought back memories from eight months ago where I challenged you on a point and you didn’t respond. Just now you wrote,

        A balance with incoming absorbed solar energy can only be restored if the photons are emitted on average from higher altitudes where CO2 molecules are sparser.

        As I did last December, I beg to differ: that’s only one possible mechanism. Another is that a higher surface temperature raises the amount of radiation through a decreasing atmospheric window until equilibrium is reached. This is a simple mechanism that is easily stated in a short sentence.

        The mechanism you refer to is singled out as the mechanism for global warming very recently in a 2011 paper by Hansen, Sato, Kharecha, and von Schuckmann, who evidently agree with you. Quoting from the third paragraph of their abstract,

        “This added opacity causes the planet’s heat radiation to space to arise from higher, colder levels in the atmosphere, thus reducing emission of heat energy to space. The temporary imbalance between the energy absorbed from the sun and heat emission to space, causes the planet to warm until planetary energy balance is restored.”

        The other mechanism involving the atmospheric window pervades sections 4.7 and 4.8 of Ray Pierrehumbert’s recent book. Unfortunately, at no point (that I could find) does Ray actually state the sentence above beginning with “Another” in so many words. (Thank god for the PDF preprint, searching the hard copy would have been a Sisyphean labor.) Yet Ray assumes throughout these two sections that the reader understands this warming mechanism intuitively. If he said it in other words somewhere I’d love the section reference. If not then it’s an unfortunate top-level omission from a book that spends much of its time drilling mind-numbingly deeply down into the sorts of gory details that allow RPH to write a 650-page book on a subject that would benefit greatly from an equally insightful but less detailed 150-page book. (The expression “wood for trees” comes to mind.)

        Now I don’t mean to imply that you [Fred] got the first mechanism from the Hansen et al paper. As evidence for this we have your comment on 12/20/2010, eight months ago and predating the Hansen et al paper, to the effect that:

        The atmospheric window surface-emitted radiation is a passive bystander – its radiating altitude can’t change as a function of CO2 concentration, but only as a function of temperature dictated by the effects of CO2 within its absorption bands. All of this means that radiation emitted by the atmosphere is the primary determinant of greenhouse effects via the change in mean radiating altitude.

        So not only did you talk about increasing cold at increasing altitudes but you also were quite explicit back then that this was the only mechanism for global warming.

        My immediately following comment replying to you disputed this as follows.

        You didn’t say anything about the two distinct distributions in that portion of the OLR [where the optical thickness is approximately one]. You seem to be implying that only those [photons] emitted by the atmosphere matter, or did I misunderstand you?

        (The two distinct distributions I was referring to were the atmospheric window where the optical thickness is much less than one, and the opaque region where optical thickness is much greater than one. The result of increasing CO2 is for the second to steadily encroach on the first, gradually shutting down the atmospheric window.)

        Usually you respond, but not to this one. My guess, correct me if I’m wrong, was that you didn’t have anything to say about my question one way or another.

        What I really want to know, Fred, is which mechanism dominates global warming. Is it primarily the increasing average altitude of radiation from the atmosphere, or the decreasing atmospheric window? Both are prime candidates for the mechanism of global warming, the only question is in what proportion? Which does the lion’s share? Or do they divide up the labor evenly?

        As a fully paid-up member of the target audience for John D. Cox’s book “Weather for Dummies” (at least in the sense that I paid full price for it), I assumed back in December that I was the last person in the world (at the time) to juxtapose the above two mechanisms in the one explanation of global warming.

        I’m now wondering if I was the first. Given that there are several thousand climate scientists it seems extremely unlikely, like winning the lottery, but who knows? (That wasn’t a rhetorical question.)

      • Vaughan, I interpret your window mechanism in two ways. The first is equivalent to raising the emitting level, because the window is closing due to nearby CO2 lines which therefore lowers the average emitting temperature. The second, if I read you right, is more controversial if you are saying that a way to compensate for the window closing is for the surface to be hotter, and emit more through the window. If that is what you mean, it implies no convective adjustment to the surface temperature change, which would go against the natural response of convection to maintain its lapse rate based on the new surface temperature,

      • Vaughan – I believe that the increasing radiating altitude and what you call the “decreasing atmospheric window” are the same phenomenon, and that Raypierre has described this effect in both his book and his Physics Today article. At the center of the 15 um CO2 absorption band, opacity is so great that raising the altitude doesn’t reduce the radiating temperature, because it places it in the low stratosphere where we actually see a temperature inversion (if you look at the atmospheric IR profile in his Physics Today article, you see this as a small “spike” in the middle of the trough centered at 15 um, which is wavenumber 667). However, as CO2 increases, the “wings” of the band extending toward 13 and 17 um become increasingly important, because they are now becoming more opaque as well. At those wavelengths, opacity is still low enough so that the radiating altitude is below the tropopause and raising it puts it at a colder altitude where warming is needed for adequate emissions. The higher the CO2 concentration, the further out into the wings this effect turns out to be significant, and the greater these regions away from 15 um contribute to the change in radiating altitude.

        You can refer to this as a “decreasing window”, but it is in these regions that are not totally transparent where the decrease matters, because emission from these regions is being forced to occur from higher, colder altitudes.

        I believe there are two RC posts entitled “a saturated gassy argument” where this is also explained. Spencer Weart did Part 1 and discussed the raised altitude effect, but Raypierre did part 2 and put this in the context of the increasing role of the wings of the CO2 absorption region, which are “window regions” in a relative sense of greater but not total transparency.

        I suppose a summary of the above would state that only wavelengths where radiation must occur from an increasingly colder altitude can contribute to the greenhouse effect, but regions of this nature with more than minimal IR absorption capacity spread further and further from the band center as CO2 increases.

      • To add to the foregoing, it is the quasi-exponential decline in the strength of the absorption coefficients as one goes from the band center wavelength outward that dictate the logarithmic relationship between CO2 concentration and forcing in the range of concentrations relevant to our climate.

      • I believe that the increasing radiating altitude and what you call the “decreasing atmospheric window” are the same phenomenon,

        Fred, you’re missing the point I made in December, that the two mechanisms produce distinct kinds of radiation in the OLR. What I should have added is that the two kinds can be distinguished observationally at satellite altitude (600+ km). Moreover satellites can do so by not just one but two characteristics, linewidths and wavelengths. This makes it more than just a theoretical distinction but one that should be easily measurable above TOA, permitting empirical validation of whatever prediction one might estimate theoretically of their proportion.

        Let’s call rising-altitude radiation RAR and narrowing window radiation NWR. OLR should be largely RAR + NWR, with radiation from high-altitude aerosols as a cross-over kind that may be harder to classify.

        RAR originates from atmospheric GHGs. It is distinguishable from NWR as follows.

        (i) It is instantly recognizable from its evident spectral lines, being from gases. Less instantly, namely decadally, the average pressure on those molecules is slowly decreasing with rising altitude, whence the spectral lines should gradually become narrower due to decreasing pressure broadening.

        (ii) Its mean wavelength increases decadally, because it is radiated by molecules that on average are cooling.

        NWR originates from solids and liquids. Solids are primarily on land or in the sky as aerosols. Liquids are primarily oceans, lakes, and clouds. Precipitation (solid hail and snow and liquid rain) will contribute only to the extent that some radiation may make its way around the edges of clouds. It is distinguishable from RAR as follows.

        (i) NWR is broadband radiation, lacking distinct spectral lines, since radiation from solids and liquids can be viewed as undergoing extreme pressure broadening.

        (ii) Its mean wavelength decreases on a decadal time frame, because it is radiated by molecules that on average are warming. The two exceptions are radiation from clouds, which should condense at the same temperature independently of global warming, and high-altitude aerosols, HAA, for which the blocking mechanism responsible for the increasing altitude of RAR may come into play and cool the average HAA radiation. IIRC surface radiation is only 6% of total OLR so NWR warming may be harder to spot than RAR cooling.

        Recognizing (ii) is dependent on (i), otherwise the mixture of warming and cooling radiation would tend to average out on the spectrum. That is, the lines constituting RAR move down in mean frequency (not the individual lines of course which are at their characteristic frequencies) while the broadband radiation from the NWR moves up.

        Given that this is harder to see than I’d realized, maybe it’s worth a short note somewhere. I’d been assuming it was so obvious that it was well known (one of the handicaps of being an amateur), but then if it were one would expect the above points to been made somewhere, and I’m starting to think maybe they haven’t been.

      • All NWR is RAR – see above.

      • I’ll be out for a few hours. I can expand on this later, but I thought it was already apparent from my earlier comments. GHGs can’t warm the planet by any mechanism other than RAR, and any narrowing of the regions of relative transparency does just that, by making those regions more opaque so that they radiated at colder altitudes.

      • any narrowing of the regions of relative transparency does just that, by making those regions more opaque so that they radiated at colder altitudes.

        This would only be true if all OLR originated from the atmosphere. As some originates from the ground and the sea, whose altitude can’t change, your argument fails.

        There are also clouds that emit upwards, and those don’t get colder, although they don’t get warmer either which is a really nasty effect regulation-wise when you think about it. Clouds work like a busted thermostat, and as such mess up calculations of climate sensitivity in a major way without any involvement of the H2O lines in the HITRAN tables or feedbacks. They’re simply a static obstacle to cooling.

      • All NWR is RAR – see above.

        Fred, I looked about and could not see any refutation of the distinction I pointed out between NWR and RAR as observed from space. Are you claiming that a satellite can’t tell the difference between broadband radiation and emission spectra?

        Why can scientists in a lab easily observe that difference when according to you a satellite can’t?

      • Vaughan – I’m not exactly sure where your misconception lies, as I see it, but I’ll try to outline my perspective as to why all GHG-mediated warming involves a higher, colder radiating altitude – what you refer to as RAR. In a window region where CO2 can’t absorb at all, added CO2 will not change the IR opacity (although a water feedback might do so subsequently). However, in the wings of the main CO2 band centered at 15 um, more CO2 will intercept more photons emanating from the surface. The CO2 was already doing some of this before it increased, and because it was emitting IR upward at a colder temperature than the surface, it was contributing some existing warming to the greenhouse effect. With more CO2, the opacity increases at these wavelengths, and so less IR escapes to space. A radiative balance can only be restored by emissions at a higher, colder altitude, associated with warming of the atmosphere and surface sufficient for that altitude to permit the impeded IR to escape. For this reason, increased opacity in these wing regions is always associated with an increased radiating altitude as part of the climate response.

        You can confirm this from the satellite data, or from one of the Modtran simulations that are available online. For example, if you look at Figure 3 of Raypierre’s Physics Today article, you’ll see that all “bites” out of the OLR are associated with temperatures that are colder than the surface, and progressively colder the greater the size of the bite.

        To summarize, wavelengths from the surface that are not intercepted by CO2 are not encroached upon by added CO2 – there is no “narrowing” of emissions from these wavelengths, which continue to escape from the surface. Those wavelengths that are intercepted are characterized by emissions radiated at altitudes that are progressively higher and colder as CO2 concentrations rise. NWR and RAR are thus two sides of the same coin.

      • One other small point, based on a statement you made much earlier. When the surface is warmed via effects in non-window regions (i.e., wavelengths absorbable to a greater or lesser extent by CO2), radiation emitted by the surface from window wavelengths rises in accordance with the S-B law for black bodies (the surface has nearly 1.0 emissivity in the IR). This is a cooling (temperature-reducing) rather than a warming (temperature-raising) phenomenon.

      • A radiative balance can only be restored by emissions at a higher, colder altitude, associated with warming of the atmosphere and surface sufficient for that altitude to permit the impeded IR to escape.

        You keep saying that, and I keep telling you it’s not true. In addition to the effect you refer to, balance can be restored by increased emissions from the surface at wavelengths within the atmospheric window. The latter increase comes about from increased warming of the surface. Even if the atmosphere did not warm at all, increased warming of the surface would still increase radiation from the ground through the atmospheric window.

        There is another way of looking at what I’m saying that might make it clearer. From this perspective, what I’ve been calling RAR and NWR might better be called gas radiation or GR and solid-liquid radiation or SLR.

        Pick any wavelength among those that radiate significantly at terrestrial temperatures, and restrict attention to radiation at that wavelength. If this is not a GHG wavelength then the associated radiation will be purely SLR and will come from the surface, clouds, aerosols, and precipitation. Increasing GHGs will have no impact on that radiation, though increasing temperature will increase it.

        If it is a GHG wavelength then GR and SLR will mix in some proportion according to how much of the relevant GHGs are present. If enough to make the atmosphere essentially opaque at that wavelength (optical depth significantly greater than unity), little or no SLR will get through and all radiation will be GR. Increasing GHGs won’t affect SLR, but will raise the average altitude from which the GR originates, thereby reducing it (assuming no change in temperature at any altitude). But increasing temperature at any given altitude will increase the GR originating from that altitude.

        On the other side of unit optical depth, namely significantly less than one, some SLR will get through and some GR will radiate as well. Increasing surface temperature will increase the SLR, while increasing GHGs will decrease the SLR (by blocking it) while also decreasing the GR (by raising its effective emission altitude to a cooler part of the atmosphere).

        For example, if you look at Figure 3 of Raypierre’s Physics Today article, you’ll see that all “bites” out of the OLR are associated with temperatures that are colder than the surface, and progressively colder the greater the size of the bite.

        Quite right. But Figure 3a (the only one relevant to Earth) shows only the impact of GHGs, it does not show how increasing surface temperature increases cooling by radiating through the atmospheric window. For example at wavenumbers from 800 to 1000 we would want to see an increase in flux with increasing surface temperature, a relevant mechanism for which he has no figure. In fact I don’t see any reference in Ray’s article to that cooling mechanism, which therefore seems like a serious omission. That Ray omits it doesn’t mean that it is correct to omit it. Perhaps he refers directly or indirectly to it elsewhere in the article, but I couldn’t find it.

        In any event, that cooling mechanism is not associated with any change in altitude, contrary to your claim that “radiative balance can only be restored by emissions at a higher, colder altitude”.

        If someone as clued in as you has this much trouble seeing what I thought originally was an obvious point, then it’s a considerably more subtle point than I’d realized.

      • When the surface is warmed via effects in non-window regions (i.e., wavelengths absorbable to a greater or lesser extent by CO2), radiation emitted by the surface from window wavelengths rises in accordance with the S-B law for black bodies (the surface has nearly 1.0 emissivity in the IR). This is a cooling (temperature-reducing) rather than a warming (temperature-raising) phenomenon.

        Yes!!! That’s the point I’ve been trying to make all along. That’s what I’ve been referring to as NWR. It is not the same thing as RAR, contrary to what you claimed earlier, because it can be distinguished from RAR using satellite telemetry.

        Now that you see it I’m sure you’ll agree with me that it’s a completely obvious point. And one that is completely missing from Ray’s Physics Today article. If it’s in there please point it out.

      • Vaughan, this is the point I tried to address above on the 23rd 12:29am. Bottom line is that you can’t heat the surface without also heating the atmosphere. Convection prevents that from happening by maintaining the lapse rate that includes the surface temperature. The tropospheric temperature is tied to the surface temperature. They can’t be considered independently.

      • The tropospheric temperature is tied to the surface temperature. They can’t be considered independently.

        Let’s accept that for the sake of argument. It does not follow that a given change dT of temperature has the same radiative impact for wavelengths in the atmospheric window as in blocked wavelengths. Differentiating SB, we have dF = 4σT^3 dT. Let dT = 0.1 K. Then at the surface where T = 288 K, we have dF = 5.42 W/m2. At 5 km where T = 238 K say, we have dF = 3.06 W/m2, which is 2.36 W/m2 less than at the surface, even though the temperature increase is the same all the way up the column. The surface dF is relevant to the atmospheric window while the 5 km dF is relevant to where 5 km is the effective altitude of radiation for a blocked wavelength.

        This shows that for a given global dT, ground emission can have a stronger cooling effect than emission from the atmosphere.

        None of this arithmetic is even hinted at in RPH’s article.

      • Vaughan – It’s possible we’ve been talking past each other. My point is that there is only a single mechanism responsible for the warming response to GHGs, not two,and that mechanism involves the rise in radiating altitude to a colder altitude that is insufficient to restore OLR but mediates atmospheric warming until it warms up sufficiently. For CO2, that involves the wings of the absorption band, and so their transparency declines and their opacity increases. If you consider that a “narrowing”, then we agree it’s happening, but accompanied by the rising altitude in all of those wavelengths. In truly “window” wavelengths (no atmospheric absorption but only emission from the surface), there is no narrowing, no encroachment. In a warming scenario, these window regions warm as part of the general surface warming, and consequently emit more radiation. That is not a contributor to warming but to cooling (or in the case of a CO2 increase, a reduced rate of warming), since it warms nothing except outer space. As long as that is not considered to be a separate warming mechanism, there is no disagreement with me, Hansen, or Pierrehumbert (from whom much of my understanding is derived). However, I don’t see anything new or unconventional about this way of thinking about the process.

      • Vaughan, I suspect you were using dT=1 K for your numbers. But, yes, I agree. To get 3.7 W/m2 more out of the top (to counter CO2 doubling) you need of order 1 K warming in the column. This is the classic no-feedback calculation.

      • Vaughan – Just to indicate where I interpreted you to mean one thing while you may have intended something else, your statement that I referred to was:

        “The mechanism you [meaning me] refer to is singled out as the mechanism for global warming”. You then went on to cite an “other mechanism”, referring to the NWR. I interpreted that as an “other” mechanism for global warming. If what you meant was simply that it was a mechanism by which a reduced OLR is returned toward normal, then there is of course no disagreement among individuals familiar with the greenhouse effect principles.

      • Dr. Pratt, I am glad I stumbled on this, I was trying to explain how the direct radiation to space increased with altitude and was not all directly from the surface. Yours is much simpler than mine.

      • Great, Dallas, glad it seemed simple. But be warned that you’re in a logical minefield here. Some of us on this blog think pretty clearly, I would say, but we don’t always agree, which can be taken to mean that we aren’t always right. Which in turn means that this stuff is really hard to get right.

        It’s rather like the situation with quantum mechanics prior to the definitive Heisenberg-Schroedinger solution in 1925-26, when people were stumbling around in the dark with different theories. Ironically Einstein, the greatest physicist of the first half of that century, was never able to reconcile himself with their solution, even decades later. Science isn’t always obvious, even after the fact.

        Incidentally direct radiation to space as a function of altitude is a bit tricky. On the one hand, GHGs at higher altitudes are at lower T, and SB says they radiate T^4 energy which is therefore a lot less. On the other, they have fewer GHGs above them to capture their photons. As a function of increasing GHGs, the former turns out to dominate (picture piling up grains of white sand on black cardboard until no black is visible, and then keep on piling more and more sand and watch it grow in height).

        Complicated stuff, to be taken with a grain of (white) salt.

      • LOL, it is complicated and I am far from fully understanding. What I was looking at is water which is far more interesting than CO2.

  77. Mr. Pratt says:”But climate is not climate change, which science attributes not to changing lapse rate but to increased GHGs and also increased aerosols at cloud altitudes.”

    This is the underlying question. Is the climate changing outside what has been seen in the past? If not then blaming CO2 for a normal rise in temperture is irresponsible.

    • This is the underlying question. Is the climate changing outside what has been seen in the past? If not then blaming CO2 for a normal rise in temperture is irresponsible.

      I fully agree, this is a key question. One can’t simply point to the last four decades of rising temperature and blame it all on CO2 without a good understanding of the natural contributors to climate change. What if 90% of that rise is naturally caused?

      Those who accept the role of CO2 in raising the surface temperature, as well as those who don’t, are at risk of seeing climate change as an all-or-nothing thing: either CO2 or nature is the cause.

      The only scientific way to approach this question is to have ways of distinguishing the natural and anthropogenic contributions. This is what I’ve been focusing on in my own analysis of the temperature and CO2 records: do they contain enough information to let one see the difference? Obviously the last four decades don’t since both CO2 and temperature have been rising in more or less a straight line and one must consider the possibility that nature has been raising the temperature on a more or less straight line too during that period (which in fact I believe to be the case). However I believe the last sixteen do, and that they show that two-thirds of the last four decades of temperature increase are of anthropogenic origin. I have a paper on this that hopefully will be ready for release soon.

      • mkelly´s statement actually makes no sense. Attribution of climate change to CO2 has nothing to do with warming being unprecedented our ¨outside what has been seen in the past.¨ It is like saying that you need to kill more people than Stalin in order to be convicted of murder.

      • Pol Pot is a better example since he was an environmentalist

      • “Attribution of climate change to CO2 has nothing to do with warming being unprecedented our ¨outside what has been seen in the past.¨

        Chris I think the whole “unprecedented” meme has been a horrible mistake on the part of people who believe in AGW. I think it has led us down the path to put more support behind some nascent science ( dendro) that it merits merely for the iconography that science can produce.. with the right graphsmanship.

      • Chris, the reason mkelly’s statement makes no sense to you is that his question is ambiguous: it could be interpreted as meaning “unprecedented” as you suggest, but then his next sentence would make no sense, as you point out.

        It could also be interpreted as asking whether the climate was changing via some new factor that had never been present before. With that reading his next sentence makes perfect sense: if all the change is via preindustrial factors then I would agree with him that blaming it on CO2 is irresponsible.

        When a question is ambiguous and the context makes one of the meanings nonsensical, I don’t accuse the questioner of not making sense, instead I pick the other meaning. In this case the other meaning makes perfect sense, and as I said I think it’s an excellent question.

      • Chris Colose says
        …”mkelly´s statement actually makes no sense. Attribution of climate change to CO2 has nothing to do with warming being unprecedented our ¨outside what has been seen in the past.¨ It is like saying that you need to kill more people than Stalin in order to be convicted of murder.”…

        A woman with a squint in her eye arrives in a village.
        Two days later a cow dies mysteriously.
        Another villager notices that the milk has gone off.
        A child catches a mysterious disease.
        All these things happen within a fortnight.
        What caused all the ill luck to befall the village?
        Well, some suggest that the woman with the evil squint might be a witch.
        A wise old man says that there is nothing unusual about the run of bad luck and he has records to prove its happened several times in the past.
        This explanation seems to complicated for some!
        Well you know all about Salem I’m sure.
        Lindzen tells us that the climate has risen by 0.7C in the last 150 years.
        Even during that period there have been several cooling periods.
        The last 15 years shows no increase.
        The modern witch finders have demonized the trace gas CO2.
        Hedge funds profit from market instability join in the wheeze and back the irrational reactionaries.
        Research scientists know which way the wind blows.
        Some sceptics make life awkward by bringing up inconvenient facts.
        The sceptics are quickly branded as liars and of low moral character and the witch finders long for the days of burning at the stake.

    • mkelly:

      “This is the underlying question. Is the climate changing outside what has been seen in the past? If not then blaming CO2 for a normal rise in temperture is irresponsible”

      This is NOT the question. erase everything mankind knows about the climate prior to 1850. You still have the question. What will be the likely outcome of our GHGs policy.

      1. we know that addiing GHGs warms the planet. It does not cool the planet. There is no evidence that it cools the planet and no WORKING theory that says it will. By working theory I mean a theory that actually gets used succesfully in engineering.

      2. we know to a first order that doubling C02 will raise the temperature approximately 1C. We know this from verified measurements of the instaneous forcing of c02 and from basic energy balance. There is no
      credible assualt on those measurements and no credible alternative
      to our understanding of energy balance

      3. We have reason to believe that the effect will be greater than 1C per doubling.

      Knowing that, it matters LITTLE what has occurred in the past. What we want to know is what happens if we keep dumping GHGs. That is the question. Its been warmer in the past and colder in past. More ice and less ice. Thats never been the question.

      • A. C. Osborn

        It must be the question, unless you can positively prove that what happened in the past has less influence than GHG increases and currently you can’t prove that.

      • by that standard ‘what happened in the past” i would have to prove that the big bang has less influence. “In the past” is vague and unspecified. Further, I do not have prove anything about yesterday to predict that if I drop a ball right now that it will fall. here is what we know:

        1. It was colder in 1850 than it is today. I assume you believe in in LIA.
        2. Energy balance physics dictates that if you increase the forcing the temperature will increase. Whether that forcing is from a more energetic sun or other sources.
        3. GHGs increase forcing. We know this from observation and theory.
        4. from 1850 to now forcing via GHGs has increased.
        5. The theory predicts warming, we see warming.

        Ruling out other causes is not a requirement for prediction or knowledge.
        I cannot rule out that some other magical force caused the ball to drop. I dont need to rule out what happened in 1000AD to explain and predict how balls drop. In fact I can use my present day know

      • steve,
        The past- you know the terretria past that has some allegedly reliable records. I am aware of no one wanting to go back to the Big Bang regarding climate.
        We see nominal warming, barely if at all, outside the range of pre-1850 or MWP temps.
        We see no trends of significant change in storms, floods, droughts, slr, etc.
        Ocean pH is not measurably or reliably changed.
        So please tell us why this great and expensive CO2 mania has been worth the substantial money and emotion expended on it.

      • A. C. Osborn

        3. We have reason to believe that the effect will be greater than 1C per doubling.
        Who’s this “We” batman, the latest studies all point to less than 1C.
        Why oh Why would you be worrried about 1C when the earth experiences swings of over 20C in some areas on an annual basis.
        Some areas can even experience that kind of swing on a daily basis.

      • steven mosher,
        The idea that we forget the well documented past because of our theory today is not going to fly with most people.

      • Steve all gases dissipate heat. Our lives depend on that idea. JCurry could not dry her hair , you could not heat your home, my car would over heat and fail. My dog’s panting would be for nothing. Gases in and of them selves cannot add heat to anything.

      • mkelly,
        My hope is that we who keep pointing this out are going to find out that we simply misunderstood what was being said, and not that what is implied by the believer’s typical reaction is what they actually believe.
        But this seems to be like the non-answer to questions about successful mitigation policies. I am tired of the AGW community either insulting skeptics for questioning AGW orthodoxy or for giving non-answers that assume that asking about the much predicted apocalypse are not for skeptics to ask.
        steven, who I respect greatly, simply brushed off my question about where the changing climate is as non-falsifiable. I asked, politely, how this could be and unless I missed the answer I got none.
        When I ask about mitigation policies, which the AGW community is so focused on, I get ignored.
        When asked about the actual avidence of OA, we get circular answers that ignore reality in favor of explanations.
        And so forth.

      • andrew adams

        hunter,

        My hope is that we who keep pointing this out are going to find out that we simply misunderstood what was being said, and not that what is implied by the believer’s typical reaction is what they actually believe.

        We have had a number of recent threads about the GHE, some with 1,000+ comments, including some very informative ones from serious experts in the field. So when Steven says “we know that adding GHGs warms the planet. It does not cool the planet” those of us who have been paying attention know very well what that means in practice, and from Steven’s previous comments I’m pretty sure he does as well. And if others insist on pointing out that “gases in and of themselves do not add heat to anything” we can only include that they not only misunderstood what was said but possibly did not make a serious attempt to do so.

      • aa,
        They warm the planet in the same sense that a blanket warms someone in bed: By moderating the loss of heat.
        No one caught on fire from a blanket being on them.
        That you immediately dive into the bs assertion that skeptics who do not agree with you are not serious says a lot more about you and the many AGW believers who seem unable to grant goodwill or sincerity to those who dare disagree with you.

      • andrew adams

        hunter,

        I think you misunderstood my comment. The point is there is NO disagreement – you, I, Steven Mosher (and AFAIK mkelly as well) all understand perfectly well that the GHE keeps the earth warm by reducing the amount of energy loss, not by adding heat to the system, yet you insist on trying to manufacture disagreement by making an entirely specious objection to Steven’s comment.

      • Andrew here is Mr. Mosher’s comment “1. we know that addiing GHGs warms the planet. It does not cool the planet.”

        My point is that there are no examples I can think of where gases do anything other than cool. They cool the surface. Other than green house effect can you name an example of a gas warming?

      • a.a.,
        It seems most of the disagreement is the use of down welling radiation with the exclusion of other means of thermodynamic heat transfer. For example during convection, warm air rising is replaced with cooler air falling to replace it. That falling air is warmer in the day than it would be at night, so more energy is transferred to the surface than if the air was cooler at night. For true down welling long wave, there are clear windows to the surface if the temperature of the body or the emission spectrum of the gas is in that spectrum. For CO2 and water vapor there is little if any emission in that window, so interaction with other molecules is a very significant part of the energy to or reduced transfer from, the surface.

        Another issue is that the majority of the energy in the atmosphere is directly from solar absorption or latent heat rising. So per the TFK paper, approximately 222 W/m^2 is due to incoming solar and 111 W/m^2 due to surface radiation and conductive (thermals) inducted convection, if you look. TFK makes no attempt to include convective replacement, the sensible heat returned with precipitation or differentiate solar versus outgoing radiation produced atmospheric temperature.

        No one disagrees that the sky has a temperature, there is just more than one way its temperature can impact surface temperature.

      • all gases dissipate heat. JCurry could not dry her hair , you could not heat your home, my car would over heat and fail. My dog’s panting would be for nothing.

        Sometimes I can find a meaning for what you say that makes sense. This wasn’t one of those times. Please try to be more ambiguous.

      • Apologize Mr. Pratt, I was responding to Mr. Mosher’s comment ( It does not cool the planet.) that gases (GHG’s) don’t cool. All gases cool (dissipate heat) and that idea is used in our everyday life and by animals. There are no examples I can think of where a gas adds heat in and of itself.

        By giving examples of heat dissipation I thought I was not being ambiguous.

      • By giving examples of heat dissipation I thought I was not being ambiguous.

        You weren’t, that was my complaint. I was struggling to see these as examples of gases dissipating heat, but if I really try I can see that the fan in a hair dryer serves to dissipate the heat in the heating coil, similarly the air circulated through a furnace dissipates the heat in the heat exchanger, the radiator fan dissipates heat in the radiator by blowing air over it, and the dog’s panting carries the high-latent-heat water vapor molecules away from its tongue (aka evaporative cooling).

        The problem I was having was that the usual ways of explaining how these mechanisms perform their intended function aren’t naturally explained as “gases dissipating heat.” The first two serve to add heat (to the hair and the home), not dissipate it, the third (unless you have an air-cooled car) uses water cooling, and evaporative cooling by panting is not ordinarily thought of as “gases dissipating heat” since even though water vapor is a gas it is not carrying away sensible heat, only latent heat.

        An example I would have had no problem with would be a CPU fan, which dissipates the (finned) CPU’s heat by blowing cold air over it. But this only works if the resulting hot air then moves away from the CPU. Similarly cooling the planet by blowing cool air over it would only work provided the resulting hot air leaves the planet; unfortunately for that theory, but fortunately for us (if we want to keep breathing), it doesn’t.

      • Steven – have you read John Eggart’s engineering approach?:

        http://johneggert.files.wordpress.com/2010/09/agw-an-alternate-look-part-1-details1.pdf

      • This is NOT the question. erase everything mankind knows about the climate prior to 1850. You still have the question.

        Quite right. That was how I was interpreting mkelly’s question: just look at 1850-now. The fluctuations in the first half of that period are out of proportion to CO2 changes back then. To my thinking this makes mkelly’s question very reasonable. To what do you attribute those significant fluctuations? And now you appear to be dismissing them as no longer significant. Why?

        (Note that I’m not asking Arfur Bryant’s question, why worry about an average rise of 0.06 °C per decade over those 160 years? This is NOT the question.)

        2. we know to a first order that doubling C02 will raise the temperature approximately 1C.

        Quite right. lb(368/326) = 0.17, whence 1C/doubling should raise the temperature 0.17 °C between 1970 (326 ppmv) and 2000 (368 ppmv), lb being binary logarithm. In fact it rose 0.5 °C. So by your reasoning up to this point, CO2 has accounted for one-third of observed warming during those three decades.

        3. We have reason to believe that the effect will be greater than 1C per doubling.

        Based on what? If on models, how do you know the models weren’t being tuned until they agreed with observation (standard practice when modeling, that’s the whole point of least-squares fitting), subject to the assumption that CO2 is 100% responsible for global warming since 1970? If on feedbacks, what do you estimate as the magnitude of those feedbacks, and why do you trust your estimate? If on something else, what?

        I believe global warming is happening, that much (notice I did not say all) of it is due to CO2, and that it’s a serious problem.

        However it should be clear from the temperature record since 1850 that we don’t understand natural climate change anywhere near well enough to say whether the global land-sea temperature rise since 1970 is 40%, 70%, 100%, or 130% due to CO2. (130% is the case when the temperature would have fallen without our CO2 contribution.)

        I would be delighted to see any argument to the contrary.

      • The warming from 1910 to 1940 matches a period when the number of sunspots almost tripled, and some estimates of the corresponding TSI change suggest that can account for the part of this warming that CO2 can’t. However since 1940 the sunspots haven’t been changing substantially until perhaps now, so I believe the solar variation, CO2 and aerosols can account for the various changes we saw over the last century.

      • Jim D said, “The warming from 1910 to 1940 matches a period when the number of sunspots almost tripled, and some estimates of the corresponding TSI change suggest that can account for the part of this warming that CO2 can’t.”

        The issue with the TSI change is that the “some estimates” have mostly been revised to show less change in TSI. That does not mean the sun didn’t do it, just that the mechanism for the sun doing it is poorly understood. This current solar cycle will shed some more light on that issue. As it stands now, some old reconstructions, Lean et al 2000 and Hoyl(?) seem to be used selectively when Wang and Lean revisions, Svalgard and a few other new reconstructions seem to agree very well that there was much less TSI variation.

      • Almost certainly, “global brightening: – increased penetration of solar irradiance to the surface due to a declining atmospheric aerosol burden – contributed to the post-1970 warming. Martin Wild has written extensively on this. Conversely, between 1950 and the late 1970s, “global dimming” due to increased aerosols masked some of the warming due to CO2.

        In the early part of the century, solar irradiance is likely to have played a larger role in the observed warming, but CO2 and other greenhouse gases were probably significant contributors as well.

      • Spoken like a true believer in the axiom, “there are no unknown physical phenomena of significance to climate change.”

        How come you left ohmic heating induced by geomagnetic secular variation off your list of candidates? That’s hardly an unknown phenomenon in the geophysical literature, nor an insignificant one either, yet it seems largely if not completely unknown in the climate science literature.

      • I think these are the main forcings (with aerosols being an albedo forcing) that between them can explain the long-term trajectory of the temperature. I am not interested on short-term fluctuations like ENSO and volcanoes, and if we don’t need more forcings, I wouldn’t look for any.

      • A quick Google Scholar search for “geomagnetic secular variation climate” turned up over 3,000 hits. So I’m not sure that it is unknown in the climate science literature.

      • Geomagnetic variation is good example of what is not included in standard analysis of solar forcing . There is indeed a wide literature in this area of investigation both with the causal mechanisms by ionization,
        energetic bremsstrahlung etc eg Chernekov,Sakharov,Zeldonovich Crutzen and others.eg K. Semeniuk et al.2010

        Ionization by energetic particle precipitation in the atmosphere is an ubiquitous feature of the Sun-Earth system. The work by Warneck (1972), Swider and Keneshea (1973) and Crutzen et al. (1975) pioneered research into influence of energetic particle precipitation on the chemistry of the atmosphere through the enhancement of NOx.
        Following this early work, Solomon and Crutzen (1981) and Solomon et al. (1981, 1983) pointed out a coupling mechanism whereby thermospheric NOx could affect the stratosphere.

        The effects of changes in Geomagnetic activity seem to cause changes in surface temeprature eg Rozanov et al 2005,A. J. G. Baumgaertner
        et al.2010

        A strong (weak) Northern Hemisphere vortex is known to be associated with a positive (negative) Northern Annular Mode (NAM) index;
        our simulations exhibit a positive NAM index for strong geomagnetic activity, and a negative NAM for weak geomagnetic activity. Such NAM anomalies have been shown to propagate to the surface, and this is also seen in the model simulations. NAM anomalies are known to lead to specific surface temperature anomalies: a positive NAM is associated with warmer than average northern Eurasia and colder than average east
        ern North Atlantic. This is also the case in our simulation. Our simulations suggest a link between geomagnetic activity, ozone loss, stratospheric cooling, the NAM, and surface temperature variability.

        This is not to say that these mechanisms are the prominent cause of temperature excursions,however they are not trivial either.Understanding non standard mechanisms may be useful in reducing the sensitivity problem which does seem to be irreducible after 30 years and with all its random consequences.

      • This is not to say that these mechanisms are the prominent cause of temperature excursions,however they are not trivial either.

        Agreed. Moreover there are interesting correlations between GSV and solar cycles (no reason to expect that!) as well as much-longer-period ocean temperature oscillations, in particular AMO and PDO.

        Aerosols play an obvious and major role in climate. However the only long-term aerosol effects I’m aware of are the ones caused by industry, including long-term mitigation efforts such as power station scrubbers, engine emission controls, etc., and possibly World War II and its aftermath, which was a significant departure from routine industry. Volcanos are for the most part short-term, no longer than El Nino events and shorter than solar cycles.

        If from the temperature record you filter out all significant frequencies with periods 11 years and less, hardly anything remains except oscillations of period half a century or more, plus a rise in temperature well correlated with the increase in fuel consumption that we have good estimates of going back centuries. There appear to be no significant temperature cycles with periods between 12 and 50 years!

        This makes it easier than one might expect to tell just how much of the recent variation is associated with fuel consumption and how much with these long-term oscillations, which are large but don’t show any obvious correlation with growth of human population and hence are very likely of natural origin.

      • Perhaps cosmic ray generated aerosols may better explain some of the climate variations?

        http://wattsupwiththat.com/2011/08/24/breaking-news-cern-experiment-confirms-cosmic-rays-influence-climate-change/#more-45793

      • Cosmic rays are yet another candidate for a mechanism for long-period oscillations we can observe in the temperature record.

        But whatever the origin of those oscillations, one cannot jump to the conclusion that they’re all that matters and that CO2 is having no impact.

      • Aerosols are problematic in the SH and particularly in the NZ record.this was noted in Wild 2008 eg

        Evidence for a decrease of Solar duration SD from the 1950s to
        1990 and a recovery thereafter was also found on the
        Southern Hemisphere at the majority of 207 sites in
        New Zealand and on South Pacific Islands [Liley, 2009].

        Liley [2009] pointed out that the dimming and brightening
        observed in New Zealand is unlikely related to the direct
        aerosol effect, since aerosol optical depth measurements
        showed too little aerosol to explain the changes. On the
        basis of sunshine duration measurements he argued that
        increasing and decreasing cloudiness could have caused
        dimming and brightening at the New Zealand sites.

        Interesting problems now arise ie the NZ T record is in a decreasing trend albeit bound by canard excursions.

      • Very true, Cosmic rays impact on clouds just adds another variable to the equation. With CO2 doubling perhaps causing a 1% change, there are quite a few things that could contribute 0.1% or so that can become significant.

      • Vaughan – John Eggert puts forward a persuasive and coherent argument that increases in CO2 above 200ppm will not impact on climate. His conclusion:

        “Above 200 ppm atmospheric concentration of CO2 there is no increase in the greenhouse affect due to CO2, and changes to human emissions of CO2 will have no affect on climate.”

        It is referenced and well written – I’d value your comments.

        http://johneggert.files.wordpress.com/2010/09/agw-an-alternate-look-part-1-details1.pdf

      • Is Eggert’s emissivity plot for a gas at constant temperature? If so then it does not apply to the atmosphere, whose temperature decreases with height at the lapse rate of around 10 °C/km.

        One result of this is that as CO2 increases, a greater proportion of the emission comes from colder parts of the atmosphere. Hence instead of converging to some limit as shown in Eggert’s graph, the apparent emissivity of the atmosphere decreases with increasing CO2. By this mechanism more heat is blocked when more CO2 is added.

        For the constant-temperature case, how did Eggert arrive at his graph? By measurement or calculation?

      • “we know that addiing GHGs warms the planet”

        No we don’t.

        We do know the planet surface got warmer while we added ghg’s, but correlation is not causation, and theory is inadequate as we don’t understand all the feedbacks, or even the main one.

  78. Chiefhydrologist,

    You said “Back radiation is a non-physical concept – only net radiative counts in energy transfer. ”

    I, 100% agree with you, Robert, that ‘only net radiative counts in energy transfer’. Whicheverway Fred Moolten looks at it, photon particles or IR wave radiation, they both have to comply with their properties when transferring energies. A weaker wave such as the backradiation (if exists) have to comply with wave properties, i.e. it can be interfered, it can be reflected, it can be absorbed, it can be refracted etc. when facing an upcoming stronger wave. Fred Moolten and other AGWers, think that they can violate these wave properties, penetrate these stronger waves and reach the stronger radiating body and heat it up, which is of course, nonscience. Whenever they (the warmists) unable to handle with waves, they resort to photons as emitted particles. Fine, but photon has mass even though extremely light, has all the particle properties, i.e. it can be reflected, deflected, absorbed etc. When a weaker photon rays facing the stronger photon rays coming towards them, they are absorbed (same wavelengths), deflected (different wavelengths), etc. long before these weaker photons reached the body of radiating stronger photons. And yet, these warmists insist that these photons can penetrate thru the stronger photon rays and reached the warmer body. This is of course unscientific violating particle theory and particles properties.

    What the warmists are also wrong is that adding more CO2 to atmosphere actually cool down the atmosphere more quickly, not warming up due to CO2 has better radiation effect and cools down N2 and O2 in the air by molecular collisions. Burning more fossil fuels actual cools down atmosphere and not warming the atmosphere. The warmists and the alarmists are very wrong in this aspect.

    Since the CO2 has increased a lot, we should prepare for the cold coming and we should stop using fossil fuel for over-cooling the Earth. No, the 400ppm or even 1000ppm, we are safe as the Earth will not be too cold.

    • Chief Hydrologist

      Sam,

      What you have to understand is that photons behave as both particles and waves. I don’t really have time right now – but I actually said that was that greehouse gases slowed cooling to space. But radation moves in all directions including downward – without violating the 2nd law of thermodynamics as long as the net flux is from the warmer to the cooler.

      When I have some more time – we can go into this a little more.

      Cheers

  79. Chief Hydrologist presented questions related to this thread in another thread. I think the questions, my answers and further comments are close to the topic that Vaughan, Fred and some others have pursued in this thread. These subthreads can be found at the following links

    http://judithcurry.com/2011/08/22/can-we-make-good-decisions-under-ignorance/#comment-103501

    http://judithcurry.com/2011/08/22/can-we-make-good-decisions-under-ignorance/#comment-103552

  80. ‘Local’ Energy Equilibrium of the ocean (W/m^2):
    Solar in = evaporation out + conduction/convection out + net radiative out = input to atmosphere
    170 = 78 + 24 + ~66-70 = ~170

    ‘Local’ Energy Equilibrium of the atmosphere (W/m^2):
    Input from the ocean below + Direct absorption from the Sun above = radiation out to space
    170 + 67 = ~235-239

    The upward and downward components of the radiative flux in the atmosphere cause convection within the atmosphere which inflates it and supports the adiabatic lapse rate. This is partly why the surface air temperature and ocean surface are at a temperature ~33C warmer than an atmosphere devoid of radiatively active gases would be (There would still be some adiabatic lapse due to the thermal difference between equator and poles which would drive a reduced atmospheric circulation, according to some).

    Surface conduction, pressure, humidity, convection, and the adiabatic lapse rate supported by the radiative flux, over the course of several billion years, has kept ocean and atmosphere in a dynamic thermal equilibrium whereby the ocean surface is slightly warmer than the surface air, to enable the rate of energy loss the ocean needs to maintain in order to remain in local energy equilibrium.

    Both the ocean surface and surface air are warmer than the upper atmosphere where radiation to space takes place at the rate demanded by solar input and the S-B BB equation because of the equilibrium demanded by adiabatic lapse. The radiative flux plays an important role in that equilibrium, but small changes in the balance of the components of the flux will not measurably change the bulk temperature of the ocean at the multi-decadal scale because the thermal inertia of the ocean and its evaporation rate lies behind the longwave radiative flux component’s differential of ~66-70W/m^2.

    The tail does not wag the dog.

    On the other hand, the Sun warms the oceans top layers very effectively and directly, and the ocean’s response time to solar input is much quicker, on the order of hours. It is therefore reasonable to assume that the empirically measured reduction in low tropical cloud cover 1980-1998 is far more likely responsible for the increase in ocean heat content and surface temperature measured by buoys, satellites and engine coolant intakes over the same period.